Das Zwillingsparadoxon der Speziellen Relativitätstheorie (kurz SRT) führt ja immer gern zur Verwirrung. Das liegt nicht nur daran, dass es auf den ersten Blick unserer Intuition widerspricht, sondern vermutlich auch, dass ganz verschiedene Erklärungen des Paradoxons kursieren, die auch nicht alle wirklich zusammenpassen.

Das “Paradoxon”

Gleich vorneweg: Das Zwillingsparadoxon trägt seinen Namen nicht zu Recht – an ihm ist nichts paradox, auch wenn es auf den ersten Blick so aussehen mag. Hier nochmal in Kurzform die Idee des Paradoxons:

Die beiden Zwillingsschwestern (oder gleichaltrigen Freundinnen) Teresa und Serena müssen sich leider voneinander verabschieden: Während Teresa auf der Erde bleibt, macht sich Serena auf eine Reise nach Alpha Centauri, mehr als vier Lichtjahre von der Erde entfernt. Sie nutzt ein Hochgeschwindigkeitsraumschiff, das mit 80% der Lichtgeschwindigkeit unterwegs ist. Bei Alpha Centauri angekommen, winkt sie einmal den dort wohnenden Aliens, stellt fest, dass es ihr dort doch nicht gefällt, und macht sich auf den Rückweg zur Erde.

Bei derartig hohen Geschwindigkeiten kommt es laut SRT zum Phänomen der Zeitdilatation: Teresa sieht Serena verlangsamt, für sie sieht es so aus, als würde die Zeit für Serena langsamer laufen, so dass Serena auch langsamer altert. Da in der SRT aber Bewegungen immer nur relativ sind, sieht auch Serena Teresa verlangsamt, für sie sieht es so aus, als würde Teresa langsamer altern. Wenn Serena auf der Erde ankommt, sollte also Teresa erwarten dass sie selbst älter ist als Serena, aber umgekehrt sah Serena ja auch Teresa immer verlangsamt, sie sollte also erwarten, dass Teresa jünger ist als sie. Aber spätestens, wenn sich die beiden wieder direkt gegenüberstehen, sollte hoffentlich eindeutig zu entscheiden sein, wer nun älter ist.

Dieser scheinbare Widerspruch ist das sogenannte Zwillingsparadoxon. Um es gleich vorwegzunehmen: Die Situation ist in der Tat nicht symmetrisch, und es ist Teresa (die auf der Erde geblieben ist), die am Ende älter ist, während Serena jünger bleibt.

The Bad

Gelegentlich (beispielsweise bei dieser “Lernhelfer”-Seite – ich hab da mal ne Nachricht hinterlassen…) liest man folgende Erklärung des Zwillingsparadoxons: Um bei Alpha Centauri umzukehren, muss Serena beschleunigen. (Soweit richtig) Diese Beschleunigung benötigt eine Kraft – Serena wird in Ihren Sessel gepresst. (Auch richtig) Diese Kraft erscheint Serena wie eine Schwerkraft, sie kann beim In-den-Sessel-Gedrückt-werden  nicht ohne Weiteres unterscheiden, ob sie durch ein Schwerefeld gedrückt wird oder durch die Beschleunigung, auch wenn sie natürlich weiß, dass sie gerade ihre Raketen gezündet hat. (Immer noch richtig) Weil diese Beschleunigung also äquivalent zu einer Schwerkraft ist, kann man das Zwillingsparadoxon nur dann auflösen, wenn man die SRT zu einer Theorie verallgemeinert, die auch die Schwerkraft enthält, also zur Allgemeinen Relativitätstheorie.

Nachtrag (21.2.22): Die Lernhelferseite wurde anscheinend korrigiert und jetzt wird dort korrekt über den Wechsel der Bezugssysteme argumentiert. Beschleunigungen werden allerdings immer noch erwähnt, trotzdem großes Lob.

Diese letzte Erklärung ist leider falsch. Die SRT hat überhaupt keine Probleme damit, beispielsweise die Zeitdilatation zu beschreiben, die auf eine Beobachterin wirkt, die gerade beschleunigt – das ist sogar vergleichsweise einfach. (Wer’s genau wissen will: Der Dilatationsfaktor ist zu jedem Zeitpunkt gegeben durch die momentane Geschwindigkeit, man muss also bei einer sich ändernden Geschwindigkeit lediglich aufintegrieren.) Nur Beschleunigungen, die durch die Schwerkraft zu Stande kommen, lassen sich mit der SRT nicht erfassen, alle anderen schon. (Das liegt daran, dass es laut ART eigentlich gar keine Schwerkraft gibt – wenn ihr mehr darüber wissen wollt, klickt rechts bei den Artikelserien, da gibt es sehr viele Artikel zur ART, die das erklären. Oder ihr geduldet euch bis kurz vor Weihnachten…) Ein anderes schönes Beispiel, wie die SRT Beschleunigungen korrekt berücksichtigt, findet ihr hier bzw. hier.

Dass die Erklärung mit der ART nicht funktioniert kann man übrigens auch auf eine ganz andere Weise sehen. Die SRT stammt aus dem Jahr 1905, erst 1907 hat Einstein die Logik des Äquivalenzprinzips erkannt, wonach sich eine gleichförmige Beschleunigung und ein Schwerefeld nicht unterscheiden lassen, und dann dauerte es bis 1915, bis die ART fertig war. Würde die Erklärung stimmen, dann hätten die Physikerinnen also 10 Jahre lang eine Theorie akzeptiert die in sich widersprüchliche Aussagen macht. Auch wenn manche Leute ja seltsame Ideen davon haben, wie Physikerinnen denken und glauben, dass die gesamte Physik mal eben schnell durch eine pensionierte Elektroingenieurin im Alleingang revolutioniert werden kann, die nie Physik studiert hat (ja, solche mails habe ich jede Menge in meinem Archiv) – mit der Realität hat das nicht viel zu tun. (Und falls ihr hier in den Kommentaren wieder einmal irgendwelche unsinnigen “Widerlegungen” der Relativitätstheorie posten wollt, lasst es einfach.) Eine Theorie, die derart eklatante innere Probleme mit sich bringt, hätte sicherlich niemand akzeptiert. Das Zwillingsparadoxon sollte sich innerhalb der SRT lösen lassen.

The Ugly

Kommen wir zu den Erklärungen, die zumindest richtig sind. Man kann schlicht und einfach die Gleichungen der SRT nehmen (in diesem Fall die sogenannten Lorentz-Transformationen) und die Sache einfach Schritt für Schritt nachrechnen. Die Rechnung findet ihr (mit einem kleinen Extra-Aufenthalt am Zielplaneten) ausführlich beispielsweise bei Relativitätsprinzip.info. (Dass ich diese Rechnung hier unter der Überschrift “Ugly” anführe, ist natürlich keine Kritik an Joachim Schulz, seiner Internetseite oder seinem Blog. Der weiß auch, dass man das intuitiver erklären kann, hat sich aber trotzdem netterweise die Mühe gemacht, mal alles formelmäßig auseinanderzudröseln.) Heraus kommt in der Tat, dass – in unserem Beispiel – Teresa am Ende älter ist als Serena. (Wer Zahlen mag: Bei 80% Lichtgeschwindigkeit und einer Entfernung von 4,3 Lichtjahren nach Alpha Centauri sind für Serena insgesamt nur 6,4 Jahre vergangen, für Teresa dagegen 10,8.)

Warum ich diese korrekte Herleitung mit dem Wort “ugly” belege? Weil ich das bloße Nachrechnen von Formeln immer wenig intuitiv finde – oft folgt man dem mathematischen Formalsimus, sieht am Ende ein Ergebnis, hat aber keine wirkliche Inuition gewonnen, was da eigentlich passiert.

Interessant ist an der Herleitung aber folgendes: Da ist tatsächlich von Beschleunigungen die Rede, die aber als unendlich schnell angenommen werden. Serena würde also bei Alpha Centauri ihre Geschwindigkeit sofort umkehren. (In der Rechnung ist tatsächlich noch ein Zwischenaufenthalt drin, der tut aber nix zur Sache.)

Man könnte also meinen, dass es tatsächlich die Beschleunigungen sind, die für den Unterschied der beiden Zeitabläufe verantwortlich ist. Das ist auf den ersten Blick auch sehr plausibel: Serena muss ja die Düsen ihres Raumschiffs zünden, wird dabei in ihren Sessel gedrückt etc. wie oben schon beschrieben. Aber ist das wirklich so? Dazu können wir uns ein kleines Gedankenexperiment ausdenken: Nehmen wir an, Serena wäre eine künstliche Intelligenz, ein Computerprogramm, das auf einem Supercomputer abläuft. (Und ja, ich nehme hier an, dass ein Computer ein Bewusstsein haben könnte – spielt für die Physik keine Rolle, macht die Sache aber anschaulicher.) Stellen wir uns einen zweiten, baugleichen Supercomputer vor, der sich auf einem Raumschiff befindet, das von einem entfernten Stern Richtung Erde unterwegs ist und zeitgleich mit Serena bei Alpha Centauri ankommt. Serena überträgt jetzt ihr Computerprogramm (ihr Bewusstsein) per Funk auf diesen anderen Computer. Dabei vergeht für sie keine Zeit, Zeit vergeht für sie nur auf den beiden Wegstrecken nach Alpha Centauri und zur Erde. Altert jetzt plötzlich ihr Bewusstsein anders, weil es nicht beschleunigt wurde? Nehmen wir an, Serena kopiert ihr Bewusstsein – Serena 1 wurde mit einem Funksignal übertragen, Serena 2 dagegen kehrt ihr Raumschiff zeitgleich um. Es wäre schon ziemlich seltsam, wenn für beide die Zeit vollkommen unterschiedlich vergehen würde und Serena 1 plötzlich irgendwie einen Zeitsprung macht. (Laut Wikipedia ist eine ähnliche Überlegung unter dem Namen “Drei-Brüder-Ansatz” bekannt.)

Nachtrag, in der Hoffnung, noch deutlicher zu machen, wo das Problem der Beschleunigung steckt: Nehmen wir an Serena fliegt auf dem Hinweg mit einem der Brüder mit (dem, der nach Alpha Centauri unterwegs ist), auf dem Rückweg mit dem anderen, der zur Erde fliegt. (Bruder 1 bleibt die ganze Zeit auf der Erde.) Zu jeder Zeit geht ihre Uhr gleich der Uhr des jeweiligen Bruders, bei dem sie an Bord ist. Die gesamte für sie verstrichene Zeit ist auf jedem der beiden Schiffe gleich der Zeit, die für den jeweiligen Bruder vergeht – in beiden Fällen sind es 3,2 Jahre, zusammen 6,4. Die gesamte für sie verstrichene Zeit ist also exakt gleich der Zeit, die man erhält, wenn man jeweils die Zeiten aufaddiert, die sie mit den Brüdern verbracht hat. Trotzdem sollen wir irgendwie argumentieren, dass es die Beschleunigung beim Übersetzen von einem Schiff aufs andere war, die dafür verantwortlich ist, dass für sie nicht die 10,8 Jahre von Teresa vergehen?

1 / 2 / Auf einer Seite lesen

Kommentare (1.039)

  1. #1 Markweger
    4. Juli 2018

    Das ist doch Schöne an der Relativitätstheorie.
    Selbst die Experten kommen nach 100 Jahren immer noch zu unterschiedlichen Erklärungen.
    Aber in ein paar hundert Jahren wird man sich geeinigt haben. Es ist überhaupt nichts zu erklären, so wenig wie beim ptolemäischen Weltbild halt auch.

  2. #2 Bbr1960
    4. Juli 2018

    Klar ist, dass „The Bad“ überflüssig ist, da das Problem innerhalb der SRT gut lösbar ist. „The Bad“ ist außerdem schlecht, weil sie den spontanten Wechsel nicht beschreiben kann. Aber ist die Erklärung deshalb auch falsch? Wenn man mit der ART ausrechnet, welche Zeitverschiebung dieses „gefühlte“ Gravitationsfeld erzeugt, müsste man doch zum selben Ergebnis kommen. Schließlich sind die Theorien ja konsistent. So nach dem Motto „warum einfach, wenn‘s auch umständlich geht“.

  3. #3 MartinB
    4. Juli 2018

    @Markweger
    Das Problem ist wie so oft, dass Versuche, physikalische Theorien und ihre Konsequenzen anschaulich zu erklären, oft nicht so gut funktionieren, wenn man nicht sehr genau aufpasst.

    @Bbr1960
    Ja, die Erklärung ist insofern falsch, weil Beschleunigungen ja irrelevant sind, wie das Beispiel mit dem Bewusstseinstransfer oder mit den drei Brüdern zeigt. Natürlich kommt man trotzdem zum richtigen Ergebnis, wenn man es durchrechnet (so steht’s ja auch bei Wiki in dem beschleunigungs-Beispiel, das könnte man sicher auch auf den ART-Formalismus übertragen), aber es geht am Kern der Sache schlicht vorbei. Entscheidend ist in diesem Fall die Interpretation der Gleichungen, die Gleichungen selbst sind unstrittig.

  4. #4 MartinB
    4. Juli 2018

    @Bbr1960
    Wntscheidend istt, dass die Rechnung im Formalismus der ART nichts neues enthält – man braucht sie nicht, um das Problem zu lösen, aber genau das behauptet ja z.B. die zitierte Internetseite.

  5. #5 Tox
    5. Juli 2018

    Naja, Kern der Sache ist, dass Serena das Bezugssystem wechselt, bzw. dass ihr System nicht während des gesamten Flugs ein Inertialsystem ist. Das ist gleichbedeutend damit, dass sich ihre Geschwindigkeit bezüglich eines Inertialsystems während des Flugs ändert. Für physikalische Objekte sind Änderungen der Geschwindigkeit stets mit Beschleunigungen verbunden. Also sind, solange Serena ein physikalisches Objekt ist, Beschleunigungen sehr wohl relevant.

    Bei den drei Brüdern bin ich mir nicht sicher, ob ich verstanden habe, worin eigentlich das vermeintliche Paradoxon bestehen soll. Beim klassischen Zwillingsparadoxon ist das vermeintliche Paradoxon doch, dass aus der Sicht jedes Zwillings die Schwester langsamer altert, so dass man (falls beide gleichwertige Beobachterinnen sind) durch Wechsel der Beobachterin unterschiedliche Resultate für den Altersvergleich am Ende erhält. Bei den drei Brüdern hat man weder den Altersvergleich am Ende noch mehrere Beobachter die die Situation (vermeintlich) unterschiedlich beschreiben würden.

  6. #6 Abdul Alhazred
    5. Juli 2018

    Zur Bezeichnung “Paradoxon”. wikipedia sagt:

    “Ein Paradoxon (…) ist eine Feststellung, die anscheinend zutrifft, doch dem allgemein Erwarteten, der herrschenden Meinung oder Ähnlichem zuwiderläuft oder beim üblichen Verständnis der betroffenen Gegenstände bzw. Begriffe zu einem Widerspruch führt. Die Analyse von Paradoxien kann zu einem tieferen Verständnis der betreffenden Gegenstände bzw. Begriffe oder Situationen führen, was den Widerspruch im besten Fall auflöst.”

    Passt doch. Dass Zwillinge ungleich alt sein sollen, widerspricht der allgemeinen Erwartung, eine Analyse der Situation führt zu einem tieferen Verständnis und löst den (vermeintlichen) Widerspruch auf. Ich hab nie verstanden, wieso immer mal wieder gesagt wird, das Zwillingsparadoxon trage seinen Namen zu Unrecht – bitte nicht “Paradoxon” mit “Antinomie” verwechseln!

  7. #7 MartinB
    5. Juli 2018

    @Tox
    Eigentlich dachte ich,ich hätte das erklärt – wie gesagt, stell dir vor, Serena würde ihr Bewusstsein übertragen. Dann wirken keine Kräfte, trotzdem nimmt sie weniger Zeit wahr. Der “Wechsel des Bezugssystems” bedeutet schlicht, dass Serenas Weltlinie einen Knick aufweist – die damit möglicherweise verbundenen Beschleunigungen sind letztlich irrelevant.
    Ist im Raum übrigens genauso: Um erst nach Norden, dann nach Osten zu gehen, muss ich ja auch beschleunigen, um die Richtung zu wechseln. Trotzdem würden wir nicht sagen, dass diese Beschleunigungen der Grund dafür sind, dass der Umweg länger ist.
    Hmm, das sollte ich vielleicht oben noch einbauen.

    @Abdul Alhazred
    Ich verstehe Paradoxon immer als Widerspruch (die zweite Variante der Wiki-Definition). Auf jeden Fall trägt das Paradoxon seinen Namen wirklich nicht zu recht – nur bei sehr naiver Anwendung der Regeln der SRT kommt man ja zu einem Widerspruch, das “Paradoxon” ist eher eine Denkfalle, man kann leicht in die Irre geführt werden, wenn man nicht genau aufpasst, aber das ist auch alles.

  8. #8 schlappohr
    5. Juli 2018

    Ich erkläre mir das Zwillingsparadoxon auf andere Weise. Ich gehe dabei von einem Denkmodell aus, von dem ich zum ersten mal in Brian Greenes Buch “Der Stoff aus dem der Kosmos ist” gelesen habe (wir haben darüber vor längerer Zeit hier schon einmal diskutiert, Florian hat das Buch auch rezensiert).
    Greene sagt, dass sich alles im Universum immer mit Lichtgeschwindigkeit durch die Raumzeit bewegt, aber diese Bewegung verläuft zum größten Teil entlang der Zeitachse. Ein ruhender Körper bewegt sich sozusagen also mit Lichtgeschwindigkeit durch die Zeit.
    Wenn sich der Körper hingegen durch den Raum bewegt, bekommt der Geschwindigkeitsvektor eine Komponente in Raumrichtung, aber der Betrag bleibt gleich (nämlich c), sodass die Komponente in Zeitrichtung kleiner wird. Im Extremfall einer Bewegung mit Lichtgeschwindigkeit durch den Raum wird die Zeitkomponente des Bewegungsvektors Null, wie es z.B. bei Photonen der Fall ist.

    Ok, nehmen wir an, Theresa und Serena beginnen bei Punkt A in der Raumzeit ihr Experiment. Serena steigt in ihr Raumschiff und fliegt weg, Teresa bleibt auf der (als ruhend angenommen) Erde. Das Experiment endet am Raumzeitpunkt B mit Serenas Rückkehr.
    Abgesehen von ihrer geringfügigen Eigenbewegung (mit dem Bus zur Arbeit etc.) bewegt sich Theresa laut Greene fast vollständig mit Lichtgeschwindigkeit entlang der Zeitachse von A nach B. In Serenas Bezugssystem hingegen hat der Vektor eine massiv große Komponente in Raumrichtung, wodurch die Komponente in Zeitrichtung kleiner wird. Serena legt daher in der Zeitrichtung eine kleinere Strecke zurück als Theresa, daher vergeht für sie subjektiv weniger Zeit auf ihrer Reise von A nach B. Erkauft hat sie sich dies mit einer Bewegung durch den Raum.

    Ich weiß nicht, ob diese Erklärung einen Sinn ergibt, aber mit Greenes Denkmodell wird es für mich sehr verständlich. Insbesondere zeigt sich hier, dass es vollkommen unsinnig ist, Raum und Zeit als zwei unabhängige Größen zu betrachten.

  9. #9 roel
    5. Juli 2018

    @Schlappohr Ich versteh deine Erklärung so, dass wenn sich alles mit Lichtgeschwindigkeit bewegt. der Lichtstrahl, dann doppelte Lichtgeschwindigkeit haben müsste. Und das geht nicht.

  10. #10 schlappohr
    5. Juli 2018

    @roel:

    Nein, ein Lichtstrahl besteht aus Photonen, die haben keine Eigenzeit, also nur eine Bewegung durch den Raum mit Lichtgeschwindigkeit.

  11. #11 MartinB
    5. Juli 2018

    @schlappohr
    Ich habe diese Erklärung schon öfter gehört, kann damit aber nicht so viel anfangen – ich hake immer an der Frage, wie man sich “durch die Zeit bewegt” – die Änderung der Zeit mit der Zeit, so wie Bewegung im Raum die Änderung der Zeit mit dem raum ist?
    Dann muss man auch wieder unterschiedliche Bezugssysteme angucken (denn für Serena ist es ja Teresa, die verlangsamt ist), und dann finde zumindest ich es einigermaßen wenig intuitiv.
    Zumal das auch nicht wirklich zu den Gleichungen passt, wo man ja eher die Vierergeschwindigkeit angucken würde, aber deren Zeitkomponente wird ja für bewegte Objekte größer, nicht kleiner. Raum und Zeit gehen in die Länge eines Vektors ja mit unterschiedlichen Vorzeichen ein, das passt mit der Greene-Erklärung nicht so gut zusammen.

    @roel
    Ich glaueb, man muss hir aufpassen, von welcher Zeit man redet – in schlappohrs Erklärung ist die Eigenzeit des Objekts gemeint, und die gibt es für ein Photon nicht.

  12. #12 roel
    5. Juli 2018

    @Schlappohr und MartinB

    “Eigenzeit” hatte ich nicht gelesen. Sorry.

  13. #13 schlappohr
    5. Juli 2018

    @MartinB

    ja, ich habe das zugegeben nicht im Detail durchdacht, aber mit dieser Erklärung erscheint es mir intuitiv irgendwie verständlich, wenn man Greenes Modell einmal voraussetzt. Was Bewegung durch die Zeit bedeutet, müsste man sicher noch genauer definieren, das hat Greene leider auch nicht gemacht. Das ist das Problem mit dieser Art von Büchern: Wenn es zu sehr ans Eingemachte geht, hören sie auf.
    Aber wird in der SRT die Zeitkoordinate nicht auch mit der Lichtgeschwindigkeit c multipliziert und bekommt dadurch die Einheit einer Länge? Angenommen, man multipliziert die Zeit t mit c und erhält daraus eine Strecke l. Wenn diese Strecke mit der Geschwindigkeit c zurückgelegt wird, dauert das genau t. Bei halber Lichtgeschwindigkeit dauert es 2t usw. Das wäre doch eine Defintion der Bewegung durch die Zeit?

  14. #14 schlappohr
    5. Juli 2018

    @roel

    Mein Fehler. Ich habe Eigenzeit gemeint aber nicht geschrieben.

  15. #15 MartinB
    5. Juli 2018

    “Das ist das Problem mit dieser Art von Büchern”
    Aber das ändert sich ja bald, muhaha 🙂

    Deine Definition mit der Zeitkoordinate verstehe ich nicht so ganz – welche Zeitkoordinate (oder besser Zeitspanne) willst du denn nehmen? Wenn z.B. für mich der Lorentzfaktor gleich 2 ist, ich also relativ zu dir um 50% verlangsamt bin, dann bewege ich mich ja nicht mit halber Lichtgeschwindigkeit – die Zeitdilatation ist ja nicht linear. Oder ich habe deine Konstruktion nicht verstanden

  16. #16 Michael
    5. Juli 2018

    Es ist nicht schwer das “Zwillingsparadoxon” zu verstehen. Hier ist eine gute Erklärung, die man sehr leicht nachvollziehen kann:

    https://www.av8n.com/physics/twins.htm

  17. #17 roel
    5. Juli 2018

    @MartinB “Aber das ändert sich ja bald, muhaha”

    Ich bin gespannt!

  18. #18 Tox
    5. Juli 2018

    @MartinB
    Die Variante mit der Bewusstseinsübertragung halte ich für problematisch, weil z.B. nicht klar ist (jedenfalls nicht mir), ob während der Übertragung (die ja nicht instantan passieren kann) ein Bewusstsein existiert. Überhaupt halte ich es für fraglich ob es sinnvoll ist ein so komplizieres Konzept wie Bewusstsein hier ins spiel zu bringen. Der Effekt tritt ja auch bei simpler unbelebter Materie auf.

    Desweiteren sage ich ja nicht, dass die Beschleunigungen “der Grund” für den Effekt sind. Sondern nur, dass es den Effekt für physikalische Objekte ohne Beschleunigungen nicht gibt, da es ohne Beschleunigungen keine Änderungen der Geschwindigkeit und damit keine Knicke in der Weltlinie gibt. Daher würde ich Beschleunigungen nicht als irrelevant betrachten.

  19. #19 MartinB
    5. Juli 2018

    @Tox
    Aber wie schon gesagt, wir betrachten auch im Raum Beschleunigungen als irrelevant obwohl ich meine Richtung im Raum auch nur ändern kann, wenn ich beschleunigt werde.
    Oder in der Geometrie von Raum und Raumzeit ausgedrückt: Eine Linie mit Knick ist was anderes als eine gerade Linie, vollkommen egal, wodurch der Knick zu stande kommt. Dass ein materielles Objekt den Knick nur machen kann, wenn es beschleunigt, ist zwar korrekt, aber für die Geometrie egal.

    @MIchael
    Sagt die Erklärung dort was anderes als meine hier?

  20. #20 Michael
    5. Juli 2018

    @MartinB

    Kann ich nicht sagen, da ich Ihren Beitrag nicht gelesen haben.

  21. #21 Michael
    5. Juli 2018

    Edit: haben > habe

  22. #22 MartinB
    5. Juli 2018

    @Michael
    Eigenwillige Art, auf nem Blog zu kommentieren, ohne in den Text zu schauen, den man kommentiert.

    @Tox
    Noch ein Nachtrag, der auch nochmal das mit den drei Brüdern klarer macht, hoffe ich: Nehmen wir an Serena fliegt auf dem hinweg mit einem der Brüder mit, auf dem Rückweg mit dem anderen. Zu jeder Zeit geht ihre Uhr gleich der Uhr des jeweiligen Bruders, bei dem sie an Bord ist. Die gesamte für sie verstrichene Zeit ist auf jedem der beiden Schiffe gleich der Zeit, die für den jeweiligen Bruder vergeht – in beiden Fällen sind es 3,2 Jahre, zusammen 6,4.

    Die gesamte für sie verstrichene Zeit ist also exakt gleich der Zeit, die man erhält, wenn man jeweils die Zeiten aufaddiert, die sie mit den Brüdern verbracht hat. Trotzdem sollen wir irgendwie argumentieren, dass es die Beschleunigung beim Übersetzen von einem Schiff aufs andere war, die dafür verantwortlich ist, dass für sie nicht die 10,8 Jahre von Teresa vergehen?

    Ergibt für mich nicht wirklich viel Sinn, das so zu sehen.

  23. #23 MartinB
    5. Juli 2018

    PS: Habe das mal in den Haupttext übernommen, ich hoffe, das macht das Problem noch etwas deutlicher.

  24. #24 Michael
    5. Juli 2018

    @ MartinB

    Eigenwillig hin oder her, wenn ich darf, dann möchte noch ein Buch von Taylor & Wheeler zum Thema empfehlen:

    Spacetime Physics : Introduction to Special Relativity

  25. #25 Bbr
    5. Juli 2018

    @MartinB bgzl. #11

    Ich bin hier mal auf eine Seite gestoßen, wo alles auf der Grundlage erklärt wird, dass man sich in der Raumzeit immer mit c bewegt.

    https://www.relativity.li/de/epstein/lesen

    So richtig weiß ich nicht, was ich davon halten soll. Was dahinter steckt ist natürlich, dass die Norm der Viergeschwindigkeit=c ist.

    Zumindest die “Herleitung” der Lorentztransformation aus dem Epstein-Diagramm ist aber ein klassischer Zirkelschluss, da man ja die Lorenztranformation braucht, um zur Norm der Vierergeschwindigkeit zu kommen.

  26. #26 MartinB
    5. Juli 2018

    @Bbr
    Ja klar, die Norm der Vierergeschwindigkeit ist c, aber bei der bewegten beobachterin ist die Nullkomponente ja gamma*c, also größer als c, deswegen passt das ja gerade nicht zu dem Bild. (Muss so sein, weil ja räumliche nd zeitliche Komponente mit umgekehrtem Vorzeichen eingehen, wenn also ne Raumkomponente dazukommt, muss die Zeitkomponente größer werden, damit die Länge c bleibt.)

    Wo genau steht denn dazu was in dem Epstein-Buch? Auf die schnelle hab ich das nicht gefunden.

  27. #27 Tox
    5. Juli 2018

    @MartinB

    Aber wie schon gesagt, wir betrachten auch im Raum Beschleunigungen als irrelevant obwohl ich meine Richtung im Raum auch nur ändern kann, wenn ich beschleunigt werde.

    Auch im Raum würde ich Beschleunigungen nicht als irrelevant betrachten, eben genau deshalb weil es ohne sie keine Richtungsänderungen gibt.

    Eine Linie mit Knick ist was anderes als eine gerade Linie, vollkommen egal, wodurch der Knick zu stande kommt.

    Selbstverständlich. Davon wie die Beschleunigung zustande kommt rede ich ja auch nicht. Ich sage nur dass der Effekt nicht auftritt, wenn es keine Beschleunigung gibt. Vielleicht verstehen wir unter “irrelevant” unterschiedliche Dinge? Ich finde es jedenfalls seltsam, eine Sache irrelevant zu nennen, wenn der zu betrachtende Effekt ohne diese Sache nicht auftritt.

    Die gesamte für sie verstrichene Zeit ist also exakt gleich der Zeit, die man erhält, wenn man jeweils die Zeiten aufaddiert, die sie mit den Brüdern verbracht hat. Trotzdem sollen wir irgendwie argumentieren, dass es die Beschleunigung beim Übersetzen von einem Schiff aufs andere war, die dafür verantwortlich ist, dass für sie nicht die 10,8 Jahre von Teresa vergehen?

    Hier verstehe ich das “trotzdem” nicht. Was genau widerspricht sich hier? Und den Ausdruck “dafür verantwortlich sein” finde ich auch problematisch. Es gibt eine ganze Reihe von Bedingungen, die erfüllt sein müssen, damit der Effekt auftritt. Das Vorhandensein einer Beschleunigung ist eine davon. Ich würde keine einzelne als für den Effekt verantwortlich bezeichnen.

    Was genau wollen wir eigentlich erklären? Wie oben angedeutet, ging ich davon aus, dass der wesentliche interessante Punkt beim Zwillingsparadoxon ist, dass die Situation eben nicht symmetrisch zwischen Teresa und Serena ist, obwohl man das vielleicht zunächst wegen des Relativitätsprinzips erwarten könnte. Und die Symmetrie wird eben genau durch die Beschleunigung gebrochen.

    Wie wäre es mit folgender Erweiterung: Eine weitere Schwester/Freundin setzt sich in eine Kiste mit undurchlässigen Wänden (wie Einsteins Fahrstuhl) und wird dann zufällig entweder von Serena mit auf die Reise genommen oder bleibt bei Teresa zu Hause. Am Schluss wird die Kiste geöffnet und sie soll entscheiden, ob sie bei Teresa oder Serena war. Anhand der Beschleunigung kann sie diese Unterscheidung treffen (und wenn sie nicht weiß, wie weit Serenas Zwischen-Reiseziel entfernt ist, ist das auch die einzige Möglichkeit).

    Daraus würde ich folgern, dass es genau die Beschleunigung ist, die die Symmetrie bricht. Und daher würde ich die Beschleunigung als relevant für den zu erklärenden Effekt betrachten.

    Wie wäre es mit folgender “Kompromissformulierung”: Der Effekt tritt ohne Beschleunigungen nicht auf, aber für das Verständnis des Effekts und eine (näherungsweise) Berechnung ist es nicht notwendig, die Beschleunigungen detailiert zu kennen (statt des vollen Beschleunigungsprofils a(t) reicht die Gesamtänderung der Geschwindigkeit Δu).

  28. #28 roel
    5. Juli 2018

    @Michael Ich empfehle dir eine Rechtschreibprüfung.

  29. #29 MartinB
    5. Juli 2018

    @Tox
    “Was genau widerspricht sich hier?”
    Wenn sie mit Bruder 2 fliegt, vergeht für sie dieselbe zeit wie für Bruder 2, nämlich 3,2 Jahre. Wenn sie mit Bruder 3 fliegt ebenfalls. In der Summe vergehen für sie also 6,4 jahre, die Summe der zeit, die für die Brüder vergeht. Wie sie von Bruder 2 zu Bruder 3 gekommen ist, spielt offensichtlich keine Rolle (solange es schnell genug ging), weil ihre Uhr immer exakt synchron zu einer der Brüder-Uhren ist und weil die für sie vergangene Zeit exakt gleich der Summe der Zeit der beiden Brüder ist.

    Ich verstehe nicht, wie man da argumentieren soll, dass “eigentlich” wie für Teresa 10,8 Jahre vergangen wären, und dass es die Beschleunigung ist, die dafür sorgt, dass es nicht so ist. Tut mir Leid, ich verstehe die Logik wirklich schlicht nicht.

    “Und die Symmetrie wird eben genau durch die Beschleunigung gebrochen.”
    Die Symmetrie wird dadurch gebrochen, dass eine Weltlinie eine gerade Linie ist, die andere nicht. Dass eine person, die versucht dieser Weltlinie zu folgen, dafür beschleunigen muss, ist in meinen Augen genauso unerheblich, wie es unerheblich ist, dass ich beschleunigen muss, wenn ich erst nach Norden und dann nach Osten gehe. Ist der Weg erst nach Norden dann nach Osten länger als der direkte Weg, weil ich einmal beschleunigen muss? Oder ist es einfach die Geometrie des Raums, die das bewirkt?

  30. #30 H.H.Voynich
    Nordrhein-Westfalen - Hilden
    5. Juli 2018

    @Tox:
    “Der Effekt tritt ohne Beschleunigungen nicht auf […]”

    Tut er aber.
    Wir können doch zwei (bzw. drei) baugleiche Uhren nehmen, die über einen Sender und Empfänger verfügen und einen Datenspeicher, in welchem sie regelmäßig ihren eigenen und den von der jeweils anderen Uhr empfangenen Zeitstempel loggen.
    U1 bleibt hier, U2 fliegt zu Proxima, wo sie sehr dicht an U3 vorbeifliegt, welche mit gleichem Geschwindigkeitsbetrag in entgegengesetzte Richtung – Ziel:Erde – unterwegs ist.
    Zum Zeitpunkt der Begegnung synchronisiert sich U3 anhand U2, was sicherlich instantan möglich sein sollte.
    Außerdem sendet U2 ihr gesamtes Log an U3, was nicht instantan passieren muss, da die gespeicherten Daten sich ja nicht mehr ändern.
    Danach kann man doch sagen, daß U3 den Zustand von U2 vollständig übernommen hat?

    U3 erreicht dann die Erde, und wir können die beiden Logs ansehen.
    Bei U1 sieht das etwa so aus (links der eigene Zeitstempel, rechts der empfangene):

    1 – 0,8
    2 – 1,6
    3 – 2,4
    4 – 3,2
    5 – 4
    6 – 4,8
    7 – 5,6
    8 – 6,4
    9 – 7,2
    10 – 8

    Das Log von U3 sieht hingegen so aus:

    1 – 0,8
    2 – 1,6
    3 – 2,4
    4 – 3,2
    5 – 7,6
    6 – 8,4
    7 – 9,2
    8 – 10

    In diesem Szenario kam keine Beschleunigung vor, das Phänomen bleibt aber das selbe.

  31. #31 MartinB
    5. Juli 2018

    @H.H. Voynich
    Das ist ja genau die Idee hinter dem 3-Brüder-Szenario.

  32. #32 MartinB
    5. Juli 2018

    @Tox
    Vielleicht liegt es auch daran, dass wir etwas unterschiedliches unter dem zu verstehenden Effekt verstehen:
    Du sagst: “Für das Zwillingsparadoxon ist entscheidend, dass eine Person einer Weltlinie mit Knick folgt, und das geht nicht ohne Beschleunigung.”
    Ich sage “Für das Zwillingsparadoxon ist entscheidend, dass eine Linie in der Raumzeit, die einen Knick hat, insgesamt eine kürzere Eigenzeit hat als eine gerade Linie. Du hast natürlich recht, dass eine *Person* die einer solchen Linie folgen will, dazu beschleunigen muss, aber es ist nicht die Beschleunigung, die für die Länge der Linie verantwortlich ist (was ja auch die 3-Brüder-Variante zeigt), sondern schlicht die Geometrie der Raumzeit.”

  33. #33 H.H.Voynich
    5. Juli 2018

    @MartinB: genau.
    Ich wollte es noch mal in eigenen Worten formulieren, um mich (und Tox) davon zu überzeugen, dass das Phänomen auch ohne Beschleunigung auftritt.
    Habe es eigentlich schon zu kompliziert gemacht: Sender, Empfänger und Log sind nicht nötig, um den Effekt zu demonstrieren.

  34. #34 Bbr1960
    5. Juli 2018

    @MartinB: Der interessante Teil des Epstein-Buchs fängt bei 4.2 an.

    Was mich etwas stört ist die verwendete Sprache mit „Mythos“ und „Dogma“. In der Naturwissenschaft hat man Postulate und keine Dogmen. Und die grundlegenden Postulate der STT sind numal das Relativitätsprinzip und die Konstanz der Lichtgeschwindigkeit. Was er als „Dogma“ bezeichnet ist daraus herleitbar.

    Wie auch immer: Ich finde Minkowsi-Diagramme alles andere als anschaulich. Erklärungen, die mit kürzesten Wegen in der Raumzeit arbeiten, sind zumindest für mich viel besser.

  35. #35 Tox
    5. Juli 2018

    @MartinB

    Ich verstehe nicht, wie man da argumentieren soll, dass “eigentlich” wie für Teresa 10,8 Jahre vergangen wären,

    Das argumentiere ich doch gar nicht. Ich wüsste auch gar nicht was das heißen soll. (Vielleicht: “Wie viel Zeit wäre für Serena vergangen, wenn sie nicht umgekehrt wäre aber trotzdem zur Erde zurückkehrt?” Solange wir im simplen Minkowski-Raum sind, wiedersprechen sich die beiden Teile, so dass die Frage nicht sinnvoll ist.)

    Die Symmetrie wird dadurch gebrochen, dass eine Weltlinie eine gerade Linie ist, die andere nicht.

    Woran erkennst du, ob eine Weltlinie gerade ist? Doch gerade daran, ob entlang ihr keine Beschleunigung wirkt. (Oder formaler ausgedrückt: Ihre Tangente wird parallel transportiert.)

    Für das Zwillingsparadoxon ist entscheidend, dass eine Linie in der Raumzeit, die einen Knick hat, insgesamt eine kürzere Eigenzeit hat als eine gerade Linie.

    Wieder stellt sich die Frage, woran du den Knick und das gerade sein erkennst, wenn nicht durch die Beschleunigung. Man könnte natürlich sagen “eine Linie in der Raumzeit ist gerade, wenn sie eine Geodäte ist”. Aber dann wird die Aussage zur Tautologie. Geodäten sind ja gerade dadurch definiert, dass sie (lokal) extremale Länge haben.

    … aber es ist nicht die Beschleunigung, die für die Länge der Linie verantwortlich ist (was ja auch die 3-Brüder-Variante zeigt), sondern schlicht die Geometrie der Raumzeit.

    Da ist wieder dieses “verantwortlich sein”…

    Sobald man für eine noch unbestimmte Kurve in der Raumzeit für jeden Wert im Parameterintervall die Beschleunigung festgelegt hat, hat man damit auch die Länge der Kurve festgelegt. Ich weiß nicht ob das eine ausreichende Verantwortlichkeit ist.

    @H.H.Voynich
    Wie ich bereits oben in Kommentar #5 schrieb, sehe ich in dieser Form das vermeintliche Paradoxon nicht. Denn es gibt ja ganz offensichtlich keine zwei unterschiedlichen Standpunkte die vermeintlich gleichwertig sind aber zu unterschiedlichen Vorhersagen führen.

  36. #36 Tox
    5. Juli 2018

    @H.H.Voynich (#33):
    Wir scheinen unterschiedliche Dinge unter “dem Phänomen” zu verstehen. Könntest du mir deine Definition des Phänomens in der Drei-Brüder-Situation mitteilen und am besten auch, warum dies vermeintlich ein Paradoxon ist?

  37. #37 H.H.Voynich
    5. Juli 2018

    @Tox:
    Offenbar gibt es verschiedene Ansichten darüber, was eigentlich das “Paradox” sei.
    In #6 fand Abdul Alhazred es m.E. schon paradox, dass Zwillinge plötzlich unterschiedlich alt sind. Darin liegt aber nicht das Problem, das ließe sich auch anders erreichen. Zum Beispiel vergeht laut ART die Zeit für den Zwilling langsamer, der sich in einem stärkeren Gravitationsfeld befindet. Da sind sich aber alle Beobachter einig darüber, wessen Uhr langsamer geht – also kein Paradox.
    Anders in der SRT: die besagt, dass die “bewegte” Uhr langsamer geht.
    Nun kann aber jeder Beobachter mit gleichem Recht den Standpunkt vertreten, selbst zu ruhen, so dass es der andere sein muss, dessen Uhr langsamer geht als die eigene – also der andere am Ende der Jüngere sein müsste. Es sieht so aus, als wäre die Situation symmetrisch – bis sich die beiden treffen und feststellen, wer von beiden nun wirklich älter ist.
    Wäre die Situation wirklich symmetrisch, dann müsste jetzt jeder der beiden sehen können, dass der jeweils andere weniger gealtert ist. Was offensichtlich nicht sein kann.

  38. #38 Tox
    5. Juli 2018

    @H.H.Voynich
    Ok, exakt das verstehe ich auch unter dem Zwillingsparadoxon.

    Aber das passiert doch bei den drei Brüdern nicht. Es gibt dort keine zwei Beobachter die sich zweimal treffen und somit keinen Uhrenvergleich entlang zweier Wege durch die Raumzeit. Es gibt stattdessen drei Beobachter, die sich paarweise jeweils einmal treffen und dabei Information austauschen.

  39. #39 H.H.Voynich
    6. Juli 2018

    @Tox:
    Also würdest Du mir bei meinem obigen Satz widersprechen:
    “Danach kann man doch sagen, daß U3 den Zustand von U2 vollständig übernommen hat?”

    Denn für das Phänomen sollte es ja unerheblich sein, ob U2 selbst zurück reist, oder ob eine 100%ige Kopie von ihr das tut …?

  40. #40 Tox
    6. Juli 2018

    @H.H.Voynich
    Der Satz stimmt dann, wenn du “Zustand” so definierst, dass die Geschwindigkeit durch die Raumzeit (bezüglich eines beliebigen Koordinatensystems) nicht Teil des Zustands ist. Denn diese wird ja offensichtlich nicht kopiert.

    Ich würde daher U3 nicht als 100%ige Kopie von U2 betrachten.

  41. #41 MartinB
    6. Juli 2018

    @Tox
    “Woran erkennst du, ob eine Weltlinie gerade ist? Doch gerade daran, ob entlang ihr keine Beschleunigung wirkt. (Oder formaler ausgedrückt: Ihre Tangente wird parallel transportiert.)”
    Richtig. Aber wie das 3-Brüder-Modell zeigt, kann man ja dieselbe Linie als aus zwei Weltlinien zusammengesetzt denken und bekommt denselben Effekt. Nicht jedes geometrische Gebilde in der Raumzeit muss ja eine Weltlinie sein. Das ist das, was ich in #32 sagen wollte.
    Natürlich kannst du argumentieren, dass ich dazu dann bereits die Tatsache als Grundlage nehme, dass die Geodäte die Linie maximaler Eigenzeit ist – aber das ist ja auch genau das, was ich argumentiere: Meiner Ansicht nach ist es am anschaulichsten und intuitivsten, wenn man die Geometrie als Basis nimmt (deswegen ja auch der Text mit Anna Meise und Teresa Mite), statt sich mit irgendwelchen Lorentz-Trafos oder ähnlichem rumzuärgern.
    Und in dieser Betrachtungsweise ist die Linie, die Serena zurücklegt, schlicht eine Linie, die aus zwei geraden Stücken besteht. Dass diese Linie länger ist, liegt schlicht an der Geometrie.

    Man kann übrigens auch daran sehen, dass diese Argumentation schlüssiger ist, weil sie auch für raumartige Abstände funktioniert, wo die Linien gar keine Weltlinien sein können, die Situation ist da ja ansolut symmetrisch (bis auf den Vorzeichenwechsel bei der Raum- und Zeitkomponente).

  42. #42 Tox
    6. Juli 2018

    @MartinB

    Aber wie das 3-Brüder-Modell zeigt, kann man ja dieselbe Linie als aus zwei Weltlinien zusammengesetzt denken und bekommt denselben Effekt.

    Wie ich bereits schrieb, bin ich der Meinung, dass man im Drei-Brüder-Modell nicht exakt denselben Effekt hat. Jedenfalls gibt es meiner Ansicht nach dort die eigentliche “Paradoxie” des Zwillingsparadoxons nicht.

    Und ob man nun einen Bereich der Kurve hat, in dem eine Beschleunigung vorliegt, oder einen Punkt in dem die Kurve nicht differenzierbar ist, spielt meiner Meinung nach keine wesentliche Rolle. Jede stetige Kurve die nur an endlich vielen Stellen nicht differenzierbar ist lässt sich schließlich beliebig genau (bezüglich einer passenden Norm) durch glatte Kurven approximieren.

    Nicht jedes geometrische Gebilde in der Raumzeit muss ja eine Weltlinie sein.

    Selbstverständlich. Ich habe nie gegenteiliges behauptet.

    Und in dieser Betrachtungsweise ist die Linie, die Serena zurücklegt, schlicht eine Linie, die aus zwei geraden Stücken besteht. Dass diese Linie länger ist, liegt schlicht an der Geometrie.

    Aber selbst wenn man die Definition einer “geraden Linie” als gegeben annimmt, hat man allein durch die Feststellung, dass Serenas Kurve aus zwei geraden Linien zusammengesetzt ist noch nicht den entscheidenden Unterschied zwischen Serenas und Teresas Kurven erfasst. Denn auch Teresas Kurve lässt sich in zwei gerade Linien zerlegen. Entscheidend ist, dass bei Teresa die beiden Teilwege die selben Richtungen/Geschwindigkeiten haben und bei Serena nicht.

    Dein “Ein Umweg hat eine andere Länge als direkte Weg” klingt für mich nach einer Null-Aussage, die den eigentlichen Effekt nicht erklärt.

    In der geometrischen Beschreibungsweise ist für mich ist der entscheidende Punkt beim Zwillingsparadoxon die Frage warum Teresas Weg der direkte Weg und Serenas Weg ein Umweg ist (wenn man einen “Umweg” für zeitartig getrennte Endpunkte als Weg definiert, der kürzer als der direkte Weg ist).

  43. #43 MartinB
    6. Juli 2018

    @Tox
    “Und ob man nun einen Bereich der Kurve hat, in dem eine Beschleunigung vorliegt, oder einen Punkt in dem die Kurve nicht differenzierbar ist, spielt meiner Meinung nach keine wesentliche Rolle. ”
    Dem stimme ich zu. Die Tatsache, dass das keine Rolle spielt, kann man aber zumindest als Hinweis darauf werten, dass das, was da genau am Knick passiert, für das Ergebnis unerheblich ist.

    “Entscheidend ist, dass bei Teresa die beiden Teilwege die selben Richtungen/Geschwindigkeiten haben und bei Serena nicht.”
    Natürlich, Aber das lässt sich für die Linien doch in jedem Koordinatensystem eindeutig herausfinden, genau wie in der gewöhnlichen Geometrie der Ebene. Es ist in jedem KS eindeutig klar, ob drei Punkte auf einer (geraden) Linie liegen oder nicht. Ist im Raum doch genauso.
    “In der geometrischen Beschreibungsweise ist für mich ist der entscheidende Punkt beim Zwillingsparadoxon die Frage warum Teresas Weg der direkte Weg und Serenas Weg ein Umweg ist”
    Würdest du das in der Geometrie der Ebene auch so sehen – warum ist der Weg entlang der Katheten eines Dreiecks ein Umweg gegenüber dem Weg entlang der Hypothenuse? Ist das nicht eine fundamentale Eigenschaft der Geometrie der Ebene (Dreiecksungleichung)? In der Geometrie ist das doch ein Axiom für jede Abstandsfunktion – fundamentaler geht’s doch eigentlich nicht.

  44. #44 Tox
    6. Juli 2018

    @MartinB

    Die Tatsache, dass das keine Rolle spielt, kann man aber zumindest als Hinweis darauf werten, dass das, was da genau am Knick passiert, für das Ergebnis unerheblich ist.

    Für mich sieht das so aus als würde man sagen “das elektrische Feld einer Punktladung ist das selbe wie das elektrische Feld im Vakuum, denn das was da genau an diesem einen Punkt passiert, ist für das Ergebnis unerheblich”. Die Beschleunigung verschwindet nicht, nur weil man sie in einem Delta-Peak versteckt.

    Aber das lässt sich für die Linien doch in jedem Koordinatensystem eindeutig herausfinden,

    Wirklich in jedem Koordinatensystem geht das nur indem man sich genau den einen Punkt mit dem Knick ansieht und feststellt, dass die Kurve dort nicht differenzierbar ist.

    Es ist in jedem KS eindeutig klar, ob drei Punkte auf einer (geraden) Linie liegen oder nicht.

    Selbstverständlich ist dies eindeutig klar. Aber ich sehe keine Methode dies festzustellen, die nicht die “Beschleunigung” entlang einer glatten Kurve, oder eine Nicht-Differenzierbarkeit einer Kurve mit Knick, oder den Vergleich der Länge zweier Kurven verwendet. Und letzteres führt zu einer Tautologie (“die längere Kurve ist die längere Kurve”).

    Würdest du das in der Geometrie der Ebene auch so sehen – warum ist der Weg entlang der Katheten eines Dreiecks ein Umweg gegenüber dem Weg entlang der Hypothenuse?

    Das ist meiner Ansicht nach eine andere Frage. Beim Zwillingsparadoxon hat man zwei Kurven in der Raumzeit mit den selben Endpunkten und es wird “physikalisch” beschrieben, wie diese Kurven konstruiert werden. Teresa bleibt auf der Erde, Serena reist nach Alpha Centauri und zurück. Es wird aber z.B. in der Problembeschreibung nicht direkt gesagt, dass Teresas Weg gerade ist und damit der längste Weg mit diesen Endpunkten. Beim Dreieck in der Ebene setzt du implizit voraus, dass die Hypothenuse eine gerade Linie und damit der kürzeste Weg zwischen ihren Endpunkten ist.

  45. #45 MartinB
    6. Juli 2018

    @Tox
    “Die Beschleunigung verschwindet nicht, nur weil man sie in einem Delta-Peak versteckt.”
    Stimmt schon, habe auch ne Weile gezögert, ob ich das schreiben soll. Die Tatsache, dass Serenas Gesamtlänge genau gleich der Summe der beiden Teillängen ist, ohne (oder mit beliebig wenig) Einfluss durch den “Knick” zeigt für mich aber schon, dass der Knick letztlich egal ist (und das wäre bei der Punktladung ja nict so).

    “Das ist meiner Ansicht nach eine andere Frage. ”
    Und das scheint der Knackpunkt für uns zu sein: Für mich ist das exakt dieselbe Frage.

    Würdest du auch im Raum sagen, dass man den Knick einer Linie nur herausfinden kann “indem man sich genau den einen Punkt mit dem Knick ansieht und feststellt, dass die Kurve dort nicht differenzierbar ist.”?

    Wenn nein, wo ist der Unterschied?

    ” Es wird aber z.B. in der Problembeschreibung nicht direkt gesagt, dass Teresas Weg gerade ist und damit der längste Weg mit diesen Endpunkten”
    Na klar wird das gesagt – Teresa ist stationär, also ist ihre Weltlinie zwangsläufig ne Gerade.

    “oder den Vergleich der Länge zweier Kurven verwendet. Und letzteres führt zu einer Tautologie (“die längere Kurve ist die längere Kurve”).”
    Vielleicht ist das ganze eine Frage, was man als fundamental ansieht: Ich versuche ja in diesem Artikel zu zeigen, dass man, wenn man die Geometrie der Raumzeit als fundamental ansieht, das Zwillingsparadoxon genau so erklären kann wie die Dreiecksungleichung im Raum. In der Geometrie ist die Dreiecksungleichung auch der Startpunkt jeder EEntfernungsdefinition – da könntest du beim Dreieck also letztlich genauso argumentieren.

  46. #46 Tox
    6. Juli 2018

    @MartinB:

    Würdest du auch im Raum sagen, dass man den Knick einer Linie nur herausfinden kann “indem man sich genau den einen Punkt mit dem Knick ansieht und feststellt, dass die Kurve dort nicht differenzierbar ist.”?

    Ja. Sofern du mit “den Knick einer Linie herausfinden” meinst “feststellen, ob eine Kurve an einem bestimmten Punkt einen Knick hat”.

    Na klar wird das gesagt – Teresa ist stationär, also ist ihre Weltlinie zwangsläufig ne Gerade.

    Das ist wohl ein weiterer Unterschied zwischen unseren Ansichten. In dieser Zwangsläufigkeit steckt meiner Meinung nach drin, dass die Beschleunigung eine Rolle spielt. Denn was heißt stationär anderes als dass die Beschleunigung gleich 0 ist.

  47. #47 MartinB
    6. Juli 2018

    @Tox
    Für mich ist der Knackpunkt der: Es gibt gerade und gekrümmte Linien in der Mikowski-Raumziet. Ob eine Linie gerade oder gekrümmt ist, ist eine rein geometrische Frage.
    Einige Linien (die zeitartigen) können auch Weltlinien sein, wenn man sich auf einer zeitartigen Weltlinie bewegt, ist eine Linie genau dann gerade, wenn man sich mit konstanter Geschwindigeit bewegt. Da es aber auch raumartige (oder lichtartige) Linien gibt, sehe ich das Konzept “Beschleunigung” nicht als fundamental an.

    “Ja. Sofern du mit “den Knick einer Linie herausfinden” meinst “feststellen, ob eine Kurve an einem bestimmten Punkt einen Knick hat”.”
    Warum kann ich nicht herausfinden, ob eine Linie einen Knick hat, indem ich an einem punkt ne tangente anlege, von dort aus mit einer Geraden Verlängere und prüfe, ob alle Punkte (in unserem Fall insbesondere der Endpunkt) auf der Linie liegt. Es geht hier ja nicht darum, herauszufinden, wo der Knick ist, sondern nur darum, dass es ein Verfahren gibt, um eindeutig zu entscheiden, ob eine Linie gerade ist. (Insofern meine ich eben nicht “an einem bestimmten Punkt”.)

    Wenn du auf der Autobahn fährst, vor dir ist ein Hügel, wo die ein Stück der Strecke nicht siehst, und danach siehst du die Autobahn in eine andere Richtung weiterlaufen, musst du auch nicht erst zum Hügel fahren um zu wissen, dass die Autobahn einen Knick macht. Wenn Serena bei der Erde in Richtung AC losfliegt, dann weiß sie, dass eine gerade Bahn sie niemals wieder zurück zur Erde führen kann. Wenn sie irgendwann wieder auf der Erde ist, weiß sie sicher, dass sie sich nicht auf einer geraden Weltlinie bewegt hat – egal ob sie während der Beschleunigungsphase bewusstlos war oder sonst etwas. Damit weiß sie, dass sie sich nicht auf einer Linie maximaler Eigenzeit bewegt hat.

  48. #48 Tox
    6. Juli 2018

    @MartinB

    Da es aber auch raumartige (oder lichtartige) Linien gibt, sehe ich das Konzept “Beschleunigung” nicht als fundamental an.

    Naja, für rein zeitartige Kurven ist die Beschleunigung einfach die zweite Ableitung des Ortes nach der Eigenzeit. Das lässt sich leicht auf solche Kurven verallgemeinern, die nirgends lichtartig sind. Und auch im lichtartigen Fall dürfte (wenn ich mich nicht täusche) zumindest die Unterscheidung “Beschleunigung gleich oder ungleich null” wohldefiniert sein.

    Warum kann ich nicht herausfinden, ob eine Linie einen Knick hat, indem ich an einem punkt ne tangente anlege, von dort aus mit einer Geraden Verlängere und prüfe, ob alle Punkte (in unserem Fall insbesondere der Endpunkt) auf der Linie liegt.

    Wie konstruiert man eine solche Tangente, oder allgemeiner: wie konstruiert man eine Gerade? Einfach die Kurve extremaler Länge zwischen zwei festen Endpunkten zu wählen finde ich äußerst unbefriedigend. Zum einen ist das praktisch natürlich nicht realisierbar (man kann schlecht alle überabzählbar vielen Kurven durchgehen), zum anderen ist es nicht lokal. Alternativ kann man die Geodätengleichung lösen, aber das würde ja wieder auf die Beschleunigung führen…

    Wenn du auf der Autobahn fährst, vor dir ist ein Hügel, wo die ein Stück der Strecke nicht siehst, und danach siehst du die Autobahn in eine andere Richtung weiterlaufen, musst du auch nicht erst zum Hügel fahren um zu wissen, dass die Autobahn einen Knick macht.

    Hier geht als Annahme ein, dass sich Licht auf geraden Linien ausbreitet, d.h. unbeschleunigt.

    Wenn Serena bei der Erde in Richtung AC losfliegt, dann weiß sie, dass eine gerade Bahn sie niemals wieder zurück zur Erde führen kann.

    Hier geht als Annahme ein, dass sich die Erde auf einer geraden Linie bewegt (und dass sich nicht identische gerade Linien höchstens einmal schneiden).

  49. #49 MartinB
    6. Juli 2018

    @Tox
    Irgendwie werde ich immer verwirrter:
    “Hier geht als Annahme ein, dass sich Licht auf geraden Linien ausbreitet, d.h. unbeschleunigt.”
    Ja, und davon gehst du doch in der Realität aus. Du kannst meinetwegen auch euklidische Geometrie in der Ebene betreiben, da kannst du auch entscheiden, ob drei Punkte kollinear sind oder nicht.

    Ich versuch’s mal andersrum:
    1. Sind wir uns einig, dass die Frage, ob drei Punkte in der Ebene kollinear sind (ganz normal im euklidischen Raum) beantwortet werden kann, sobald man das Konzept “gerade Linie” definiert hat?
    2. Sind wir uns einig, dass die Definition einer “Geraden” in der ebenen Geometrie fundamental ist? (Siehe Euklids 1. und 2. Axiom von hier:
    https://de.wikipedia.org/wiki/Euklidische_Geometrie)

  50. #50 Tox
    6. Juli 2018

    Ja, und davon gehst du doch in der Realität aus.

    Mir ging es darum, dass du in diesem Beispiel Licht verwendest um Geraden zu realisieren. Dass das funktioniert ist meiner Meinung nach keine Trivialität, sondern eine Aussage mit physikalischem Gehalt (und daher experimentell überprüfbar und zu überprüfen).

    1. Sind wir uns einig, dass die Frage, ob drei Punkte in der Ebene kollinear sind (ganz normal im euklidischen Raum) beantwortet werden kann, sobald man das Konzept “gerade Linie” definiert hat?

    Ja, wenn wir z.B. gegeben haben, dass durch zwei unterschiedliche Punkte genau eine Gerade geht, und dass feststellbar ist, ob ein Punkt auf einer Geraden liegt.

    2. Sind wir uns einig, dass die Definition einer “Geraden” in der ebenen Geometrie fundamental ist?

    Ich bin mir nicht sicher, ob wir uns da einig sind.

    Ja, man kann mit den Euklidischen Axiomen eine Geometrie aufbauen. Und wenn man noch ein paar Axiome dazu nimmt, kann man im Grunde den R^2 mit der üblichen Vektorraumstruktur herleiten. Alternativ kann man mit dem R^2 als Punktemenge starten, definieren was Geraden sind, und dann die Euklidischen Axiome herleiten (bzw. zeigen dass diese erfüllt sind). Beide Herangehensweisen sind also im Prinzip äquivalent. Persönlich würde ich den R^2 als das fundamentalere Objekt ansehen und die Geraden als eine Konstruktion darauf.

  51. #51 MartinB
    6. Juli 2018

    @Tox
    “dass du in diesem Beispiel Licht verwendest um Geraden zu realisieren. ”
    Das spielt für das Beispiel ja keine Rolle, du kannst die Geradheit ja auch anders messen (lineal oder sonstwie).

    ” Persönlich würde ich den R^2 als das fundamentalere Objekt ansehen und die Geraden als eine Konstruktion darauf.”
    Das soll mir recht sein. Wie genau definierst du eine Gerade im R^2? Über eine lineare Beziehung? (Alle Punkte der Form a + s*b, mit a und b Vektoren?)

    Können wir dann mit diesen Definitionen problemlos feststellen, ob drei Punkte auf einer Geraden liegen?

    Wenn ja, warum gilt das deiner Ansicht nach im Minkowski-Raum nicht, wo die Situation bis auf die andere Metrik doch genau identisch ist?

  52. #52 Tox
    6. Juli 2018

    @MartinB

    Das spielt für das Beispiel ja keine Rolle, du kannst die Geradheit ja auch anders messen (lineal oder sonstwie).

    Es spielt schon eine Rolle. Wenn ich mich richtig erinnere, wolltest du mit dem Beispiel zeigen, wie man eine gegebene Kurve auf “Geradheit” testen kann, ohne auf Änderungen der Geschwindigkeit entlang der Kurve zu testen. Und im Prinzip verschiebst du das Problem nur, indem du die zu testende Kurve mit einer anderen Kurve vergleichst. Dass diese Vergleichskurve aber eine Gerade ist, zeigst du nicht.

    Wie genau definierst du eine Gerade im R^2? Über eine lineare Beziehung?

    Gerne.

    Können wir dann mit diesen Definitionen problemlos feststellen, ob drei Punkte auf einer Geraden liegen?

    Ja. (Wenn die Koordinaten der Punkte bekannt sind.)

    Wenn ja, warum gilt das deiner Ansicht nach im Minkowski-Raum nicht, wo die Situation bis auf die andere Metrik doch genau identisch ist?

    Ich bin mir nicht sicher, ob ich das exakt so behauptet habe.

    Das Problem diesen Kolinearitätstests beim Zwillingsparadoxon direkt anzuwenden ist, dass wir ja die Koordinaten der drei Ereignisse — Start bei der Erde, Wende bei Alpha Centauri, Ankunft an der Erde — benötigen. Es bietet sich natürlich an, die Ortskoordinaten des ersten und dritten Ereignisses identisch zu wählen, aber mit dieser Wahl setzt man ja schon voraus, dass sich Teresa auf einer Geraden bewegt.

  53. #53 Niels
    7. Juli 2018

    @MartinB @Tox
    Ich verstehe euer Problem nicht.

    In der Riemannsche Geometrie ist (vereinfacht zusammengefasst) äquivalent:
    1) Eine Kurve ist die kürzeste Verbindungslinie
    2) Eine Kurve erfüllt die Geodätengleichung (d.h. die “Beschleunigung” ist Null.)

    MartinB, du findet es besonders intuitiv, wenn man sich 1) anschaut, richtig?

    Tox, du bist der Meinung, man müsse dringend [stattdessen? auch?] 2) betrachten, 1) alleine empfindest du als unvollständig zur Beschreibung?

    Wir sind uns schon einig, dass 1) und 2) äquivalent sind und daher beide als Definition für den Geodäten-Begriff verwendet werden können, oder?
    Damit ist weder 1) grundlegender als zwei 2) noch 2) als 1), korrekt?

    Worüber genau seit ihr jetzt unterschiedlicher Meinung?

  54. #54 Niels
    7. Juli 2018

    Nachtrag:

    “Die Beschleunigung verschwindet nicht, nur weil man sie in einem Delta-Peak versteckt.”
    Stimmt schon, habe auch ne Weile gezögert, ob ich das schreiben soll. Die Tatsache, dass Serenas Gesamtlänge genau gleich der Summe der beiden Teillängen ist, ohne (oder mit beliebig wenig) Einfluss durch den “Knick” zeigt für mich aber schon, dass der Knick letztlich egal ist (und das wäre bei der Punktladung ja nict so)Punktladung ja nict so).

    Nö, der Knick ist nicht egal.
    Nach 2) ist er genau der Grund, dass die Kurve die Geodätengleichung nicht erfüllt und damit keine Geodäte sein kann.
    Er ist also absolut entscheidend.

    Man kann aber natürlich auch über die äquivalente Aussage 1) abwägen, ob eine Geodöte vorliegt.
    Dann sind die von dir genannten Aspekte zu prüfen.

    Für mich ist der Knackpunkt der: Es gibt gerade und gekrümmte Linien in der Mikowski-Raumziet. Ob eine Linie gerade oder gekrümmt ist, ist eine rein geometrische Frage.
    Einige Linien (die zeitartigen) können auch Weltlinien sein, wenn man sich auf einer zeitartigen Weltlinie bewegt, ist eine Linie genau dann gerade, wenn man sich mit konstanter Geschwindigeit bewegt. Da es aber auch raumartige (oder lichtartige) Linien gibt, sehe ich das Konzept “Beschleunigung” nicht als fundamental an.

    Wieder das Gleiche.
    Auch raumartige und lichtartige Kurven erfüllen natürlich sowohl 1) als auch 2), wenn sie Geodäten sind.
    Keine der äquivalenten Definitionen kann doch grundlegender sein.
    Der Parameter “Beschleunigung” in 2) hat dann einfach nichts mehr mit dem Bewegungszustands eines Körpers zu tun und sollte anders genannt werden. Dadurch wird aber doch 1) trotzdem nicht grundlegender als 2).

    Wobei das aber selbstverständlich eine “rein geometrische Frage” ist, solange wir nur die Mathematik betrachten. Es geht schließlich den mathematischen Teilbereich der riemannschen Geometrie.

    Experimentell könnte man unterscheiden, dass man für einen Test von 1) die Eigenzeit und für einen Test von 2) die Beschleunigung messen sollte.
    Da beides mathematisch äquivalent ist, ist so eine Abgrenzung aber eigentlich nicht so wahnsinnig weiterführend.
    Für raum- und lichtartige Kurven natürlich erst recht nicht.

    “Ja. Sofern du mit “den Knick einer Linie herausfinden” meinst “feststellen, ob eine Kurve an einem bestimmten Punkt einen Knick hat”.”
    Warum kann ich nicht herausfinden, ob eine Linie einen Knick hat, indem ich an einem punkt ne tangente anlege, von dort aus mit einer Geraden Verlängere und prüfe, ob alle Punkte (in unserem Fall insbesondere der Endpunkt) auf der Linie liegt

    Das kannst du natürlich, allerdings ist das doch ganz exakt ein Test darauf, ob die Linie die Geodätengleichung erfüllt?
    Kann also nicht zeigen, dass 1) gegenüber 2) zu bevorzugen ist?

  55. #55 MartinB
    7. Juli 2018

    @Niels
    “2) Eine Kurve erfüllt die Geodätengleichung (d.h. die “Beschleunigung” ist Null.) ”
    Jein. Denn auch eine raumartige Kruve kann eindeutig als gerade oder gekrümmt gemessen werden, ohne dass da irgendwas beschleunigt.
    “Auch raumartige und lichtartige Kurven erfüllen natürlich sowohl 1) als auch 2), wenn sie Geodäten sind.”
    Aber bei ner raumartigen Kurve ergibt es doch keinen Sinn, von einer Beschleunigung zu reden?

    @Niels und Tox
    Ich versuche nochmal, die Analogie im raum schärfer zu fassen:
    Wir haben ein Dreieck im Raum mit Hypothenuse AB und drittem Punkt C. Wir legen jetzt drei absolut gerade Stahldrähte entlang der drei Seiten und sehen, dass die drähte AC und CB zusammen länger sind als AB. Jetzt nehme ich einen vierten Draht, der so lang ist wie AC und CB zusammen. Ich mache mit der Zange einen Knick in diesen vierten Draht und lege ihn passend auf das Dreieck.
    Niemand würde vermutlich sagen, dass es der Eingriff der Zange war, der dafür sorgt, dass dieser Draht länger ist als AB.

    Die Sprechweise “Die Beschleunigung ist verantwortlich” suggeriert aber in meinen Augen genau das in der Raumzeit – wird ja gern noch mit den Effekten der Beschleunigung illustriert, “Serena wird in ihren Sitz gedrückt” usw. Das wäre dann so, als würde ich sagen: “Dass der Draht ACB länger ist als AB liegt daran, dass ich ihn mit der Zange verbogen habe, das merkt man ja auch daran, dass ich dazu eine Kraft brauche.”

    Das ist vielleicht mein Problem mit dem Begriff der Beschleunigung hier: Beschleunigung erfordert (in der SRT) eine Kraft – wer also sagt, “Ich brauche eine Beschleunigung” sagt, “ich brauche eine Kraft”, und das klingt so, also würde diese Beschleunigung oder Kraft dann irgendwie die Länge der Weltlinie beeinflussen oder verändern. Auch um die Frage zu beantworten, ob eine Linie eine Geodäte ist, brauche ich keine Kräfte oder Beschleunigungen zu messen (mit nem Beschleunigungssensor oder so), ich kann einfach die Punkte auf der Linie geometrisch auswerten. Für mich ist der Begriff “Beschleunigung” eher ein physikalischer Begriff, kein mathematischer.

    Vielleicht seht Ihr das einfach anders oder trennt das weniger strikt, und deshalb sind wir uns in der Interpretation nicht einig.

  56. #56 MartinB
    7. Juli 2018

    PS:
    Wie wäre es denn, wen ich Serena durch ein Lichtsignal ersetze, das bei Alpha Centauri an einem Spiegel reflektiert wird? Würdet ihr dann auch sagen: “Dass die Eigenzeit des Lichtsignals auf dem Weg Erde-AC-Erde gleich Null ist, liegt an der Beschleunigung bei AC” statt “Dass die Eigenzeit des Lichtsignals Null ist, liegt daran, dass es aus zwei Stücken mit Eigenzeit Nullzusammengesetzt ist?”

  57. #57 MartinB
    7. Juli 2018

    PPS @Niels (Sorry, aber im Kopf rotiert es gerade):
    Was die Geodätengleichung angeht: Ist es nich tmathematisch klarre zus agen, die Geodätengleichung enthält die zweite Ablewitung nach einem affinen Parameter, der die Kurve parametrisiert? In manchen Fällen (zeitartig) ist es möglich, die Zeit als affinen Parameter zu nehmen, und wenn man das tut, dann kann man diese Größe mit der Beschleunigung identifizieren – aber ich kann die Kurve ja auch aders parametrisieren oder ne raumartige Kurve nehmen, und die Geodätengleichung funktioniert trotzdem.

    Und schließlich
    PPPS @Niels und Tox
    Ich wollte nur nochmal sagen, dass ich die Diskussion sehr interessan tfinde. Es geht mir auch nicht darum euch zu zeigen, dass ich “recht” habe (hab ich natürlich ;.) ), sondern ich möchte gern möglichst genau verstehen, wo unsere Intuitionen auseinanderlaufen. Ich finde es immer wieder faszinierend, dass unterschiedliche Personen dieselben physikalischen Gleichungen, Begriffe und Phänomene anscheinend manchmal doch intuitiv etwas anders interpretieren/assoziieren.

  58. #58 erik||e oder wie auch immer . . . ..
    7. Juli 2018

    @MartinB
    „Intuitiv interpretieren/assoziieren . . . ..“ 🙂
    . . . .. zur Zeit können Sie sich Inspiration bei Politikern wie Seehofer holen. Es scheint, das eine Sachlage interpretiert wird, die Logik in Bezug zur Sachlage fehlt, aber auf emotionaler Ebene (Bezugswechsel – vielleicht in eine Geometrie von emotionaler Wechselwirkung) irgendwelche Deals ausgehandelt werden, welche für die Öffentlichkeit nicht „messbar“/sichtbar werden.
    . . . .. dann heben Sie Bewusstsein nicht in eine physikalische/mathematische Ebene – und somit fehlt die damit verbundene Intuition
    . . . .. Sie suchen nach einem Phänomen, welches Sie mir gelegentlich vorgeworfen haben 🙂

  59. #59 Niels
    7. Juli 2018

    @MartinB

    Was die Geodätengleichung angeht: Ist es nich tmathematisch klarre zus agen, die Geodätengleichung enthält die zweite Ablewitung nach einem affinen Parameter, der die Kurve parametrisiert? In manchen Fällen (zeitartig) ist es möglich, die Zeit als affinen Parameter zu nehmen, und wenn man das tut, dann kann man diese Größe mit der Beschleunigung identifizieren – aber ich kann die Kurve ja auch aders parametrisieren oder ne raumartige Kurve nehmen, und die Geodätengleichung funktioniert trotzdem

    Doch, natürlich.

    Ich schrieb ja auch:
    Auch raumartige und lichtartige Kurven erfüllen natürlich sowohl 1) als auch 2), wenn sie Geodäten sind. […]
    Der Parameter “Beschleunigung” in 2) hat dann einfach nichts mehr mit dem Bewegungszustands eines Körpers zu tun und sollte anders genannt werden.

    Da sowohl Tox als auch du mehrmals Beschleunigung in Anführungszeichen verwendet habt, bin ich davon ausgegangen, dass ihr euch damit stillschweigend darauf geeinigt habt, die zweite Ableitung dieses affinen Parameter einfach immer “Beschleunigung” zu nennen.

    Wenn das nicht so gemeint war, welche Bedeutung haben dann jeweils die Anführungszeichen um Beschleunigung?

    Für mich ist der Begriff “Beschleunigung” eher ein physikalischer Begriff, kein mathematischer.

    Na ja, mathematisch ist eine Geodäte nach 2) eine Kurve, in der jeder Tangentialvektor zum parallel (entlang der Kurve) verschobenen Tangentialvektor gleich ist.
    Das kann man jetzt noch über die Definition der Parallelverschiebung umformulieren, also mit Hilfe eines affinen Parameter und der kovarianten Ableitung.

    Für zeitartige Kurven übersetzen sich die kovarianten Ableitungen dieses affinen Parameters eben in das Geschwindigkeitsvektorfeld, das für Geodäten längs der Kurve konstant ist und in die Beschleunigung, die Null sein muss.

    Da steckt meiner Meinung nach noch keine Physik drin.

    Klar, man muss voraussetzen, dass unsere pseudo-riemannsche Mannigfaltigkeit eine Zeit-Dimension hat.
    Aber wenn wir das nicht tun, können wir auch nicht zwischen licht-, raum- und zeitartig unterscheiden oder überhaupt sinnvoll über Licht und Bewegung sprechen.

    Niemand würde vermutlich sagen, dass es der Eingriff der Zange war, der dafür sorgt, dass dieser Draht länger ist als AB.

    Na ja, das Beispiel mit der Zange ist aber nur intuitiv, weil es hier keine Zeitdimension gibt.
    Dann gibt es in 2) natürlich auch keine Ableitungen nach einer Zeit und keine Beschleunigung.

    Übersetzt auf die Raumzeit vergleichst du in deinem Beispiel die Länge einer zeitartigen Kurve mit der einer nicht-zeitartigen Kurve.

    Menschen, die die Sprechweise “Die Beschleunigung ist verantwortlich” verwenden, vergleichen aber natürlich immer ausschließlich zeitartige Kurven mit anderen zeitartigen Kurven.

    Die Sprechweise “Die Beschleunigung ist verantwortlich” suggeriert aber in meinen Augen genau das in der Raumzeit

    Ich reite hier ja die ganze Zeit auf der Äquivalenz der beiden Aussagen herum.
    Jetzt können wir uns darüber streiten, ob eine Äquivalenz von A und B bedeutet, dass A verantwortlich für B ist.
    Meiner Meinung nach eher nicht, aber “verantwortlich” ist mathematisch auch gar nicht sauber definiert, oder? 😉

    “Ich brauche eine Beschleunigung” sagt, “ich brauche eine Kraft”, und das klingt so, also würde diese Beschleunigung oder Kraft dann irgendwie die Länge der Weltlinie beeinflussen oder verändern.

    Ich würde “Ich brauche eine Beschleunigung” erst einmal so verstehen, dass eine Beschleunigung vorhanden sein muss. (Und sei sie irgendwie in einer Unstetigkeit versteckt.)
    Das ist für zeitartige Kurven ja auch immer so.
    (Für andere Kurven muss eben die zweite Ableitung eines passend gewählten Parameters irgendwo nicht verschwinden.)

    Wenn wir uns zeitartige Kurven und damit Bewegungen von Körpern anschauen:
    Was ist denn die Ursache, dass sich Körper nicht auf Geodäten bewegen, wenn nicht eine Beschleunigung und damit eine Kraft?
    (Für nicht-zeitartige Kurven haben wir dafür einfach nicht das passende intuitiv verständliche Vokabular, weil solche Kurven für unsere Alltagsumgebung eben keinerlei Rolle spielen.)

    Immer dann, wenn eine Kraft bzw. eine Beschleunigung vorliegt, ist eine Weltlinie länger als eine Geodäte zwischen zwei Ereignissen auf der Weltlinie.

    Warum kann man dann nicht sagen,
    diese Beschleunigung oder Kraft würde dann irgendwie die Länge der Weltlinie beeinflussen oder verändern”?
    Was beeinflusst oder verändert die Weltlinie des Körpers denn sonst weg von der geodätischen Linie?

    Auch um die Frage zu beantworten, ob eine Linie eine Geodäte ist, brauche ich keine Kräfte oder Beschleunigungen zu messen (mit nem Beschleunigungssensor oder so), ich kann einfach die Punkte auf der Linie geometrisch auswerten.

    Nein, man muss keine Kräfte oder Beschleunigungen messen. Man kann aber.

    Ich sage ja gerade nicht, dass 2) fundamentaler als 1) wäre.
    Man muss eben auch nicht die Punkte auf der Linie geometrisch auswerten.

    Warum findest du es “physikalischer”, eine Kraft bzw. Beschleunigung zu messen als eine Strecke oder eine Zeit?

    Geht es nur darum, dass man für 2) die Zusatzannahme treffen muss, dass die Ableitung nach der Zeit eine Geschwindigkeit ist und die Ableitung der Geschwindigkeit eine Beschleunigung?

    Okay, diese Zusatzannahme braucht man für 1) nicht, aber das ist schon ein sehr philosophischer Unterschied, oder?
    .

    PS:

    Würdet ihr dann auch sagen: “Dass die Eigenzeit des Lichtsignals[…]

    Ich bin mal Gentleman und ignoriere, dass du eine angebliche Eigenzeit von Licht erwähnt hast. 😉
    .

    PPS:

    ich möchte gern möglichst genau verstehen, wo unsere Intuitionen auseinanderlaufen

    Tun sie aber gar nicht.
    Mit deinem Artikel habe ich überhaupt kein Problem. Ich habe eben nur nicht verstanden, worüber Tox und du euch uneinig wart.

    Ich glaube mittlerweile, Tox findet es einfach nicht hilfreich oder weniger intuitiv, Kurven zu betrachten, die nicht zeitartig sind. Richtig, Tox?
    Da dagegen findest es nicht hilfreich oder weniger intuitiv, überhaupt über Beschleunigung zu sprechen, weil du auch nicht zeitartige Kurven betrachten willst, für die man eben keine physikalisch sinnvolle “Beschleunigung” definieren kann. Richtig?

    Ich vertrete da eher eine dritte Position.
    In der Regel ist es nicht besonders sinnvoll, über nicht zeitartige Kurven zu sprechen. Allerdings sind lichtartige Kurven dann doch manchmal ne schicke Sache. Man läuft nur eben schnell in Gefahr, dass es dann unanschaulich wird, weil wir eben kein Bauchgefühl für solche Kurven haben.

  60. #60 MartinB
    7. Juli 2018

    @Niels
    “Da sowohl Tox als auch du mehrmals “Beschleunigung” in Anführungszeichen verwendet habt, bin ich davon ausgegangen, dass ihr euch damit stillschweigend darauf geeinigt habt, die zweite Ableitung dieses affinen Parameter einfach immer “Beschleunigung” zu nennen.”
    Aha – ich sag#s ja, immer wieder interessant zu sehen,w ie unertschiedliche Leute diesleben Begriffe nutzen. Nein, auf die Idee, die Ableitung nach irgendeinem affinen Parameter “Beschleunigung” zu nennen, würde ich nicht kommen, weil das ja mit der physikalischen Beschleunigung nix zu tun hat.

    “Da steckt meiner Meinung nach noch keine Physik drin. ”
    Naja, von einer Geschwindigkeit würde ich erst reden, wenn es tatsächlich ein objekt gibt, dass sich auf der Weltlinie bewegt.

    “Na ja, das Beispiel mit der Zange ist aber nur intuitiv, weil es hier keine Zeitdimension gibt.
    Dann gibt es in 2) natürlich auch keine Ableitungen nach einer Zeit und keine Beschleunigung.”
    Das verstehe ich jetzt nicht mehr. Zum einen gibt es genauso einen affinen Parameter wie in der Raumzeit, zum anderen brauche ich nur ins Blockuniversumsbild zu gehen, und der Unterschied verschwindet.

    “Was beeinflusst oder verändert die Weltlinie des Körpers denn sonst weg von der geodätischen Linie?”
    Natprlich brauche ich eine echte, physikkalische Beschleunigung, um einen Körper von einer Geodäte zu entfernen, das ist klar. Aber diese Beschleunigung sorgt eben nur dafür, dass die Weltlinie des Körpers nicht mehr eine Geodäte ist, ich würde die nicht verantwortlich für die Zeitdilatation machen. (siehe unten)

    “Warum findest du es “physikalischer”, eine Kraft bzw. Beschleunigung zu messen als eine Strecke oder eine Zeit?”
    In dem Sinne, dass ich eine Kraft auf ein bestimmtes Objekt messen muss – eine Strecke kann ich aus mehreren Streckenstücken zusammensetzen, von denen keine dem Weg eines Objekts entspricht, von denen einige raum- andere zeitartig sind usw.

    “Ich bin mal Gentleman und ignoriere, dass du eine angebliche Eigenzeit von Licht erwähnt hast. ”
    Du hast oben selbst von lichtartigen Linien gesprochen. Nenn’ es nicht “Eigenzeit” sondern einfach “raumzeitliche Länge”, wenn dir das lieber ist.

    “Da dagegen findest es nicht hilfreich oder weniger intuitiv, überhaupt über Beschleunigung zu sprechen, weil du auch nicht zeitartige Kurven betrachten willst, für die man eben keine physikalisch sinnvolle “Beschleunigung” definieren kann. Richtig?”
    Oder weil man auch drei Brüder haben kann, bei denen auch nichts beschleunigt, oder sonstige beliebige Konstruktionen, richtig. Wenn Dinge wie Zeitdilatation eine Eigenschaft der Geometrie der Raumzeit sind, dann sind sie unabhängig davon, ob konkrete physikalische Objekte die jeweiligen Linien abklappern oder nicht, das ist mein punkt.

    “In der Regel ist es nicht besonders sinnvoll, über nicht zeitartige Kurven zu sprechen. ”
    Wirklich nicht? Aber Entfernungen, die man zu einer Zeit misst, sind doch genau das und sind doch ziemlich wichtig, oder nicht? Wen wir uns fragem, wie groß das beobachtbare Universum ist oder so, reden wir doch genau über den raumartigen Abstand von Punkten.

    @Tox und Niels
    Ich mach das noch mal an einem anderen Beispiel fest. Zusätzlich zu Serena nehmen wir noch Uhura hinzu. Die startet ein paar Lichtjahre von der Erd entfernt genau gegenüber von Alpha centauri, fliegt mit konstanter Geschwindigkeit an der Erde vorbei und weiter nach AC, wo sie dann umkehrt. Beim trip Erde-AC-Erde fliegt sie genau parallel zu Serena, auch ihre Beschleunigung bei AC ist exakt dieselbe. Trotzdem hat sie am Ende ne andere /größere) zeitdilatation gegenüber ihrer Zwillingsschwester. Dieselbe Beschleunigung ist also in dieser Sprechweise verantwortlich für zwei ganz unterschiedliche Größen der Zeitdilatation. Das finde ich schon seltsam…

    @Tox
    Hast du Beschleunigung auch wie Niels schlicht als Ableitung nach nem affinen Parameter gemeint?

  61. #61 Niels
    7. Juli 2018

    @MartinB

    Welche Bedeutung hatten denn dann deine Anführungszeichen um Beschleunigung?

    Wirklich nicht? Aber Entfernungen, die man zu einer Zeit misst, sind doch genau das und sind doch ziemlich wichtig, oder nicht?

    Ja, das war totaler Schwachsinn. Offenbar wollte mein Gehirn kurzzeitig nicht mehr verstehen, was eigentlich raumartige Kurven sind. Ist ja auch ziemlich schwer, der Name liefert schließlich überhaupt keinen Hinweis…

    auf die Idee, die Ableitung nach irgendeinem affinen Parameter “Beschleunigung” zu nennen, würde ich nicht kommen

    Ich dagegen bin aus irgendwelchen Gründen nicht darauf gekommen, dass du mit der Eigenzeit des Lichtsignals einfach auf das Linienelement hinaus wolltest und habe deswegen die ganze Argumentation nicht mehr verstanden.
    Da hat mich der (rein formal falsch verwendete) Begriff völlig verwirrt, obwohl es eigentlich hätte klar sein müssen, dass das einfach nur ein bisschen flapsig ausgedrückt war.

    Du hast oben selbst von lichtartigen Linien gesprochen. Nenn’ es nicht “Eigenzeit” sondern einfach “raumzeitliche Länge”, wenn dir das lieber ist.

    Sorry.
    Wie gesagt, ich hab einfach nicht kapiert, was gemeint war.

    Naja, von einer Geschwindigkeit würde ich erst reden, wenn es tatsächlich ein objekt gibt, dass sich auf der Weltlinie bewegt.

    Kann man so vertreten, finde ich auch einleuchtender.
    Ich kann aber auch verstehen, wenn man den Standpunkt vertritt, dass man auch nur dann von Eigenzeit und Zeitdilatation sprechen sollte, wenn es Objekte gibt.

    Wenn Dinge wie Zeitdilatation eine Eigenschaft der Geometrie der Raumzeit sind, dann sind sie unabhängig davon, ob konkrete physikalische Objekte die jeweiligen Linien abklappern oder nicht, das ist mein punkt.

    Sind sie das denn?

    Ganz allgemein kann man die SRT doch auch als “relational theory” (wie heißt das auf Deutsch?) auffassen.
    Dann sind eben nur Objekte und die Abstände, Geschwindigkeiten usw. von Objekten relativ zu anderen Objekten das Fundamentale.
    https://en.wikipedia.org/wiki/Relational_theory

    Erst für die ART bekommt man mit dieser Sichtweise Probleme.

    Die Sichtweise mit Hilfe der Geometrie der Raumzeit ist damit zumindest in der SRT nicht richtiger oder weniger richtig als die Sichtweise über Körper und Beschleunigungen.

    Welche Sichtweise fürs Verständnis brauchbarer ist, ist wieder eine ganz andere Frage.

    Natprlich brauche ich eine echte, physikkalische Beschleunigung, um einen Körper von einer Geodäte zu entfernen, das ist klar. Aber diese Beschleunigung sorgt eben nur dafür, dass die Weltlinie des Körpers nicht mehr eine Geodäte ist, ich würde die nicht verantwortlich für die Zeitdilatation machen

    Mir ist wie schon im letzten Post erwähnt nicht ganz klar, was du eigentlich mit verantwortlich meinst.

    Wenn man den zeitlichen Verlauf der Geschwindigkeit kennt, kann man die Eigenzeit und die Zeitdilatation berechnen.
    Den zeitlichen Verlauf der Geschwindigkeit kann man aber natürlich auch mit Hilfe des zeitlichen Verlaufs der Beschleunigung (plus Randbedingung) ausdrücken.

    Ist die Geschwindigkeit verantwortlich für Eigenzeit und Zeitdilatation?
    Ist die Beschleunigung zusammen mit der Anfangsgeschwindigkeit verantwortlich?
    Wäre für mich eine eher seltsame Aussage.

    Ich kann die Eigenzeit aber auch rein geometrisch mit Hilfe des Linienelements berechnen, dann braucht man weder Geschwindigkeiten noch Beschleunigung.

    Ist das Linienelement und damit die Kurvenlänge verantwortlich für Eigenzeit und Zeitdilatation?
    Wäre für mich eine ebenfalls seltsame Aussage.

    Trotzdem hat sie am Ende ne andere /größere) zeitdilatation gegenüber ihrer Zwillingsschwester. Dieselbe Beschleunigung ist also in dieser Sprechweise verantwortlich für zwei ganz unterschiedliche Größen der Zeitdilatation.

    Was ist denn jetzt deiner Sprechweise nach ganz genau das Verantwortliche für die unterschiedlichen Eigenzeiten?
    (Wenn du allgemein mit “die Geometrie der Raumzeit” antwortest, antworte ich auch allgemein.
    Nicht die Beschleunigung ist verantwortlich, sondern die Form der Relativbewegungen der Zwillingsschwestern zueinander. (Unter Berücksichtigung der Konsequenzen des Prinzips der Konstanz der Lichtgeschwindigkeit.))
    .

    PS:
    Ich glaube nicht, das wir besonders weit auseinander liegen.
    Vermutlich hakt es nur daran, dass wir uns über die Bedeutung eines bestimmten Begriffes nicht einig sind und es nicht merken.

  62. #62 MartinB
    8. Juli 2018

    @Niels
    Ich habe “Beschleunigung” eigentlich doch nur in Anführungsstriche gesetzt, wenn ich über den Begriff geredet habe – es solte an der Stelle nur verdeutlichen, dass ich die Definition des Begriffs meine.

    Ja, das war totaler Schwachsinn.

    Sowas würde mir natürlich *nie* passieren, absolut *nie* [unschuldig pfeif]

    Ich dagegen bin aus irgendwelchen Gründen nicht darauf gekommen, dass du mit der Eigenzeit des Lichtsignals einfach auf das Linienelement hinaus wolltest und habe deswegen die ganze Argumentation nicht mehr verstanden.

    Mcht ja nix. Würdest du denn nun sagen, dass ein Lichtsignal an einem Spiegel beschleunigt wird?

    Die Sichtweise mit Hilfe der Geometrie der Raumzeit ist damit zumindest in der SRT nicht richtiger oder weniger richtig als die Sichtweise über Körper und Beschleunigungen.

    Stimmt. Aber wie du schon sagst, erstens ist es in der ART wesentlich einfacher, es so zu sehen. Zweitens ist es meiner Ansicht nach auch in der SRT wesentlich einfacher, Die Rrelationen zwischen Ereignissen schlicht als geometrisch aufzufassen, so wie wir es auch im Raum tun. Dann sieht man Dinge wie das zwillingspardoxon sofort (siehe meinen letzten Artikel). Da sind wir uns ja anscheinend einig, wenn ich diesen Satz “Welche Sichtweise fürs Verständnis brauchbarer ist, ist wieder eine ganz andere Frage.” richtig verstehe. Und man sieht eben, dass es nicht der physikalische Prozess der Beschleunigung ist, der für’s Zwillingsparadoxon entscheidend ist (im Sinne von: da müssen Kräfte wirken etc.), sondern nur die Tatsache, dass wir eine Linie betrachten, die aus zwei Stücken zusammengesetzt ist, egal wie.

    Ist das Linienelement und damit die Kurvenlänge verantwortlich für Eigenzeit und Zeitdilatation?
    Wäre für mich eine ebenfalls seltsame Aussage.

    Würde ich eigentlich schon so sehen.

    Worum es mir eigentlich geht ist, dass die Aussage “das zwillingsparadoxon wird dadurch erklärt, dass Serena beschleunigen muss” suggeriert, dass Beschleunigungen per se den zeitablauf beeinflussen. Dass das nicht funktioniert, habe ich in dem Beispiel mit Uhura am Ende von #60 ja nochmal deutlich zu machen versucht, und das ist ja auch die Idee hinter dem 3-Brüder-Argument.

    Was ist denn jetzt deiner Sprechweise nach ganz genau das Verantwortliche für die unterschiedlichen Eigenzeiten?

    Das gleiche, was dafür verantwortlich ist, dass die beiden Katheten eines Dreiceks zusammen länger sind als die Hypothenuse. Ich würde in der Tat von der Geometrie reden, oder, wenn’s konkreter sein soll, von der Metrik der Raumzeit.

    Nicht die Beschleunigung ist verantwortlich, sondern die Form der Relativbewegungen der Zwillingsschwestern zueinander.

    “Form der Relativbewegung” finde ich etwas unscharf, aber prinzipiell find eich den Satz o.k.

    Vermutlich hakt es nur daran, dass wir uns über die Bedeutung eines bestimmten Begriffes nicht einig sind und es nicht merken.

    Hakt es denn jetzt noch? Ich verstehe dich so, dass du nicht ganz so dogmatisch wie ich die Raumzeit-Geometrie in den Vordergrund stellen würdest, weil man die SRT prinzipiell auch ohne Raumzeit-Geometrie erklären/formulieren kann, soll ein gewisser Herr Einstein ja auch mal so gemacht haben. Kann ich akzeptieren, aber für mich ist die geometrische Sichtweise wesentlich einleuchtender und intuitiver und macht insbesondere aus Dingen wie dem Zwillingsparadoxon kein Geheimnis, die wir im Raum (Dreiecksungleichung) auch selbstverständlich akzeptieren.

  63. #63 erik||e oder wie auch immer . . . ..
    8. Juli 2018

    * „Geometrie der Raumzeit“ [+1] -> [Mathematik]
    * „Nicht die Beschleunigung“ [-1] -> [Physik] „ist verantwortlich, sondern die Form der Relativbewegungen der Zwillingsschwestern zueinander“
    * Exp(i*π) = [-1] und statische Form der Geometrie der Raumzeit [+1] ergeben in ihren Beträgen immer [=0]
    * [=0] ist die Bedingung für c=const . . . .. und dem Energieerhaltungssatz 🙂

    Interessant ist: Warum hat sich dieses Gleichgewicht gebildet und zu unserem Universum entwickelt?

    PS: Alles in „“Gesetztes sind Zitate aus #62.

  64. #64 Niels
    8. Juli 2018

    @MartinB

    Ich habe “Beschleunigung” eigentlich doch nur in Anführungsstriche gesetzt, wenn ich über den Begriff geredet habe – es solte an der Stelle nur verdeutlichen, dass ich die Definition des Begriffs meine.

    Okay, war mir überhaupt nicht klar.

    Und man sieht eben, dass es nicht der physikalische Prozess der Beschleunigung ist, der für’s Zwillingsparadoxon entscheidend ist (im Sinne von: da müssen Kräfte wirken etc.), sondern nur die Tatsache, dass wir eine Linie betrachten, die aus zwei Stücken zusammengesetzt ist, egal wie.

    Nenn mich verrückt, aber mit “ist entscheidend” habe ich überhaupt keine Probleme.
    Mit “ist verantwortlich”, wie du es sonst formulierst, irgendwie schon.

    Ich komme aber nicht darauf, warum eigentlich?
    (Vielleicht ist es einfach zu heiß. ;-))

    Würdest du denn nun sagen, dass ein Lichtsignal an einem Spiegel beschleunigt wird?

    Wenn du das irgendwo geschrieben hättest, hätte ich kommentiert, dass das Unsinn ist. 😉

    Nee, wir haben ja jetzt herausgearbeitet, dass zumindest bei dir die Verwendung von Beschleunigung eben nicht meint, dass es eine zweite Ableitung ungleich Null gibt, wodurch die Geodätengleichung dann nicht mehr erfüllt wird.

    Jetzt kommt es noch darauf an, wie Tox Beschleunigung versteht.

    Tox schrieb ja:
    für rein zeitartige Kurven ist die Beschleunigung einfach die zweite Ableitung des Ortes nach der Eigenzeit. Das lässt sich leicht auf solche Kurven verallgemeinern, die nirgends lichtartig sind. Und auch im lichtartigen Fall dürfte (wenn ich mich nicht täusche) zumindest die Unterscheidung “Beschleunigung gleich oder ungleich null” wohldefiniert sein.

    Deswegen bin ich mir ziemlich sicher, dass Tox Beschleunigung eben mindestens von da an als “Verallgemeinerung” gemeint hat, nämlich als die besagte zweite Ableitung nach dem passenden affinen Parameter.
    Oder kann man dieses Zitat auch anders verstehen?

    Und weil du Tox Definition nie ausdrücklich abgelehnt hast und Beschleunigung verwendet hast, war ich der Meinung, du würdest dich diesem Spachgebrauch anschließen.

    Wodurch deine Bemerkungen dann für mich so klangen, als hättest du die Äquivalenz von 1) und 2) nicht bedacht oder würdest sie nicht anerkennen und hieltest 1) für irgendwie mathematisch bedeutsamer.

    Hakt es denn jetzt noch? Ich verstehe dich so, dass du nicht ganz so dogmatisch wie ich die Raumzeit-Geometrie in den Vordergrund stellen würdest, weil man die SRT prinzipiell auch ohne Raumzeit-Geometrie erklären/formulieren kann.

    So hatte ich es gemeint, ja.

    Hauptsächlich hat es aber nach meinem momentanen Verständnis wie gesagt daran gehakt, dass soweit ich es sehe, du und Tox den Begriff Beschleunigung unterschiedlich verwendet habt und ich das nicht gemerkt habe.
    Dadurch haben du und ich diesen Begriff dann ebenfalls unterschiedlich verwendet und wussten dadurch nicht, was der andere eigentlich meint.

    Kann ich akzeptieren, aber für mich ist die geometrische Sichtweise wesentlich einleuchtender und intuitiver

    Für mich auch.
    Könnte aber nur daran liegen, dass wir beide uns viel mit der ART beschäftigt haben.
    Dort wird einem diese Sichtweise ja eigentlich aufgezwungen. Dadurch wird es dann völlig normal und natürlich, auch die SRT so zu betrachten.

  65. #65 MartinB
    8. Juli 2018

    @Niels
    “du und Tox den Begriff Beschleunigung unterschiedlich verwendet habt und ich das nicht gemerkt habe.”
    Ich glaube, das haben wir alle nicht wirklich gemerkt, aber wie es aussieht, sind wir uns dann ja einig…

    ” Dadurch wird es dann völlig normal und natürlich, auch die SRT so zu betrachten.”
    Hat der olle Weyl aber ja scho 1908 so gesehen in dem berühmten “von Studn an…”-Zitat, da sind wir also in guter Gesellschaft.

  66. #66 Niels
    8. Juli 2018

    @MartinB

    Hat der olle Weyl aber ja scho 1908 so gesehen in dem berühmten “von Studn an…”-Zitat, da sind wir also in guter Gesellschaft.

    Weyl hatte aus unglaublich drauf und ist meiner Meinung nach unverdient ziemlich in Vergessenheit geraten.

    Ich kenne aber nur das Minkowski-Zitat mit “von Stund an”?
    Von Stund an sollen Raum und Zeit für sich völlig zu Schatten herabsinken, und nur noch eine Union der beiden soll Selbständigkeit bewahren.

  67. #67 MartinB
    8. Juli 2018

    Mist, hab ich mal wieder verwechselt, Weyl war der mit dem Nahewirkungsprinzip…
    Hab ich natürlich nur gemacht, damit du nicht der einige bist, der sich hier in den Kommentaren mal vertan hat, reine Nettigkeit von mir ;-))))

    Aber ja, Weyl war voll der Checker, das denke ich auch.

  68. #68 Niels
    8. Juli 2018

    Von Weyl hab ich etwas Ähnliches gefunden, allerdings von 1927 und ziemlich theatralisch:

    Der Schauplatz der Wirklichkeit ist nicht ein stehender dreidimensionaler Raum, in dem die Dinge in zeitlicher Entwicklung begriffen sind, sondern die vierdimensionale Welt, in welcher Raum und Zeit unlöslich miteinander verwachsen sind.
    Diese objektive Welt geschieht nicht, sondern sie ist – schlechthin; ein vierdimensionales Kontinuum, aber weder Raum noch Zeit.
    Nur vor dem Blick des in den Weltlinien der Leiber emporkriechenden Bewußtseins lebt ein Ausschnitt dieser Welt auf und zieht an ihm vorüber als räumliches, in zeitlicher Wandlung begriffenes Bild.

    Wundert mich nicht so wahnsinnig, dass in der Regel stattdessen Minkowski zitiert wird…

  69. #69 MartinB
    8. Juli 2018

    @Niels
    schick, zumal da mal explizit das Blockuniversum erklärt wird. Und “unlöslich miteinander verwachsen” ist etwas schwülstig, aber auch schön.

  70. #70 Niels
    8. Juli 2018

    Ja, die ersten beiden Sätze sind ziemlich klasse.
    Der letzt versaut es dann ein bisschen, ist aber natürlich Geschmacksache.
    War 1927 aber vielleicht völlig normal, sich so auszudrücken und wäre dann nur schlecht gealtert.

    Hab ich natürlich nur gemacht, damit du nicht der einige bist, der sich hier in den Kommentaren mal vertan hat

    Sehr großzügig. 😉

  71. #71 erik||e oder wie auch immer . . . ..
    8. Juli 2018

    . . . .. der dritte/letzte Satz liefert doch das Salz in der Suppe 🙂
    . . . .. mal konkret betrachtet: Ihre drei Leiber und mein Leib und das in uns emporkriechende Bewusstsein !!! – entlang unserer (!!!) Weltlinien
    . . . .. meine Herren – viel Bewegung in den Kommentaren, um eigenes Denken fit zu machen – und den durch unsere Leiber verlaufenden Weltlinien zu entsprechen (Quelle unseren Willens?)

    PS: . . . .. Danke für die Inspiration

  72. […] Beispiel aus verschiedenen Perspektiven setzen würde. Nachdem Martin mit seinem Artikel The Good, the Bad, and the Ugly – die Erklärung(en) des Zwillingsparadoxons einen faktischen Undercut vorgelegt hat, in dem insbesondere das Rechnen eines Beispiels sehr […]

  73. #73 Christopher
    10. Juli 2018

    Toller Beitrag!
    Da es bisher noch niemand gepostet hat, hier der Link zu einer ebenfalls guten Erklärung des Zwillingsparadoxons mittels Raumzeitdiagrammen von Josef M. Gaßner auf YouTube: https://www.youtube.com/watch?v=wvvngeHEq2M&t=18m37s

  74. […] Grund ist nicht etwa, wie schon bei Martin erklärt, ein Effekt der allgemeinen Relativitätstheorie, wie man gelegentlich hört, der gemäß die Zeit […]

  75. #75 Tox
    12. Juli 2018

    @MartinB, Niels

    Ich hatte die letzten paar Tage leider keine Zeit mich dieser Diskussion zu widmen, und inzwischen ist mir auch das Interesse vergangen. Wenn es uns in knapp 50 Kommentaren jeweils nicht gelungen ist die Sichtweise des Gegenüber zu verstehen, dann werden weitere 50 Kommentare daran auch nichts mehr ändern.

    Sollte es dennoch für irgendjemanden von Interesse sein, hier eine Kurzfassung meiner Position: Unter dem Zwillingsparadoxon verstehe ich den Effekt, dass für zwei Beobachter, die sich in zwei unterschiedlichen Punkten der Raumzeit treffen, zwischen diesen Punkten unterschiedliche Eigenzeiten vergehen können. Im Beispiel altert Serena weniger als Teresa. Und dies obwohl nach einem naiven Verständnis des Relativitätsprinzips die beiden Beobachter äquivalent sein sollten. Dieser Effekt tritt in der SRT nur auf, wenn mindestens einer der beiden Beobachter unterwegs beschleunigt wird (und sei es in einem Delta-Peak). Daher halte ich es für falsch zu sagen dass Beschleunigungen für das Zwillingsparadoxon irrelevant sind.

    Dass MartinB das anders sieht, kann ich mir nur so erklären, dass er entweder etwas anderes unter dem Zwillingsparadoxon versteht, oder dass Relevanz für ihn etwas anderes bedeutet.

    Alles andere waren meines Erachtens nach weniger wichtige Nebenschauplätze.

    Und ich halte die Frage danach, was denn nun für den Effekt verantwortlich ist, für nicht sinnvoll. Jedenfalls solange “Verantwortlichkeit” nicht klar definiert ist.

  76. #76 MartinB
    12. Juli 2018

    @Tox
    Wie oben gesagt, der Knackpunkt liegt unserer Ansicht nach in der genauen Definition von “Beschleunigung”.

  77. #77 Tox
    12. Juli 2018

    @MartinB
    Mit ist keine sinnvolle Definition von “Beschleunigung” bekannt für die meine Aussage aus Kommentar #75 falsch ist.

  78. #78 MartinB
    12. Juli 2018

    @Tox
    Siehe die Diskussion oben zu affinen Parametern etc..
    Für deine Einschränkung (betrachte nur 2 Beobachter, schließe also explizit Szenarien wie das 3-Brüder-Paradoxon aus), ist deine #75 korrekt, in meinen Augen aber irreführend, wie oben erläutert.

  79. #79 Tox
    12. Juli 2018

    @MartinB
    Das ist was ich damit meinte, dass du etwas anderes unter dem “Zwillingsparadoxon” verstehst. Wie ich bereits mehrfach schrieb, sehe ich im Fall der drei Brüder kein (scheinbares) Paradoxon.

  80. #80 MartinB
    12. Juli 2018

    @Tox
    Ja, dann hat es vielleicht keinen Sinn, weiterzudiskutieren, weil du irgendwas anders siehst als ich (und ich nicht wirklich verstehe, was das ist).

  81. #81 Tox
    12. Juli 2018

    @MartinB
    Ich sehe exakt das was ich in Kommentar #75 (und weiteren Kommentaren weiter oben, beginnend mit #5) beschrieben habe. Wenn ich mich richtig erinnere, hast du leider bisher noch nicht erklärt, was genau du unter dem “Zwillingsparadoxon” verstehst und was genau in der Situation mit den drei Brüdern das scheinbare Paradoxon sein soll (siehe dazu auch die Diskussion zwischen H.H.Voynich und mir).

  82. #82 MartinB
    12. Juli 2018

    @Tox
    “was genau du unter dem “Zwillingsparadoxon” verstehst”
    Das Paradoxon ist zunächst mal das, was du auch darunter verstehst. Vergleicht man die Situation aber mit den 3 Brüdern, sieht man, dass beide Fälle letztlich dieselben sind – es wird jeweils die Summe zweier Strecken mit einer dritten verglichen, die Zahlen sind jeweils identisch. Sobald die drei Strecken ein Dreieck bilden, kommt dabei was anderes raus, genau wie bei der Dreiecksungleichung in der Ebene. Das nicht auf die Geometrie sondern auf die physikalische Beschleunigung (also das Wirken von Kräften etc.) zurückzuführen, halte ich wie erklärt für irreführend. (Siehe auch das Beispiel mit der Zange.)

  83. #83 Tox
    12. Juli 2018

    @MartinB
    Ein allerletzter Versuch:

    Du erklärst meiner Meinung nach überhaupt nicht, was an der Situation mit den drei Brüdern “paradox” sein soll. Seit Kommentar #5 warte ich darauf, dass mir das jemand erklärt. Aber wenn ich nichts übersehen habe, sind die einzigen Antworten darauf bisher, dass die Situation im wesentlichen die selbe wie bei den zwei Schwestern sei. Aber wie man zu dieser Überzeugung gelangt wurde bisher nicht erklärt.

    Oder ausführlicher: Warum ist es sinnvoll, die beiden Eigenzeiten zu addieren? Warum ist es sinnvoll, diese Summe mit der Eigenzeit des anderen Bruders zu vergleichen? Und am wichtigsten: Warum ist das Resultat dieses Vergleichs “paradox”? Bei der Standardsituation mit den zwei Schwestern sind die Antworten auf (die Äquivalente) all dieser Fragen offensichtlich. Bei den drei Brüdern ist das meiner Meinung nach überhaupt nicht der Fall.

    Aus diesen Gründen sind die beiden Situationen meiner Meinung nach nicht “letztlich dieselben”. Was sich auch darin äußert, dass man sonst ja die drei-Brüder-Situation nicht hätte erfinden müssen.

    … zurückzuführen …

    Das ist wieder so ein schwammiges Wort, das ich (wenn ich mich richtig erinnere) in dieser Diskussion selbst nie verwendet habe. Ein letztes Mal: Meiner Ansicht nach tritt das Zwillingsparadoxon ohne Beschleunigungen nicht auf, daher können Beschleunigungen für das Zwillingsparadoxon nicht irrelevant sein.

  84. #84 MartinB
    12. Juli 2018

    @Tox
    “Du erklärst meiner Meinung nach überhaupt nicht, was an der Situation mit den drei Brüdern “paradox” sein soll. ”
    Daran ist ja auch nichts paradox. Und da die Situation des echten Zwillingsparadoxons identisch ist (es werden exakt dieselben Entfernungen in der Raumzeit und dieselben punktre betrachtet) ist auch daran nichts paradox.

    “Oder ausführlicher: Warum ist es sinnvoll, die beiden Eigenzeiten zu addieren? ”
    Weil das dieselben zeiten sind, die auch Serena misst wenn sie erst neben dem einen und dann neben dem anderen Bruder herfliegt. Wenn also das dreiBrüder-Szenario nicht paradox ist, und die beiden Zeiten, die wir da addieren, jeweils identisch zu den zeiten von Serena sind, dann ist auch an Serenas Situation nichts paradox oder irgendwie seltsam.
    Oder anders gesagt: Wenn uns klar ist, dass die Summe der beiden Brüderzeiten natürlich nicht gleich der Zeit von teresa ist, dann muss dasselbe auch für Serena gelten, weil wir genau dieselben zeiten addieren – wenn Serena bei Alpha Centauri ihre Geschewindigkeit instantan umkehrt, dann ist ihre Zeit immer synchron zur Zeit des einen bzw. anderen Bruders.
    Natürlich muss Serena ihre Geschwindigkeit dazu umkehren, wenn wir darauf bestehen, dass die Summe der Zeiten gleich der Eigenzeit einer Beobachterin ist – aber das Szenario zeigt eben, dass diese Forderung eigentlich nur obendraufgepackt ist, die hat mit den verstrichenen Zeiten nichts zu tun.

  85. #85 Matthias U
    Nürnberg
    15. Juli 2018

    “Wenn ich einen Kilometer nach Norden gehe, dann einen nach Osten, dann habe ich zwei Kilometer zurückgelegt, aber die Entfernung zwischen Start- und Zielpunkt beträgt eben nicht zwei Kilometer.”

    Auf einer 2km großen Kugel schon. 🙂

  86. #86 Matthias U
    15. Juli 2018

    Ein weiterer Grund, warum Beschleunigungen irrelevant sind: ich muss sie nicht mal bemerken. Wenn ein vorbeirasendes schwarzes Loch mich in Richtung Alpha Centauri katapultiert, und mich ein zufällig dort herumliegender und von mir knapp verfehlter Neutronenstern umkehren lässt, dann bin ich bei meiner Rückkunft trotzdem jünger als mein Zwillingsbruder – obwohl ich von irgendeinem Wechsel der Bezugssysteme, egal ob verursacht durch Beschleunigung oder sonstwas, nicht das Geringste gespürt habe.

  87. #87 MartinB
    15. Juli 2018

    @Matthias U
    1. Wow. Das gibt die goldene Rasierklinge am Zopf für die Haarspaltung der Woche.
    2. Schon – aber da würde dann die Beschleunigungsfraktion sicher argumentieren, dass da dann ja Effekte der ART ins Spiel kommen (was in dem fall ja auch rchtig wäre, wenn du hinreichend dicht am Neutronenstern vorbeisaust, kommt ja die Zeitdilatation der ART hinzu.)

  88. #88 PS
    26. Juli 2018

    Ich finde, Tox’s Anliegen ua in #75 hat etwas für sich. Die Zeit vergeht in Bezug auf Materie, nicht in Bezug auf das Nichts und nicht in Bezug auf die Ausbreitung von Licht. Das Minkowski-Diagramm ist der in Geometrie übersetzte mathematische Formalismus der Lorentz-Transformation. Es ist die Abbildung, nicht die Kausalursache des unterschiedlichen Verbrauchs an Eigenzeiten beim Zwillingsparadoxon. Ich bin bei Tox, dass als Kausalursache die Beschleunigung in Frage kommt. Letztlich ist (lineare) Beschleunigung im Rahmen der SRT durch Rückstoß (ev mit Photonen) möglich. Der Rückstoß ist ein Nullsummenspiel, sind es in letzter Hinsicht auch die Zeitverschiebungen? Die Sinnhaftigkeit solcher Fragen kann man nur einschätzen, wenn man die Rolle der Beschleunigung bei Zeitverschiebungen kennt. Wenn man alles auf Geometrie reduziert, lässt man derartige Fragen – mE ohne Berechtigung – von vornherein nicht zu.

  89. #89 Lulu
    26. Juli 2018

    Das Minkowski-Diagramm zeigt klar, dass sich der Altersunterschied bei doppelter Reiselänge verdoppelt. Unabhängig von der Beschleunigung.

  90. #90 PS
    26. Juli 2018

    @ Lulu
    Da sich in Deinem Beispiel nur die “Reiselänge” (offenbar räumlich und zeitlich vom ruhenden Zwilling aus gesehen), nicht aber die Reisegeschwindigkeit und die Beschleunigung an den Umkehrpunkten ändert, ist der Umstand, dass sich der Altersunterschied verdoppelt, kein Indiz dafür, dass er von der Beschleunigung unabhängig wäre.

  91. #91 Lulu
    27. Juli 2018

    Äh – doch???

  92. #92 PS
    27. Juli 2018

    Denkt man sich die Reisegeschwindigkeit weg, gibt es keinen Altersunterschied. Denkt man sich die Beschleunigung weg, gibt es auch keinen Altersunterschied.

    Aber ich verstehe Dein Anliegen. Das Ausmaß des Effekts wird auch dann größer, wenn gar keine (zusätzliche) Beschleunigung ins Spiel gebracht wird. Das hat eben damit zu tun, dass beide Zwillinge sich gegenseitig langsamer altern sehen, jedoch der beschleunigte Zwilling die Symmetrie durch Beschleunigung bricht und sein Zeitguthaben “nach Hause bringen kann”. Die Größe des Zeitguthabens ist auf die Reisedauer und die durchschnittliche Reisegeschwindigkeit zurückzuführen, seine Verwirklichung auf die Beschleunigung.

  93. #93 Lulu
    27. Juli 2018

    Interessant. Angenommen, das Universum ist rund, so dass man, wenn man immer geradeaus fliegt, wieder am Ausgangspunkt ankommt. Wie ist dann der Altersunterschied? Ist dann eine Beschleunigung involviert?

  94. #94 Lulu
    27. Juli 2018

    Und: Könnte man die Situation dann mit einem auf einen Zylinder gewickelten Minkowski-Diagramm modellieren?

  95. #95 PS
    27. Juli 2018

    Meine Kenntnisse der ART reichen nicht aus, diese Fragen zu beantworten. Sollten zwei Uhren, die sich mit hoher gleichförmiger Geschwindigkeit voneinander entfernen, wirklich irgendwann wieder aneinander vorbeikommen, so sagt meine Intuition, dass sie die gleiche Zeit anzeigen sollten. Da aber – bei Ausblendung gravitativer Wirkungen – jede Uhr hinter den Anzeigen der Uhren eines anderen Systems zurückbleibt, an denen sie vorbeikommt, entstünde die paradoxe Situation, dass beim besagten Treffen auf der Uhr 1 weniger Zeit vergangen ist als auf der Uhr 2, und auf der Uhr 2 weniger Zeit vergangen ist als auf der Uhr 1.
    Ich vermute, dass die Prämisse Deiner Fragen nicht den tatsächlichen Raumverhältnissen der ART entspricht. Sie können aber dahingestellt bleiben, bis über die Rolle der Beschleunigung in der SRT Einigkeit besteht.

  96. #96 Lulu
    27. Juli 2018

    Meine Prämisse führt zu einer Fülle von Widersprüchen. In Mathematik wäre das ein Beweis dafür, dass sie falsch ist.

  97. #97 MartinB
    28. Juli 2018

    @lulu
    Bin gerade im urlaub und lese nur spoadisch mit.
    Wenn ds univrsum z.b. ein zylinder wäre, dann würde serena beim losfliegen vor sich eine weitre vrsion von teresa sehen, das haben wir neulich diskutiert,michbglaube, bei alderamins post zum paradoxon, hab die quelle gerade nicht hier.

  98. #98 Alderamin
    28. Juli 2018

    @Martin, Lulu

    Ab hier bis #30, Austausch zwischen Daniel Rehbein und mir.

  99. #99 Laie
    30. Juli 2018

    Herr Gassner erklärt dies sehr schön über das RZ Diagramm.
    https://youtu.be/wvvngeHEq2M

    Die Zeit sieht man beim “Heimflug” nicht verlangsamt vergehen sondern beschleunigt weil man ja auf die Erde zufliegt.
    Vergl. Blauverschiebung

  100. #100 thomas
    LU
    10. August 2018

    Sehr interessanter Blog, Kompliment!
    Aber mal eine dumme Laienfrage zu “Entscheidend ist aber – und das macht dieses Diagramm sehr schön deutlich -, dass es einen weiten Bereich von Ereignissen bei Teresa gibt, die für Serena mit gar nichts gleichzeitig sind.”
    Das Kernproblem scheint mir die endliche Übertragungsgeschwindigkeit eines Zeitsignals zu sein.
    Kann mir gut vorstellen, dass es bei Schall (natürlich nicht im Weltraum) ganz ähnliche Probleme mit dem Begriff “gleichzeitig” gibt.

    Was aber, wenn die “Jahressignale” (zumindest Nr. 4-6) nicht per Photon, sondern per Quantenverschränkung übertragen werden? Kann man ja vorher verabreden und Experimente aufbauen.
    Heisst das, Serena kann die Signale/Verschränkung gar nicht bemerken? Oder kommen die gar zu willkürlichen Zeitpunkten bei Serena an?

  101. #101 MartinB
    10. August 2018

    @thomas
    Mit Quantenverschränkung kann man keine Signale übertragen, Information lässt sich da erst gewinnen, wenn Signale auf klassischem Wege übertragen werden, so dass die Partner ihre Messungen vergleichen können. Siehe meine Texte “Quantenmechanik verstehen” bei den Artikelserien, da erkläre ich das im Detail,

    Die endliche Übertragungsgeschwindigkeit (natürlich braucht das Licht von der Erde auch noch Zeit) wird bei diesen Überlegungen immer herausgerechnet. Im Bild oben ist A2 etwa gleichzeitig mit Serenas Umkehr, aber bis das Licht von A2 Serena erreicht, vergeht natürlich auch nochmal einige Zeit (Lichtsignale kann man im Diagramm unter 45° einzeichnen, wenn Entfernungen in Lichtjahren gemessen sind). D.h. Serena bekomt irgendwann das Lichtsignal von A2 und rechnet dann zurück, wann das Signal ausgesandt wurde. Das zeichnet man wie gesagt nie mit, sonst wäre es total verwirrend.

  102. #102 thomas
    12. August 2018

    Hallo Martin,

    die eigentliche Information würde natürlich mit dem vorher aufgebauten Experiment übertragen. (Enthält ein vorherbestimmtes Ereignis überhaupt eine Information?)
    Es diente mir eher als Hilfsmittel für den Begriff der Gleichzeitigkeit.
    Auch wenn man Experimente erst nachträglich verifizieren kann,
    sie haben (zu einer gewissen Eigenzeit, die sich in ein anderes System umrechnen lässt) stattgefunden.

    Aber jetzt schreibst Du, A2 wäre gleichzeitig mit Serenas Umkehr.
    Das verstehe ich nicht.
    Bei A2 sehe ich Teresa nach 3 Jahren bzw. Serena bei (ca.) 2.5 Jahren.
    Serenas Umkehr aber bei 4 “Serena-Jahren” bzw. 5 “Teresa-Jahren” (=”absoluten” Jahren auf der Zeitachse).
    Vielleicht kann ich das Diagramm nicht lesen?

  103. #103 MartinB
    12. August 2018

    Die farbigen Linien geben an, welche Ereignisse in Serenas Bezugssystem gleichzitig sind, näherungsweise (passt nicht ganz) sind das A2 und B.
    Für teresa sind beim A2 etwas mehr als 3 jahre vergangen, für Serena etwa 4 (Serena sieht ja auf dem Hinflug teresa auch verlangsamt).
    Serenas Umkehr ist dann in dem Bild bei 4 jahren ihrer Zeit, für Teresa sind das 5 Jahre.
    In dem Moment, wo Serena umkehrt “springen” die Linien ihrer Gleichzeitigkeit, jetzt ist für sie A3 gleichzeitig mit ihrer aktuellen zeit von 4 Jahren.

    Das mit der “Gleichzitigkeit” beim Quantenexperiment sehe ich nicht – man kann ja den Kollaps der QWellenfunktion nicht messen, also weiß man trotzdem nicht, was gleichzeitig ist. (Siehe diesen Artikel am Ende:
    https://scienceblogs.de/hier-wohnen-drachen/2012/10/10/quantenmechanik-und-realitat/

  104. #104 thomas
    Ludwigshafen
    13. August 2018

    Hallo Martin,

    ich habe den Eindruck, Du definierst Gleichzeitigkeit allein durch die Ankunft des Signals.
    A2 sei gleichzeitig mit B und B sei gleichzeitig mit A3.
    Ok, kann man machen…
    Zwischen A2 und A3 liegen aber ein paar Jahre. Folglich keine Transitivität bei “gleichzeitig”.
    Auch die Atomuhr der PTB (immerhin die gesetzliche Zeit) liefe für einen unregelmäßig bewegten Beobachter etwas erratisch.

    Man bräuchte wahrscheinlich mehrere verschiedene Worte und Bedeutungen für “gleichzeitig”.
    Der Duden gibt das (noch) nicht her.

    Wie denken denn die “Kollegen” aus der Astronomie?
    Ist das am 14.9.2015 empfangene Signal verschmelzender Schwarzer Löcher am 14.9.2015 passiert oder viel früher?
    Wenn “gleichzeitig” mit der Detektion, fände ich die angegebene Entfernung von 1.3 Milliarden Lichtjahren rätselhaft.
    (Rätselhaft jedenfalls aus irdischer Sicht, die lichtschnellen/zeitlosen Gravitonen mögen dagegen ihr eigenes Bezugssystem haben.)

  105. #105 MartinB
    13. August 2018

    @Thomas
    Nein, gleichzeitig sind alle Ereignisse, die für einen Beobachter gleichzeitig sind. Das lässt sich mit etwas Mühe auch eindeutig und sauber definieren. Die farbigen Linien oben im Bild sind nicht die Lichtsignale.

    Zwischen A2 und A3 liegen für Serena keine paar Jahre, für sie sind sie unmittelbar aufeinanderfolgend und somit fast gleichzeitig. (Was an ihrem Bezugssystemwechsel liegt.)

    Die Astronomen haben soweit ich weiß als konvention, dass sie das Messdatum als Ereignisdatum nehmen,s ie sagen, also, dass die Verschmelzung der SL am 14.9.2015 war, auch wenn die real natürlich vor 1,3 Mrd Jahren war. (In der Astronomie ist es mit der gleichzeitigkeit eigentlich einfacher, weil man das mit der Expansion mitbewegte bezugssystem verwenden kann, das ist eindeutig und legt eine Zeit fest.)

  106. #106 Anonym_2018
    13. August 2018

    weil man das mit der Expansion mitbewegte bezugssystem verwenden kann

    Darauf basiert auch eine Studie der NASA zum Warp-Antrieb: https://ntrs.nasa.gov/archive/nasa/casi.ntrs.nasa.gov/20110015936.pdf

  107. #107 Alderamin
    13. August 2018

    @MartinB

    In der Astronomie ist es mit der gleichzeitigkeit eigentlich einfacher, weil man das mit der Expansion mitbewegte bezugssystem verwenden kann, das ist eindeutig und legt eine Zeit fest.

    Ist es nicht, weil die Entfernung oft schwer zu bestimmen ist, wie der Fall der “Galaxie ohne Dunkle Materie” zeigt. Man kann oft froh sein, wenn man die Entfernung auf 20% genau bestimmen kann. Großräumig dominiert die Expansion (für die man glaubt, die Parameter ganz gut zu kennen, obwohl H0 fürs junge und alte Universum im Rahmen der Fehlerbalken nicht zusammenpassen wollen), aber lokal die Individualbewegungen der Galaxien, die dann keine einfache Zuordnung der Rotverschiebung zur Entfernung erlauben. Und ohne Entfernung ist es müßig, über den vergangenen Zeitpunkt eines heute beobachteten Ereignisses zu reden. Insbesondere, wo es manchmal auf hohe Präzision bei Zeitangaben ankommt (z.B. Pulsar-Timing)

    Die Astronomie ist zudem viel, viel älter als die Geschichte ihrer Entfernungsbestimmung und daher macht es sehr viel Sinn, dass sie bei den Rohdaten bleibt (haben wir auch im Physik-Praktikum so gelernt, Rohdaten beim Experiment aufschreiben, nicht abgeleitete Größen) – z für die Rotverschiebung und das Beobachtungsdatum für den Zeitpunkt. (Außerdem rechnen Astronomen furchtbar gerne in cgs-Einheit, aber das ist ein anderes Thema…)

  108. #108 MartinB
    14. August 2018

    @Alderamin
    Das ist aber nur, weil die ollen Astronominnen nicht richtig messen können – theoretisch ist es einfach 😛

    @Anonym
    Der Alcubierre-Antrieb hat mit der Expansion des Alls aber nix zu tun, der nutzt exotische Materie, um den Raum lokal aufzublasen bzw. zu schrumpfen. Ob das selbst theoretisch funktionieren kann, ist aber sehr umstritten.

  109. #109 thomas
    14. August 2018

    OT, aber das liegt mir auf der Zunge: Wenn die Physiker richtig messen würden, bräuchte es den ganzen Quantenmist nicht. 😉

  110. #110 Mike
    14. August 2018

    Ich habe an verschiedenen Stellen sowohl von Physikern wie auch von Nichtphysikern Diverses zum Zwillingsparadoxon gelesen. Dabei habe ich für mich festgestellt, dass auch unter Physikern sehr unterschiedlich erklärt wird – glücklicherweise am Ende mit selbem Ergebnis.

    Die Sache mit der Beschleunigung. Da wird dann ausgeführt, dass der eine Zwilling sich vom anderen dadurch auszeichnet, dass er Beschleunigungen erfährt. Mal geht es dann mit sich ändernden Bezugssystemen weiter, mal mit der Diskussion, ob ART dann nötig wird oder eben nicht (wie hier richtig erklärt). Mal geht es zu den drei Brüdern. Für mich selbst fand ich hilfreich die Abwandlung des Zwillingsparadoxons, wo beide die gleichen Beschleunigungen erfahren, der eine halt nur viel früher zurückkommt. Das finde ich in den Erklärungen ansonsten eher selten. Welchen Grund hat das?

    Der Zugang über die Raumzeit. Das habe ich als dermaßen grundlegend empfunden, dass dabei die Aha-Effekte viel stärker waren. Bei Epstein / Eckstein findet man dazu sinngemäß den Satz: “Da der am Ende jüngere Zwilling einen Teil dieser Raumzeit-Strecke für den Raum beansprucht, bleibt bei ihm weniger für die Bewegung in der Zeit übrig.” Das leuchtet mir ein. Lese ich ja auch hier, nur irgendwie nicht ganz so einfach ausgedrückt.

    Apropos – das Märchen über die Raumzeit – absolut genial. So hätte ich das früher gerne aufbereitet bekommen. Wann gibt es diese ganzen Blogs endlich als Buch? Ok, vielleicht eine rhetorische Frage – ich hoffe, “Isaac oder Die Entdeckung der Raumzeit” ist so etwas.

  111. #111 MartinB
    14. August 2018

    @Mike
    Ja, die verschiedneen Ansätze zur Erklärung sind hier ja auch Thema – man stellt immer wieder fest, dass Physikerinnen nicht immer so gut darin sind, ihre Gleichungen zu erklären wie darin, sie aufzustellen und mit ihnen zu rechnen. Hier beim Zwillingsparadoxon sind verschiedene Erklärungen “richtig” – es ist immer eine Frage der Sichtweise und der genauen Wortbedeutungen, siehe die Diskussion oben zur Frage was genau eine Beschleunigung ist.

    Das Buch ist keine Sammlung von Blogartikeln, sondern wirklich ein Buch zur ART.

  112. #112 Mike
    14. August 2018

    Wenn Du Dir da ähnlich Mühe gibst bezüglich begreifbarem und so anschaulich als möglichem Zugang wie im Blog, sollte der vorhandene Eintrag auf meiner Bücherliste “trotzdem” gerechtfertigt sein. Allerdings kaufe ich immer erst, wenn es ein paar Rezensionen gegeben hat.

  113. #113 MartinB
    14. August 2018

    @Mike
    Der Zugang ist zumindest teilweise – nun ja, anders.
    Das mit den renzensionen ist generell ein guter Ansatz, auch wenn es natürlich Kants kategorischem Imperativ widerspricht (denn wenn das alle machen würden…) 🙂

    Wenn das Buch rauskommt (soll jetzt tatsächlich schon im Herbst passieren) gibt’s hier mehr Infos dazu.

  114. #114 Mike
    14. August 2018

    @MartinB

    Du musst nur genug kostenfreie Rezensionsexpemplare verteilen 😉 Nee, das wird schon klappen. Gerade ist ein Buch auf meiner Liste gelandet, das hat Sabine Hossenfelder gestern rezensiert, auch ein Rezensionsfreiexemplar. Ohne die Rezension wäre ich nie darauf gekommen.

    Anders finde ich gut. Ich habe vor vielen Jahren studiert. Es gab kein Internet, und an Büchern nur das, was in der Unibibliothek stand. Dinge auch mal anders dargeboten zu bekommen, ging im Studium allein aus Zeitgründen nicht. So hat man (Be)Rechnen gelernt ohne Ende, ganz viel rein “mechanisch”, das Verständnis blieb trotz bestandener Prüfungen manches Mal ein wenig auf der Strecke – zumindest habe ich das so empfunden. Später habe ich nichts mehr damit zu tun gehabt, viel ist nicht hängengeblieben. Mit ein klein wenig Grundverständnis Dinge anders aufbereitet zu lesen, auch unterhaltender, ist da genau das Richtige. Wenn man sich zwar sehr interessiert, aber genau weiß, Wissen und Methoden auf Studiumslevel sich nicht mehr aneignen zu können.

    Bei Dir habe ich ab und an das Gefühl, dass Du durch Dein Blog-Schreiben auch Dir selbst Dinge klarer machen möchtest. Was vielleicht mit dem Wissen im Kopf, für Leser zu schreiben, die nicht alle Physiker oder Mathematiker sind, ganz gut hinhaut auch für die Leser.

    Genug offtopic – wir reden dann übers Buch, wenn es da ist.

  115. #115 MartinB
    14. August 2018

    @Mike
    ” das Verständnis blieb trotz bestandener Prüfungen manches Mal ein wenig auf der Strecke – zumindest habe ich das so empfunden.”
    So ging es mir auch.
    “Bei Dir habe ich ab und an das Gefühl, dass Du durch Dein Blog-Schreiben auch Dir selbst Dinge klarer machen möchtest. ”
    Klar, daraus habe ich nie ein Geheimnis gemacht. Das Buch ist mein Versuch, die ART zu verstehen (so wie diverse Blogartikel vorher, die da natürlich eingeflossen sind). Mit meiner QFT-Serie war es ähnlich.

  116. #116 Markweger
    17. August 2018

    Und wenn beide symmetrisch in die Gegenrichtung reisen und zurück dann kommt jeder dem anderen gegenüber jünger zurück.
    Auch nicht schlecht.

  117. #117 MartinB
    17. August 2018

    @Markweger
    Ja, wir wissen jetzt alle, dass du das Zwillingsparadoxon nicht verstanden hast – es ist wirklich nicht notwendig, uns deine Ignoranz permanent wieder vor Augen zu führen, wir haben das verstanden.

    Aber danke für’s mitspielen…

  118. #118 roel
    17. August 2018

    @MartinB
    “Das Buch ist mein Versuch, die ART zu verstehen (so wie diverse Blogartikel vorher, die da natürlich eingeflossen sind).”

    Jetzt kommt die Frage, die bei so einer Aussage fast zwangsläufig kommen muß: Ist der Versuch geglückt?

  119. #119 MartinB
    17. August 2018

    @roel
    Ich verstehe jetzt sehr viel mehr – klar, manche Details in technischen papern hängen mich immer noch ab, aber ich glaube, ich habe für die wesentlichen Effekte eine gute Intuition entwickelt, die ich auch zu erklären versuche…

  120. #120 roel
    17. August 2018

    @MartinB
    “manche Details in technischen papern hängen mich immer noch ab”

    Ich kann mir das nur so vorstellen, dass diese Details halt nicht gut erklärt werden. Das ist vielleicht auch nicht der Sinn dieser paper, aber es würde sie sinnvoller machen.

    “aber ich glaube, ich habe für die wesentlichen Effekte eine gute Intuition entwickelt, die ich auch zu erklären versuche…”

    Da bin ich mir sehr sicher! Der voraussichtliche Veröffentlichungstermin, habe ich gesehen, ist jetzt der 04.11.18. Für alle, die noch keine Weihnachtsgeschenke haben…

  121. #121 MartinB
    17. August 2018

    @roel
    “Ich kann mir das nur so vorstellen, dass diese Details halt nicht gut erklärt werden. Das ist vielleicht auch nicht der Sinn dieser paper, aber es würde sie sinnvoller machen.”
    Naja, wenn’s sehr technisch wird, bin ich halt nicht mehr so versiert (gerade in Sachen Rechentechniken usw.) und da gibt es jede Menge spezielle Dinge, in die man sich erst reinarbeiten müsste.

  122. #122 Toni
    13. Dezember 2018

    Sie sagen, dass der Wechsel des Bezugssystems und nicht die Beschleunigung verantwortlich für die Auflösung des Paradoxons ist. Jedoch wenn Sie sich in einem beschleunigten System befinden, wechseln Sie Ihr Bezugsystem zu jedem Zeitpunkt. Bei Zwillingsparadoxon ist der Wechsel des Bezugssystems eng verknüpft mit Beschleunigung. Selbst wenn Sie mit einer konstanten Geschwindigkeit fliegen, und plötzlich Ihre Bewegungsrichtung ändern, führen Sie eine beschleunigte Bewegung aus, sowie Kreisbewegung oder freier Fall von ISS um die Erde aus der Sicht eines Beobachters auf der Erde.

    Angenommen Serena würde beschleunigt einfach nur Geradeaus fliegen, sie erhält eben durch diese Beschleunigung eine gravitative Zeitdilatation (Zeit läuft für sie langsamer).

    Auch in der englischen Wikipedia sehen Sie eine sehr schöne Rechnung bezüglich des Zwillingsparadoxons, in der das ganze mit Beschleunigung gerechnet wurde.

    Die beste Erklärung für das Zwillingsparadoxon ist, dass die Symmetrie, die man in SRT hat, nun gebrochen ist. Man vergleicht eben nicht zwei gleichwertige Systeme miteinander.

    Zur Vervollständigung SRT kann nur beschleunigte Systeme rechnen, wenn der Beobachter sich selbst in einem Inertialsystem befindet.

  123. #123 MartinB
    13. Dezember 2018

    @Toni
    “Angenommen Serena würde beschleunigt einfach nur Geradeaus fliegen, sie erhält eben durch diese Beschleunigung eine gravitative Zeitdilatation (Zeit läuft für sie langsamer).”
    Nein. Die Dilatation ist gleich der Dilatation, die durch die jeweilige Geschwindigkeit zustande kommt, muss halt nur über die sich ständig ändernde Geschwindigkeit aufintegriert werden. Es gibt aber keinen zusätzlichen Effekt durch die Beschleunigung.

    “Jedoch wenn Sie sich in einem beschleunigten System befinden, wechseln Sie Ihr Bezugsystem zu jedem Zeitpunkt. ”
    Jede Beschleunigung wechselt das Inertialsystem im Sinne der SRT. Aber nicht jeder Wechsel des Inertialsystems ist eine echte Beschleunigung, wie man ja auch an der “Drei-Brüder-Form” des Paradoxons sieht.

    Den letzten Satz verstehe ich nicht – die SRT kann natürlich auch berechnen, was ein beschleunigter Beobachter sieht oder misst.

  124. #124 Toni
    13. Dezember 2018

    “Nein. Die Dilatation ist gleich der Dilatation, die durch die jeweilige Geschwindigkeit zustande kommt, muss halt nur über die sich ständig ändernde Geschwindigkeit aufintegriert werden”

    In SRT gibt es keine reale Zeitdilatation (sie ist beobachterabhängig und eine reine Illusion). Die echte (gravitative) Zeitdilatation kommt nur durch Beschleunigung (Gravitation) zustande, weil die Symmetrie in SRT nun gebrochen ist, wenn Beschleunigung da ist. Mein Punkt ist, dass dieser Wechsel des Bezugssystems, von dem Sie reden, kommt gerade durch Beschleunigung zustande und beide zwei Seiten des gleichen Medallions sind.

    “Den letzten Satz verstehe ich nicht – die SRT kann natürlich auch berechnen, was ein beschleunigter Beobachter sieht oder misst.”

    Die SRT kann natürlich auch berechnen, was ein beschleunigter Beobachter sieht oder misst, WENN derjenige, der diese Rechnung macht, sich selbst in einem Inertialsystem befindet.

  125. #125 MartinB
    13. Dezember 2018

    @Toni
    Das mit der Beschleunigung halte ich für falsch – solange ich nur relativ zu dir beschleunige, ist die Dilatation dadurch doch nicht “echter” als wenn ich mit konstanter Geschwindigkeit fliege.

    Mathematisch siehe dazu
    https://en.wikipedia.org/wiki/Rindler_coordinates
    Da sieht man doch direkt am dt²-Term, dass da letztlich die momentane Geschwindigkeit eingeht.

    In der ART ist das nur deswegen anders, weil es Dilatationseffekte auch dann gibt, wenn zwei Beobachterinnen relativ zueinander ruhen, so dass sie durch Uhrenvergleich Einigkeit erzielen können, wessen Uhr langsamer geht.

    Den letzten Satz halte ich auch für falsch – sonst könnte man Beschleunigungen dadurch messen, dass man guckt, ob man in der Lage ist, eine Rechnung durchzuführen? Oder was genau meinst du?

  126. #126 Toni
    13. Dezember 2018

    “Das mit der Beschleunigung halte ich für falsch”
    Der Wechsel des Bezugssystems gibt es, weil man beschleunigt, sonst kann man nicht von einem Wechsel des Bezugssystems reden. Das ist ganz klar.

    Hier die Rechnung mit Beschleunigung:
    https://en.wikipedia.org/wiki/Twin_paradox

    ” Oder was genau meinst du?”
    Ich meine, dass SRT nur aus einem Inertialsystem heraus angewendet werden darf. Der Beobachter, der die Zeit für einen anderen beschleunigten Beobachter mittels SRT rechnet, muss sich entwender in Ruhe befinden oder mit konstanter Geschwindigkeit bewegen. Das ist auch ganz klar.

  127. #127 MartinB
    13. Dezember 2018

    @Toni
    Die Rechnung kenne ich, und ja, jede Beschleunigung kann man im Sinne der SRT als Wechsel des bezugssystems auffassen. Aber die Formulierung mit den drei Brüdern oder die anderen Szenarien, die wir oben in den Kommenatren diskutiert haben, zeigen, dass umgekehrt nicht unbedingt jeder Wechsel eines Bezugssystems physikalisch als Beschleunigung interpretiert werden kann.

    Nehmen wir an, ich sitze in einem permanent beschleunigenden Raumschiff. Was hindert mich daran, dieslebe Rechnung durchzuführen, mit der du in einem nicht beschleunigten Raumschiff meine Bewegung berechnest? Verstehe ich wirklich nicht.

  128. #128 Toni
    13. Dezember 2018

    “Was hindert mich daran, dieslebe Rechnung durchzuführen, mit der du in einem nicht beschleunigten Raumschiff meine Bewegung berechnest?”

    Das Relativitätsprinzip der SRT hindert Sie daran.
    Sie können nicht so rechnen wie Sie wollen. Sie sollen beachten, in welchem System Sie sich befinden.

    Die Rechnung mit Atomuhren für Nachweis von Zeitdilatation (eine im Flugzeug und eine auf der Bodenstation) macht man aus der Sicht eines dritten Systems, das Inertial ist, damit man SRT anwenden kann, sonst geht das nicht.

  129. #129 MartinB
    13. Dezember 2018

    @Toni
    Ich raff’s nicht: Ein Buch, in dem eine korrekte Rechnugn drinsteht, sieht doch nich tplötzlich anders aus, nur weil ich es beschleunige und die Rechnung wird auch nicht falsch… Du meinst sicher irgendwas anderes, aber ich weiß nicht was.

  130. #130 Toni
    13. Dezember 2018

    Ja, ich meine was anderes.

    SRT lässt sich auf beschleunigte System anwenden, wenn man selbst NICHT in einem beschleunigten System ist. Das ist die Kernaussage.

    Übrigens die Idee mit der Beschleunigung für Zwillingsparadoxon kam von Einstein selbst.

  131. #131 MartinB
    13. Dezember 2018

    “SRT lässt sich auf beschleunigte System anwenden, wenn man selbst NICHT in einem beschleunigten System ist. Das ist die Kernaussage.”
    Ich verstehe diese Aussage nicht. Wieso soll die Frage, wo die Person sitzt, die irgendwas berechnet, relevant sein? Wer ist “man selbst”? Warum soll jemand in einem beschleunigten Bezugssystem nicht einfach z.B. mit der Rindler-Metrik rechnen können?

  132. #132 Toni
    13. Dezember 2018

    “Ich verstehe diese Aussage nicht. Wieso soll die Frage, wo die Person sitzt, die irgendwas berechnet, relevant sein? Wer ist “man selbst”? ”

    Um die Gesetze der SRT anwenden zu können (egal af welche Systeme), muss der Beobachter sich entweder in Ruhe befinden oder sich mit konstanter Bewegung fortbewegen. Wenn der Beobachter sich selbst in einem beschleunigten System befindet, kann er in diesem System nicht die Gesetze der SRT anwenden, um zu rechnen, was bei anderen Beobachter passiert.

    Die Zeit von Serena auf der Reise wird mittels SRT aus der Sicht eines Beobachters in einem Inertialsystem berechnet. Die Annahme ist, dass die Zwillingsschwester auf der Erde in einem Inertialsystem ist.

  133. #133 Anonym_2018
    13. Dezember 2018

    @Toni #126

    Der Wechsel des Bezugssystems gibt es, weil man beschleunigt

    Der Wechsel des Bezugssystems kann für die Zeitmessung auch ohne Beschleunigung erfolgen:

    Man kann sich statt des Zwillingsparadoxons das sogenannte Uhrenparadoxon ansehen, in dem überhaupt keine Beschleunigungen vorkommen. Statt des wegfliegenden und zurückkehrenden Zwillings hat man eine wegfliegende und eine auf die Erde zufliegende Uhr, wobei letztere in dem Moment, wo sie an der wegfliegenden Uhr vorbeifliegt, auf dieselbe Zeit gestellt wird wie die wegfliegende.

    https://www.wissenschaft-im-dialog.de/projekte/wieso/artikel/beitrag/was-hat-die-beschleunigung-mit-einsteins-zwillingsparadoxon-zu-tun/

    Dieses Argument steht in diesem Artikel auch schon drin, durch den Link zum “Drei-Brüder-Ansatz”.

    Außerdem wurde das experimentell untersucht:

    In den weiter oben genannten Experimenten wurde die Zeitdilatation während des Teilchenzerfalls gemessen, als die Teilchen keiner Beschleunigung ausgesetzt waren. Hingegen Bailey et al. (1977) bestätigten auch die sogenannte „Uhrenhypothese“, wonach die Beschleunigung die Zeitdilatation nicht beeinflusst. In ihrem Experiment waren die Myonen einer dauernden transversalen Beschleunigung von bis zu ∼10¹⁸ g ausgesetzt, trotzdem erhielten sie den gleichen Wert für die Zeitdilatation wie in den anderen Experimenten.

    Quelle:
    https://de.wikipedia.org/wiki/Zeitdilatation_bewegter_Teilchen#Uhrenhypothese

  134. #134 Toni
    13. Dezember 2018

    @Anonym_2018:

    Die fliegende Atomuhr im Flugzeug befindet sich in einem System, das aus der Sicht eines ruhenden Beobachters, ein beschleunigtes System ist. Das ist analog zu einem Satellit, der im freien Fall ist, und ein beschleunigtes System darstellt aus der Sicht eines ruhenden Beobachters auf der Erde (dazu kannst du mehr im Buch von Torsten Fließbach lesen).

    “Man kann sich statt des Zwillingsparadoxons das sogenannte Uhrenparadoxon ansehen, in dem überhaupt keine Beschleunigungen vorkommen”

    In Uhrenparadoxon, die wegfliegende Uhr stellt eine beschleunigte Bewegung aus der Sicht eines ruhenden Beobachters dar. Kreisbewegungen sind beschleunigte Bewegungen, weil sich der Geschwindigkeitsvektor ständig ändert, und das kann nur dann passieren, wenn eine äußere Kraft wirkt, in dem Fall Zentripetalkraft. Auch hier wirkt die Beschleunigung (Gravitation) und ein ständiges Wechsel des Bezugssystems.

    Die Zeitdilatation gibt es sobald die Symmetrie der völlig gleichberechtigten Systeme (das Relativitäsprinzip) gebrochen ist.

    “Außerdem wurde das experimentell untersucht:”
    SRT Effekte sind ALLE SCHEINEFFEKTE, weil sie beobachterabhängig sind. Es ist nur die Beschleunigung, die die Zeit verlangsamt.

  135. #135 Anonym_2018
    13. Dezember 2018

    @Toni #134

    In Uhrenparadoxon, die wegfliegende Uhr stellt eine beschleunigte Bewegung aus der Sicht eines ruhenden Beobachters dar. Kreisbewegungen sind beschleunigte Bewegungen

    Man kann sich ja beide Uhren als grandlinig-gleichförmig bewegte Atomuhren mit Digitalanzeige vorstellen. Dann bewegt sich nichts im Kreis.

  136. #136 Toni
    13. Dezember 2018

    “Man kann sich ja beide Uhren als grandlinig-gleichförmig bewegte Atomuhren mit Digitalanzeige vorstellen. Dann bewegt sich nichts im Kreis.”

    Dann misst man auch keine Zeitdilatation.
    Das mit der Atomuhr kannst du im Buch von Roman Sexl (Raum -Zeit Relativität) genauer lesen.

  137. #137 Anonym_2018
    13. Dezember 2018

    @Toni #136

    Dann misst man auch keine Zeitdilatation.

    Und wie kann man dann, ohne real existierende SRT-Zeitdilatation, erklären, dass ein Lichtstrahl im Vakuum in jedem grandlinig-gleichförmig Bezugssystem die gleiche Geschwindigkeit von ca. 300.000 km/s hat?

  138. #138 Toni
    13. Dezember 2018

    Licht hat kein eigenes Bezugsystem.

    Das Relativitätsprinzip macht sehr deutlich, dass alle Effekte aufgrund der Geschwindigkeit Effekte sind, die scheinbar sind und vom Beobachter abhängen. Aber wenn Menschen annehmen, dass Geschwindigkeit eine intrinsische Eigenschaft ist, erwarten sie fälschlicherweise echte physikalische Veränderungen am sich bewegenden Objekt selbst.

    Die Zeitdilatation durch Beschleunigung (oder Schwerkraft) ist eindeutig. Eine Uhr auf einem Planeten ist langsamer als eine Uhr, die weit entfernt von jeglicher Schwerkraft ist, unabhängig davon, wer die Uhren beobachtet.

    Aber die Zeitdilatation, die durch die Relativgeschwindigkeit verursacht wird, ist so etwas wie eine Illusion, aber es ist keine Illusion der Augen. Für jeden in Bewegung befindlichen Beobachter relativ zueinander ist die Uhr des anderen Beobachters langsamer. Dies ist eine Einschränkung der Beobachtungsfähigkeit, da sie nicht beide langsamer sein können als die anderen.

    Die konstanz der Lichtgeschwindigkeit ist ein Postulat. Sie ist nicht die Konsequenz der scheinbaren Zeitdilatation in SRT.

  139. #139 Niels
    13. Dezember 2018

    @Toni

    Um die Gesetze der SRT anwenden zu können (egal af welche Systeme), muss der Beobachter sich entweder in Ruhe befinden oder sich mit konstanter Bewegung fortbewegen. Wenn der Beobachter sich selbst in einem beschleunigten System befindet, kann er in diesem System nicht die Gesetze der SRT anwenden, um zu rechnen, was bei anderen Beobachter passiert.

    Du hast doch in #126 selbst auf den Wiki-Artikel verlinkt, in dem genau diese angeblich unmögliche Rechnung vorgeführt wird?
    Siehe dort unter:
    Difference in elapsed times: how to calculate it from the ship

    Dort könntest du dir dann auch gleich noch den Abschnitt “Role of acceleration” durchlesen:

    others note that the effect also arises if one imagines separate outward-going and inward-coming travellers, who pass each other and synchronize their clocks at the point corresponding to “turnaround” of a single traveller.
    In this version, physical acceleration of the travelling clock plays no direct role;
    “the issue is how long the world-lines are, not how bent”

  140. #140 Niels
    13. Dezember 2018

    Sorry, da hatte mein Browser offenbar Schluckauf…

  141. #141 Toni
    13. Dezember 2018

    @Niels:
    Die Rechnung mit Beschleunigung ist nicht unmöglich. Sie ist ganz real.

    “others note that …”
    Die Betonung liegt auf andere. Das ist ein Versuch Beschleunigung zu umgehen. Man sollte das Äquivalenzprinzip nicht versuchen zu umgehen

  142. #142 Anonym_2018
    13. Dezember 2018

    @Toni #138

    Dies ist eine Einschränkung der Beobachtungsfähigkeit, da sie nicht beide langsamer sein können als die anderen.

    Beide können langsamer sein als die anderen, genauso wie auch zwei Ereignisse gleichzeitig und nicht-gleichzeitigt sein können. Grund: Das ist relativ und damit Beobachter-abhängig.

    Die konstanz der Lichtgeschwindigkeit ist ein Postulat. Sie ist nicht die Konsequenz der scheinbaren Zeitdilatation in SRT.

    Aus diesem und dem anderen Postulat folgt, dass die Galilei-Transformation durch die Lorentz-Transformation ersetzt werden muss. Aus dieser folgen Längenkontraktion und Zeitdilatation, und dass folgendes falsch ist:

    #136

    Dann misst man auch keine Zeitdilatation.

  143. #143 Anonym_2018
    13. Dezember 2018

    @Niels #144

    Sorry, da hatte mein Browser offenbar Schluckauf…

    Das liegt nicht an deinem Browser, sondern an der Technik des Blogs. Ich beobachte heute Abend auch, dass die Veröffentlichung meiner Kommentare mit ca. 5 bis 10 Minuten Verspätung nach dem Abschicken erfolgt. D.h. mal sollte nicht mehrfach auf “Abschicken” klicken, sondern etwas abwarten.

  144. #144 Toni
    13. Dezember 2018

    @Anonym_2018:

    “Beide können langsamer sein als die anderen, genauso wie auch zwei Ereignisse gleichzeitig und nicht-gleichzeitigt sein können. Grund: Das ist relativ und damit Beobachter-abhängig.”

    Du wiederholst mich. Gerade aus diesem Grund sind ja SRT Effekte Schein Effekte.

    Man misst die Zeitdilatation, wenn man das Prinzip der VÖLLIGEN GLEICHBERECHTIGUNG der Inertialsysteme außer Kraft setzt. Ansonsten is es nur eine ILLUSION.

  145. #145 Toni
    13. Dezember 2018

    @Anonym_2018
    “Beide können langsamer sein als die anderen,”
    Dann hast du das Paradox im Zwillignsparadoxon offenbar nicht verstanden. Das Paradox ist ja, dass BEIDE nicht langsamer als der andere sein können, wenn wir nur SRT zugrunde legen. Und weil hier die Symmetrie gebrochen wird, misst man eine echte Zeitdilatationeffekt.

  146. #146 Anonym_2018
    13. Dezember 2018

    Man misst die Zeitdilatation, wenn man das Prinzip der VÖLLIGEN GLEICHBERECHTIGUNG der Inertialsysteme außer Kraft setzt.

    Bei dem Uhrenparadoxon wird das Problem umgangen durch Verwendung von drei Intertialsystemen, die paarweise gleichberechtigt sind, in denen sich jeweils eine Uhr befindet. Die Uhr A wird als ruhend angenommen. Dann fliegt die Uhr B auf diese zu und übernimmt beim Vorbeifliegen deren Uhrzeiteinstellung. Viel später kommt der Uhr B die Uhr C entgegen und diese übernimmt beim Vorbeifliegen deren Uhrzeiteinstellung. Wieder viel später trifft Uhr C die Uhr A und ein Uhrenvergleicht zeigt die (reale) Zeitdilatation.

    Direkt gemessen wurde die SRT-Zeitdilatation z.B. beim transversalen Doppler-Effekt:

    Jedoch besagt die Relativitätstheorie, dass jedes Objekt aufgrund seiner Bewegung einer Zeitdilatation unterliegt, aufgrund der die Frequenz ebenfalls verringert wird. Diesen Effekt bezeichnet man als transversalen Doppler-Effekt.

    Quelle:
    https://de.wikipedia.org/wiki/Doppler-Effekt#Transversaler_Doppler-Effekt

  147. #147 Toni
    14. Dezember 2018

    @Anonym_2018

    Bei Uhrenparadoxon gibt es diese völligen Gleichberechtigung der Systeme nicht mehr. Wenn es gäbe, würde dies das Relativitätsprinzip der SRT außer Kraft setzen. Bei Uhrenparadoxon ist die SYMMETRIE GEBROCHEN. Wenn sie gebrochen wird, wo in welchem Experiment auch immer, misst man DANN eine ECHTE Zeitdilatation.

    Auch Längenkontraktion ist eine Illusion, wenn man das Relativitätsprinzip der SRT zugrunde legt.

  148. #148 Toni
    14. Dezember 2018

    @Anonym_2018
    “umgangen durch Verwendung von drei Intertialsystemen, die paarweise gleichberechtigt sind, in denen sich jeweils eine Uhr befindet.”

    Die Uhr A befindet sich in einem Inertialsystem, die beiden andere sind gegenüber Uhr A beschleunigt. Somit ist die Symmetrie gebrochen.

  149. #149 Niels
    14. Dezember 2018

    @Toni

    In SRT gibt es keine reale Zeitdilatation

    Aber die Zeitdilatation, die durch die Relativgeschwindigkeit verursacht wird, ist so etwas wie eine Illusion

    SRT Effekte sind ALLE SCHEINEFFEKTE, weil sie beobachterabhängig sind. Es ist nur die Beschleunigung, die die Zeit verlangsamt.

    Versteh ich nicht.
    Die in der Atmosphäre entstehenden Myonen erreichen also doch nicht die Erdoberfläche?
    Was hab ich dann damals im Praktikum gemessen?

    Die Rechnung mit Beschleunigung ist nicht unmöglich. Sie ist ganz real.

    Genau?

    Du schriebst:

    Wenn der Beobachter sich selbst in einem beschleunigten System befindet, kann er in diesem System nicht die Gesetze der SRT anwenden, um zu rechnen, was bei anderen Beobachter passiert.

    Im Wiki-Artikel wird aber konkret ausformuliert, wie der Beobachter im beschleunigten System die Gesetze der SRT anwenden kann, um zu rechnen, was bei anderen Beobachtern passiert.
    Dazu benötigt er nämlich nur Messgrößen, die er in seinem eigenen Bezugssystem ermitteln kann.

    @Anonym_2018
    Oh, immer mal wieder was Neues?
    Früher bedeutete bei den scienceblogs ein Nichtauftauchen eines Kommentars ohne Meldung, dass der Spamfilter das Ganze für immer ins Nirvana verschwinden ließ.

  150. #150 Toni
    14. Dezember 2018

    @Niels:

    “Die in der Atmosphäre entstehenden Myonen erreichen also doch nicht die Erdoberfläche?”

    Die Myonen sind geladen und werden in der Atmosphäre beschleunigt. Auch hier ist die Symmetrie gebrochen.

    Wo man SRT anwenden kann und wo nicht steht ganz klar im Buch von Torsten Fließbach “Die Allgemeine Relativitätstheorie”. Die Formeln der SRT gelten für Inertialsysteme, d.h. mit SRT rechnet man aus einem Inertialsystem heraus für welch ein anderes System auch immer.

  151. #151 Toni
    14. Dezember 2018

    @Niels:
    “Im Wiki-Artikel wird aber konkret ausformuliert, wie der Beobachter im beschleunigten System die Gesetze der SRT anwenden kann, um zu rechnen, was bei anderen Beobachtern passiert.”

    Man nimmt an, dass sich der Beobachter auf der Erde in einem Inertialsystem befindet. Er ist also gar nicht beschleunigt. Auch hier ist die Symmetrie gebrochen.

  152. #152 Niels
    14. Dezember 2018

    @Toni

    “Die in der Atmosphäre entstehenden Myonen erreichen also doch nicht die Erdoberfläche?”

    Die Myonen sind geladen und werden in der Atmosphäre beschleunigt. Auch hier ist die Symmetrie gebrochen.

    Deiner Meinung nach zerfallen für uns Z-Bosonen, die sich für uns mit 0,999999c bewegen, also genau gleich schnell wie für uns ruhende Z-Bosonen?
    Weil sie ungeladen sind?

    Man nimmt an, dass sich der Beobachter auf der Erde in einem Inertialsystem befindet. Er ist also gar nicht beschleunigt.
    […]
    Die Formeln der SRT gelten für Inertialsysteme, d.h. mit SRT rechnet man aus einem Inertialsystem heraus für welch ein anderes System auch immer.

    Du bist also der Meinung, dass man, wenn man als die beiden Beobachter zwei beschleunigende Raumschiffe betrachtet, mit der allgemeinen Relativitätstheorie rechnen muss?
    Weil keiner der beiden Beobachter unbeschleunigt ist?

  153. #153 MartinB
    14. Dezember 2018

    @Niels
    Ich habe oben mal aufgeräumt, keine AHnung, was da wieder los war (aber wir haben seit einigen Tagen einen neuen Server, vielleicht hakt es da…)

    @Toni
    Langsam verstehe ich gar nicht mehr, was du sagen willst.

  154. #154 Anonym_2018
    14. Dezember 2018

    @Toni #148

    Die Uhr A befindet sich in einem Inertialsystem, die beiden andere sind gegenüber Uhr A beschleunigt. Somit ist die Symmetrie gebrochen.

    Wie kommst du darauf, dass die beiden anderen beschleunigt sind? Ich habe in #146 von drei Intertialsystemen geschrieben. Das sind grandlinig-gleichförmig bewegte Systeme. Eine von Null verschiedene messbare Zeitdilatation kann man mit zwei Lorentztransformationen (A-B und A-C) und Addition von Eigenzeit-Intervallen der Uhren B und C berechnen.

  155. #155 Anonym_2018
    14. Dezember 2018

    @Toni #148

    Bei Uhrenparadoxon ist die SYMMETRIE GEBROCHEN. Wenn sie gebrochen wird, wo in welchem Experiment auch immer, misst man DANN eine ECHTE Zeitdilatation.

    Das stimmt. Allerdings ist die Symmetrie in dem Uhrenparadoxon (=“Drei-Brüder-Ansatz”) nicht duch Beschleunigung gebrochen.

    Sie ist vielmehr dadurch gebrochen, dass eine Zeitdilatation nur gegenüber dem Bezugssystem A messbar ist. Die Sequenz der (zusammentreff-)Ereignisse erlaubt es aber nicht, die Zeitdilatationen gegenüber den Bezugssystemen der Uhren B oder C zu messen. In diesen Bezugssystemen können bei diesem Ablauf keine zwei notwendigen Uhrenvergleichen am selben Ort durchgeführt werden. Das kann auch garnicht anders sein, sonst würden logischen Widersprüche entstehen.

    Die messbare Zeitdilatation gegenüber A kommt aber nicht durch Beschleunigung zustande, sondern nur durch gradling-gleichförmige (relative) Geschwindigkeiten.

  156. #156 Anonym_2018
    14. Dezember 2018

    @Toni
    Bei dem transversalen Dopplereffekt aufgrund der Zeitdilatation ist die Symmetrie nicht gebrochen, und man kann trotzdem gegenseitig die SRT-Zeitdilatation messen.

    Beispiel:
    Ein Eisenbahnzug (ICE6) überholt mit der Geschwindigkeit 0,999 c auf gerader Strecke einen ICE5, der nur mit der Geschwindigkeit 0,5 c fährt. In beiden Zügen sitzen Personen, die mit jeweils einem Laserpointer transversal zur Zug-Geschwindigkeit nach draussen leuchten.
    Zu dem Zeitpunkt, zu dem beide Personen nebeneinander sind, sehen sie bei dem Laserpointer des jeweils anderen eine Rotverschiebung aufgrund der Zeitdilatation im Vergleich zum eigenen (gleichartigen) Laserpointer.

    Das ist in diesem Beispiel ohner Symmetriebrechung möglich, weil hier keine identischen Zeiträume verglichen werden. Das wäre auch bei nur einem Zusammentreffen aufgrund der Relativität der Gleichzeitigkeit nicht möglich.

  157. #157 Toni
    14. Dezember 2018

    @Niels :

    “Du bist also der Meinung, dass man, wenn man als die beiden Beobachter zwei beschleunigende Raumschiffe betrachtet, mit der allgemeinen Relativitätstheorie rechnen muss?
    Weil keiner der beiden Beobachter unbeschleunigt ist?”

    Die Formeln der SRT lassen sich nur bezüglich der Inertialsysteme auf egal welche andere Systeme (beschleunigt oder unbeschleunigt) anwenden.
    Wenn ich mich selbst in einem beschleunigten System befinde, ist das äquivalent mit, als wenn ich mich in Ruhe in einem Gravitationsfeld befinden würde. Das ist das ÄQUIVALENZPRINZIP.

    Wie gesagt, steht alles sauber im Buch von Torsten Fließbach. Falls du Physiker bist, musst du es kennen.

  158. #158 Toni
    14. Dezember 2018

    @Anonym_2018:

    “Wie kommst du darauf, dass die beiden anderen beschleunigt sind? Ich habe in #146 von drei Intertialsystemen geschrieben.”

    Weil die Uhr im Flugzeug sich um die Erde bewegt, und eine Kreisbewegung ausführt und somit eine beschleunigte Bewegung aus der Sicht der ruhenden Uhr A in seinem Inertialsystem darstellt. DIESE 3 SYSTEME SIND NICHT ÄQUIVALENT!

    “Das stimmt. Allerdings ist die Symmetrie in dem Uhrenparadoxon (=“Drei-Brüder-Ansatz”) nicht duch Beschleunigung gebrochen.”

    Ja schön, dass du das endlich akzeptiert hast (zumindest ein klener Schritt). Also wenn die völligen gleichberechtigung der Systeme gebrochen ist, dann misst man eine echte Zeitdilatation. Falls nicht, dann sind die SRT SCHEIN EFFEKTE.

    NOCHMAL Effekte, die nur von relativen Geschwindigkeit abhängen und beobachterabhängig sind keine echte Effekte.

  159. #159 Toni
    14. Dezember 2018

    @Niels, @Anonym_2018, @MartinB

    Ich empfehle euch drei einen Blick auf diesen Link zu werfen. Da wird ganz gut beschrieben, was man alles in SRT nicht ganz genau verstanden hat.

    https://www.quora.com/What-do-most-people-misunderstand-about-Einsteins-Theory-of-Relativity

    Sucht nach einem Viktor Toth.

  160. #160 Anonym_2018
    14. Dezember 2018

    Weil die Uhr im Flugzeug sich um die Erde bewegt, und eine Kreisbewegung ausführt

    Bei dem Uhrenparadoxon, das ich in #146 beschrieben habe, steht nichts von einer Erde oder von einen Flugzeug. Das spielt sich weit entfernt von jedem Himmelskörper ab (keine Gravitation, keine Beschleunigungen). Die Uhren fliegen einfach gradlinig-gleichförmig durchs Weltall. Man könnte sie sich auch im inneren von Raketen vorstellen, wenn das anschaulicher ist.

    NOCHMAL Effekte, die nur von relativen Geschwindigkeit abhängen und beobachterabhängig sind keine echte Effekte.

    Falsch. Die relativen Effekt beim genannten Uhrenparadoxon oder beim transversalen Dopplereffekt sind messbar und damit echt.

  161. #161 Toni
    14. Dezember 2018

    @Anonym_2018:
    “Bei dem Uhrenparadoxon, das ich in #146 beschrieben habe, steht nichts von einer Erde oder von einen Flugzeug. Das spielt sich weit entfernt von jedem Himmelskörper ab (keine Gravitation, keine Beschleunigungen). Die Uhren fliegen einfach gradlinig-gleichförmig durchs Weltall. Man könnte sie sich auch im inneren von Raketen vorstellen, wenn das anschaulicher ist.”

    Ich rede vom Experiment, der von Hafele und Keating 1971 mit Atomuhren gemacht wurde, mit Erde, Flugzeug, Bodenstation und alles drum und dran. Wenn es ECHTE Effekte geben sollte, die von der relativen Geschwindigkeit abhängen sollte, dann ist das Relativitätsprinzip der SRT verletzt!!!

    Wenn die Symmetrie gebrochen ist, dann ist das Relativitätsprinzip der SRT außer Kraft gesetzt worden, und somit die Zeitdilatation, die man misst, hat nichts mehr mit SRT zu tun. Das ist analog zu Zwillingsparadoxon.

    Wenn du weiß, was genau das paradoxe im Zwillingsparadox ist, dann wirst du es verstehen, was ich oben geschrieben habe.

  162. #162 Anonym_2018
    14. Dezember 2018

    Wenn die Symmetrie gebrochen ist, dann ist das Relativitätsprinzip der SRT außer Kraft gesetzt worden

    Nein. Bei dem Uhrenparadoxon im Sinne von Kommentar #146 (=Version des “Drei-Brüder-Ansatz”, nur ohne Erde), besteht zwar Symmetrie zwischen jedem Paar der drei Inertialsysteme, aber eine Asymmetrie in dem Sinne, dass die Eigenzeit eines Inertialsystems mit der Summe der Eigenzeiten von zwei anderen Inertialsystemen verglichen wird. Das Relativitätsprinzip wird nicht ausser Kraft gesetzt.

  163. #163 Anonym_2018
    14. Dezember 2018

    Ich rede vom Experiment, der von Hafele und Keating 1971

    Der dortige Beitrag zur Zeitdilatation aus dem Sagnac-Effekt ist z.B. ein SRT-Effekt.

  164. #164 Toni
    14. Dezember 2018

    @Anonym_2018:

    Wie gesagt, ich rede vom vom Experiment von Hafele und Keating 1971. Das ist auch beschrieben im Buch von Roman Sexl Raum-Zeit-Relativität Seite 40-42.

    “dass die Eigenzeit eines Inertialsystems mit der Summe der Eigenzeiten von zwei anderen Inertialsystemen verglichen wird”

    “Diese Variante mit drei Personen demonstriert, dass nicht die Dauer der Beschleunigung das Zwillingsparadoxon auflöst (denn diese kann im Vergleich zur inertialen Flugzeit beliebig klein gemacht werden), sondern der Umstand, dass das Geschehen während der Hin- und Rückreise in unterschiedlichen Inertialsystemen stattfindet,”

    Wenn es ein Wechsel der Inertialsysteme gibt, dann ist eben die Symmetrie gebrochen. Mein Punkt ist eben, dass diese 3 Inertialsysteme nicht äquivalent sind, weil es ein Wechsel der Inertialsysteme stattfindet.

    Du kannst nicht ohne diese Symmetriebrechung eine echte Zeitdilatation messen. Das ist der Punkt.

  165. #165 MartinB
    14. Dezember 2018

    @Toni
    Nichts von dem, was da steht, widerspricht in irgendeiner Weise, die ich sehen kann, dem, was Niels oder Anonym_2018 der ich selbst hier geschrieben haben.

  166. #166 Toni
    14. Dezember 2018

    @MartinB:

    Ihr wollt es nicht wahrhaben, dass es diese Zeitdilatation beim Zwillingsparadoxon gibt, weil die Symmetrie gebrochen ist, durch was auch immer; ich denke sie ist durch Beschleunigung gebrochen, ihr könnt gerne anderer Meinung sein, ist okay.

    Man misst einen Effekt, weil die völligen Gleichberechtigung der Systeme gebrochen wird, und somit das Relativitätsprinzip der SRT.

    Aus Einsteins Arbeit von 1918 geht klar und unmissverständlich hervor, dass er das
    Phänomen der Zeitdilatation nicht unter Zugriff auf die SRT, sondern ausschließlich mit dem Formalismus der ART erklärt.

    Diesen Wechsel der Inertialsysteme (wie in deinem Blog beschrieben) erachte ich als äquivalent zu Beschleunigung; du kannst gerne anderer Meinung sein.

  167. #167 MartinB
    14. Dezember 2018

    @Toni
    Ich habe nach wie vor den Eindruck, dass du den Begriff “Zeitdilatation” in etwas unüblicher Weise verwendest, wenn du sagst, dass z.B der Effekt bei relativ zueinander bewegten Inertialsystemen keine “echte” Zeitdilatation sei. (Siehe Niels Myonen-Beispiel, und nein, die werden nicht nennenswert in der Atmosphäre gebremst, der Effekt hat damit nichts zu tun.)
    Dass das zwillingsparadoxon vollständig innerhalb der SRT aufgelöst werden kann, ist aber hoffentlich unbestritten.

    “Diesen Wechsel der Inertialsysteme (wie in deinem Blog beschrieben) erachte ich als äquivalent zu Beschleunigung”
    Damit stehst du ja nicht allein – wurde ja oben ausführlich diskutiert. Kann man so sehen – je nachdem, wie genau man Beschleunigung (und in deinem Satz das Wort ‘äquivalent’) auffasst

  168. #168 Toni
    14. Dezember 2018

    @MartinB:

    “Ich habe nach wie vor den Eindruck, dass du den Begriff “Zeitdilatation” in etwas unüblicher Weise verwendest, wenn du sagst, dass z.B der Effekt bei relativ zueinander bewegten Inertialsystemen keine “echte” Zeitdilatation sei.”

    Fakt ist, dass ohne Symmetriebrechung (wie auch immer) keine echte messbare Zeitdilatation möglich ist.

    “Damit stehst du ja nicht allein – wurde ja oben ausführlich diskutiert. Kann man so sehen – je nachdem, wie genau man Beschleunigung (und in deinem Satz das Wort ‘äquivalent’) auffasst”

    Okay, dann sind wir gleicher Meinung. Natürlich kann das Zwillingsparadoxon ohne Beschleunigung innerhalb der SRT aufgelöst werden. Wie schon aber oben gesagt, sehe ich diesen Wechsel der Inertialsysteme beim 3-Brüder Ansatz als Beschleunigung, weil es eine Richtungsänderung stattfindet. Dann ist alles gut.

  169. #169 Anonym_2018
    14. Dezember 2018

    Die Symmetrieproblematik wird auf der 2. Seite dieses Artikels sehr gut aufgelöst:

    Entscheidend ist aber – und das macht dieses Diagramm sehr schön deutlich -, dass es einen weiten Bereich von Ereignissen bei Teresa gibt, die für Serena mit gar nichts gleichzeitig sind. Und das hat nichts mit irgendwelchen Beschleunigungen zu tun, sondern nur damit, dass Serena ihr Bezugssystem wechselt.

  170. #170 Toni
    14. Dezember 2018

    @Anonym_2018:

    “Und das hat nichts mit irgendwelchen Beschleunigungen zu tun, sondern nur damit, dass Serena ihr Bezugssystem wechselt.”

    Wie oben erwähnt, aus meiner Sicht, sobald man sein Inertiasystem wechselt (wie auch immer), hat man die völlige glechberechtigung der Inertialsysteme verletzt. Ich verbinde diesen Wechsel oder Richtungsänderung der Inertialsysteme bei 3-Brüdern-Ansatz mit Beschleunigung.

  171. #171 Anonym_2018
    14. Dezember 2018

    Fakt ist, dass ohne Symmetriebrechung (wie auch immer) keine echte messbare Zeitdilatation möglich ist.

    Dem stimme ich bzgl. des diskutierten Zwillingsparadoxons und Varianten davon zu. Das kann man aber nicht verallgemeinern. Das trifft z.B. nicht beim transversalen Dopplereffekt zu. Dort ist die SRT-Zeitdillatation auch ohne Symmetriebrechung messbar.

  172. #172 Toni
    14. Dezember 2018

    @Anonym_2018:

    “Dort ist die SRT-Zeitdillatation auch ohne Symmetriebrechung messbar.”

    Auch hier muss man eine Symmetriebrechung finden. Ansonsten wird das Relativitätsprinzip der SRT verletzt.

  173. #173 MartinB
    14. Dezember 2018

    @Toni
    “Fakt ist, dass ohne Symmetriebrechung (wie auch immer) keine echte messbare Zeitdilatation möglich ist.”
    Ich habe keine Ahnung, wie du das genau meinst. Dass auf der Erdoberfläche Myonen ankommen oder dass Teilchen in nem Linearbeschleuniger mehr Strecke zurücklegen können als ohne Zeitdilatation, ist doch auch deiner Ansicht nach korrekt, oder?

  174. #174 Toni
    14. Dezember 2018

    @MartinB

    “Ich habe keine Ahnung, wie du das genau meinst. Dass auf der Erdoberfläche Myonen ankommen oder dass Teilchen in nem Linearbeschleuniger mehr Strecke zurücklegen können als ohne Zeitdilatation, ist doch auch deiner Ansicht nach korrekt, oder?”

    Ich betrachte den Beobachter auf der Erde in seinem Inertialsystem und Myonen als nicht gleichwertige Systeme! Durch diese Symmetriebrechung der Gleichwertigkeit kommt die messbare echte Zeitdilatation zustande.

  175. #175 Niels
    14. Dezember 2018

    @Toni

    Könntest du noch einmal konkreter auf die Fragen in #152 eingehen?

    Ich glaube, daran kann man am besten unsere unterschiedlichen Auffassungen herausarbeiten.
    .

    1) Deiner Meinung nach zerfallen für uns Z-Bosonen, die sich für uns mit 0,999999c bewegen, also genau gleich schnell wie für uns ruhende Z-Bosonen?
    Weil sie ungeladen sind?

    Oder anders:
    Wenn man die atmosphärischen Myonen durch ungeladene, aber sonst identische Teilchen ersetzten würde (selben Masse, selbe Geschwindigkeit, selbe Halbwertszeit,…), was verändert sich dann?
    .

    2)

    Wenn ich mich selbst in einem beschleunigten System befinde, ist das äquivalent mit, als wenn ich mich in Ruhe in einem Gravitationsfeld befinden würde. Das ist das ÄQUIVALENZPRINZIP.

    Du bist also tatsächlich der Meinung, dass man für beschleunigte Systeme immer die ART verwenden muss?
    Auch wenn der Raum flach ist?

    Konkret:
    Wenn man zwei beschleunigende Raumschiffe betrachtet, muss man zwangsläufig die ART verwenden?

  176. #176 Anonym_2018
    14. Dezember 2018

    @Toni #172

    Auch hier muss man eine Symmetriebrechung finden.

    Hast du schon eine gefunden?

    Beschreibung eines Experiments:

    Langsam bewegte Uhren

    Inzwischen ist es gelungen, die Zeitdilatation optischer Atomuhren auch bei alltäglichen Geschwindigkeiten nachzuweisen. Chou et al. (2010) benutzten dafür Aluminiumionen, die in einem 75 m langen, phasenstabilisierten Lichtwellenleiter hin- und her bewegt wurden, und Signale einer bestimmten Frequenz übermittelten, wobei die Genauigkeit dieser Uhren ∼10⁻¹⁷ betrug. Dadurch konnte die bei Geschwindigkeiten von unter 36 km/h (kleiner 10 m/s) auftretende Verschiebung von ∼10⁻¹⁶ gemäß der relativistischen Zeitdilatation, durch Vergleich der Frequenz von bewegten und ruhenden Ionen gemessen werden.[18]

  177. #177 MartinB
    14. Dezember 2018

    @Toni
    “Ich betrachte den Beobachter auf der Erde in seinem Inertialsystem und Myonen als nicht gleichwertige Systeme! Durch diese Symmetriebrechung der Gleichwertigkeit kommt die messbare echte Zeitdilatation zustande.”
    Und wie begründest du das?
    Ich kann ja meinen Myonendetektor und das Target auch starr im Weltall montieren, wenn dich die Erde stört – das wird nichts ändern. Dann haben wir zwei Inertialsysteme – das (nach seiner Entstehung) mit konstanter Geschwindigkeit fliegende Myon und Target+Detektor, die relativ zueinander ruhen. Wo/wie ist da die Symmetrie gebrochen (was immer du nun wieder in diesem Zusammenhang darunter genau verstehst)?

  178. #178 Toni
    14. Dezember 2018

    @MartinB:

    Das Myon Experiment ist analog zu Zwillingsparadoxon. Ich betrachte die Myonen als beschleunigt gegenüber dem Detektor.

  179. #179 Toni
    14. Dezember 2018

    “Chou et al. (2010) benutzten dafür Aluminiumionen, die in einem 75 m langen, phasenstabilisierten Lichtwellenleiter hin- und her bewegt wurden”

    Hin und her, d.h. Wechsel des Bezugssystems, und somit Symmetriebrechung.

  180. #180 MartinB
    14. Dezember 2018

    @Toni
    Ein Myon passiert detektor 1, fliegt von da aus mit konstanter Geschwindigkeit weiter und passiert dann Detektor 2, den es an Hand seiner Lebensdauer eigentlich nicht hätte erreichen dürfen. Wo ist da die Beschleunigung?

  181. #181 Toni
    14. Dezember 2018

    @Niels:

    “1) Deiner Meinung nach zerfallen für uns Z-Bosonen, die sich für uns mit 0,999999c bewegen, also genau gleich schnell wie für uns ruhende Z-Bosonen?
    Weil sie ungeladen sind?

    Oder anders:
    Wenn man die atmosphärischen Myonen durch ungeladene, aber sonst identische Teilchen ersetzten würde (selben Masse, selbe Geschwindigkeit, selbe Halbwertszeit,…), was verändert sich dann?”

    Egal welche Situation man auch immer betrachtet, eine echte messbare Zeitdilatation misst, wenn man davon ausgeht, dass die 2 Systeme nicht völlig gleichwertig sind. Das ist mein Standpunkt.

    “Du bist also tatsächlich der Meinung, dass man für beschleunigte Systeme immer die ART verwenden muss? Auch wenn der Raum flach ist?”

    Ich kann rechnen, wie die Zeit für ein beschleunigtes System zu verlaufen hat, wenn ich von einem Inertialsystem (nicht-beshleunigtes System) ausgehe. Die RaumZEIT ist immer lokal flach.

    “Wenn man zwei beschleunigende Raumschiffe betrachtet, muss man zwangsläufig die ART verwenden?”

    Das hat zumindest Einstein (1918) gemacht. Die Kernaussage ist die, dass durch eine Symmetriebrechung im Zwillingsparadoxon der Altersunterschied zustand kommt, und somit das Relativitätsprinzip der SRT verletzt sein muss.

  182. #182 Niels
    14. Dezember 2018

    @Toni
    Hast du #175 gesehen?
    Nach Abschicken hat es bei mir über zwanzig Minuten gedauert, bis der Kommentar zu sehen war.

    Ich hab momentan dieses Wochenende nur wenig Zeit, deswegen werfe ich einfach mal zwei Links zu 2), Beschleunigung in der SRT, in den Raum:
    .

    Can Special Relativity Handle Acceleration?


    Acceleration (special relativity)

    .

    Zu 1), Zeitdilatation konnte ich auf die schnelle nichts Passendes finden.
    An Teilchenbeschleunigern wird naturgemäß eher nicht mit schnell bewegten, ungeladenen Teilchen hantiert.

    (@MartinB @Anonym_2018
    Kennt ihr ein Experiment, in dem die Halbwertszeit von ungeladenen, schnell bewegten Teilchen gemessen wurde?
    Also ein Experiment, das konkret zeigt, dass es bei der Zeitdilatation nicht auf die Ladung ankommt?)

    Es müsste aber eigentlich schon auffallen, dass in Rechnungen für die Lebensdauer immer nur simpelste Lornetz-Trafos verwendet werden.
    Da geht nirgendwo Ladung oder Beschleunigung in die Formel ein.

    Anonym_2018 hatte in #133 allerdings schon auf Bailey et al. (1977) hingeweisen.
    Bailey et al. (1977) bestätigten auch die sogenannte „Uhrenhypothese“, wonach die Beschleunigung die Zeitdilatation nicht beeinflusst.

    Auch dazu stelle ich einfach mal unkommentiert einen Link ein:

    Does a clock’s acceleration affect its timing rate?

    (Meines Wissens war das in der ganzen Geschichte der SRT auch niemals umstritten.)

  183. #183 Toni
    14. Dezember 2018

    @MartinB:

    “Wo ist da die Beschleunigung?”

    Dort wo sie auf nahezu Lichtgeschwindigkeit beschleunigt werden, selbst wenn diese Phase kurz sein sollte, auch hier findet ein Wechsel des Bezugssystems statt. Meine Aussage ist allgemein, und bezieht sich nicht auf ein bestimmtes Experiment. Ohne Symmetriebrechung keine echte Zeitdilatation.

  184. #184 Anonym_2018
    14. Dezember 2018

    @Toni #179

    Hin und her, d.h. Wechsel des Bezugssystems, und somit Symmetriebrechung.

    Das Hin und her spielt in diesem Experiment (siehe Fig. 2) keine Rolle, da die Strahung und ihre Frequenz senkrecht zur Bewegung der aus einem Atom bestehenden Aluminium-Ionen-Uhr ausgewertet werden und zwar genau in der Mitte zwischen den Umkehrpunkten, also bei nahezu konstanter Geschwindigkeit.

  185. #185 MartinB
    14. Dezember 2018

    @Toni #183
    Du hast den Aufbau nicht verstanden:
    Ich erzeuge teilchen am Ort A.
    Die Teilchen fliegen von dort aus geradlinig mit v=const=0.99…c an zwei detektoren B und C vorbei. Der Abstand zwischen B und C ist so groß, dass die Teilchen eigentlich zerfallen sollten.
    Wo ist in diesem Aufbau bei der Strecke von B nach C irgendeine Beschleunigung oder eine “Symmetriebrechung”?

    @Niels
    Ich denke, beim Zerfall von neutralen Pionen oder kaonen würde man das sicher bemerkt haben, wenn die im Detektor nie so weit kommen wie geladene Teilchen. Ne Quelle dafür habe ich nicht, aber ich bin ziemlich sicher, dass man das aus den unzähligen Beschleunigerexperimenten herauskitzeln könnte.

  186. #186 Toni
    14. Dezember 2018

    Trotzdem findet hier beim Experiment ein Wechsel des Bezugssystems statt.

    “also bei nahezu konstanter Geschwindigkeit.”

    Das allein reicht, um die Symmetrie zu brechen (ich rede ja auch immer von VÖLLIG gleichberechtigte Systeme, und nicht nahezu). Selbst wenn die Symmetrie sehr sehr leicht gebrochen ist, ist sie gebrochen. Je stärker sie gebrochen ist, desto größer der Zeitunterschied.

    Die Aussage ist allgemeiner Natur. Ohne Symmetriebrechung keine Zeitdilatation.

    Du sagst man kann diese Aussage nicht verallgemeinern. Dann schicke mir bitte Links zu diesem Dopplereffekt (bitte nicht die Wikipedia), damit ich die Symmetriebrechung finde.

  187. #187 Anonym_2018
    14. Dezember 2018

    @Toni #186

    Du sagst man kann diese Aussage nicht verallgemeinern. Dann schicke mir bitte Links zu diesem Dopplereffekt (bitte nicht die Wikipedia), damit ich die Symmetriebrechung finde.

    Schon geschehen, siehe Kommentar #184.

  188. #188 Toni
    14. Dezember 2018

    @MartinB:

    Die Teilchen müssen schon voher auf nahezu Lichtgeschwindigkeit beschleunigt werden. Das ist auch beim Zwillingsparadoxon so, Teilabschnitte der Weltlinie, in denen sich Serena mit konstanter Geschwindigkeit bewegt, hat sie keinen echten Zeitgewinn gegenüber ihrer Schwester.

    Wenn es keine Symmetriebrechung gibt, dann das was man gemessen hat, verstößt gegen das Relativitätsprinzip. Das Teilchensystem und Beobachter können beim Experiment nicht absolut gleichwertig sein. Sie müssen leicht ungleichwertig sein, um das Experiment erklären zu können.

  189. #189 Anonym_2018
    14. Dezember 2018

    @Toni #186
    Ergänzung
    Hier ist noch ein Link zu dem Experiment:
    https://science.sciencemag.org/content/329/5999/1630

    Ich kann den vollen Text allerdings nicht heunterladen, da man sich dort einloggen muss.

  190. #190 Toni
    14. Dezember 2018

    @Anonym_2018:

    https://ws680.nist.gov/publication/get_pdf.cfm?pub_id=905055

    “between the two probe directions, which corresponds
    to the ion moving at a speed of (1.8 plus/minus 1.1) nm/s in the lab frame.”

    Also keine konstante Geschwindigkeit und wie oben erwähnt keine VÖLLIGE Gleichberechtigung der Systeme. Also wie beim Zwillingsparadoxon und 3-Brüder Ansatz.

  191. #191 Anonym_2018
    14. Dezember 2018

    @Niels #182

    @… Anonym_2018
    Kennt ihr ein Experiment, in dem die Halbwertszeit von ungeladenen, schnell bewegten Teilchen gemessen wurde?

    Sorry, ich kenne keins.

  192. #192 MartinB
    14. Dezember 2018

    @Toni #188
    Dass die Myonen irgendwann vor Beginn der Messung beschleunigt werden, schafft doch keine Asymmetrie. Zwischen den Eriegnissen Messung an Detektor B und Messung an Detektor C haben si konstante Geschwindigkeit, das ist alles, was zählt. Anonsten müsste man in der SRT ja immer die gesamte lebensgeshcichte eines Objekts kennen, um Effekte berücksichtigen zu können.
    Und im Übrigen ist es natürlich völlig egal, ob das Myon beschleunigt wird oder nicht. Wenn das Myon als ruhendes Teilchen entsteht, aber die detektoren mit nahezu c dran vorbeifliegen, gibt es die Dilatation ja genauso (heißt ja schließlich Relativitätstheorie). Du kannst doch nicht ernsthaft der Ansicht sein, dass die Tatsache, dass die detektoren vielleicht mal vor 100000 Jahren beschleunigt wurden, einen Einfluss auf die Messung heute hat?

  193. #193 Anonym_2018
    14. Dezember 2018

    @Toni #190

    Also keine konstante Geschwindigkeit und wie oben erwähnt keine VÖLLIGE Gleichberechtigung der Systeme.

    Du hast den folgenden Satz weggelassen, dass dieser Fehler im Experiment kompensiert wird:

    However, the clock rate is not significantly affected by a velocity of this magnitude, because it is derived from an average of the two opposite laser-probe directions.

  194. #194 MartinB
    14. Dezember 2018

    PS:
    du kannst das, was passiert, doch berechnen oder mit Minkowksi-Diagrammen untersuchen. Da geht nirgends die beschleunigung des Teilchens oder der Detektoren ein. Sorry, aber ich fürchte, du hast die SRT falsch verstanden.

  195. #195 Toni
    14. Dezember 2018

    @MartinB:

    Ich denke ihr achtet nicht auf den Unterschied zwischen völlige und nahezu gleichberechtigte Systeme.

    “aber ich fürchte, du hast die SRT falsch verstanden.”
    Dann müsste ja Einstein auch selbst SRT falsch verstanden haben, denn erklärt das in seinem Paper von 1918 mit Beschleunigung.

  196. #196 Toni
    14. Dezember 2018

    “du kannst das, was passiert, doch berechnen oder mit Minkowksi-Diagrammen untersuchen.”

    Eine Richtungsänderung ist eine Beschleunigung, denn ohne sie gibt es auch keine Richtungsänderung. Siehe Satelliten um die Erde, sie bewegen sich auch mit konstanter Geschwindigkeit, sind aber beschleunigt aus der Sicht eines Beobachters auf der Erde.

  197. #197 MartinB
    14. Dezember 2018

    @Toni

    “Ich denke ihr achtet nicht auf den Unterschied zwischen völlige und nahezu gleichberechtigte Systeme.”
    Erklär mal an dem eben beschriebenen Aufbau (oder an einem Minkowski-Diagramm), wie du dir das denkst.

    Gib mal ne genaue Referenz auf das Einstein-Zitat, das du meinst – ich vermute, dass du das zitat falsch verstehst (das passiert bei Einstein-Zitaten des öfteren, mal davon abgesehen, dass Einstein auch manchmal Dinge missverständlich geschrieben hat).

  198. #198 Toni
    14. Dezember 2018

    @Anonym_2018:

    “Du hast den folgenden Satz weggelassen, dass dieser Fehler im Experiment kompensiert wird:”

    Man kann Richtungsänderung und somit Wechsel des Inertialsystems nicht kompensieren.

  199. #199 Anonym_2018
    14. Dezember 2018

    @Toni #190
    Ergänzung

    Also keine konstante Geschwindigkeit und wie oben erwähnt keine VÖLLIGE Gleichberechtigung der Systeme. Also wie beim Zwillingsparadoxon und 3-Brüder Ansatz.

    Um deine Aussage zu belegen, dass grundsätzlich eine Symmetriebrechung notwendig ist, um die SRT-Zeitdilatation messen zu können, reicht es nicht, Ungenauigkeiten in den Experimenten zu finden, die es immer geben wird.
    Bei dem Zwillingsparadoxon geht es um eine grundsätzliche Symmetriebrechung. Daher müsstest du z.B. bei dem transversalen Dopplereffekt eine grundsätzliche Symmetriebrechung finden, die auch unter Idealabedingungen unvermeidbar ist, um die SRT-Zeitdilatation zu messen.

  200. #200 Toni
    14. Dezember 2018

    @MartinB #192
    Die Uhren von Teilchen und Detektor können doch nicht in der Realität beide langsamer sein als die andere. Ich betrachte die Beschleunigung auf nahezu Lichtgeschwindigkeit als wichtig.

    Die Geschichte mit Myonen sehe ich analog zu Zwillingsparadoxon. Beschleunigung beim Umkehren. Genau da gibt es einen echten Zeitgewinn für Serena. Beschleunigung macht das ganze beobachterUNabhängig, während SRT Effekte beobachterabhängig sind.

    Wenn SRT Effekte echt sind, warum sind sie dann beobachterabhängig?

  201. #201 Toni
    14. Dezember 2018

    “Bei dem Zwillingsparadoxon geht es um eine grundsätzliche Symmetriebrechung. Daher müsstest du z.B. bei dem transversalen Dopplereffekt eine grundsätzliche Symmetriebrechung finden, die auch unter Idealabedingungen unvermeidbar ist, um die SRT-Zeitdilatation zu messen.”

    Habe ich dir doch 1000 mal eine grundsätzliche Symmetriebrechung gegeben, nähmlich Richtungswechsel im Experiment, das du mir geschickt hast.

  202. #202 Anonym_2018
    14. Dezember 2018

    @Toni #201
    Vielen Dank für die Suche!

    Mich stört dieser Fehler von ca. 2 Nanometer pro Sekunde jedenfalls nicht. Ich halte das Experiment für einen Beweis, dass die SRT-Zeitdilatation unabhängig von Beschleunigungen oder Wechseln von Inertialsystemen messbar ist.

  203. #203 MartinB
    14. Dezember 2018

    @Toni
    “Die Uhren von Teilchen und Detektor können doch nicht in der Realität beide langsamer sein als die andere. ”
    Doch natürlich. Jeder Beobachter sieht die Uhr des anderen als langsamer. Es entsteht kein Widerspruch, weil es zwei unterschiedliche Uhren bei den beiden Detektoren sind. Du solltest dir wirklich mal ein Minkwoski-Diagramm des ganzen angucken, dann siehst du, wie es geht.

    “Wenn SRT Effekte echt sind, warum sind sie dann beobachterabhängig?”
    Solche Pauschalaussagen wie “Effekte sind beobachterabhängig” sind immer wenig hilfreich.
    Beobachterabhängig sind dinge wie der räumliche oder der zeitliche Abstand zweier Ereignisse, das ist richtig.
    Und was bedeutet “echt”? Beobachter beobachten Dinge, die lassen sich konsistent beschreiben – von mir aus gesehen geht deine Uhr langsamer, wenn du dich bewegst, von dir aus gesehen meine. Dass das direkt messbare Konsequenzen haben kann, sieht man z.B. an den Myonen – aber es braucht wirklich keine Beschleunigungen, um das zu sehen.

  204. #204 Niels
    14. Dezember 2018

    @Toni

    Wenn SRT Effekte echt sind, warum sind sie dann beobachterabhängig?

    Die Feinstruktur des Wasserstoffspektrums, die gelbliche Farbe von Gold, der Aggregatzustand von Quecksilber bei Standardbedingungen (flüssig), die Bandstrukturen von Halbleitern und allgemeiner und prinzipieller alle Vorhersagen der Dirac-Gleichung oder direkt der relativistischen Quantenfeldtheorien sind also gar kein echten Effekte?
    Oder zumindest keine SRT Effekte? (Was denn sonst?)

    So wie Myonen für dich offenbar nur scheinbar die Erdoberfläche erreichen?
    Bzw. das nur für bestimmte Beobachter richtig ist?
    Oder inwiefern ist dieser Effekt sonst Beobachter abhängig?
    .

    [@MartinB
    Dazu hattest du vor ewigen Zeiten mal einen ausführlichen Artikel, oder? Könntest du ja mal verlinken, mir fällt nur leider gerade der Name nicht mehr ein.]

    .

    Ich glaube, es wäe hilfreich, wenn du mal definieren, was für dich ein “SRT Effekt” ist.
    Am besten auch noch gleich, was du unter einem “echten” Effekt versteht.

    Oder hängt für dich alles oben genannte irgendwie immer mit Beschleunigungen zusammen?
    Wodurch es dann zwangsläufig “ART-Effekte” werden und keine “SRT-Effekte” mehr sein können?

    Ich verstehe leider immer weniger, worauf du eigentlich hinaus willst?
    (Hast du dir eigentlich die Links in #182 mal angeschaut?)

  205. #205 Toni
    14. Dezember 2018

    Anonym_2018:

    “die SRT-Zeitdilatation unabhängig von Beschleunigungen oder Wechseln von Inertialsystemen messbar ist.”

    Wenn es so sein sollte, dann ist die Zeit der jeweiligen Beobachter langsamer als der andere Beobachter. Das führt zu Inkonsistenz.

    Zeit und Längenkontraktion sind beide eng miteinander verknüpft. Du kannst im Buch von Roman Sexl Raum-Zeit-Relativität nachlesen, dass die Längenkontraktion gar unsichtbar ist. Ich empfehle dir echt das Buch zu lesen, bevor andere vorzuwerden SRT nicht verstanden zu haben.

  206. #206 Toni
    14. Dezember 2018

    @Niels:

    Lese meine Kommentare, dass weißt du worauf ich hinaus will.

    Die myonen erreichen die Erdoberfläche, weil sie sich nicht in einem gleichwertigen System wie ein Beobachter auf der Erde in seinem Intertialsystem (ohne Gravitationspotental) befinden. Das habe ich schon gefühlt 1000 mal gesagt. Die Symmetrie muss gebrochen sein, um eine echt Zeitdilatation zu messen, weil die Zeit der jeweiligen Beobachter langsamer als der andere Beobachter sein kann. Das führt zu Inkonsistenz. Ich stelle ja das Myon-Experiment selbst nicht in Frage. Bitte genau lesen!

    Zeit und Längenkontraktion sind beide eng miteinander verknüpft. Du kannst im Buch von Roman Sexl Raum-Zeit-Relativität nachlesen, dass die Längenkontraktion gar unsichtbar ist. Ich empfehle dir echt das Buch zu lesen, bevor andere vorzuwerden SRT nicht verstanden zu haben.

    PS: die hoch gelobte QFT kann ja Feinstruktur nicht herleiten wie die Elektronmasse und Ladung. Über die Singularitäten möchte ich gar nicht sprechen.

  207. #207 Toni
    14. Dezember 2018

    MartinB:

    “Und was bedeutet “echt”? Beobachter beobachten Dinge, die lassen sich konsistent beschreiben – von mir aus gesehen geht deine Uhr langsamer, wenn du dich bewegst, von dir aus gesehen meine. Dass das direkt messbare Konsequenzen haben kann, sieht man z.B. an den Myonen – aber es braucht wirklich keine Beschleunigungen, um das zu sehen.”

    Das ist doch genau das Paradoxe im Zwillingsparadoxon. Entweder ist deine Uhr in WIRKLICHKEIT langsamer oder meine, aber nicht beide, wenn man sie vergleicht. Darauf ruht der scheinbare Widerspruch im Zwillingsparadoxon. Das ist genau, das was ich meine mit Schein Effekte. Um eben das Experiment erklären zu können, muss man dann die Gleichwertigkeit der Systeme brechen.

  208. #208 Toni
    14. Dezember 2018

    Niels:

    Korrektur:
    “weil die Zeit der jeweiligen Beobachter langsamer als der andere Beobachter sein kann”

    weil die Zeit der jeweiligen Beobachter NICHT langsamer als der andere Beobachter sein kann, wenn man sie verleicht (Zwillingsparadoxon).

  209. #209 Anonym_2018
    14. Dezember 2018

    @Toni #205

    Wenn es so sein sollte, dann ist die Zeit der jeweiligen Beobachter langsamer als der andere Beobachter. Das führt zu Inkonsistenz.

    Das führt nicht zu Inkonsistenzen, weil die Gleichzeitigkeit auch relativ ist. In dem Experiment mit dem transversalen Dopplereffekt werden ja nur zu bestimmten Zeitpunkten die Ganggeschwindigkeiten von Uhren verglichen, nicht die zeitlichen Abstände zweier konkreter Ereignisse wie beim Zwillingsparadoxon.

  210. #210 Toni
    14. Dezember 2018

    ok, damit ich mir das Ergebniss ALLER Experimente unter einem Hut bringen kann, muss ich es dann so formulieren:

    Wo die Symmetrie der Gleichwertigkeit der Systeme gebrochen wird, hat man eine Zeitdilatation wegen Beschleunigung (ART), und wo nicht, hat man eine Zeitdilatation wegen SRT. Damit kann ich dann leben.

  211. #211 Niels
    14. Dezember 2018

    Wo die Symmetrie der Gleichwertigkeit der Systeme gebrochen wird, hat man eine Zeitdilatation wegen Beschleunigung (ART), und wo nicht, hat man eine Zeitdilatation wegen SRT. Damit kann ich dann leben.

    Das ist nur leider schlicht und ergreifend völliger Quatsch.

  212. #212 Toni
    14. Dezember 2018

    @Niels:

    “Das ist nur leider schlicht und ergreifend völliger Quatsch.”

    In wie fern Herr Professor?

  213. #213 Anonym_2018
    14. Dezember 2018

    @Toni #166

    Aus Einsteins Arbeit von 1918 geht klar und unmissverständlich hervor, dass er das Phänomen der Zeitdilatation nicht unter Zugriff auf die SRT, sondern ausschließlich mit dem Formalismus der ART erklärt.

    Aufgrund der Quellenangabe “7.” im Wikipedia-Artikel über das Zwillingsparadoxon

    1918 beschrieb Albert Einstein das Paradoxon auch mit Hilfe der Allgemeinen Relativitätstheorie. Siehe A. Einstein: Dialog über Einwände gegen die Relativitätstheorie. In: Die Naturwissenschaften. Heft 48, S. 697–702 (1918)”

    … vermute ich, dass du folgende Einstein-Veröffentlichung meinst:

    Einstein: Dialog über Einwände gegen die Relativitätstheorie.

  214. #214 Toni
    14. Dezember 2018

    @Anonym_2018:

    “vermute ich, dass du folgende Einstein-Veröffentlichung meinst:”
    Ja, so ist das. Aber wie ich oben geschrieben habe:

    Wo die Symmetrie der Gleichwertigkeit der Systeme gebrochen wird, hat man eine echte Zeitdilatation wegen Beschleunigung (ART), und wo nicht, hat man eine echt Zeitdilatation wegen SRT. Somit hat es sich für mich erledigt. So kann man dann alle Experimente vernünftig erklären.

  215. #215 Anonym_2018
    15. Dezember 2018

    @Toni #214

    Wo die Symmetrie der Gleichwertigkeit der Systeme gebrochen wird, hat man eine echte Zeitdilatation wegen Beschleunigung (ART), …

    Dieser Teilsatz ist flasch. In dem Paper von Einstein steht nichts von einer Zeitdilatation wegen Beschleunigung.

    Einstein beschreibt in dem Paper das Zwillingsparadoxon qualitativ aus zwei Perspektiven:

    1) Zuerst beschreibt Einstein das Zwillingsparadoxon aus der Perspektive des Bezugssystems von Teresa. Die Geschwindigkeit von Serena verlangsamt deren Uhr (SRT). Einstein schreibt nichts dazu, ob ihre Beschleunigung am Umkehrpunkt einen direkten Beitrag zur Zeitdilatation liefert. (Laut dem Artikel von MartinB ist der direkte Beitrag der Beschleunigung jedenfalls Null.)

    2) Dann beschreibt Einstein das Zwillingsparadoxon aus der Perspektive des Bezugssystems von Serena. Die relative Geschwindigkeit von Teresa verlangsamt zwar deren Uhr, ihr Gravitationspotential am Umkehrpunkt hat aber, in entgegengesetzter Richtung, einen doppelt so hohen Einfluss auf die Zeitdilatation wie die SRT in den Phasen ohne Gravitation. Dadurch altert Teresa im Durchschnitt des gesamten Zeitraums der Trennung der Zwillinge schneller als Serena.

  216. #216 Anonym_2018
    15. Dezember 2018

    @MartinB

    Zitat aus 2. Seite dieses Artikels:

    Entscheidend ist aber – und das macht dieses Diagramm sehr schön deutlich -, dass es einen weiten Bereich von Ereignissen bei Teresa gibt, die für Serena mit gar nichts gleichzeitig sind.

    Das ist ein Artefakt, weil der entscheidende Zeitraum aus dem Leben von Serena in dieser Story übersprungen wurde: die beschleunigte Umkehr der Flugrichtung. Wenn man das nicht-Inertialsystem von Serena als Bezugssystem wählt (was nach ART erlaubt ist), erfährt Teresa in diesem Zeitraum ein extrem hohes äquivalentes Gravitationspotential, das sie extrem altern lässt (aus Sicht von Serena, siehe auch Kommentar #215).

  217. #217 Toni
    15. Dezember 2018

    “2) Dann beschreibt Einstein das Zwillingsparadoxon aus der Perspektive des Bezugssystems von Serena. Die relative Geschwindigkeit von Teresa verlangsamt zwar deren Uhr, ihr Gravitationspotential am Umkehrpunkt hat aber, in entgegengesetzter Richtung, einen doppelt so hohen Einfluss auf die Zeitdilatation wie die SRT in den Phasen ohne Gravitation. Dadurch altert Teresa im Durchschnitt des gesamten Zeitraums der Trennung der Zwillinge schneller als Serena.”

    “Dieser Teilsatz ist flasch. In dem Paper von Einstein steht nichts von einer Zeitdilatation wegen Beschleunigung.”

    Das sagt es Einstein ja nicht explizit, aber er meint das eindeutig. Du kannst umkehren, weil da eine äußere Kraft ausgeübt wird, das ist analog zu Satelliten in der Umlaufbahn, diese äußere Kraft ist dort Zentripetalkraft. Jede Richtungsänderung verursacht eine Beschleunigung. Von daher weiß ich nicht, was in meinem Satz falsch sein sollte.

    Was ist hier konkret falsch?
    “Wo die Symmetrie der Gleichwertigkeit der Systeme gebrochen wird, hat man eine echte Zeitdilatation wegen Beschleunigung (ART),”

    Symmetriebrechung ist eindeutig, und wie du selber sagst:
    “weil der entscheidende Zeitraum aus dem Leben von Serena in dieser Story übersprungen wurde: die beschleunigte Umkehr der Flugrichtung.”

  218. #218 MartinB
    15. Dezember 2018

    @Niels
    Du meinst sicher den hier:
    https://scienceblogs.de/hier-wohnen-drachen/2010/12/27/spezielle-relativitatstheorie/

    @Toni
    “Wo die Symmetrie der Gleichwertigkeit der Systeme gebrochen wird, hat man eine Zeitdilatation wegen Beschleunigung (ART), und wo nicht, hat man eine Zeitdilatation wegen SRT. Damit kann ich dann leben.”
    Kannst du bitte an meinem Beispiel mit den Myonen oben (erzeugt bei A, v=const zwischen B und C) mal erklären, was du dir da unter einer gebrochenen Symmetrie vorstellst oder in wie weit du “nicht echte” Effekte meinst.

    Du brignst in meinen Augen ein paar Dinge durcheinander: Wenn zwei Beobachter sich relativ zueinander bewegen, sieht jeder die Uhr des anderen verlangsamt, und das ist ein relativer Effekt (es lässt sich nicht sagen, wer von beiden “recht” hat).
    Im ZP ist es dagegen so, dass die Situation durch den Wechsel des Beugssystems nicht symmetrisch ist. Hier werden aber die Uhren zweier Beobachter an zwei Ereignissen direkt verglichen (Serena startet – Serena kehrt zurück), für beide Beobachter finden beide Ereignisse am sleben Ort (der Erde) statt.

    In Experimenten wie dem Myonenzerfall wird ebenfalls der zeitliche Abstand zweier Ereignisse gemessen, hier ist es aber so, dass diese ereignisse in einem der Bezugssysteme nicht am selben Ort stattfinden. Was wir messen, sind deshalb schlicht die zeitlichen Komponenten eines Raumzeitbstands, und die können sich natürlich unterscheiden. Das ist ein anderes Problem als das ZP.

  219. #219 Toni
    15. Dezember 2018

    @MartinB:

    “Kannst du bitte an meinem Beispiel mit den Myonen oben (erzeugt bei A, v=const zwischen B und C) mal erklären, was du dir da unter einer gebrochenen Symmetrie vorstellst oder in wie weit du “nicht echte” Effekte meinst.”

    Deshalb habe ich den obigen Satz formuliert.
    Mit den Myonen oben ist die Symmetrie der völligen Gleichwertigkeit der Inertialsysteme (Myon und Detektor) erhalten, und somit die Zeitdilatation durch SRT auch tatsächlich gegeben, wenn man die beiden Zeiten, Uhr an einem Myon und Uhr beim Detektor am selben Ort nicht vergleicht. Das ist das, was ich meine mit einem Scheineffekt der SRT. Es ist ein Scheineffekt, weil (noch mal) beobachterabhängig ist. Diese Beobachterabhängigkeit stört mich extrem. Nun, wenn man dies als nur reiner relativer beobachterabhängiger Effekt betrachtet, dann kann man von einem “echten” Effekt reden, was ja auch gemessen wurde.

    “Wenn zwei Beobachter sich relativ zueinander bewegen, sieht jeder die Uhr des anderen verlangsamt, und das ist ein relativer Effekt (es lässt sich nicht sagen, wer von beiden “recht” hat).”

    Ja, eben und genau das verleitet mich zu sagen, dass SRT Effekte “Scheineffekte” sind. Aber man hat sie ja beim Myonen Zerfall gemessen.

    Beim ZP oder 3-Brüder-Ansatz ist diese Symmetrie der Gleichwertigkeit der Systeme eben nicht mehr gegeben. Ich betrachte den Wechsel der Inertialsysteme am Umkehrpunkt als Beschleunigung. Am Umkehrpunkt bricht einfach diese Symmetrie, und somit sehe ich die echte beobachterUNabhängige Zeitdilatation als Folge von Richtungsänderung der Rakete durch Beschleunigung.

  220. #220 MartinB
    15. Dezember 2018

    @Toni
    “Es ist ein Scheineffekt, weil (noch mal) beobachterabhängig ist. Diese Beobachterabhängigkeit stört mich extrem”
    Mich nicht. Die SRT tut das, was eine physikalische Theorie tun soll: Sie beschreibt, das was, gemessen wird, korrekt. Dass unterschiedliche Leute unterschiedlice dinge messen, macht die Messungen deshalb nicht zu “scheineffekten”
    Ist aber wie es aussieht ein rein semantisches Problem – über die Physik scheinen wir uns ja jetzt einig zu sein.
    “Am Umkehrpunkt bricht einfach diese Symmetrie”
    Halte ich nach wie vor für eine sehr unglückliche Formulierung – der Begriff Symmetriebrechung bedeutet etwas anderes.

  221. #221 Anonym_2018
    15. Dezember 2018

    @Toni #217
    In der RT gibt es Zeitdillatation nur aufgrund von Geschwindigkeit und aufgrund von Gravitationspotential, nicht aufgrund von Beschleunigung.

    Im Bezugssystem von Teresa gibt es die Beschleunigung von Serena, aber nicht die dazu äquivalente Gravitation (Es ist ein Inertialsystem).

    Im Bezugssystem von Serena gibt es diese äquivalente Gravitation.

    Im Bezugssystem von Teresa wird die Zeitdillatation daher ausschließlich durch die SRT verursacht.

    Im Bezugssystem von Serena wird die Zeitdillatation teilweise durch die SRT und teilweise durch die ART verursacht. Der ART-Anteil ist genauso groß, dass er das Paradoxon aufhebt.

    Was ist hier konkret falsch?
    “Wo die Symmetrie der Gleichwertigkeit der Systeme gebrochen wird, hat man eine echte Zeitdilatation wegen Beschleunigung (ART),”

    Siehe oben. Höchstens wegen Gravitation, aber nicht wegen Beschleunigung.

  222. #222 Toni
    15. Dezember 2018

    @Anonym_2018:

    “Siehe oben. Höchstens wegen Gravitation, aber nicht wegen Beschleunigung.”

    Gravitationsfeld und Beschleunigung sind ja äquivalent.

    “Der ART-Anteil ist genauso groß, dass er das Paradoxon aufhebt.”

    Verstehe nicht, wenn der ART-Anteil genauso groß ist wie der SRT-Anteil, dann müssen die beiden Schwestern nach der Rückkehr gleich alt sein, was ja nicht der Fall ist. Nach dem Umkekrpunkt, gewinnt Serena schlagartig an Zeit, die wenig Verlust durch SRT (nach Umkehren) erleidet.

    “Im Bezugssystem von Teresa gibt es die Beschleunigung von Serena, aber nicht die dazu äquivalente Gravitation (Es ist ein Inertialsystem).”

    Das ist egal. Beschleunigung (Gravitation) erfährt ja in der Tat nur Serena.

    “Im Bezugssystem von Serena gibt es diese äquivalente Gravitation.”

    Ja, eben, und somit Beschleunigung ist äquivalent zu Gravitation.

    Das bestätigt ja meine Formulierung bezüglich Symmetrie von oben.

  223. #223 Anonym_2018
    15. Dezember 2018

    … wenn der ART-Anteil genauso groß ist wie der SRT-Anteil ..

    Das habe ich nicht geschrieben.

    Wie hoch der ART-Anteil ist, habe ich mit anderen Worten auch noch in #215 unter Punkt 2) beschrieben.

  224. #224 Toni
    15. Dezember 2018

    @MartinB

    “Ist aber wie es aussieht ein rein semantisches Problem – über die Physik scheinen wir uns ja jetzt einig zu sein.”

    Ja, in der Tat.

    “Halte ich nach wie vor für eine sehr unglückliche Formulierung – der Begriff Symmetriebrechung bedeutet etwas anderes.”

    Mit Symmetriebrechung meine ich nur, dass die völlige Gleichberechtigung der Inertialsystem verloren geht. Man kann es auch anders bezeichnen.

    Aber diese Richtungsänderung der Rakete beim ZP (Wechsel der Inertialsysteme) siehst du immer noch nicht als ein Beschleunigungseffekt?

  225. #225 Toni
    15. Dezember 2018

    @Anonym_2018:

    Also was ist hier konkret falsch, wenn Serena eine Beschleunigung erfährt, die äquivalent ist zu Gravitation:

    “Wo die Symmetrie der Gleichwertigkeit der Systeme gebrochen wird, hat man eine echte Zeitdilatation wegen Beschleunigung (ART),”

  226. #226 Anonym_2018
    15. Dezember 2018

    @Toni #225
    Antwort: siehe am Ende von Kommentar #221

    Wenn man dann “Beschleunigung” durch “Gravitation” ersetzt hat, ist diese Aussage immer noch falsch, wenn z.B. die Zeitdilatation von Serena im Bezugssystem von Teresa beschrieben wird.

    Ich verstehe zwar, was du mit Symmetriebrechung und echter Zeitdilatation sagen willst, das dürfte aber für außenstehende missverständlich oder unverständlich sein.

  227. #227 Toni
    15. Dezember 2018

    @MartinB:

    Zitat aus deinem Blog:

    “Eine Theorie, die derart eklatante innere Probleme mit sich bringt, hätte sicherlich niemand akzeptiert. Das Zwillingsparadoxon sollte sich innerhalb der SRT lösen lassen.”

    Das Problem ist ja, dass die beiden Systemen im ZP nicht gleichberechtigt sind. Auf der anderen Seite wird in SRT diese Gleichberechtigung so vehement verlangt. Deshalb sehe ich so, dass man dieses Problem unter dem Aspekt Beschleunigung lösen kann, und nicht mit reiner SRT.

  228. #228 Toni
    15. Dezember 2018

    @Anonym_2018

    “wenn z.B. die Zeitdilatation von Serena im Bezugssystem von Teresa beschrieben wird.”

    Ich sagte ja, der Bezugssystem von Teresa ist egal, weil sie ja GAR KEINE Beschleunigung erfährt.

    Meine obige Formulierung ist allgemeiner Natur, und bezieht sich nicht auf ein bestimmtes Bezugssystem. Sobald man eine echte Beschleunigung erfährt, bricht die Symmetrie. Wenn nicht, dann nicht.

    “Ich verstehe zwar, was du mit Symmetriebrechung und echter Zeitdilatation sagen willst, das dürfte”

    Ja, das versuche ich seit gestern mitzuteilen.

  229. #229 Toni
    15. Dezember 2018

    @Anonym_2018:

    Ich füge noch hinzu, es ist egal wo in welchem System die Symmetrie bricht, sondern, dass sie bricht.

  230. #230 MartinB
    15. Dezember 2018

    @Toni
    “Aber diese Richtungsänderung der Rakete beim ZP (Wechsel der Inertialsysteme) siehst du immer noch nicht als ein Beschleunigungseffekt?”
    Nein, siehe das 3-Brüder-Paradoxon. Ich verstehe unter Beschleunigung, dass ein physikalisches Objekt eine Geschwindigkeitsänderung erfährt.

    Dass die Beschleunigung selbst nicht wirklich relevant ist, sieht man ja auch daran, dass es selbst wenn Serena tatsächlich umkehrt nahezu volkommen irrelevant ist, wie sie das tut – sie kann stark beschleunigen oder schwach, sie könnte auch in Richtung Erde beschleunigen, dann nochmal kurz umkehren, weil sie was vergessen hat, dann wieder zurückbeschleunigen, für das Endergebnis spielt das praktisch keine Rolle

    Hmm, das ist eigentlich ein ziemlich gutes Argument dafür, dass die Beschleunigung selbst nicht relevant ist: Wir erweitern das Szenario oben so, dass Serena bei AC innerhalb von z.B. 1 Sekunde beschleunigt und wieder Richtugn Erde steuert. Dann beschleunigt sie nochmal so, dass sie wieder in die alte Richtung fliegt und dann wieder zurück. Die Zeitdilatation am Ende beeinflusst das alles nicht.

    Und nein, mit der ART hat das alles nichts zu tun – die Formulierung, dass Gravitation und Beschleunigung äquivalent sind, wird zwar gern verwendet, ist aber sehr missverständlich, weil man eigentlich erst mal klären muss, was genau man mit “äquivalent” meint.

  231. #231 Anonym_2018
    15. Dezember 2018

    @Toni #228

    Sobald man eine echte Beschleunigung erfährt, bricht die Symmetrie.

    Nein. Gegenbeispiel: Teresa könnte sich von einem Inertialsystem aus nach links bewegen und dann umkehren, Selena könnte sich von diesem Inertialsystem aus nach rechts bewegen und dann umkehren. Dann würden beide eine Beschleunigung erfahren und das könnte völlig symmetrisch angelegt werden.

    Meiner Ansicht nach ist das Zwillingsparadoxon zu komplex, als dass man es richtig in einem Satz zusammenfassen könnte.

  232. #232 Toni
    15. Dezember 2018

    @MartinB:

    “Nein, siehe das 3-Brüder-Paradoxon. Ich verstehe unter Beschleunigung, dass ein physikalisches Objekt eine Geschwindigkeitsänderung erfährt.”

    Für mich ist Beschleuigung verbunden mit Änderung der Geschwindigkeit UND der Richtung, weil sie eine vektorielle Größe ist.

    “Und nein, mit der ART hat das alles nichts zu tun – die Formulierung, dass Gravitation und Beschleunigung äquivalent sind, wird zwar gern verwendet, ist aber sehr missverständlich, weil man eigentlich erst mal klären muss, was genau man mit “äquivalent” meint.”

    Mit ART lässt sich ZP aber AUCH auflösen. Ja äquivalent bedeutet für mich, dass man die Zeitverzögerung, die man durch Massen erhalten kann, auch durch Beschleunigung erhalten kann.

    Die Raumzeit in einer Beschleunigten Rakete im All ist auch gekrümmt. Siehe die gekrümmte Bahn eines Laserstrahls in so einer beschleunigten Rakete.

  233. #233 MartinB
    15. Dezember 2018

    @Toni
    “Mit ART lässt sich ZP aber AUCH auflösen.”
    Die Rechnung würde ich gern sehen.
    “dass man die Zeitverzögerung, die man durch Massen erhalten kann, auch durch Beschleunigung erhalten kann.”
    Und das ist in dieser Form falsch. Woher nimmst du das? Vermutlich daher:
    “Die Raumzeit in einer Beschleunigten Rakete im All ist auch gekrümmt.”
    Nein. Siehe den Wikipedia-Eintrag für die Rindler-metrik:
    “In relativistic physics, the coordinates of a hyperbolically accelerated reference frame constitute an important and useful coordinate chart representing part of flat Minkowski spacetime”
    Wenn wir über die ART reden, musst du zwischen Koordinatensystemen und Metriken unterscheiden, sonst gibt es Chaos. Wenn ein beschleunigter Beobachter die Raumzeit misst, misst er sie flach.

  234. #234 Toni
    15. Dezember 2018

    @Anonym_2018

    “Dann würden beide eine Beschleunigung erfahren und das könnte völlig symmetrisch angelegt werden.”

    Dieser Fall bzw diese Symmetrie ist trivial, es geht um Vorhandsein einer Beschleunigung bei einem der Systeme. Wenn beide die gleiche Beschleunigung erfahren, dann werden sie beide gleich alt sein.

    Man konnte natürlich meine Formulierung verfeinern. In der Formulierung sehe ich grundsätzlich nichts falsches.

    “Meiner Ansicht nach ist das Zwillingsparadoxon zu komplex, als dass man es richtig in einem Satz zusammenfassen könnte.”

    Kommt oft in der Physik vor, dass man komplexe Vorgänge mit nur einem Satz zusammenfasst.

  235. #235 Toni
    15. Dezember 2018

    @MartinB:

    “Die Rechnung würde ich gern sehen.”

    Hier siehst du zumindest die ART Erklärung:
    https://www.wissenschaft-im-dialog.de/projekte/wieso/artikel/beitrag/was-hat-die-beschleunigung-mit-einsteins-zwillingsparadoxon-zu-tun/

    Die ART Erklärung siehst du gegen Ende des Beitrages.

    “Und das ist in dieser Form falsch. Woher nimmst du das? Vermutlich daher:”

    In welcher Form ist sie korrekt?

    Die Bahn eines Laserstrahls in einer beschleunigten Rakete von AUẞEN gesehen ist gekrümmt, weil die beschleunigung die Raumzeit krümmt. Natürlich lokal ist sie flach.

  236. #236 MartinB
    15. Dezember 2018

    @Toni
    Sorry, aber den text halte ich zumidnest am Ende für falsch bzw,. extrem irreführend (der teil über die SRT ist richtig,soweit ich sehe, der stimmt aber auch genau mit dem überein, was ich hier schreibe), denn da steht:
    “Wenn er seine Raketentriebwerke einschaltet, tritt ein Gravitationsfeld auf, das die Erde abbremst und auf ihn zu beschleunigt ”
    In der üblichen geometrischen Sichtweise der ART gibt es letztlich keine Gravitationsfelder – wenn man sie einführen will, dann entsprechen sie der Raumzeitkrümmung – und auch ein beschleunigstes Bezugssystem bewegt sich durch eine flache Raumzeit. Im übrigen wird da von einem homogenen Gravitationsfeld gesprochen (logisch, weil die relative Beschleunigung überall dieselbe ist) – dummerweise gibt es in der ART aber kein homogenes Gravitationsfeld (es gibt keine Metrik der Raumzeit, die zu einer überall konstanten Schwerebschleunigung führen würde). Leider hat die Seite keine Kommentarfunktion…

    Niels, siehst du das auch so (falls du hier noch mitliest)?

  237. #237 MartinB
    15. Dezember 2018

    Nachtrag: Habe gerade nochmal drüber nachgedacht, wie das gemeint sein könnte. ich vermute, gemeint ist folgendes: Wenn Serena alles in ihrem KS so beschreibt, dass sie immer in ruhe ist, erfährt sie beim beschleunigen zwei Effekte: Die Kraft, die sie beschleunigt, sowie die koordinatenbeschleunigung, die das kompensiert, so dass sie trotzdem in ihrem KS in Ruhe ist. Wenn sie die Bewegung der Erde beschreibt, erfährt die Erde eine koordinatenbeschleunigung, und man kann den Effekt des ZP (das will ch der Seite gern glauben) über diese Koordinatenbeschleunigung berechnen. Diese aber als eine Art “Gravitationsfeld” zu interpretieren, halte ich nach wie vor für irreführend bis falsch.
    Das Äquivalenzprinzip sagt nicht, dass Gravitation und Beschleunigung dasselbe sind.

  238. #238 Toni
    15. Dezember 2018

    @MartinB:

    “(der teil über die SRT ist richtig,soweit ich sehe, der stimmt aber auch genau mit dem überein, was ich hier schreibe),”

    Er sagt aber, dass Beschleunigung beim ZP eine untergeordnete Rolle spielt. Bei dir kommt sie gar nicht vor.

    Vielleicht gibt das eine bessere Erklärung des ZP innerhalb ART: https://www.youtube.com/watch?v=bjHLboK2M1g

  239. #239 Toni
    15. Dezember 2018

    @MartinB:

    “Wenn sie die Bewegung der Erde beschreibt, erfährt die Erde eine koordinatenbeschleunigung, und man kann den Effekt des ZP (das will ch der Seite gern glauben) über diese Koordinatenbeschleunigung berechnen.”

    So ähnlich rechnet auch Torsten Fließbach in seinem Buch allgemeine Relativitätstheorie Seite 56.

    “Das Äquivalenzprinzip sagt nicht, dass Gravitation und Beschleunigung dasselbe sind.”

    Wikipedia sagt dazu:
    https://de.wikipedia.org/wiki/%C3%84quivalenzprinzip_(Physik)

    “Nach dem starken Äquivalenzprinzip gilt, dass Gravitations- und Trägheitskräfte auf kleinen Abstands- und Zeitskalen in dem Sinn äquivalent sind, dass sie an ihren Wirkungen weder mit mechanischen noch irgendwelchen anderen Beobachtungen unterschieden werden können.”

    So meinte ich auch.

  240. #240 Anonym_2018
    15. Dezember 2018

    Das Äquivalenzprinzip sagt nicht, dass Gravitation und Beschleunigung dasselbe sind.

    Es besagt aber, dass die Beschleunigung von Serena im Bezugssystem von Teresa äquivalent ist zu einem Gravitationsfeld im Bezugssystem von Serena.

    Das schrieb A. Einstein 1918 bzgl. des Zwillingsparadoxons genauso:

    Nach der allgemeinen Relativitätstheorie geht nämlich eine Uhr desto schneller, je höher das Gravitatiohs-Potential an dem Orte ist, an dem sie sich befindet, und es befindet sich während des Teilprozesses 3 U2 tatsächlich an einem Orte höheren Gravitations-Potentials als U1. Die Rechnung ergibt, daß dies Vorauseilen gerade doppelt so viel ausmacht, als das Zurückbleiben während der Teilprozesse 2 und 4. Durch diese Betrachtung wird das von dir angeführte Paradoxon vollständig aufgeklärt.

    https://wikilivres.org/wiki/Dialog_%C3%BCber_Einw%C3%A4nde_gegen_die_Relativit%C3%A4tstheorie

  241. #241 MartinB
    15. Dezember 2018

    @Anonym_2018
    Danke für das Zitat – interessant, das Einstein das so gesehen hat.
    Ich halte es nach wie vor für eine ziemlich problematische Sicht der Dinge (weil es den Begriff “Gravitationsfeld” auf meiner Ansicht nach sehr ungeschickte Weise verwendet.), aber ich muss dann wohl zugeben, dass man es tatsächlich so sehen kann.

    @Toni&Anonym:
    In Sachen Äq-Prinzip sehe ich es so::
    Teiclhen im freien Fall folgen ihren Geodäten. Deshalb ist ein kräftefreies System im Vakuum (lokal) äquivalent zu einem System im freien Fall im “Schwerefeld”. Ebenso wird ein Objekt, das stationär im Schwerefeld verharrt, von seiner geodäten wegbeschleunigt, so dass diese Situation äquivalent ist zu einer, in der ein System in Abwesenheit eines Schwerefelds seine Geschwindigkeit relativ zu einem Inertialsystem ändert, also beschleunigt.

    Das ist die Aussage des Äq-Prinzips. Es sagt aber nicht, dass man eine Beschleunigung im Minkowski-Raum als Bewegung in einer gekrümmten Raumzeit (einem Schwerefeld) auffassen kann, was viele Formulierungen nahelegen. So wie Einstein von einem Gravitationsfeld zu reden, wenn die Raumzeit ungekrümmt ist, ist in meinen Augen wie gesagt irreführend: Das “Gravitationsfeld” identifiziert man in der ART mit der Krümmung der Raumzeit (ich glaube im ART-Buch von Weinberg gibt es dazu ne Diskussion, womit man genau es sinnvollerweise identifizieren sollte), und in einem beschleunigten Bezugssystem ist die Raumzeit ja ungekrümmt.

    @Toni
    Danke für den Hinweis auf den fließbach, aber da steht doch nichts zum ZP sondern nur die übliche Formel der Geodätengleichung oder habe ich was übersehen?

  242. #242 Toni
    15. Dezember 2018

    @MartinB:

    “und in einem beschleunigten Bezugssystem ist die Raumzeit ja ungekrümmt.”

    Warum ist dann die Bahn eines Laserstrahls in einem beschleunigen Fahrstuhl gekrümmt?
    Siehe die Bilder rechts in Wikipedia : https://de.wikipedia.org/wiki/%C3%84quivalenzprinzip_(Physik)

    “aber da steht doch nichts zum ZP sondern nur die übliche Formel der Geodätengleichung oder habe ich was übersehen?”

    Sorry, Seite 46 bitte lesen. Ich habe dir die Seitenangabe des PDF Programms gegeben.

    Nun wenn man ZP in ART auflösen kann, sind dann diese ganze Tricks in SRT, um Beschleunigung zu eliminieren, nicht überflüssig? Gerade, weil Einstein selbst das ganze innerhalb der ART gelöst hat.

  243. #243 Anonym_2018
    15. Dezember 2018

    @MarinB #241

    aber ich muss dann wohl zugeben, dass man es tatsächlich so sehen kann.

    Man kann ein Bezugssystem für Serena nur festlegen, wenn man für die Umkehr auch eine endliche Zeit und eine Beschleunigung (aus Sicht des Bezugssystems von Teresa) einplant.

    Wenn man das macht, dann kann man in der Mitte des Diagramms auf Seite 2 dieses Artikels zusätzliche Zuordnungslinien eintragen. Dieser bisherige “Zeitsprung” von Teresa entsteht im Bezugssystem von Serena durch Teresa’s schnelle gravitative Alterung, während das Gravitationsfeld (bzw. der Rakentenantrieb) eingeschaltet ist.

  244. #244 MartinB
    15. Dezember 2018

    @Toni
    “Warum ist dann die Bahn eines Laserstrahls in einem beschleunigen Fahrstuhl gekrümmt?”
    Weil das Koordinatensystem entsprechend gewählt ist – wie gesagt, du musst unterscheiden zwischen der raumzeit und den koordinaten, mit dneen du sie beschreibst. Die Geodätengleichung des Lichtsrahls enthält einen term für die koordinatenbeschleunigung. In einem anderen System (Inertialsystem der SRT) ist die Bahn ja ungekrümmt.

    Das gilt auch für die rechnung im fließbach, soweit ich sehe. Dass der metrische Tensor eine komplizierte Form hat, heißt nicht, dass die raumzeit gekrümmt ist.

    “Nun wenn man ZP in ART auflösen kann, sind dann diese ganze Tricks in SRT, um Beschleunigung zu eliminieren, nicht überflüssig?”
    Ich sehe eher die Betrachtung in der ART als “Trick” – denn in der SRT ist das wie erläutert nichts als die vertraute Dreiecksungleichung. Einfacher geht es nicht.

  245. #245 Toni
    15. Dezember 2018

    @MartinB:

    Wenn du dich erinnerst, ich habe dir geschrieben, dass man SRT Gesetze nur von einem Inertialsystem aus anwenden kann (angewandt egal auf welche andere Systeme, beschleunigt oder nicht beschleunigt).

    Im Fließbach steht das auch sehr schön geschrieben, dass man in einem beschleunigten System nicht mehr SRT Gesetze anwenden darf.

  246. #246 Toni
    15. Dezember 2018

    @MartinB:

    “Weil das Koordinatensystem entsprechend gewählt ist – wie gesagt, du musst unterscheiden zwischen der raumzeit und den koordinaten, mit dneen du sie beschreibst. Die Geodätengleichung des Lichtsrahls enthält einen term für die koordinatenbeschleunigung. In einem anderen System (Inertialsystem der SRT) ist die Bahn ja ungekrümmt.”

    Nach Äq-Prinzip müsste ja die Raumzeit ECHT gekrümmt sein, wenn man beschleunigt. Wenn man ein Gravitationsfeld durch Koordinatentrafos zum verschwinden bringt, dann hatte man ja kein echtes Gravtationsfeld. Aber hier beschleunigt man ja wirklich.

    “Ich sehe eher die Betrachtung in der ART als “Trick” – denn in der SRT ist das wie erläutert nichts als die vertraute Dreiecksungleichung. Einfacher geht es nicht.”

    Ja, wenn man in SRT die Richtungsänderung beim ZP unter dem Teppich kehrt. Dann ja.

  247. #247 MartinB
    15. Dezember 2018

    @Toni
    Wo genau?
    Auf S. 43 steht doch explizit:
    “Nicht-IS sind also zulässig, in ihnen haben die Gesetze aber nicht mehr ihre gewohnte Form”
    Die ganze Rechnung in dem Abschnitt verwendet doch nur die SRT und hantiert mit beschleunigten Systemen.

  248. #248 MartinB
    15. Dezember 2018

    @Toni
    “Nach Äq-Prinzip müsste ja die Raumzeit ECHT gekrümmt sein, wenn man beschleunigt.”
    Nein, das ist eindeutig falsch. Ob die Raumzeit gekrümmt ist oder nicht, kann nicht vom KS abhängen. Sonst wäre es ja überhaupt nicht sinnvoll von einer Raumzeit zu reden.

  249. #249 Toni
    15. Dezember 2018

    @MartinB:

    Seite 46 :

    Damit ist in KS die Berechnung der Uhrzeiten komplizierter;
    insbesondere gilt (3.29) nicht für KS’. Die Verwendung von SRT-Gesetzen (wie (3.29)) in KS’ ist vergleichbar mit der Erwartung eines Billardspielers, auch auf einem Karussell wie gewohnt Billard spielen zu können.

    Du kannst ohne eine Koordinaten-Tranformation keine SRT Gesetze in einem beschleunigten System anwenden.

    “Ob die Raumzeit gekrümmt ist oder nicht, kann nicht vom KS abhängen”

    Das ist richtig. Du kannst einen flachen Metrik in Polarkoordinaten darstellen und einen koordinatenabhängige Metrik kriegen, der immer noch FLACH ist, aber der Punkt ist, dass man bei der Rakete tatsächlich beschleunigt, hier müsste die Raumzeit in der Rakete echt gekrümmt sein, damit der Lichstrahl einen gekrümmten Weg einschlagen kann.

  250. #250 Toni
    15. Dezember 2018

    @MartinB:
    ok, wie auch im Fließbach steht:
    Der metrische Tensor von KS’ ist durch die vorgegebene Beschleunigung nicht vollständig festgelegt. Vielmehr können willkürliche Koordinatentransformationen die tatsächliche Form von gμν ändern.

    Dann ist alles gut.

  251. #251 Anonym_2018
    15. Dezember 2018

    @MartinB #233

    “Mit ART lässt sich ZP aber AUCH auflösen.”
    Die Rechnung würde ich gern sehen.

    Hier ist die Rechnung am Beispiel der Zahlen, die unter dem Diagramm auf der 2. Seite dieses Artikels angegeben sind:

    Serena fliegt mit der Geschwindigkeit 0,6 c weg und ist am Umkehrpunkt 3 Lichtjahre von Teresa entfernt.

    D.h. die Beschleunigung muss ihre Geschwindigkeit von 0,6 c auf -0,6 c, also um 1,2 c ändern.

    Nun berechne ich mit dieser Formel, um welche Zeit die Uhren von Serena und Teresa während der Beschleunigung auseinnanderlaufen:

    ∆t = v/c² * L

    In unserem Beispiel:
    ∆t = 1,2c / c² * 3 Lichtjahre = 3,6/c * (c * 1 Jahr)
    = 3,6 Jahre.

    Wenn auf Serena’s Uhr für die Dauer der Beschleunigung vergleichsweise wenig Zeit benötigt wird, z.B. weniger als 10 Tage, dann passen die 3,6 Jahre zu dem mittleren Abschnitt auf der senkrechten Achse, der nicht über die roten und blauen Linien entsprechend der SRT zugeordnet wird.

  252. #252 Niels
    15. Dezember 2018

    @MartinB

    Niels, siehst du das auch so (falls du hier noch mitliest)?

    Klar? Hätte ich jetzt auch als völlig unstrittige Grundlagen eingeordnet?
    .

    Ich bin mir eigentlich sehr sicher, dass es hier bei weitem nicht nur um ein “rein semantisches Problem” geht.

    Toni hat schon bei der SRT etwas ganz grundegendes nicht verstanden.
    Bei den Myonen seid ihr schließlich immer noch nicht weitergekommen, oder?

    Dazu, dass jetzt auf einmal die Raumzeit ECHT gekrümmt ist, wenn man beschleunigt, fällt mir deswegen nicht besonders viel ein.
    Das ist schließlich sehr offensichtlich in Widerspruch zu praktisch jeder einzigen Textstelle zur ART, die man egal wo auftreiben kann.
    Darüber hinaus habe ich schon in #182 zwei sehr ausführliche Links zur Beschleunigung in der SRT geliefert, der eine ist sogar ein erstaunlich langer Wiki-Artikel, der dann sogar Acceleration (special relativity) heißt.

    Wenn man danach immer noch behauptet, dass die Raumzeit ECHT gekrümmt ist, wenn man beschleunigt. weiß ich echt nicht, wie ich sinvoll weiter argumentieren könnte.
    .

    Ich weiß auch nicht, ob es wirklich viel bringt, hier weiter über die ART zu sprechen, wenn man das Zwillingsparadoxon der SRT nicht versteht.
    Und über das Zwillingsparadoxon der SRT, solange man die Zeitdilatation der SRT völlig falsch versteht.
    Und die Zeitdilatation gehört dann eben wirklich einfach zu den absoluten Grundlagen, die man sich irgendwann selbst aneignen muss.

    im übrigen wird da von einem homogenen Gravitationsfeld gesprochen (logisch, weil die relative Beschleunigung überall dieselbe ist) – dummerweise gibt es in der ART aber kein homogenes Gravitationsfeld

    Das ist eben praktisch eins zu eins die Originalerklärung von Einstein, wie das Zwillingsparadoxon mit Hilfe der ART aufzulösen sei.
    Einstein argumentiert in seinen frühen Arbeiten über die ART allgemein gerne mit “homogenen Gravitationsfeldern”.

    Ist kein großer Zufall, dass man das heute nicht mehr macht. Folgerichtig findet man auch in keinem einzigen Lehrbuch Einsteins ART-Auflösung dieses Paradoxons.

    Ich sehe auch wirklich keinen nicht irreführenden Weg, in der ART mit Gravitationsfeldern oder wie hier geschehen sogar außerdem mit Gravitationspotentialen(!) zu argumentieren.

    Das Äquivalenzprinzip sagt nicht, dass Gravitation und Beschleunigung dasselbe sind.

    Das hat auch nicht einmal Einstein in seinen unglücklichsten Formulierungen so behauptet.
    Er spricht davon, dass ein bestimmter Beobachter beides unter bestimmten Bedingungen durch bestimmte Experimente nicht in den Auswirkungen auf diese Experimente unterscheiden kann.

    Die knackigst Formulierung es Äquivalenzprinzips ist meiner Meinung nach:
    Lokal gelten die Gesetze der SRT.

    “Mit ART lässt sich ZP aber AUCH auflösen.”
    Die Rechnung würde ich gern sehen.

    Hm?
    Da nehme ich die SRT-Rechnung und schreibe ART drüber?
    Wenn ich Geld dafür bekomme, schreibe ich dir bei Bedarf auch noch ne Metrik und ne Geodätengleichung dazu und setzte dann überall Einser und Nuller ein?

  253. #253 Toni
    15. Dezember 2018

    @Niels:

    Du wirkst auf mich sehr sehr sehr hochnäsig und herabschauend. Und ganz ehrlich deine Lebenratschläge brauche ich nicht. Es ist kein Verbrechen über irgendetwas zu diskutieren und anderer Meinug zu sein. Es ist auch kein Verbrechen irgendetwas nicht korrekt verstanden zu haben, dass Beschleunigung die Raumzeit nicht krümmt.

    Deine menschliche und soziale Charakterqualitäten lassen sehr zu wünschen übrig. Wenn man nicht wüßte, würde man denken, dass du Intelligenz mit Löffel gefressen hast. Geh bitte wonaders dein dummes Zeug zu verteilen.

    Von Martin kannst du auf jeden Fall GAAAAAANZ VIEEEEEL lernen. Ein typisches Arschloch!!!

  254. #254 Anonym_2018
    15. Dezember 2018

    Folgerichtig findet man auch in keinem einzigen Lehrbuch Einsteins ART-Auflösung dieses Paradoxons.

    Das finde ich sehr schade. Ich halte es für didaktisch sinnvoll, wenn man das Zwillingsparadoxon sowohl im Bezugssystem von Teresa (Intertialsystem, SRT) als auch im Bezugssystem von Serena (nicht-Intertialsystem, SRT+ART) berechnet.

    Das verdeutlicht auch gut den logischen Zusammenhang zwischen SRT und ART.

    @Toni: Vielen Dank für den Hinweis auf das Einstein-Paper!

  255. #255 Anonym_2018
    15. Dezember 2018

    @Niels
    Eine Verständnisfrage:
    Wenn man die gravitative Zeitdilatation zwischen zwei Objekten, die z.B. in einem homogenen Gravitationsfeld 3 Lichtjahre voneinander entfernt sind, berechnen will, gehört das zur SRT, zur ART oder zu keiner von beiden?

  256. #256 Niels
    16. Dezember 2018

    @Anonym_2018

    Ich verstehe die Frage nicht.
    Gravitative Zeitdilatation zwischen zwei Beobachtern im selben homogenen Gravitationsfeld?

    (Zuerst musst du einmal genau bestimmen, was du mit Gravitationsfeld eigentlich meinst? Weder in der SRT noch in der ART ist das ein definierter Begriff.
    Raumzeitkrümmung?
    Diese komischen Gravitationspotentiale in Einsteins Arbeit von 1918?)

    Allgemein ist der Begriff gravitative Zeitdilatation aber doch genau so definiert, dass man damit einen ART-Effekt meint?
    Siehe wiki:
    Gravitational time dilation is a form of time dilation, an actual difference of elapsed time between two events as measured by observers situated at varying distances from a gravitating mass.

    Natürlich sind alle SRT-Effekte aber immer auch ART-Effekte, die SRT ist schließlich ein Grenzfall der ART.

    Wenn man mit der ART Zeitdilatation berechnet, berücksichtigt das immer alle möglichen Beiträge.
    Wenn man kompliziertere Raumzeiten betrachtet, im Extremfall so Sachen wie das Gödel-Universum, ergibt eine Aufspaltung in SRT-Effekte und ART-Effekte wahrscheinlich nicht mehr so wahnsinnig viel Sinn.

    Man kann natürlich immer sowas wie linearized gravity oder post-Newtonian expansion machen und sich anschauen, welche Ordnungen welche Beiträge beisteuern. Aber eine Unterteilung des Ergebnisses in SRT-Effekte und ART-Effekte ist das auch nicht unbedingt.
    .

    Die SRT nähert die ART immer dann genau genug, wenn der betrachtete Raumzeit-Teil genügend dem Minkowski-Raum gleicht.
    Was das jetzt im Allgemeinen genau bedeuten soll ist gar nicht so einfach zu sagen, das muss man sich eigentlich immer für jeden Einzelfall genauer anschauen.

    Ich halte es für didaktisch sinnvoll, wenn man das Zwillingsparadoxon sowohl im Bezugssystem von Teresa (Intertialsystem, SRT) als auch im Bezugssystem von Serena (nicht-Intertialsystem, SRT+ART) berechnet.

    Man berechnet es aber doch sehr häufig auch für beide Beobachter. (Die Rechnung für nicht-inertiale Beobachter ist allerdings naturgemäß hässlicher, deswegen wird das manchmal weggelassen.)

    Nur das eben sowohl im Intertialsystem als auch im Nicht-Intertialsystem nur mit der SRT gerechnet wird.
    Siehe dazu den schon oben verlinkten Abschnitt aus dem englischen Wiki-Eintrag zum Zwillingsparadoxon:
    Difference in elapsed times: how to calculate it from the ship

    Wenn die Raumzeit der Minkowski-Raum ist braucht man die ART grundsätzlich nicht.
    Bzw. die ART-Rechnung ist in diesem Fall eben immer exakt identisch zur SRT-Rechnung.
    ART angewendet auf den Minkowski-Raum ergibt die SRT.
    Egal, ob es um Inertial- oder Nicht-Inertialbeobachter geht.

    .

    @Toni

    Das war jetzt alles wegen Das ist nur leider schlicht und ergreifend völliger Quatsch?

    Oder wegen
    Ich weiß auch nicht, ob es wirklich viel bringt, hier weiter über die ART zu sprechen, wenn man das Zwillingsparadoxon der SRT nicht versteht.
    Und über das Zwillingsparadoxon der SRT, solange man die Zeitdilatation der SRT völlig falsch versteht.
    ?
    .

    Persönlich angegriffen habe ich dich nämlich nie!
    Quatsch muss man auch mal Quatsch nennen dürfen.
    Das muss man meiner Meinung nach auch mal aushalten können, wenn man wiederholt Unsinn redet.

    Und ganz ehrlich deine Lebenratschläge brauche ich nicht.

    Die da gewesen wären?
    Kann auch beim nochmaligen Lesen nichts finden…

    .

    @MartinB

    Ich hatte heute einen richtig schlechten Tag und habe deswegen zugegeben schärfer und wohl auch arroganter formuliert als sonst.
    Momentan sehe ich aber eigentlich nichts, für das ich mich zwingend entschuldigen müsste.

    Wenn du das anders einschätzt mach ichs aber natürlich trotzdem, ist schließlich dein Blog.
    (Vielleicht mach ichs morgen sowieso, wenn ich besser drauf bin. Ich bemühe mich jedenfalls, in Zukunft wieder weniger unleidlich zu schreiben.)

  257. #257 Toni
    16. Dezember 2018

    @Niels:

    “Oder wegen Ich weiß auch nicht, ob es wirklich viel bringt, hier weiter über die ART zu sprechen, wenn man das Zwillingsparadoxon der SRT nicht versteht.
    Und über das Zwillingsparadoxon der SRT, solange man die Zeitdilatation der SRT völlig falsch versteht.?”

    Ich habe mehrmals erwähnt, dass ich ein Problem damit habe, beobachterabhängige Effekte als echt anzusehen. Das hat sich ja aber schon längst erledigt. Wenn du die Kommentare lesen würdest, würdest du es wissen. SRT Zeitdilatation ist wirklich nichts schweres, dass man sie nicht versteht, es ging nur um Echtheit, um Objektivität! Martin hat mich überzeugt, dass sie eben echt sind, was du nicht geschafft hast!

    “Das ist nur leider schlicht und ergreifend völliger Quatsch?”

    Solche Sätze sind absolut nicht hilfreich. Mein Denkfehler, war dass ich Beschleunigung gleich Gravitation gesetzt habe. Das ist, das was ich mir ankreide, was ja kein Verbrechen ist!!!

    “Die da gewesen wären?”
    solche :
    “Und die Zeitdilatation gehört dann eben wirklich einfach zu den absoluten Grundlagen, die man sich irgendwann selbst aneignen muss.”

    Das kannst du gar nicht beurteilen, ich habe schon mehrere Bücher, original Paper von Einstein über SRT und ART gelesen und auch hier angegeben. Keine Sorge, habe mir schon einiges angeignet, aber man lernt es ja nie aus, und SRT und ART bieten viel Interpretationsmöglichkeiten und es werden viele falsche Interpretationen verbreitet. Nicht umsonst diskutiert man heute noch über die beiden Theorien.

    “Das muss man meiner Meinung nach auch mal aushalten können, wenn man wiederholt Unsinn redet.”

    Das ist deine Meinung. Keiner hat dich gezwungen an der Diskussion teizunehmen. Ich habe DICH ja nichts gefragt. Hast du dich aus heiterem Himmel in die Diskussion eingemischt.

    Du erinnerst mich stark an Mathematik-Studenten, die immer sehr besserwisserisch unterwegs waren. Wie gesagt, versuch mal von Martin zu lernen, wie man ordentlich diskutiert, auch wenn man anderer Meinung ist. Ich bin jedenfalls nicht dein Schlagkissen. Deinen Frust kannst du woanders ablassen.

  258. #258 Anonym_2018
    16. Dezember 2018

    @Niels #256

    Zuerst musst du einmal genau bestimmen, was du mit Gravitationsfeld eigentlich meinst?

    Gravitationsfeld, Schwerefeld, ein vektorielles Kraftfeld, das die Gravitationswechselwirkung vermittelt.

    Quelle:
    https://www.spektrum.de/lexikon/physik/gravitationsfeld/6090

    Ich verstehe die Frage nicht.
    Gravitative Zeitdilatation zwischen zwei Beobachtern im selben homogenen Gravitationsfeld?

    Ja. Gravitative Zeitdilatation entsteht in einem homogenen Gravitationsfeld zwischen Orten mit unterschiedlichem Gravitationspotential.

    Siehe dazu den schon oben verlinkten Abschnitt aus dem englischen Wiki-Eintrag zum Zwillingsparadoxon:
    Difference in elapsed times: how to calculate it from the ship

    Man kann die Berechnung im Bezugssystem von Serena gegenüber der verlinkten Rechnung stark vereinfachen, wenn man, wie im Paper von A. Einstein, nur von abwechselnd grandlinig-gleichförmigen (=SRT-Zeitdilatation bei Teresa) oder von konstant beschleunigten (=gravitative Zeitdilatation bei Teresa, siehe Kommentar #251) Bewegungen von Serena im Bezugssystem von Teresa ausgeht. Die sich daraus ergebenden Zeitabläufe bei Teresa müssen dann zusammenaddiert werden.

  259. #259 MartinB
    16. Dezember 2018

    @Toni
    #246
    ” Die Verwendung von SRT-Gesetzen (wie (3.29)) in KS’ ist vergleichbar mit der Erwartung eines Billardspielers, auch auf einem Karussell wie gewohnt Billard spielen zu können.”
    Klar, aber das heißt ja nicht, dass die SRT da nicht anwendbar wäre oder so.
    “hier müsste die Raumzeit in der Rakete echt gekrümmt sein, damit der Lichstrahl einen gekrümmten Weg einschlagen kann.”
    Das ist und bleibt falsch, sorry. Du verwechselst immer noch Koordinaten mit den geometrischen gebilden, die sie beschreiben. Das Beipsiel mit den Polarkoordinaten ist gut: Eine gerade Linie erscheint im phi-r-KS gekrümmt, dadurch ist die Ebene aber immer noch flach.

    #247
    Wie gesagt, man muss genau unterscheiden zwischen dem metrischen tensor und der Krümmung des Raums. Dass der metrische Tensor ne komplizierte Form hat, sagt per se nichts über die RZK, dazu musst du den Riemann-Tensor berechnen (oder entsprechende andere Größen).

    @Anonym #251
    Danke, interessant. Ich würde das immer noch nicht als “gravitativ” beschreiben, weil das total irreführend ist, aber nett, dass das aus den Formeln so rauskommt.

    @Niels#252
    “Ich sehe auch wirklich keinen nicht irreführenden Weg, in der ART mit Gravitationsfeldern oder wie hier geschehen sogar außerdem mit Gravitationspotentialen(!) zu argumentieren.”
    Danke, dann bin ich ja beruhigt. Ich dachte schon, mich hätte jetzt der Arroganzanfall gepackt, dass ich ne Erklärung von Einstein für schlecht halte…

    @Toni #253
    Auch wenn ich mich ungern in Streitereien einmische – verwechsle nicht Kritik an deinen Argumenten (wie die von Niels in #252 oder weiter oben) mit Kritik an deiner Person. Deine Beleidigungen von Niels haben hier nichts zu suchen, da wäre eigentlich ne entschuldigung fällig…

    @Anonym #255
    Es gibt in der ART kein homogenes Gravitationsfeld – es gibt keine Metrik der Raumzeit, die dazu führt, dass man eine überall im raum (oder Halbraum) konstante Schwerebeschleunigung bekommt. Dazu gibt es sehr nette papers. Das ist ja ein Grund, warum ich Einsteins Formulierung für so unglaublich unglücklich halte.

    @Niels 256
    In meinen Augen alles gut.

    @Toni257
    Eine Idee oder Aussage als “Quatsch” zu bezeichnen, ist kein Angriff einer Person, sondern normale Diskussion in der Wissenschaft.

    @Anonym #258
    “Gravitationsfeld, Schwerefeld, ein vektorielles Kraftfeld, das die Gravitationswechselwirkung vermittel”
    Sowas gibt es in der geometrischen Formulierung der ART aber nicht.

  260. #260 Anonym_2018
    16. Dezember 2018

    @MartinB #259

    Es gibt in der ART kein homogenes Gravitationsfeld – es gibt keine Metrik der Raumzeit, die dazu führt, dass man eine überall im raum (oder Halbraum) konstante Schwerebeschleunigung bekommt.

    O.K. Dann kann ich “ART” als Antwort auf meine Frage in #255 schon mal ausschließen. Zur SRT dürfte das wohl auch nicht gehören, weil das Gravitationspotential in einem homogenen Gravitationsfeld nicht räumlich konstant ist. Dann bleibt also nur “keine von beiden”.

    Das verbietet es ja einem (hoffentlich) trotzdem nicht, Berechnungen zu Gedankenexperimenten (z.B. Zwillingsparadoxon oder beschleunigter Fahrstuhl) mit einem homogenen Gravitationsfeld auszuführen 🙂

  261. #261 MartinB
    16. Dezember 2018

    @Anonym_2018
    Du kannst natürlich irgendwas berechnen, und es sollte auch durchaus möglich sein, ein Feld zu konstruieren, dass näherungsweise über einen endlichen Bereich konstant ist (das gilt auf der erdoberfläche ja auch) – du kannst dir ja eine riesig große Masse in genüender Entfernung vorstellen. Dann ist die Zeitdilatation auch entsprechend einfach zu berechnen, so wie auf der Erdoberfläche, pro Meter gibt es eine Dilatation von g/c^2, wen ich mich gerade nicht vertue (auf der Erde 0.11fs/Meter).

  262. #262 Niels
    16. Dezember 2018

    @Anonym_2018

    Warum gehst du davon aus, dass deine Rechnung aus #251 das richtige Ergebnis liefert?

    In der Wikipedia findet man unter Specific example:

    Consider a space ship traveling from Earth to the nearest star system:
    a distance d = 4 light years away, at a speed v = 0.8c

    Nach SRT-Rechnung:
    Teresa 10 Jahre, Serena 6 Jahre
    Differenz 4 Jahre

    Nach deiner Rechnung:
    Differenz ∆t = 1,6c / c² * 4 Lichtjahre = 6,4 Jahre.

    Das verbietet es ja einem (hoffentlich) trotzdem nicht, Berechnungen zu Gedankenexperimenten (z.B. Zwillingsparadoxon oder beschleunigter Fahrstuhl) mit einem homogenen Gravitationsfeld auszuführen

    Klar, kann man machen. Es muss einem nur klar sein, was man eigentlich tut.

    Dann verwendet man nämlich eine newtonsche Näherung der Zeitdilatation, in die man Gravitationspotentiale einsetzen kann. Die Formel ist leicht zu finden.

    Aber wo kommt sie eigentich her?
    Zur Herleitung verwendet man die Schwarzschild-Metrik und führt dann letztlich eine Reihenentwicklung der ART-Formel durch, die man schon bei frühstmöglicher Ordnung abbricht.

    (Wobei wirklich niemand von der Schwarzschild-Metrik behaupten kann, dass sie global ein homogenes Gravitationsfeld beschreibt.)

    Letztlich läuft es eben doch auf die ART hinaus.
    .

    Für die klassische Mechanik ist man doch eigentlich in der selben Situation.
    In deren Gültigkeitsbereich erhält man mit ausreichender Genauigkeit das selbe Ergebnis wie bei exakten Berechnung mit Hilfe der SRT.
    Ist aber trotzdem nur eine Näherung, die nur unter bestimmten Voraussetzungen anwendbar ist.

    Damit kann ich analog zu deinem #255 fragen:
    Wenn ich einen zentralen Stoss beschreiben will, gehört das dann zur klassischen Mechanik, zur SRT oder zu keiner von beiden?
    .

    @MartinB
    Ich persönlich finde alle Arbeiten von Einstein schrecklich zu lesen und fürs Verständnis nicht so wahnsinnig hilfreich.
    Sollte man meiner Meinung nach nur aus historischem Interesse lesen und nie, um die Physik zu verstehen.

    Ist aber auch kein Wunder.
    Wäre schließlich schrecklich, wenn es seit 100 Jahren keine didaktischen Fortschritte gegeben hätte.

  263. #263 MartinB
    16. Dezember 2018

    @Niels
    Also ich erinnere mich, dass ich einige Einstein-Arbeiten ganz o.k. fand und die halbwegs allgemeinverständliche Einführung in die RT auch nicht so schlecht.
    Aber gerade begrifflich hat sich halt viel getan.

  264. #264 Anonym_2018
    16. Dezember 2018

    @Niels #262

    Nach SRT-Rechnung:
    Teresa 10 Jahre, Serena 6 Jahre
    Differenz 4 Jahre

    Nach deiner Rechnung:
    Differenz ∆t = 1,6c / c² * 4 Lichtjahre = 6,4 Jahre.

    Das ist kein Widerspruch. Die Uhrendifferenz von 6,4 Jahren in meiner Rechnung bezieht sich nur auf den relativ kurzen Zeitraum der Umkehr mit aktivem Raketenantrieb. Im Kommentar #251 habe ich geschrieben und angenommen, dass das auf der Uhr von Serena nur weniger als 10 Tage benötigt. D.h. meine 6,4 Jahre (+10Tage) sind Teil der obigen 10 Jahre von Teresa und die 10 Tage sind Teil der obigen 6 Jahre von Serena.

    Wenn ich einen zentralen Stoss beschreiben will, gehört das dann zur klassischen Mechanik, zur SRT oder zu keiner von beiden?

    zur SRT.

  265. #265 Anonym_2018
    16. Dezember 2018

    @Niels #262

    Ergänzung zu #264

    Earth perspective

    In this case ε = 0.6 and the travelers will have aged only 0.6 × 10 = 6 years when they return.

    In den Phasen ohne Beschleunigung ist das Bezugssystem von Serena ein Interialsystem (aber auf Hin- und Rückweg ein unterschiedliches). Aus Symmetriegründen kann Serena ganauso rechnen wie Teresa: Aus Serena’s Sicht altert Teresa in dieses Zeitabschnitten um nur 0.6 x 6 = 3,6 Jahre.
    Was kommt heraus, wenn man dazu noch meine 6,4 Jahre (+ 10 Tage) addiert?

  266. #266 Niels
    16. Dezember 2018

    @Anonym_2018
    Ich verstehe nicht richtig, worum es gerade geht.

    Mir ging es um:

    “Mit ART lässt sich ZP aber AUCH auflösen.”
    Die Rechnung würde ich gern sehen.

    Darauif antwortetest du mit deiner Rechnung.

    Wie genau löst du das Zwillingsparadoxon jetzt mit Hilfe der gravitativen Zeitdilatation auf?
    Kannst du das mal ganz konkret (z.B. anhand der Zahlen des wiki-Beispiels) vorrechnen?

    Oder geht es dir doch um etwas ganz anderes?

    Wenn ich einen zentralen Stoss beschreiben will, gehört das dann zur klassischen Mechanik, zur SRT oder zu keiner von beiden?

    zur SRT.

    Okay?
    Dann gehört alles, was nicht Quantenmechanik ist, zur ART. Auch der zentrale Stoss.

  267. #267 MartinB
    16. Dezember 2018

    @Niels
    “Wie genau löst du das Zwillingsparadoxon jetzt mit Hilfe der gravitativen Zeitdilatation auf?”
    Das ist doch die Rechnung von Einstein, die oben verlinkt wurde – oder stehe ich jetzt völlig auf dem Schlauch?

  268. #268 Anonym_2018
    16. Dezember 2018

    @Niels #266

    Kannst du das mal ganz konkret (z.B. anhand der Zahlen des wiki-Beispiels) vorrechnen?

    Schon geschehen: in den Kommentare #264 und #265.

    Was dort noch fehlte: Bei der letzten Addition in #265 kommen 10 Jahre (+10 Tage) heraus. Diese Addition löst das Paradoxon auf.

  269. #269 Niels
    16. Dezember 2018

    @MartinB @Anonym_2018
    Ich verstehe schlicht nicht, was Anonym_2018 eigentlich rechnet?

    Was bedeutet diese “Uhrendifferenz von 6,4 Jahren”?
    Wer misst sie wann?
    Warum addiert man ausgerechnet 6 lorentztransformierte Jahre zu dieser Uhrendifferenz?

    Da steh ich völlig auf dem Schlau, sorry.

    Und warum braucht man die SRT angeblich nicht, wenn jetzt andauernd Lorentztrafos durchgeführt werden?

  270. #270 Anonym_2018
    16. Dezember 2018

    @Niels #269
    Kein Problem! Ich schage vor, dass du dir zunächst meinen Kommentar #215 anschaust, um einen Überblick zu bekommen. Dort verlinke ich zunächst das Paper von Einstein und fasse dann den wesentlichen Inhalt kurz zusammen.

    Und warum braucht man die SRT angeblich nicht

    Auch (!) die braucht man. Im Bezugssystem von Serena altert Teresa zunächst langsamer als Serena (SRT) danach kurzzeitig viiieel schneller (gravitative Zeitdilatation) und im Anschluss wieder langsamer (SRT).

    In der Summe stimmt dann die Alterung von Teresa im Bezugssystem von Serena mit der im Bezugssystem von Teresa überein.

  271. #271 MartinB
    16. Dezember 2018

    @Niels
    Ich habe die Rechnung nicht nachvollzogen und schlicht angenommen, dass es dieselbe ist wie in Einsteins 1918-Erklärung.

    @Anonym
    Vielleict kannst dues aber doch noch mal im Detail vorrechnen wie du es meinst?

  272. #272 Anonym_2018
    16. Dezember 2018

    @Niels #269

    Ich weiss zwar, dass MartinB keine Videos guckt, aber vielleicht du. Dann schau dir am besten mal dieses 10 Minuten-Video an:
    Twin Paradox in General Relativity

    Quelle: Kommentar #238 von Toni

  273. #273 MartinB
    16. Dezember 2018

    @Niels+Anonym
    So, jetzt habe ich mal beim Joggen drüber nachgedacht. ich nehme die Zahlen von Wiki:
    Teresa sieht Serena 5 jahre ewgfliegen, 5 Jahre zurückfliegen.
    Dilatationsfaktor ist 0,6 bei v=0,8c, macht 3+3=6Jahre Serena-Zeit.

    Serena sieht Teresa 3 Jahre von sich weg und auf sich zufliegen. Dilatationsfaktor 0,6 macht 3,6Jahre teresa-Zeit, dazu kommen die 6,4 Jahre durch die “gravitative Dilatation”, macht 10 Jahre. Soweit passt es also.

    Ich verstehe das Einstein-Argument jetzt so: In Teresas Bezugssystem ist sie während der Beschleunigungsphase in Ruhe, spürt aber eine Kraft, die sie in Richtung Teresa beschleunigt. Die Situatio ist damit *äquivalent* zu einer Situation, wo Serena (und Teresa) stationär in einem homogenen Gravitationsfeld sind. Wegen des Äq-Prinzips darf Serena deshalb auch die entsprechenden Formeln anwenden.

    Das heißt aber nicht, dass die physikalische Situation der Beschleunigung in beiden Fällen identisch ist oder dass diese Äquivalenz so interpretiert werden kann, als seien Serna und Teresa während des Umkehrens tatsächlich in einem Gravitationsfeld. Dass das nicht so sein kann, können wir schon daran sehen, dass wir uns eine dritte beobachterin Udine denken können, die zwischen Serena und Teresa sitzt und die permanent relativ zu Teresa in Ruhe ist. Serena ist der Ansicht, dass Udine während der Beschleunigungsphase eine zeitdilatation relativ zu Teresa erfährt, das ist aber natürlich nicht der Fall, Teresas und Udines Uhren gehen die ganze Zeit synchron, die beiden merken ja nichts von teresas Beschleunigung.

    Insofern ist die Situation aus teresas Sicht (und nur aus der) für genau diese rechnung *mathematisch äquivalent* zu einem Aufenthalt in einem homogenen G-Feld, aber auch nicht mehr. Ansonsten würde nämlich Teresa vorhersagen, dass Udines uhr relativ zu Teresas am Ende anders gehen sollte, was ja nicht der Fall ist, wie beide leicht durch Abgleichen von Lichtsignalen sicherstellen können.

    Insgesamt ist meiner Ansicht nach Einsteins Argument damit korrekt, aber unglaublich irreführend.

  274. #274 Anonym_2018
    16. Dezember 2018

    @MartinB #271, Niels #269

    Vielleict kannst dues aber doch noch mal im Detail vorrechnen wie du es meinst?

    O.K. Ich nehme das Specific_example von Wikipedia:

    Consider a space ship traveling from Earth to the nearest star system: a distance d = 4 light years away, at a speed v = 0.8c (i.e., 80 percent of the speed of light).

    Earth perspective = Teresa’s Bezugssystem
    The Earth-based mission control reasons about the journey this way: the round trip will take t = 2d/v = 10 years in Earth time (i.e. everybody on Earth will be 10 years older when the ship returns). The amount of time as measured on the ship’s clocks and the aging of the travelers during their trip will be reduced by the factor
    ϵ = √(1-v²/c²), the reciprocal of the Lorentz factor (time dilation). In this case ε = 0.6 and the travelers will have aged only 0.6 × 10 = 6 years when they return.

    1) Zwillingsparadoxon aus der Perspektive des Bezugssystems von Teresa (=Inertialsystem, idealisiert angenommen): siehe Wikipedia-Zitat oben.

    2) Zwillingsparadoxon aus der Perspektive des Bezugssystems von Serena:

    2a) – gradlinig-glechförmig –
    Aus Sicht dieses Bezugssystems fliegt die Erde mit Teresa gleichmäßig-gradlinig weg. Teresa altert aus Symmetriegründen 0,6 mal so schnell wie Serena. Wenn der Umkehrpunkt erreicht ist, ist Serena 3 Jahre älter als zum Start, Teresa 0,6 x 3 = 1,8 Jahre.

    2b) – beschleunigt –
    Serena dreht das (ruhende) Raumschiff um 180 Grad und zündet das Triebwerk. Das Raumschiff bleibt in Ruhe, weil ein homogenes Gravitationsfeld eingeschaltet wird, das die Kraft des Triebwerks neutralisiert. Das Triebwerk bleibt solange eingeschaltet, bis sich die Geschwindigeit der Erde von 0,8 c in -0,8 c geändert hat, also um 1,6c. Annahme (ohne signifikanten Einfluss auf die Rechnung): für 10 Tage.
    In diesen 10 Tage läuft die Uhr von Teresa um
    ∆t = 1,6c / c² * 4 Lichtjahre = 6,4 Jahre vor (gegenüber 10 Tagen bei Serena).

    2c) – gradlinig-glechförmig –
    Aus Sicht dieses Bezugssystems fliegt die Erde mit Teresa wieder gleichmäßig-gradlinig heran. Teresa altert aus Symmetriegründen 0,6 mal so schnell wie Serena. Wenn die Ankunft erreicht ist, ist Serena 3 Jahre älter als zum Umkehrzeitpunkt, Teresa 0,6 x 3 = 1,8 Jahre.

    Summe der fettgedruckten Jahre: 10 Jahre wird Teresa älter, wie in deren Bezugssystem unter 1).

  275. #275 MartinB
    16. Dezember 2018

    @Anonym
    Das hat sich dann wohl überschnitten, trotzdem danke, dann sind wir uns zumindest bezüglich der Rechnung ja einig.

  276. #276 MartinB
    16. Dezember 2018

    @Anonym+Toni
    Nochmal zum Begriff Äq-Prinzip:
    Freier Fall auf eine Masse zu und homogene Bewegung mit v=const im vakuum sind äquivalent, weil ich mich in beiden Fällen entlang einer Geodäten bewege.
    Beschleunigung im vakuum und statioäres Verweilen auf einer Planetenoberfläche sind äquivalent, weil ich in beiden fällen von einer Geodäten wegbeschleunigt werde.
    Solange ich nur lokale Messungen mache, lassen sich die Situationen nicht unterscheiden (lokal ist die Raumzeit flach und es gilt die SRT).
    Die Situationen sind aber nicht physikalisch identisch, weil die geodäten in einem fall gerade sind, im anderen nicht (die Raumzeit ist gekrümmt). Das lässt sich allerdings nur durch nichtlokale Messungen feststellen (weswegen ja in den riemannschen Krümmungstensor auch die Ableitungen der Metrik eingehen, nicht nur die metrik selbst.)

  277. #277 Toni
    16. Dezember 2018

    @MartinB:

    Ich habe die ganze Zeit mit Beschleunigung, das was Einstein in seinem Paper schrieb, im Sinn gehabt, habe aber die Beschleunigung FÄLSCHLICHERWEISE mit Raumzeit Krümmung gleichgesetzt, weil ich beim Beschleunigten Fahrstuhl einen physikalischen Grund für Biegung des Laserstrahls suchte.

    Offenbar wird durch Beschleunigung im Fahrstuhl ein künstliches “Gravitationsfeld”, das nichts mit der geometrischen Interpretation der Gravitation zu tun, erzeugt (ein Kraftfeld OHNE Quelle). Ich habe diesbezüglich folgende Erklärung gefunden:

    acceleration bends spacetime​, but does not curve it. It’s an important but subtle difference: bent spacetime is still flat; curved spacetime is not.

    oder diese:

    Acceleration is a world line curvature, with respect to geometry of enveloping spacetime.

    Ich möchte noch erwähnen, dass generell Beschleunigungen Störungen in Raumzeit verursachen können, wie Gravitationswellen, die das LIGO Team gemessen hat.

    Da man ZP AUCH im Rahmen ART behandeln kann, dann war meine Formulierung des ZP doch nicht völliger Quatsch:

    Wo die Symmetrie der Gleichwertigkeit der Systeme gebrochen wird, hat man eine Zeitdilatation wegen Beschleunigung (“Gravitationsfeld”—–> nicht im Sinne von Raumzeit Krümmung), und wo nicht, hat man eine Zeitdilatation wegen SRT.

    Aber ok, man kann diese Formulierung unglücklich finden.

  278. #278 MartinB
    16. Dezember 2018

    @Toni
    “acceleration bends spacetime​, but does not curve it. It’s an important but subtle difference: bent spacetime is still flat; curved spacetime is not.”
    Finde ich sehr unglücklich formuliert (und weiß ehrlich gesagt nicht, wie ich das genau verstehen soll – etwas zu biegen, gibt eine extrinische Krümmung, keine intrinsische, aber das ergibt wenig Sinn).

    Unterscheide einfach zwischen Koordinatenbeschleunigungen, die durch die ungeschickte Wahl des KS zu stande kommen, und Beschleunigungen durch Kräfte, damit fährst du besser.

    “Ich möchte noch erwähnen, dass generell Beschleunigungen Störungen in Raumzeit verursachen können, wie Gravitationswellen, die das LIGO Team gemessen hat.”
    Aber nicht jede beschleunigung tut das, und das hat ja auch mit dem ZP nichts zu tun, weil das ZP unabhängig davon ist, welche Masse die Objekte haben.

    Und der Begriff “Symmetriebrechung” wird in der Physik wirklich wirklich wirklich anders verwendet (oder sollte es zumindest, es gibt ja viele schlampige Formulierungen…):
    https://de.wikipedia.org/wiki/Symmetriebrechung

  279. #279 MartinB
    16. Dezember 2018

    PS
    Nicht alles, was bei quora steht, ist verlässlich…

    Und ja, wen man Massen beschleunigt (oder generell bewegt) hat das einen Einfluss auf die Krümmung der Raumzeit – die ist aber wie gesagt abhängig von der Masse, während es für das ZP völlig egal ist, ob du ein Gramm oder 1000 Sonnemassen hast…

  280. #280 Toni
    16. Dezember 2018

    @MartinB:

    Wie erklärt man nun PHYSIKALISCH diese Biegung des Laserstarhls im Fahrstuhl, bist du einverstanden mit wirkung der Beschleunigung als eine Art künstliches “Gravitationsfeld” auf Licht?

    “Und ja, wen man Massen beschleunigt (oder generell bewegt) hat das einen Einfluss auf die Krümmung der Raumzeit – die ist aber wie gesagt abhängig von der Masse, während es für das ZP völlig egal ist, ob du ein Gramm oder 1000 Sonnemassen hast…”

    Ja, ist schon klar. Wollte nur sagen, dass Beschleunigung der Massen dann doch einen Effekt auf Raumzeit auslösen können, so wie bei rotierenden Schwarzen Löchern (Frame Dragging effect).

    “Nicht alles, was bei quora steht, ist verlässlich…”
    Ja, aber da sind auch viele renommierte Wissenschaftler unterwegs, die man woanders schwer erreichen kann.

  281. #281 Niels
    16. Dezember 2018

    @MartinB @Anonym_2018

    Danke nochmals.
    #273 und #276 kann ich dann auch komplett so unterschreiben.
    .

    Mein Knackpunkt lag darin, dass ich nicht verstanden habe, was mit einer “ART-Rechnung” des Zwillingsparadoxons gemeint sein soll.

    Wenn ich alles ineinander einsetze, kommt nämlich immer die SRT-Formel raus.
    Da sind wir uns einig, oder?

    Man kann diese Gleichung auf recht umständliche Weise aus Teresas Sicht als homogenes Gravitationsfeld interpretieren, das sehe ich ein.

    Dann wäre das Ganze aber auch nur eine ART-Interpretation einer SRT-Rechnung, keine ART-Rechnung.
    Richtig?

    Von diesem Streit um den genauen Bedeutungsinhalt der Begriffe Rechnung und Interpretation mal abgesehen:
    Bei mir hat es aber dann außerdem vor allem daran gehakt, dass es den Begriff “homogenes Gravitationsfeld” in der ART schlicht nicht gibt und er auch nicht sinnvoll definiert werden kann.

    Deswegen kann man das Ganze meiner Meinung nach eigentlich nicht einmal eine Interpretation im Rahmen der ART nennen.
    Homogene Gravitationsfelder sind newtonsche Gravitationstheorie, nicht ART.
    ART beschäftigt sich mit Raumzeitkrümmung und diese spielt im Zwillingsparadoxon keine Rolle.

  282. #282 Toni
    16. Dezember 2018

    @MartinB:

    Innerhalb der ART interpretiert man ja Gravitation als Raumzeit Krümmung, nun in quantisierte Feldtheorien, konnte man auch offenbar zumindest einige vorhersagen der ART wie Perihelbewegung von Merkur innerhalb der QFT in einer FLACHEN Raumzeit zu reproduzieren. Hier mal ein Link: https://arxiv.org/abs/gr-qc/9912003

    Es gibt auch eine alte und wenig bekannte Theorie von Julian Schwinger, der Übervater von QFT, names Source Theory, mit man auch solche Rechnungen und Ergebnisse OHNE Krümmung der Raumzeit kriegen konnte. Das quält mich seit graumer Zeit, dass man in QFT offenbar zumindest einige wichtige Ergebnisse der ART ohne jegliche Raumzeit Krümmung erhalten kann. Das wirft zumindest die Frage, ob Raumzeit wirklich gekrümmt ist oder spinnen die Quantenphysiker?

    Würde gerne deine Meinung diesbezüglich wissen.

  283. #283 Anonym_2018
    17. Dezember 2018

    @MartinB #273

    … als seien Serna und Teresa während des Umkehrens tatsächlich in einem Gravitationsfeld. Dass das nicht so sein kann, können wir schon daran sehen, dass wir uns eine dritte beobachterin Udine denken können …

    Das Argument mit Udine zieht meiner Ansicht nach nicht.

    Grund: Nehmen wir an, Teresa, Udine und Serena seien Drillinge. Teresa und Serena trennen sich, als Teresa ihren 20. Geburtstag feiert. Im Bezugssystem von Serena, die noch bei Teresa ist, feiert Udine ihren 20. Geburtstag nur dann zum selben Zeitpunkt wie Teresa den ihren, wenn sie (Serena) relativ zu Teresa ruht, ansonsten zieht die Relativität der Gleichzeitigkeit.
    Falls aber Serena am Ort von Teresa ruht, muss sie für den Start ihrer Reise bereits kurz beschleunigen (richtiger: mit dem Triebwerk auch die Gravitation einschalten). Das lässt Teresa nicht gravitativ altern, weil der Abstand noch nahezu Null ist. Das lässt aber wohl Udine gravitativ altern, weil der Abstand zu ihr beim Start bereits 2 Lichtjahre beträgt. Dann kommt es später noch einmal zu einer Beschleunigung (richtiger: Gravitation) usw …

    Meiner Ansicht nach handelt es sich im Bezugssystem von der Wirkung auf passive Massen um ein Gravitationsfeld. Und dass es nicht sinnvoll ist, relative Effekte als “unecht” zu klassifizieren, hast du schon Toni gesagt.

  284. #284 Anonym_2018
    17. Dezember 2018

    @MartinB #273

    Ergänzung zu #283:

    Ich nehme jetzt mal an, dass Serena vor Beginn der Reise und nach Ende der Reise jeweils bei Teresa ist und relativ zu ihr ruht. Nur dann kann sie feststellen, dass Teresa und die 2 Lichtjahre entfernte ruhende Udine die 20. Geburtstage zum Beginn der Reise und die 30. Gebutrstage zum Ende der Reise gleichzeitig feiern und nicht die Relativität der Gleichzeitigkeit zieht.

    Dann sind 3 Beschleunigungen (die durch Gravitation kompensiert werden) erforderlich, die immer in Richtung Udine gerichtet sind, immer im Abstand von 2 Lichtjahren.

    Die 1. Beschleunigung von 0 auf 0,8 c (Start) lässt Udine um 1,6 Jahre gravitativ altern.

    Die 2. Beschleunigung von 0,8 c auf -0,8 c (Umkehr) lässt Udine um 3,2 Jahre gravitativ altern.

    Die 3. Beschleunigung von -0,8 c auf 0 (Landung) lässt Udine um 1,6 Jahre gravitativ altern.

    Summe: 6,4 Jahre, wie bei Teresa, die wegen der Abstände nur von der 2. Beschleunigung, aber im Abstand von 4 Lichtjahren betroffen ist.

  285. #285 MartinB
    17. Dezember 2018

    @Toni
    “Wie erklärt man nun PHYSIKALISCH diese Biegung des Laserstarhls im Fahrstuhl, bist du einverstanden mit wirkung der Beschleunigung als eine Art künstliches “Gravitationsfeld” auf Licht?”
    Nein. Es ist eine Koordinatenbeschleunigung, die direkt aus der Geodätengleichung folgt. Genauso wie eine gerade Linie in der Ebene in Polarkoordinaten im phi,r-Raum gekrümmt erscheint. Das hat mit Gravitation, Raumkrümmung oder so erst mal nichts zu tun und wäre schließlich schon in der Newtonschen Physik so.

    ” konnte man auch offenbar zumindest einige vorhersagen der ART wie Perihelbewegung von Merkur innerhalb der QFT in einer FLACHEN Raumzeit zu reproduzieren”
    Natürlich kann man die gesamte ART ohne RZK formulieren, das ist die Grundlage in den Büchern von Weinberg oder Feynman und nichts Neues. (Wenn man will, als klassischen Grenzfall einer QFT eines masselosen Spin-2-Teilchens.) Am Ende kommt aber eine Theorie heraus, bei der die Raumzeit gekrümmt erscheint.
    Dazu gab’s hier mal ne Artikelserie: Ist die Raumzeit gekrümmt, siehe die Liste der Serien rechts.

    Mathematisch ist das zunächst mal äquivalent, am Ende kommt aber auch bei dieser Formulierung eine Theorie heraus, bei der alle Messungen als Messungen innerhalb einer gekrümmten Raumzeit interpretiert werden können. Detailliert wie gesagt im Buch von Weinberg oder Feynman erklärt.

    @Anonym
    Aber Serena könnt ja seit ewiger Zeit mit konstanter Geschwindigkeit unterwegs sein, dann funktioniert dein Argument nicht. Du kannst ja nicht einfach eine Zusatzannahme reinbasteln…

    “Und dass es nicht sinnvoll ist, relative Effekte als “unecht” zu klassifizieren, hast du schon Toni gesagt”
    Nur weil es nicht richtig ist, einige Effekte als “unecht” zu kennzeichnen, heißt das nicht, dass es nie richtig ist, Effekte als unecht zu bezeichnen. Serena müsste in der Gravitationsfeld-Analogie auch z.B. vorhersagen, dass Lichtsignale, die Teresa und Udine austauschen, rot- bzw- blauverschoben sind…

    @Niels
    “Dann wäre das Ganze aber auch nur eine ART-Interpretation einer SRT-Rechnung, keine ART-Rechnung.
    Richtig?”
    Ich denke schon.

  286. #286 Anonym_2018
    17. Dezember 2018

    @MartinB #285

    Aber Serena könnt ja seit ewiger Zeit mit konstanter Geschwindigkeit unterwegs sein, dann funktioniert dein Argument nicht. Du kannst ja nicht einfach eine Zusatzannahme reinbasteln…

    Wenn Serena ewige Zeit unterwegs ist, stimmt aber das folgende nicht im Bezugssystem von Serena:

    #273

    Teresas und Udines Uhren gehen die ganze Zeit synchron

    Grund: Die Gleichzeitigkeit im Bezugssystem Teresa/Udine ist nicht-gleichzeitig im dazu bewegten Bezugssystem von Serena (Relativität der Gleichzeitigkeit!).

    Wir betrachten das ja gerade im Bezugssystem von Serena. Man darf nicht in der Argumentation die Bezugssysteme wechseln/mixen.

  287. #287 Anonym_2018
    17. Dezember 2018

    @MartinB #285

    Ergänzung zu #286

    .. und auf dem Rückweg ist das Bezugssystem von Serena in einem anderen Inertialsystem als auf dem Hinweg. Dann vertauscht sich die zeitliche Reihenfolge der “gleichzeitigen” Ereignisse bei Teresa und Udine

  288. #288 MartinB
    17. Dezember 2018

    @Anonym
    Teresas und Udines Uhren gehen aus ihrer Sicht immer synchron, vor und nach der Beschleunigung. Das sollte aus Serenas Sicht unmöglich sein, wenn Teresa eine stärkere Verzögerung erfährt als Udine.
    Natürlich würde alles wieder korrekt rauskommen, wenn Serena bei Teresa wieder anhält – dann bekommt Udine die Extra-Zeitdilatation dazu.
    Trotzdem würde Serena meiner Ansicht nach vorhersagen, dass während ihres Rückflugs Teresa und Udine bei einem Uhrenvergleich unterschiedliche Zeiten messen, aber ich hab’s nicht durchgerechnet, den letztlich ist es egal: Dass man die Beschleunigung allenfalls als rechnerische Äquivalenz über Gravitations-Dilatation erklären kann, haben wir ja hoffentlich geklärt.

  289. #289 Anonym_2018
    17. Dezember 2018

    @MartinB #285

    Serena müsste in der Gravitationsfeld-Analogie auch z.B. vorhersagen, dass Lichtsignale, die Teresa und Udine austauschen, rot- bzw- blauverschoben sind…

    Genau das ist auch wegen der gravitativen Zeitdilatation der Fall. Das ist ein Gravitationsfeld, keine Analogie.

  290. #290 Anonym_2018
    17. Dezember 2018

    @MartinB #288

    Teresas und Udines Uhren gehen aus ihrer Sicht immer synchron, vor und nach der Beschleunigung.

    Das widerspricht bei ewiger Bewegung von Serena nicht dem:

    Das sollte aus Serenas Sicht unmöglich sein, wenn Teresa eine stärkere Verzögerung erfährt als Udine.

    Diese Synchonität im Bezugssystem von Teresa gibt es nicht im Bezugssystem (=nacheinander 2 verschiedenen Inertialsysteme) von Serena. Wegen der Relativität der Gleichzeitigkeit ist so etwas völlig normal.

  291. #291 MartinB
    17. Dezember 2018

    @Anonym_2018
    Ich bin mir trotzdem nicht sicher, ob Serena die richtigen Vorhersagen für einen Signalaustausch zwischen Teresa und Udine macht. Müsste man durchrechnen, ist mir aber nicht wichtig genug weil ich diese Sichtweise des ZP ohnehin ziemlich künstlich und umständlich finde.
    Selbst wenn es richtig rauskommt (was durchaus sein kann): Ja, ist nett, dass man das so rechnen kann, führt aber – wie man ja auch hier sieht – zu endloser Verwirrung und der Idee, dass Beschleunigungen tatsächlich Gravitation und damit ne RZK erzeugen. Niels hat schon recht: es gibt einen Grund, warum diese Rechnung in neueren Büchern nicht drin ist…

  292. #292 Anonym_2018
    17. Dezember 2018

    @MartinB #291

    Laut Äquivalenzprinzip ist das in dem beschleunigten Bezugssystem Gravitation. Auch die ART beruht auf diesem Äquivalenzprinzip. Man nennt die ART ja eine Theorie der Gravitation und nicht eine Theorie der Analogie.

  293. #293 Anonym_2018
    17. Dezember 2018

    @MartinB #285

    Du kannst ja nicht einfach eine Zusatzannahme reinbasteln…

    Das musste ich sogar, um sicherzustellen, dass die Uhren von Teresa und Udine zumindest vor und nach der Reise auch im Bezugssystem von Serena synchron sind.

  294. #294 MartinB
    17. Dezember 2018

    @Anonym
    Nein. Gravitation ist Raumzeitkrümmung, und nur weil jemand beschleunigt, entsteht keine Gravitation und keine Raumzeitkrümmung. Wenn du es nicht glaubst, kannst du den Krümmungstensor in der Rindlermetrik ausrechnen.

    Sonst wäre die SRT für sich allein auch eine inkonsistente Theorie, was nicht der Fall ist. Denk dir einfach ein Universum, in dem Massen nicht die Raumzeit krümmen, da könnte trotzdem die SRT gelten.

    Das Äq-Prinzip sagt nicht dass Beschleunigungen Gravitation *sind*, sondern dass sie (in gewisser Hinsicht, wie oben erklärt) “äquivalent” (d.h. bezüglich *lokaler* Messungen ununterscheidbar) sind. Äquivalenz bedeutet nicht Identität. Siehe #276.

    Geometrisch liegt das schlicht daran, dass jede gekrümmte Mannigfaltigkeit lokal flach ist, so wie auch die Erde lokal flach ist.

  295. #295 Anonym_2018
    17. Dezember 2018

    @MartinB #294

    Gravitation ist Raumzeitkrümmung

    Eine solche Definition von Gravitation habe ich bisher nirgendwo gefunden.

    Laut dem Äquivalenzprinzip könnte man es so sehen, dass Gravitation eine geometrische Ursache hat. Diese geometrische Ursache könnte eine Raumzeitkrümmung sein oder (im Falle eines homogenen Gravitationsfeldes) die Beschleunigung des Bezugssystems.

  296. #296 MartinB
    17. Dezember 2018

    @ANonym
    “Eine solche Definition von Gravitation habe ich bisher nirgendwo gefunden.”
    Schon Wiki sagt:
    “Sie deutet Gravitation als geometrische Eigenschaft der gekrümmten vierdimensionalen Raumzeit.”
    bzw. auf englisch
    “General relativity is the geometric theory of gravitation”

    Selbstverständlich ist Gravitation=RZK; was denn sonst? (Es sei denn, du nimmst die Alternativformulierung über Felder, die landet aber am Ende beim selben Ergebnis, siehe den Kommentar oben.)

    Den zweiten Teil verstehe ich nicht – welches geometrische Maß soll sich durch eine Beschleunigung ändern? Die Rindler-Metrik beschreibt wie gesagt eine flache Raumzeit.

  297. #297 Anonym_2018
    17. Dezember 2018

    @MartinB #296

    “General relativity is the geometric theory of gravitation”

    Die ART kann allerdings nur einen Teil der Gravitationseffekte beschreiben, weil sie gerade kein homogenes Gravitationsfeld unterstüzt. Diesbezüglich ist sie eingeschränkter als z.B. die Newton’sche Gravitationstheorie. Die kann das.

    welches geometrische Maß soll sich durch eine Beschleunigung ändern?

    Ich habe von geometrischer Ursache geschrieben, nicht von einem geometrische Maß. Mit der Beschleunigung wird die Bewegung des Bezugssystems rein geometrisch beschrieben, siehe auch:

    Die Kinematik (…) ist ein Gebiet der Mechanik, das die Bewegung von Körpern rein geometrisch beschreibt mit den Größen Zeit, Ort, Geschwindigkeit und Beschleunigung.

    Quelle:
    https://de.wikipedia.org/wiki/Kinematik

  298. #298 MartinB
    17. Dezember 2018

    @Anonym_2018
    “Die ART kann allerdings nur einen Teil der Gravitationseffekte beschreiben, weil sie gerade kein homogenes Gravitationsfeld unterstüzt”
    Das hast du völlig falsch verstanden: Es kan in unserem Universum kein homogenes G-Feld geben, das ist nicht möglich, ein solches Feld verletzt immer die Einsteingleichung. Dass die Newton-Theorie so etwas annehmen kann, zeigt eben nur, dass sie nicht korrekt ist, weil ein solches Feld schlicht nicht mit der ART und SRT vereinbar wäre.

    Und dass die Kinematik die Bewegung von Körpern geometrisch beschreibt, weiß ich, was hat das mit einer gekrümmten Raumzeit zu tun?

    Langsam wird die Diskussion ein wenig abstrus…

  299. #299 Anonym_2018
    17. Dezember 2018

    @MartinB #298

    Letztlich ist es für die physikalischen Gegebenheiten auch unwichtig, wie man das Wort “Gravitation” formal definiert. Heutzutage scheinen die Physiker das anders zu tun als z.B. vor 100 Jahren A. Einstein. Ob das wirklich eine Verbesserung ist, sei dahingestellt.

  300. #300 Toni
    17. Dezember 2018

    @MartinB

    “Mathematisch ist das zunächst mal äquivalent, am Ende kommt aber auch bei dieser Formulierung eine Theorie heraus, bei der alle Messungen als Messungen innerhalb einer gekrümmten Raumzeit interpretiert werden können. Detailliert wie gesagt im Buch von Weinberg oder Feynman erklärt.”

    Ich dachte immer, dass man in der quantenfeldtheoretische Behandlung der Gravitation die Raumzeitkrümmung irgendwie eliminiert, und man das ganze wie gehabt in einer flachen Raumzeit rechnet.

    Ok, dann wäre ja die quantisierte Theorie der Gravitation nichts anderes als QFT in gekrümmten Raumzeit. Seltsam denn in Stringtheorie habe ich nichts von Raumzeit Krümmung gesehen.

  301. #301 MartinB
    17. Dezember 2018

    @Anonym
    ” Heutzutage scheinen die Physiker das anders zu tun als z.B. vor 100 Jahren A. Einstein”
    Richtig, Einstein war halt in vieler Hinsicht noch Ideen verhaftet, die man heute so einfach nicht mehr nutzt, weil sie eher irreführend sind, wie ja auch diese Diskussion zeigt.

    @Toni
    “Ich dachte immer, dass man in der quantenfeldtheoretische Behandlung der Gravitation die Raumzeitkrümmung irgendwie eliminiert, und man das ganze wie gehabt in einer flachen Raumzeit rechnet.”
    Es ist andersrum: Man startet mit einer flachen Minkowski-Raumzeit und heraus kommen Effekte, die sich von den Auswirkungen einer gekrümmten Raumzeit nicht unterscheiden lassen. Ich verweise mal diskret auf mein demnächst erscheinendes Buch, da steht das auch drin…

  302. #302 Anonym_2018
    17. Dezember 2018

    @MartinB #301

    Einstein war halt in vieler Hinsicht noch Ideen verhaftet, die man heute so einfach nicht mehr nutzt, weil sie eher irreführend sind, wie ja auch diese Diskussion zeigt.

    Einstein hat das Zwillingsparadoxon (u.a.) im Bezugssystem von Serena berechnet. Das ist aus meiner Sicht nicht verwerflich. Ich würde auch kein Problem darin sehen, wenn antiquierte Begriffe von Einstein durch modere ersetzt würden. Dann könnte man eine Übersetzungstabelle erstellen und mit deren Hilfe den antiquierten Text in einen Text mit aktuellen Begriffen übersetzen. Das eigentliche Problem sehe ich nun darin, dass es für einige physikalische Erscheinungen heute gar kein zulässiges Wort mehr zu geben scheint, um die Erscheinung zutreffend zu beschreiben.

    Beispiel:

    Ich habe in den letzten Tage hier Formulierungen verwendet wie etwa “Die gravitative Zeitdilatation bei Teresa beträgt im Bezugssystem von Serena …”.

    Das Adjektiv “gravitativ” vor Zeitdilatation schein heutzutage in diesem Kontext unzulässig oder irreführend zu sein. Mir ist aber kein alternatives besseres Adjektiv eingefallen, welches diese Art von Zeitdilatation zeitgmäß, verständlich und korrekt beschreiben würde. Für Vorschläge dazu bin ich jederzeit offen.

  303. #303 MartinB
    17. Dezember 2018

    @Anonym2018
    “Garivative Zeitdilatation” verwendet man halt am besten für Zeitdilatation in den Fällen, wo die Raumzeit auch tatsächlich gekrümmt ist. Wenn man nur ein beschleunigtes Bezugssystem verwendet, passt der Begriff “gravitativ” halt nicht – so wie auch in der Newtonschen Physik in einer Zentrifuge scheinbar eine Kraft nach außen wirkt, man aber trotzdem nicht von einer Gravitationskraft reden würde, sondern von einer Scheinkraft.

  304. #304 Anonym_2018
    17. Dezember 2018

    passt der Begriff “gravitativ” halt nicht

    Das hatte ich schon so verstanden. Konstruktiver als was nicht passt wäre für mich eine Information, was passen würde in diesem Kontext.

  305. #305 MartinB
    17. Dezember 2018

    @Anonym
    Ja, hab ich verstanden, ich weiß aber nicht, ob es einen Begriff gibt, der genau für diesen fall zugeschnitten ist, weil es eben eine sehr spezielle Betrachtungsweise eines speziellen Phänomens ist.

  306. #306 Anonym_2018
    17. Dezember 2018

    @MartinB #305

    ich weiß aber nicht, ob es einen Begriff gibt, der genau für diesen fall zugeschnitten ist

    O.K. Ich bewerte das dann, falls es keinen Begriff gibt, als eine Rückschritt in der Physik-Forschung in den letzten 100 Jahren. Grund: Einstein hatte damals einen Begriff, der in der damaligen Zeit auf Akzeptanz stieß, und heute, 100 Jahre später, fehlt ein Begriff, der heute auf Akzeptanz stößt.

    Die Folge: Solche interessanten physikalischen Effekte werden unter den Tisch gekehrt und garnicht mehr analysiert, weil und uns die Worte fehlen, das zu beschreiben.

    Und wenn das dann in Lehrbüchern fehlt, ist es plausibel, dass es Irritationen gibt, wenn jemand das unerwartet rauskramt und mit den einzig verfügbaren, den “alten” Worten, beschreibt.

    Trotzdem vielen Dank für die Klarstellung!

  307. #307 MartinB
    17. Dezember 2018

    @Anonym
    Es ist eine spezielle Betrachtngsweise eines ganz bestimmten problems. Allgemein kannst du das sicher mit dem Begriff “Koordinatenbeschleunigung” erfassen, aber es ist nichts Grundlegendes, und einen “Effekt” im eigentlichen Sinn gibt es ja nciht – man kann etwas unter Ausnutzung des Äq-Prinzips analog zu einer gravitativen Dilatation berechnen, schön, aber (wie die Diskussion zeigt) eher irreführend als wichtig.

  308. #308 Toni
    17. Dezember 2018

    @Anonym_2018:
    ich weiß es nicht, ob dich das interessiert, aber hier ein Link zu allen Einsteins Arbeiten, die aus meiner Sicht sehr interessant sind, und auch Zugang zu Einsteins Gedankengänge geben.
    https://myweb.rz.uni-augsburg.de/~eckern/adp/history/Einstein-in-AdP.htm

    Ich finde es auch schade, dass man vieles nicht wieder in den Lehrbüchern findet.

    Ich wußte zum Beispiel bis vor kurzem nicht, dass die Schreibweise und viele mehr, die man in Quantenmechanik verwendet, von Pascal Jordan (ein Doktorand) und Max Born entwickelt wurde, und das VOR Schrödinger. Auch da empfehele ich dir die Original Literatur.

  309. #309 MartinB
    17. Dezember 2018

    @Toni
    “Ich wußte zum Beispiel bis vor kurzem nicht, dass die Schreibweise und viele mehr, die man in Quantenmechanik verwendet, von Pascal Jordan (ein Doktorand) und Max Born entwickelt wurde, und das VOR Schrödinger.”
    Dass Born und jordan da viel gemacht haben, ist eigentlich schon Allgemeinwissen (welche Schreibweise genau meinst du?) – aber man muss in der Physik nicht immer die gesamte Geschichte wiederholen, das ist in meinen Augen etwas, das gerade in populärwissenschaftlciehn Darstellungen gern gemacht wird und oft zur Veriwrrung führt (beispiel etwa die Dirac-See…)

  310. #310 Anonym_2018
    17. Dezember 2018

    @MartinB #307

    einen “Effekt” im eigentlichen Sinn gibt es ja nciht

    Nach dem Relativitätsprinzip darf ich doch ein Serena-Bezugssystem als ruhend betrachten. In diesem System gibt es Effekte, z.B. eine Zeitdilatation zwischen verschiedenene Potentialen des (Schein?-)Gravitationsfeldes. Dieser Anteil der Zeitdilatation von Teresa ist real, wie ein Vergleich mit der Rechnung in deren Bezugssystem zeigt.

    Eine Beschleunigung von Serena gibt es logischerweise im Bezugssystem von Serena nicht.

  311. #311 Toni
    17. Dezember 2018

    @MartinB:

    “Dass Born und jordan da viel gemacht haben, ist eigentlich schon Allgemeinwissen”

    Die algebraische Schreibweise … und die wenigsten wissen, dass sie Matrizenmechanik von Heisenberg (1926) weiterentwickelt haben.

    Die wenigsten kennen Jordan oder Teichmüller …

  312. #312 MartinB
    17. Dezember 2018

    @Anonym_2018
    ” In diesem System gibt es Effekte, z.B. eine Zeitdilatation zwischen verschiedenene Potentialen des (Schein?-)Gravitationsfeldes. ”
    Klar, und das kannst du alles mit den passenden koordinatenbeschleunigungen etc. berechnen, beispielsweise mit Hilfe der Rindler-Metrik. Ist nur nicht besonders erhellend.

    @Toni
    “die wenigsten wissen, dass sie Matrizenmechanik von Heisenberg (1926) weiterentwickelt haben.”
    Also ich habe das soweit ich mich entsinne im Studium gelernt…

  313. #313 Toni
    17. Dezember 2018

    “Also ich habe das soweit ich mich entsinne im Studium gelernt…”

    Ich meine nicht das Lernen der QM, sondern wer es gemacht hat. Von Pascal Jordan sieht man heute noch kaum was. Heisenberg Matrizenmechanik wird nicht wirklich gelehrt, das ist genauso wie bohmsche Mechanik. Davon kriegst du im Studium kein Wind.

  314. #314 MartinB
    17. Dezember 2018

    @Toni
    Man kann Geschichte der Physik studieren, wenn einen das interessiert. Ist aber nicht der Sinn eines Physikstudiums.
    Matritzenmechanik wird natürlich gelehrt, aber mit einem modernen Formalismus. Wir müssen ja auch die Maxwellgleichungen nicht mehr immer in Komponenten ausschreiben, nur weil Maxwell noch keine praktische Kurznotation hatte.

    Ganz ehrlich: Für mich lesen sich diese Beschwerden irgendwo zwischen unnötiger Personenverehrung und “früher war alles besser”. Klar, manchmal ist es hilfreich, zu wissen, wie etwas mal erfunden oder früher dargestellt wurde – oft aber auch nicht.

  315. #315 Anonym_2018
    17. Dezember 2018

    @MartinB #312

    Klar, und das kannst du alles mit den passenden koordinatenbeschleunigungen etc. berechnen

    Das denke ich auch, jetzt habe ich dazu etwas bei Einstein gefunden, was ich nicht beurteilen kann:

    Allerdings können als Realursachen für das Feld nicht die beschleunigten Koordinatensysteme herangezogen werden, welche Meinung ein humorvoller Kritiker mir einmal zuschreiben zu müssen glaubte. Aber es können alle Sterne, welche im Weltall sind, als an der Erzeugung des Gravitationsfeldes beteiligt aufgefaßt werden; denn sie sind während der Beschleunigungsphasen des Koordinatensystem K’ relativ zu letzterem beschleunigt und können dadurch ein Gravitationsfeld induzieren, ähnlich wie beschleunigt bewegte elektrische Ladungen ein elektrisches Feld induzieren. Angenäherte Integration der Gravitationsgleichungen hat in der Tat ergeben, daß derartige Indukfionswirkungen beschleunigt bewegter Massen wirklich auftreten müssen.

    Quelle:
    Paper von Einstein

    Gibt es dafür heutzutage Indizien?

  316. #316 Toni
    17. Dezember 2018

    Anonym_2018
    “Gibt es dafür heutzutage Indizien?”

    Es gab in den 50er Jahren ein Experiment von dem Physiker Burkhard Heim, mit dem er versuchte ein Gravitationsfeld zu erzeugen. Suche nach dem Paper “Prinzip der dynamischen Kontrabarie”.

    Falls es dicht ineterssiert, er hat auch eine Theorie enwickelt, die die ART quantisiert, und damit schafft er einige unglaubliche Vorhersagen zu treffen, wie Vorhersage der Massen der Elementarteilchen OHNE freie Parameter, oder Herleitung von Feinstruktur und elektrische Ladung, etc…

  317. #317 Anonym_2018
    17. Dezember 2018

    @Toni #316

    Bei mir überwiegt die Skepsis, weil ich das für spekulativ halte:

    Burkhard Heims Feldtheorie und die Radionik

    Spiegelbildlich zur materiellen Raumzeit R₄ schließt der R₁₂ nach oben mit dem G₄ Raum ab. Es ist ein rein geistiger Raum, in dem die mathematisch darstellbaren Aspekte der Bewusstseinsaktivitäten der Lebewesen und des Universums abgebildet werden.

    Es wird als Graviphoton bezeichnet und vermittelt die Wechselwirkung zwischen Gravitation und der elektromagnetischen Wechselwirkung (Photonen).

    Quelle:
    https://www.raum-und-zeit.com/naturwissenschaft/hyperraum/

  318. #318 Toni
    17. Dezember 2018

    @Anonym_2018:

    Ich habe von ihm nur seine Feldtheorie gemeint, und das er auch intensiv danach geforscht hat, ein Gravitationsfeld zu erzeugen, nicht was er sonst noch gemacht hat. Seine Ideen sind recht interessant, ob sie richtig sind, sei dahingestellt.

    Wer weiß, vlt gibt es weitaus mehr als uns die moderne Physik sagt.

    “Bei mir überwiegt die Skepsis, weil ich das für spekulativ halte:”

    Ich halte auch Stringtheorie und Quantengravitationstheorien auch für spekulativ. Gerade ST, die nachweislich durch CERN widerlegt wurde, da man keine Supersymmetrie gefunden hat.

  319. #319 MartinB
    18. Dezember 2018

    @Anonym
    Einstein war ja ein starker Anhänger der mach-Idee, dass die Massen im Universum auch für die träge Masse verantwortlich sind, ich denke, daher kommt diese Aussage. Nein, Hinweise gibt es darauf eigentlich keine, auch wenn die Idee prinzipiell natürlich attraktiv ist.

    Und danke für das schöne Zitat:
    “Allerdings können als Realursachen für das Feld nicht die beschleunigten Koordinatensysteme herangezogen werden”

    Raum udn Zeit ist übrigens wirklich keine seriöse Quelle, und spekulative Ideen (wie die von Heim) sind noch keine Physik.

    @Toni
    “Ich halte auch Stringtheorie und Quantengravitationstheorien auch für spekulativ.”
    Ich auch. Aber wie heißt es so schön: “just because there are problems with aircraft design, that doesn’t mean that magic carpets really fly.”

  320. #320 Toni
    18. Dezember 2018

    @MartinB

    “just because there are problems with aircraft design, that doesn’t mean that magic carpets really fly.”

    Das ist zu einfach gesagt. Die Probleme von ST oder QG sind keine Design-Probleme, sie sind fundamentaler Natur. An Supersymmetrie hängt ganz schön viel ab, und ja Vorhersagen lassen auch grüßen. Es wird spannend, wie es in Physik weitergehen wird, ob man noch an ST festhält oder doch ganz neue Ansätze suchen wird.

  321. #321 MartinB
    18. Dezember 2018

    @Toni
    Was ich mit dem Zitat sagen will, ist nur, dass die Tatsache, dass Dinge wie Stringtehorie Probleme haben, kein Beleg ist, dass irgendeine Alternative korrekt ist und kein Grund, sich auf nur unscharf definierte Konzepte wie die von Heim einzulassen.

  322. #322 Toni
    18. Dezember 2018

    @MartinB:

    “Was ich mit dem Zitat sagen will, ist nur, dass die Tatsache, dass Dinge wie Stringtehorie Probleme haben, kein Beleg ist, dass irgendeine Alternative korrekt ist und kein Grund, sich auf nur unscharf definierte Konzepte wie die von Heim einzulassen.”

    Ich habe nie gesagt, Heim hätte recht oder ähnliches. Der Punkt ist, dass ST oder ähnliches viel Forschungsgeld gekostet haben, und die Physik (nicht die Mathematik) nicht wirklich weitergebracht haben. Heim hatte interessante Ideen, zum Bespiel wie er ART quantisiert (ob das so stimmt, weiß ich nicht).
    https://zfn.mpdl.mpg.de/data/Reihe_A/32/ZNA-1977-32a-0233.pdf

    Es ist nun mal so, dass andersdenkende gerade bezüglich ST schon angegriffen werden, siehe Lee Smolin. Die Physik community wird schon von ST-Physiker dominiert, die als Gutachter fast überall zu finden sind. Wehe, wenn man was dagegen sagt.

  323. #323 MartinB
    18. Dezember 2018

    @Toni
    Siehe das kürzlich rezensierte Buch von Sabine Hossenfelder, sicher.
    Heims Ideen habe ich im Detail nicht angeguckt.

  324. #324 Adasaz
    19. Dezember 2018

    Geschafft.

    Die Endlösung der Zwillings-Problematik:

    https://mahag.com/neufor/viewtopic.php?f=6&t=843&p=129555#p129482
    Wie oben schon stand – lasst es einfach mit euren albernen Widerlegungen der SRT… Mal davon abgesehen, dass das Wort “Endlösung” vielleicht auch eins ist, dass man eher nicht benutzen sollte… MartinB

  325. #325 Adasaz
    19. Dezember 2018

    Ach Martin,

    Das reicht jetzt nicht mehr aus (aussperren, diffamieren etc.)
    Es sitzt jetzt. Unwiderlegbar.

  326. #326 MartinB
    19. Dezember 2018

    Aber sicher…

  327. #327 Adasaz
    19. Dezember 2018

    So wie es ausschaut, ist die Sache geritzt.
    Man wird dich ignorieren, man wird dich auslachen, man wird dich aussperren, man wird mit dir kämpfen, und dann gewinnst Du.

  328. #328 MartinB
    19. Dezember 2018

    Aber sicher. Du bist der neue Galilei und all das. Das muss jetzt aber nicht hier diskutiert werden, deinen Link durftest du abladen, wer den Unsinn lesen will, kann das tun, weitere Posts werde ich löschen.

  329. #329 MartinB
    19. Dezember 2018

    Kommentar gelöscht – Sockenpuppen sind hier besonders unerwünscht. Peinlich, wenn man den namen wechseln muss, um so zu tun, als würde irgendwer anders einem zustimmen…

  330. #330 Anonym_2018
    19. Dezember 2018

    @Toni #138

    Die Zeitdilatation durch Beschleunigung (oder Schwerkraft) ist eindeutig. Eine Uhr auf einem Planeten ist langsamer als eine Uhr, die weit entfernt von jeglicher Schwerkraft ist, unabhängig davon, wer die Uhren beobachtet.

    Nein, das ist nicht eindeutig. Aus der Perspektive eines Beobachters, der in der Schwerkraft dieses Planeten senkrecht auf die “verlangsamte” am Boden befindliche Uhr zufällt, hat diese in seinem Bezugssystem keine gravitative Zeitdilatation, mangels Gravitationsfeld. Bei diesem Bezugssystem handelt es sich (lokal) um ein Intertialsystem. In diesem haben die Uhr auf dem Planeten und die weit entfernte Uhr, wo keine Schwerkraft ist, ggf. eine SRT-Zeitdilatation aufgrund der relativen Geschwindigkeiten.

    Eine Zeitdilatation kann in einem Bezugssystem eine ART-Zeitdilatation und in einem anderen Bezugssystem eine SRT-Zeitdilatation sein.

    Eine ART-Zeitdilatation ist nicht “echter” oder “unechter”, als eine SRT-Zeitdilatation.

  331. #331 MartinB
    19. Dezember 2018

    @Anonym_2018
    ” Aus der Perspektive eines Beobachters, der in der Schwerkraft dieses Planeten senkrecht auf die “verlangsamte” am Boden befindliche Uhr zufällt, hat diese in seinem Bezugssystem keine gravitative Zeitdilatation, mangels Gravitationsfeld. ”
    Das halte ich für falsch. Folgendes Gedankenexperiment: Di bist direkt am Ereignishorizont eines SL, ich bin weit entfernt und in Ruhe weil die Raketen meines Raumschiffs mit Volllast laufen, sehe deine Signale also extrem rotverschoben. Jetzt schalte ich die Triebwerke aus. Ich bin ab jetzt im freien Fall, müsste also nach deiner Logik keine Rotverschiebung sehen, obwohl ich genau zu diesem Zeitpunkt noch in Ruhe bin.
    Damit könnte ich sogar ein perpetuum mobile bauen.

    Generell kann ich auch durch jedes Ereignis ein lokales Lorentz-System legen, das im freien Fall ist und in dem ich dasselbe beobachte – für jemanden, der in Ruhe in einem G-Feld ist ist das das System eines nach oben geworfenen frei fallenden Teilchens, das genau jetzt und hier am Umkehrpunkt ist. Für einen Moment sind beide Systeme ununterscheidbar, also kann nicht der eine die Rotverschiebung sehen, der andere nicht.

    Nein, rotverschobene Signale aus einem G-Feld sind rotverschoben – wenn du auf das SL zufällst, siehst du natürlich zusätzlich die Blauverschiebung durch deine Bewegung, aber das muss sich nicht kompensieren und ist ein zusätzliche Effekt.

  332. #332 Anonym_2018
    19. Dezember 2018

    @MartinB #331

    Mein Beispiel meinte ich nur sehr nahe an der Oberfläche des Planten, wo das Schwerefeld nahezu homogen ist.

  333. #333 Toni
    19. Dezember 2018

    @Anonym_2018:

    “Die Zeitdilatation durch Beschleunigung (oder Schwerkraft) ist eindeutig. Eine Uhr auf einem Planeten ist langsamer als eine Uhr, die weit entfernt von jeglicher Schwerkraft ist, unabhängig davon, wer die Uhren beobachtet.”

    Diese Aussage ist korrekt. Das war auch der Grund, warum ich mich so schwer getan habe mit beobachterabhängigkeit der SRT Effekte. Zeitdilatation durch Gravitation ist nicht symmetrisch wie bei SRT für verschiedene Beobachter.

  334. #334 Anonym_2018
    19. Dezember 2018

    rotverschobene Signale aus einem G-Feld sind rotverschoben – wenn du auf das SL zufällst, siehst du natürlich zusätzlich die Blauverschiebung durch deine Bewegung, aber das muss sich nicht kompensieren und ist ein zusätzliche Effekt.

    Ich kann aber das Raumschiff stärker durch eine zusätzlichen Rakentenantrieb beschleunigen, so dass die Rot- und Blauverschiebung sich exakt kompensieren. Falls die Uhr exakt auf dem Ereignishorizont liegt (Frequenz 0 Hz), brauche ich halt theoretische ein unendliche Beschleunigung. Aber es geht ja nur um das theoretische Prinzip.

  335. #335 MartinB
    19. Dezember 2018

    @Anonym #331
    Spielt keine Rolle. Wen ich auf einer Plattfor stehe und neben mir ein Ball hochfliegt, der seinen Scheitelpunkt genau neben mir hat, sind wir für einen Moment im selben Lorentz-System (lokal ist die raumzeit immer flach), also müssen wir auch dasselbe sehen.

    “Ich kann aber das Raumschiff stärker durch eine zusätzlichen Rakentenantrieb beschleunigen, so dass die Rot- und Blauverschiebung sich exakt kompensieren. ”
    Klar kannst du das – aber das ist ja ein Spezialfall, nicht der Allgemeinfall.

    Das Äq-Prinzip in der Form “lokal ist die Raumzeit flach” ist in meinen Augen die beste Argumentationshilfe in solchen Fragen.

  336. #336 Anonym_2018
    19. Dezember 2018

    @MartinB #335

    Wen ich auf einer Plattfor stehe und neben mir ein Ball hochfliegt, der seinen Scheitelpunkt genau neben mir hat, sind wir für einen Moment im selben Lorentz-System (lokal ist die raumzeit immer flach), also müssen wir auch dasselbe sehen.

    Der Ball sieht schon wärend er hochfliegt die gravitave Zeitdilatation der Uhr nicht, weil er auch da schon nach unten beschleunigt wird.

  337. #337 Anonym_2018
    20. Dezember 2018

    @MartinB #335

    Wen ich auf einer Plattfor stehe und neben mir ein Ball hochfliegt, der seinen Scheitelpunkt genau neben mir hat, sind wir für einen Moment im selben Lorentz-System (lokal ist die raumzeit immer flach), also müssen wir auch dasselbe sehen.

    Ergänzung:
    Wen ich auf einer Plattform stehe und neben mir ein Ball hochfliegt, der seinen Scheitelpunkt genau neben mir hat, dann sind der Ball und ich für auch für den Moment in unterschiedlichen Lorentz-Systemen. Ich bin in einem lokal flachen Lorentz-System mit fast homogener Erdgravitation, der Ball ist in einem lokal flachen Lorentz-System ohne resultierende Gravitation, weil die durch die Beschleunigung seines Bezugssystems verursachte homogene (Pseudo?-)Gravitation, die die Erde und den dahinter liegenden Teil des Universums auf ihn zu beschleunigt, und die Erd-Gravitation sich gegenseitig neutralisieren (vom Betrag her gleiche Gravitationskräfte mit entgegengesetzter Richtung). Das ist genau das Resultat des Äquivalenzprinzips.

    Wir sehen unterschiedliche Sachen.

    Aus Sicht des Balls hat eine unter ihm auf dem Boden liegende Uhr keine gravitative Zeitdilatation, mangels resultierendem Gravitationsfeld zwischen ihm und der Uhr. (Zumindest, wenn der Ball nur wenige Meter hochgeflogen ist und nicht 100 km oder so.)
    Ich dagegen sehe auf der Uhr eine gravitative Zeitdilatation proportional zur Höhe der Plattform.

  338. #338 Anonym_2018
    20. Dezember 2018

    @Toni #333

    Diese Aussage ist korrekt

    Dass die Zeitdilatation durch Beschleunigung (oder Schwerkraft) nicht ist eindeutig ist, sieht man auch am Zwillingsparadoxon:

    Die gravitative Zeitdilatation spielt eine Rolle im Bezugssystem von Serena, aber keine Rolle im Bezugssystem von Teresa.

    In beiden Bezugssystemen kann man die resultierende Altersdifferenz der Zwillinge berechnen. Nach dem allgemeinen Relativitätsprinzip sind beide Bezugssysteme gleichberechtigt.

    Ob eine gravitative Zeitdilatation für die resultierende Altersdifferenz eine Rolle spielt oder nicht, ist damit relativ.

  339. #339 Niels
    20. Dezember 2018

    @Anonym_2018

    Wen ich auf einer Plattfor stehe und neben mir ein Ball hochfliegt, der seinen Scheitelpunkt genau neben mir hat, sind wir für einen Moment im selben Lorentz-System (lokal ist die raumzeit immer flach), also müssen wir auch dasselbe sehen

    Wir sehen unterschiedliche Sachen.

    Aus Sicht des Balls hat eine unter ihm auf dem Boden liegende Uhr keine gravitative Zeitdilatation, mangels resultierendem Gravitationsfeld zwischen ihm und der Uhr. (Zumindest, wenn der Ball nur wenige Meter hochgeflogen ist und nicht 100 km oder so.)
    Ich dagegen sehe auf der Uhr eine gravitative Zeitdilatation proportional zur Höhe der Plattform.

    Huch?
    Ball und Plattform ruhen zum Zeitpunkt T auf der Uhr der Plattform.
    Beide befinden sich gerade nebeneinander in 200 Meter über dem Ereignishorizont eines schwarzen Lochs.

    Deiner Meinung nach ist ein Photon, dass jetzt von 1 Meter Höhe über dem Ereignishorizont abgestrahlt wird, für die ruhende Plattform zum Zeitpunkt T rotverschoben.

    Für den ruhenden Ball aber nicht?
    Weil er eine andere Vergangenheit als die Plattform hat?
    Verstehe ich dich da richtig?

    Ich bin in einem lokal flachen Lorentz-System mit fast homogener Erdgravitation, der Ball ist in einem lokal flachen Lorentz-System ohne resultierende Gravitation

    Das ist eine unzulässige Vermischung.
    Für Lorentz-Systeme spielt nur die Geschwindigkeit eine Rolle. Die ist sowohl für Ball als auch Plattform in diesem Moment Null.

    .

    Eine Zeitdilatation kann in einem Bezugssystem eine ART-Zeitdilatation und in einem anderen Bezugssystem eine SRT-Zeitdilatation sein.

    Eine ART-Zeitdilatation ist nicht “echter” oder “unechter”, als eine SRT-Zeitdilatation.

    Das ist keine sinnvolle Unterscheidung.

    Um festzustellen, wie viel Zeit für einen Beobachter zwischen zwei Ereignissen (also zwei Raumzeitpunkten) vergeht, berechnet man in der ART schlicht die für den Beobachter vergangene Eigenzeit zwischen diesen beiden Ereignissen.

    Mit dem Linienelement ds^2 gilt für die Eigenzeit dτ
    dτ^2 = ds^2/c^2
    c ist die Lichtgeschwindigkeit

    Wenn ich für zwei Beobachter den Unterschied in der für sie vergangenen Zeit zwischen Ereignis A und Ereignis B wissen will, berechne ich einfach die Zeitdifferenz
    Δt(1,2)= τ(A,B)-τ(A,B)

    τ1(A,B) ist die vergangene Eigenzeit für Beobachter 1 zwischen Ereignis A und Ereignis B,
    τ2(A,B) für Beobachter 2.
    .

    Um Festzustellen, ob Zeitdilatation zwischen dem Ball und der Plattform vorliegt, nimmt man jetzt einfach das zur passenden Metrik gehörende Linienelement und berechnet Δτ.

    Wenn ich dagegen wissen will, was Beobachter 1 auf einer von Beobachter 2 mitgeführten Uhr sieht, muss ich mir zusätzlich die Lichtlaufzeiten in dieser Raumzeit anschauen.
    .

    Das Ergebnis ist einfach ein ART-Ergebnis. Dass kann man nicht irgendwie sinnvoll in ART und SRT aufspalten.

    Dass die Zeitdilatation durch Beschleunigung (oder Schwerkraft) nicht ist eindeutig ist, sieht man auch am Zwillingsparadoxon:

    Die gravitative Zeitdilatation spielt eine Rolle im Bezugssystem von Serena, aber keine Rolle im Bezugssystem von Teresa.

    Jetzt sind wir wieder ganz, ganz am Anfang angekommen…

    Es gibt keine Zeitdilatation durch Beschleunigung.
    Siehe ab MartinB #123.

    Nach dem allgemeinen Relativitätsprinzip sind beide Bezugssysteme gleichberechtigt.

    Nach dem allgemeinen Relativitätsprinzip sind grundsätzlich alle Bezugssysteme gleichberechtigt.

    In beiden Bezugssystemen kann man die resultierende Altersdifferenz der Zwillinge berechnen.

    Das Relativitätsprinzip besagt, dass man grundsätzlich immer alles in allen Bezugssystemen berechnen kann.

    Auch schon in der SRT. Auch dort gibt es schon ein Relativitätsprinzip.

    Das hat nichts mit dem Äquivalenzprinzip zu tun.

    Eine resultierende Altersdifferenz kann doch außerdem offensichtlich niemals einen beobachterabhängigen Effekt darstellen?

    Ob eine gravitative Zeitdilatation für die resultierende Altersdifferenz eine Rolle spielt oder nicht, ist damit relativ.

    Nein. Nach der der Definition von gravitativer Zeitdilatation nicht.

    Laut wiki etwa:
    Gravitational time dilation is a form of time dilation, an actual difference of elapsed time between two events as measured by observers situated at varying distances from a gravitating mass.

    Beim Zwillingsparadoxon gibt es keine gravitierende Masse und daher auch keine gravitative Zeitdilatation.

  340. #340 Anonym_2018
    20. Dezember 2018

    @Niels #340

    Beide befinden sich gerade nebeneinander in 200 Meter über dem Ereignishorizont eines schwarzen Lochs.

    So wie ich MartinB verstanden habe, befinden sich Plattform und Ball ca. 5 Meter über dem Boden einer Turnhalle auf der Erde. Die Uhr liegt am Boden unter dem Ball. Jedenfalls ist meine Voraussetzung, dass in der Turnhalle das Graviationsfeld der Erde nahezu homogen ist.

    Deiner Meinung nach ist ein Photon, dass jetzt von 1 Meter Höhe über dem Ereignishorizont abgestrahlt wird, für die ruhende Plattform zum Zeitpunkt T rotverschoben.
    Für den ruhenden Ball aber nicht?

    Meiner Meinung nach ist ein Photon, das von der LED-Anzeige der Uhr abgestrahlt wird, für die Plattform rotverschoben, für den im Scheitelpunkt befindlichen Ball aber nicht. Grund: Für den Ball gilt:

    Das einsteinsche starke Äquivalenzprinzip besagt, dass ein Beobachter in einem geschlossenen Labor ohne Wechselwirkung mit der Umgebung durch überhaupt kein Experiment feststellen kann, ob er sich in der Schwerelosigkeit fernab von Massen befindet oder im freien Fall nahe einer Masse.

    Quelle:
    https://de.wikipedia.org/wiki/%C3%84quivalenzprinzip_(Physik)#%C3%84quivalenzprinzip_in_der_allgemeinen_Relativit%C3%A4tstheorie

    Ob das bei einem schwarzen Loch bei 200m die Homogenität näherungsweise erfüllt ist, weiss ich nicht.

    Das ist eine unzulässige Vermischung.
    Für Lorentz-Systeme spielt nur die Geschwindigkeit eine Rolle. Die ist sowohl für Ball als auch Plattform in diesem Moment Null.

    Nein. Für das Lorentz-System spielt auch eine Rolle, ob es ein Inertialsystem ist.

    Das ist keine sinnvolle Unterscheidung.

    Das unterscheide ich, weil bei der Darstellung des Zwillingsparadoxons meistens nur im Bezugssystem von Teresa und mit SRT gerechnet wird.

    Jetzt sind wir wieder ganz, ganz am Anfang angekommen…
    Es gibt keine Zeitdilatation durch Beschleunigung.
    Siehe ab MartinB #123.

    Das ist zwar richtig, es gibt aber in einem beschleunigten Bezugssystem eine äquivalente (Pseudo?-)Gravitation mit einer gravitativen Zeitdilatation. Beispiel: ein im Weltraum nach oben beschleunigter Aufzug. Für dessen Insassen ist zwischen Decke und Boden eine solche Zeitdilatation messbar.

    Eine resultierende Altersdifferenz kann doch außerdem offensichtlich niemals einen beobachterabhängigen Effekt darstellen?

    Zumindest nicht, wenn die Zwillinge zum Zeitpunkt des Altersvergleichs z.B. am selben Ort sind.

    Beim Zwillingsparadoxon gibt es keine gravitierende Masse und daher auch keine gravitative Zeitdilatation.

    Ich habe mich nicht an die Definition von Wikipedia sondern an die von A. Einstein 1918 gehalten. Hast du ein besseres Adjektiv als “gravitativ” für die Zeitdilatation zwischen Decke und Boden eines im Weltall nach oben beschleunigten Aufzuges?

  341. #341 MartinB
    20. Dezember 2018

    @Anonym_2018
    ” dann sind der Ball und ich für auch für den Moment in unterschiedlichen Lorentz-Systemen. Ich bin in einem lokal flachen Lorentz-System mit fast homogener Erdgravitation, der Ball ist in einem lokal flachen Lorentz-System ohne resultierende Gravitation, weil die durch die Beschleunigung seines Bezugssystems verursachte homogene (Pseudo?-)Gravitation, die die Erde und den dahinter liegenden Teil des Universums auf ihn zu beschleunigt, und die Erd-Gravitation sich gegenseitig neutralisieren (vom Betrag her gleiche Gravitationskräfte mit entgegengesetzter Richtung). Das ist genau das Resultat des Äquivalenzprinzips.”
    Nein. Du hast das Äq-Prinzip bzw. die Aussage “lokal ist die Raumzeit flach” nicht verstanden, wenn du das glaubst.
    Die Weltlinien der beiden Beobachter sind für einen Moment identisch, sie berühren sich (die eine ist parabelförmig, die andere ist gerade (z.B. in ruhenden Erdkoordinaten)), also sind sie im betrachteten Moment identisch bezüglich Ort und Richtung (aber nicht bezüglich 2. Ableitung=Beschleunigung). Jetzt schlägt das Äq-Prinzip zu, nach dem in einer hinreichend kleinen Umgebung eines Punktes auf der Weltlinie die Raumzeit lokal flach ist, oder anders gesagt, Beschleunigungen keine Rolle spielen. (Die führen zu Abweichungen von Geodäten, das merkt man aber erst in zweiter Ordnung, nicht in erster.) Die Logik ist exakt dieselbe wie bei einer gekrümmten Fläche im Raum: Lokal darf ich auf der Erde so tun, als wäre die Erdoberfläche flach, Abweichungen merke ich erst, wenn ich hinreichend große Strecken gehe. Das ist der Kern der ART.

    “Aus Sicht des Balls hat eine unter ihm auf dem Boden liegende Uhr keine gravitative Zeitdilatation, mangels resultierendem Gravitationsfeld zwischen ihm und der Uhr. ”
    Wir haben also zwei Beobachter, die sich am selben Ort mit derselben Geschwindigkeit befinden und unterschiedliche Sachen sehen? Das widerspricht dem Äq-Prinzip.

    Niels hat ja alles weitere schon ausführlich erklärt, aber hier unterliegst du wirklich einem Denkfehler.

    “Ich habe mich nicht an die Definition von Wikipedia sondern an die von A. Einstein 1918 gehalten.”
    Dass Einsteins Nomenklatur irreführend und teilweise veraltet ist und aus gutem Grund nicht mehr so verwendet wird, haben wir aber oben schon bis zum Abwinken diskutiert.

    “Meiner Meinung nach ist ein Photon, das von der LED-Anzeige der Uhr abgestrahlt wird, für die Plattform rotverschoben, für den im Scheitelpunkt befindlichen Ball aber nich”
    Toll, dan lass uns ein perpetuum mobile konstruieren: Die Uhr absorbiert das Photon, zieht damit eine Feder auf. Da Uhr und Plattform im Moment relativ zueinander in Ruhe sind, kann ich die Feder kräftefrei auf die Plattform schieben. Der Beobachter dort nimmt einen Teil der Energie der Feder, erzeugt damit ein rotes Photon, schickt das nach unten (wo es ein blaues Photon wird), so dass unten netto keine Arbeit geleistet wird. Der Beobachter auf der Plattform hat jetzt aber Energie übrig, weil die Feder mit einem blauen Photon gespannt wurde. Und nein, das Gegenargument lautet nicht, dass in der ART die Energie nicht zwangsläufig erhalten ist.

    ” Beispiel: ein im Weltraum nach oben beschleunigter Aufzug. Für dessen Insassen ist zwischen Decke und Boden eine solche Zeitdilatation messbar.”
    Aber das ist außer in Einsteins veralteter (und wie man an dieser Diskussion sieht extrem irreführender) Sprechweise kein gravitationsfeld, weil wir uns in einer flachen Raumzeit befinden (Rindler-Metrik).

  342. #342 MartinB
    20. Dezember 2018

    PS
    Alleind ie Formulierung
    “in einem lokal flachen Lorentz-System mit fast homogener Erdgravitation” ist unsinnig: in einem lokal flachen Lorentzsystem kann es keine Gravitation geben, dann ist es ja nicht mehr flach…

  343. #343 Anonym_2018
    20. Dezember 2018

    @MartinB #342

    in einem lokal flachen Lorentzsystem kann es keine Gravitation geben, dann ist es ja nicht mehr flach…

    Das widerspricht aber folgender Aussage, von da habe ich das nur übernommen:

    Wen ich auf einer Plattfor stehe und neben mir ein Ball hochfliegt, der seinen Scheitelpunkt genau neben mir hat, sind wir für einen Moment im selben Lorentz-System (lokal ist die raumzeit immer flach), also müssen wir auch dasselbe sehen.

  344. #344 MartinB
    20. Dezember 2018

    @Anonym_2018
    Nein, das widerspricht sich nicht, das ist genau der Kern der Aussage.

  345. #345 Anonym_2018
    20. Dezember 2018

    @MartinB #341

    (aber nicht bezüglich 2. Ableitung=Beschleunigung). Jetzt schlägt das Äq-Prinzip zu, nach dem in einer hinreichend kleinen Umgebung eines Punktes auf der Weltlinie die Raumzeit lokal flach ist, oder anders gesagt, Beschleunigungen keine Rolle spielen.

    Wie kann das System der Plattform lokal flach sein, wenn dort die Erdgravitation wirkt? Aus meiner Sicht spielt die 2. Ableitung=Beschleunigung die entscheidende Rolle.

    Wir haben also zwei Beobachter, die sich am selben Ort mit derselben Geschwindigkeit befinden und unterschiedliche Sachen sehen? Das widerspricht dem Äq-Prinzip.

    Nein, wie gesagt, aus meiner Sicht spielt die 2. Ableitung=Beschleunigung die entscheidende Rolle.

    Toll, dan lass uns ein perpetuum mobile konstruieren

    Leider funktioniert das nicht. Wenn man die Feler aufzieht, ist danach Energie “E” in ihr gespeichert. D.h. dann hat sie mindestens eine Masse von E/c².

    Da Uhr und Plattform im Moment relativ zueinander in Ruhe sind, kann ich die Feder kräftefrei auf die Plattform schieben.

    Eine Masse von E/c² kann man im Gravitationsfeld der Erde nicht kräftefrei nach oben schieben.

  346. #346 Anonym_2018
    20. Dezember 2018

    Aber das ist außer in Einsteins veralteter (und wie man an dieser Diskussion sieht extrem irreführender) Sprechweise kein gravitationsfeld, weil wir uns in einer flachen Raumzeit befinden (Rindler-Metrik).

    Die Rindler-Metrik ist aber nur pseudo-flach, weil sie eine x-Koordinaten-abhängige Vakuum-Lichtgeschwindigkeit hat. Da, wo normalerweise “c” steht, steht in Rindler-Koordinaten “α * x”:

    ds² = – (α x)² dt² + dx² + dy² + dz²

    Quelle:
    https://en.wikipedia.org/wiki/Rindler_coordinates#Variants_of_transformation_formulas

  347. #347 MartinB
    20. Dezember 2018

    @Anonym
    ” Aus meiner Sicht spielt die 2. Ableitung=Beschleunigung die entscheidende Rolle.”
    Ja, für Effekte zweiter Ordnung (Krümmungseffekte), aber nicht im lokalen Lorentz-System.
    Wenn du das nicht siehst, dann hast du leider was Grundlegendes noch nicht verstanden, sorry.

    “Leider funktioniert das nicht. Wenn man die Feler aufzieht, ist danach Energie “E” in ihr gespeichert. D.h. dann hat sie mindestens eine Masse von E/c². ”
    Na und? Gespeicherte Energie ist gespeicherte Energie. Das blaue Photon oben hat ja auch ne höhere effektive Masse als das rote. Es kann nicht sein, dass ein beobachter ein rotes Photon absorbiert (also weniger Energie gewinnt) und der andere ein blaues (also mehr energie gewinnt) wenn es dasselbe Photon ist und die beiden Beobachter momentan in Ruhe sind.

    “Die Rindler-Metrik ist aber nur pseudo-flach”
    Was meinst du mit “pseudo-flach” – der RKT ist Null, oder nicht, also ist die Raumzeit flach.

  348. #348 Anonym_2018
    20. Dezember 2018

    @MartinB #347

    Ja, für Effekte zweiter Ordnung (Krümmungseffekte), aber nicht im lokalen Lorentz-System.

    Wenn mit “lokalem Lorentz-System” ein ungekrümmtes System gemeint ist, ändere ich die Formulierung:

    Das Bezugssystem der Plattform ist kein lokal flaches System, weil es das Erd-Gravitationsfeld spürt.

    Das Bezugssystem des Balls ist ein lokal flaches System, weil es sich im nahezu homogenen Erd-Gravitationsfeld auf einer Geodäte bewegt, und so in diesem Bezugssystem das Erd-gravitationsfeld nicht zu spüren ist.

    Es kann nicht sein, dass ein beobachter ein rotes Photon absorbiert (also weniger Energie gewinnt) und der andere ein blaues (also mehr energie gewinnt) wenn es dasselbe Photon ist und die beiden Beobachter momentan in Ruhe sind.

    Doch, das kann sein, weil man die Welt nur aus der Perspektive des einen Bezugssystems (nicht flach) oder aus der des anderen Bezugssystems (flach) beschreiben darf. Man darf aber in einer Argumentationskette nicht beide Bezugssysteme mixen.

    Was meinst du mit “pseudo-flach” – der RKT ist Null, oder nicht, also ist die Raumzeit flach.

    Sie ist nur flach in einer nur für Rechenvereinfachungen geschaffenen merkwürdigen Metrik. Wenn Menschen in dem im Weltraum nach oben beschleunigten Aufzug sitzen, halten sie den Aufzug für ruhend in einem Gravitationsfeld und sehen zwischen Boden und Decke eine Rot-/Blauverschiebung, weil sie intuitiv eine normale Metrik anwenden.

  349. #349 Toni
    20. Dezember 2018

    @Anonym_2018 #338

    “Die gravitative Zeitdilatation spielt eine Rolle im Bezugssystem von Serena, aber keine Rolle im Bezugssystem von Teresa.”

    Eben, der Effekt der “gravitative” Zeitdilatation ist eindeutig, und beobachterUNabhängig. Der Beobachter ganz weit weg im All in einer gravitationsfreien Umgebung sieht, dass die Zeit für einen anderen Beobachter nahe SL langsamer geht, und der Beobachter nahe SL, sieht, dass die Zeit für den Beobachter im unendlichen schneller läuft als seine eigene. Das war gemeint mit Eindeutigkeit und Beobachterunabhängigkeit. Dagegen in SRT jeder sieht die Zeit für den anderen langsamer laufen, und somit geht die Objektivität der Beurteilung verloren. Vielleicht verstehst du unter eindeutig was anderes.

    “Wenn Menschen in dem im Weltraum nach oben beschleunigten Aufzug sitzen, halten sie den Aufzug für ruhend in einem Gravitationsfeld und sehen zwischen Boden und Decke eine Rot-/Blauverschiebung, weil sie intuitiv eine normale Metrik anwenden.”

    Für beschleunigte Systeme nimmt man keine normale Metrik, sondern eine koordinatenabhängige Metrik g_mu_nu (x), was es NICHT heißt, dass die Raumzeit im beschleunigten System gekrümmt sein muss. Im beschleunigten System muss man die lokale flache Metrik durch eine flache koordinatenabhängige Metrik ersetzen.

  350. #350 MartinB
    20. Dezember 2018

    @Anonym_2018
    “Das Bezugssystem der Plattform ist kein lokal flaches System, weil es das Erd-Gravitationsfeld spürt.”
    Das ist so leider nicht richtig, es wird auch durch beliebig häufiges Wiederholen in leicht veränderter Formulierung nicht wahrer.
    ” Man darf aber in einer Argumentationskette nicht beide Bezugssysteme mixen.”
    Muss man auch nicht. Ein Beobachter speichert die Energie, reicht sie an den anderen weiter – fertig. Was passiert denn z.B., wenn der Beobachter im Parabelflug nen Spiegel aufstellt und das für ihn blaue Photon horizontal rüberreflektiert? Und was passiert, wenn vor den beiden Beobachter ein Blaufilter ist – dann kommt das Photon durch oder nicht, je nachdem, wer nachguckt? So relativ kann auch die RT nicht sein.

    “Sie ist nur flach in einer nur für Rechenvereinfachungen geschaffenen merkwürdigen Metrik”
    Ob die Raumzeit flach ist oder nicht, hängt nicht von der Wahl der Koordinaten ab. Wenn es ein KS gibt, in dem der RKT verschwindet, dann tut er das in allen KS.

    Du würfelst KS, Metrik und RKT etwas durcheinander.

  351. #351 Anonym_2018
    20. Dezember 2018

    @MartinB #350

    Das ist so leider nicht richtig

    Was ist den richtig bzgl. des Bezugssystems der Plattform (flach oder nicht-flach? – Gravitation vorhanden oder nicht?)

    ” Man darf aber in einer Argumentationskette nicht beide Bezugssysteme mixen.”
    Muss man auch nicht. Ein Beobachter speichert die Energie, reicht sie an den anderen weiter – fertig.

    Man muss aufpassen, wenn man von “Beobachter” spricht. Man muss unterscheiden, ob man diese Person meint oder das Bezugssystem dieser Person.
    Die Energie, die weitergereicht wird oder die Frequenz, die von einem “Blau”-Filter durchgelassen wird, können in einem Bezugssystem anders sein als in dem anderen.

  352. #352 MartinB
    20. Dezember 2018

    @Anonym_2018
    Man kann *immer* ein lokal (räumlich und zeitlich begrenzt) flaches System finden, in dem die RZK beliebig klein ist und entsprechend keinen Einfluss hat. Dass ist der Kern der Aussage, dass die Gravitation durch die Krümmung der Raumzeit bestimmt wird und der Kern des Äq-Prinzips. (Siehe das Buch von Weinberg, wo er aus dem Äq-Prinzip das Konzept der RZK ableitet.)

    “die Frequenz, die von einem “Blau”-Filter durchgelassen wird, können in einem Bezugssystem anders sein als in dem anderen.”
    Nein. Wenn hinter dem Filter ein Schalter ist, der die Erde explodieren lässt und der ausgelöst wird, wenn ein Photon durchkommt, dann kann das wohl kaum beobachterabhängig sein. Und dass die Frequenz eines Farbfilters von der Schwerebeschleunigung abhängt, ist schlicht nicht korrekt.

  353. #353 Anonym_2018
    20. Dezember 2018

    @MartinB #352

    Man kann *immer* ein lokal (räumlich und zeitlich begrenzt) flaches System finden, in dem die RZK beliebig klein ist und entsprechend keinen Einfluss hat.

    Gilt das nur für Bezugssysteme, die sich auf Geodäten bewegen (z.B. Ball), oder auch für Bezugssysteme, die durch Einwirkung von mechanischer Kraft von den Geodäten weggedrückt werden (z.B. Plattform)?

    Hintergrund der Frage ist der folgende Text:

    Ein Teil der Gravitation ist ein Artefakt, abhängig vom Bezugssystem des jeweiligen Beobachters. Dieser Teil der Gravitation lässt sich zum Verschwinden bringen, wenn man zu einem frei fallenden Bezugssystem übergeht. Was wir hier auf der Erde an Gravitationseffekten beobachten können – im wesentlichen: Körper, die zu Boden fallen – gehört zu dieser Art von “relativer Gravitation”. Der Rest an Gravitation, man könnte ihn “intrinsische Gravitation” nennen, macht sich durch Gezeitenkräfte bemerkbar und hängt direkt mit einer ganz bestimmten geometrischen Eigenschaft zusammen: der Krümmung der Raumzeit.

    Quelle:
    https://www.einstein-online.info/vertiefung/FahrstuhlKruemmung@set_language=de.html

    Danach wäre ein Bezugssystem, das nur ein homogenes Gravitationsfeld enthält, “flach”.

  354. #354 MartinB
    20. Dezember 2018

    @Anonym
    “oder auch für Bezugssysteme, die durch Einwirkung von mechanischer Kraft von den Geodäten weggedrückt werden (z.B. Plattform)?”
    Das gilt immer. Weil das weggedrücktwerden von der geodäten ja immer ein Effekt 2. Ordnung ist.

    “. Der Rest an Gravitation, man könnte ihn “intrinsische Gravitation” nennen, macht sich durch Gezeitenkräfte bemerkbar und hängt direkt mit einer ganz bestimmten geometrischen Eigenschaft zusammen: der Krümmung der Raumzeit.”
    Das ist genau was ich die ganze Zeit sage.

    “Danach wäre ein Bezugssystem, das nur ein homogenes Gravitationsfeld enthält, “flach”.”
    Es gibt aber kein homogenes Gravitationsfeld in der ART.

  355. #355 MartinB
    20. Dezember 2018

    PS
    Und natürlich ist auch in einem (näherungsweise) homogenen Gravitatonsfeld die Raumzeit gekrümmt, siehe z.B. die Konstruktion eines “Vierecks” in den Feynman Lectures II-42.

  356. #356 Niels
    20. Dezember 2018

    @Anonym_2018

    Man muss aufpassen, wenn man von “Beobachter” spricht. Man muss unterscheiden, ob man diese Person meint oder das Bezugssystem dieser Person.

    Nach der in der Relativitätstheorie verwendeten Definition von Bezugssystem kann man hier nicht sinnvoll unterscheiden.

    Nein. Für das Lorentz-System spielt auch eine Rolle, ob es ein Inertialsystem ist.

    Wie definierst du Lorentz-System?
    Nach der üblichen Definition müssen Lorentz-Systeme Inertialsysteme sein.

    Danach wäre ein Bezugssystem, das nur ein homogenes Gravitationsfeld enthält, “flach”.

    Nein.
    Es gibt keine wissenschaftliche Theorie, die Aussagen zur Krümmung der Raumzeit macht und in der man gleichzeitig sinnvoll den Begriff Gravitationsfeld verwenden kann. Homogenes Gravitationsfeld schon gar nicht.

    Wenn du homogene Gravitationsfelder willst, musst du die newtonsche Gravitationstheorie verwenden.
    Dann musst du auf Raumzeit und Krümmung verzichten.

    Wenn du Raumzeit und Krümmung willst, fallen Gravitationsfelder und Potentiale weg.
    Diese werden ja gerade durch das Konzept der gekrümmten Raumzeit ersetzt.

    Da kann man nicht einfach beides gleichzeitig verwenden.

    Wie kann das System der Plattform lokal flach sein, wenn dort die Erdgravitation wirkt?

    Das ist die Aussage des Äquivalenzprinzip!
    Jedes System ist lokal flach, wenn man unter lokal einen genügend kleinen Bereich versteht.

    Das “geschlossene Labor” aus deiner Formulierung des Äquivalenzprinzip muss immer als klein genug angenommen werden.
    Auf der Erdoberfläche kann das Labor kilometergroß sein, auf dem Neutronenstern eben nur noch metergroß.

    Doch, das kann sein, weil man die Welt nur aus der Perspektive des einen Bezugssystems (nicht flach) oder aus der des anderen Bezugssystems (flach) beschreiben darf.

    Ob die Raumzeit flach ist, ist keine Frage der Perspektive!
    Das kann man messen.
    Als einfaches Beispiel male ich ein Dreieck und schaue mir die Winkelsumme im Dreieck an. Wenn da etwas anderes als 180 rauskommt, ist der Raum gekrümmt.

    Sie ist nur flach in einer nur für Rechenvereinfachungen geschaffenen merkwürdigen Metrik.

    Nein.
    Die Krümmung ist beobachterunabhängig. Deswegen wird sie mit Hilfe von Tensoren angegeben.
    Der ganze Witz bei Tensoren ist, dass sie koordinatenunabhängig sind.
    Jeder Beobachter in jedem Bezugssystem berechnet daher in jedem Koordinatensystem den selben Krümmungstensor.

    Wenn Menschen in dem im Weltraum nach oben beschleunigten Aufzug sitzen, halten sie den Aufzug für ruhend in einem Gravitationsfeld und sehen zwischen Boden und Decke eine Rot-/Blauverschiebung, weil sie intuitiv eine normale Metrik anwenden.

    Richtig.
    Intiuitiv denkt man in Scheinkräften wie etwa auch der Zentrifugalkraft und der Corioliskraft.
    Sind das dann auch ART-Effekte?

    Braucht man dann für die Erklärung, was der Beobachter in der Achterbahn wahrnimmt, ebenfalls das Äquivalenzprinzip und die ART?

    Das einsteinsche starke Äquivalenzprinzip besagt, dass ein Beobachter in einem geschlossenen Labor ohne Wechselwirkung mit der Umgebung durch überhaupt kein Experiment feststellen kann, ob er sich in der Schwerelosigkeit fernab von Massen befindet oder im freien Fall nahe einer Masse.

    Ob das bei einem schwarzen Loch bei 200m die Homogenität näherungsweise erfüllt ist, weiss ich nicht.

    In dieser Formulierung des Äquivalenzprinzips kommt Homogenität aber doch nirgends vor, oder?
    (Wäre auch schlimm wenn doch, schließlich gibt in in der Natur so etwas wie homogene Gravitationsfelder gar nicht.)

    Das Äquivalenzprinzips gilt also natürlich sowohl am Ereignishorizont eines schwarzen Loches als auch auf dem Mond.

    Das unterscheide ich, weil bei der Darstellung des Zwillingsparadoxons meistens nur im Bezugssystem von Teresa und mit SRT gerechnet wird.

    Das Zwillingsparadoxons kann man in allen denkbaren Bezugssystemen berechnen.

    Da keine gravitierenden Massen auftauchen, ist die SRT-Rechnung buchstäblich identisch zur ART-Rechnung.

    Wenn man mit irgendwelchen Gravitationspotentialen und homogenen Gravitationsfeldern rechnet, ist das keine ART-Rechnung.
    Es ist eine Vermischung von Newtonscher Gravitationstheorie und SRT unter zusätzlicher Annahme der Gültigkeit des Äquivalenzprinzips .

    Das ist zwar richtig, es gibt aber in einem beschleunigten Bezugssystem eine äquivalente (Pseudo?-)Gravitation mit einer gravitativen Zeitdilatation. Beispiel: ein im Weltraum nach oben beschleunigter Aufzug. Für dessen Insassen ist zwischen Decke und Boden eine solche Zeitdilatation messbar.

    Nehmen wir einfach mal an, gravitierende Massen würden die Raumzeit nicht krümmen.
    Das Einsteinsche Äquivalenzprinzip gilt in diesem Universum nicht.

    Für die Gültigkeit der SRT ist die Gültigkeit des Einsteinschen Äquivalenzprinzip keine Voraussetzung.

    In diesem Universum gilt die SRT.

    Dieses Universum kann man über das Zwillingsparadoxon, über die Zeitdilatation und über alle anderen Effekte der SRT nicht von unserem Universum unterscheiden.

    In diesem Universum kann man diese Effekte dann aber offensichtlich nicht mit Hilfe von homogenen Gravitationsfeldern, Gravitationspotentialen und dem Einsteinschen Äquivalenzprinzip interpretieren, oder?
    Das Einsteinsche Äquivalenzprinzip ist ja wie gesagt ungültig.
    .

    Ergibt es Sinn, dass du in unserem Universum Effekte beim Zwillingsparadoxon ja nach Bezugssystem einmal als ART-Effekt und einmal als SRT-Effekt ansiehst?

    Wenn du im anderen Universum dieselben Effekte für alle Bezugssysteme als reine SRT-Effekte ansehen musst?
    Dort gibt es schließlich gar keine ART-Effekte.

    Wobei beide Universen wie gesagt in allen Fällen zum selben Ergebnis kommen.

    Kann die Einstufung als ART-Effekte in unserem Universum dann wirklich sinnvoll sein?

  357. #357 Selina
    20. Dezember 2018

    #276
    “weil ich in beiden fällen von einer Geodäten wegbeschleunigt werde.”

    Sorry für die Störung.
    Was bedeutet hier das Wegbeschleunigen entlang einer Geodäte? War es nicht so, dass die Massen entlang einer Geodäte kräftefrei unterwegs sind?

  358. #358 Anonym_2018
    20. Dezember 2018

    @Niels #356

    Nach der in der Relativitätstheorie verwendeten Definition von Bezugssystem kann man hier nicht sinnvoll unterscheiden.

    Aus meiner Sicht muss man Person und Bezugssystem unterscheiden. Man muss immer sagen über welche Person man reden und zusätzlich, in welchem Bezugssystem man über diese Person redet.

    Beispiele Zwillingsparadoxon (siehe Berechnung #274):

    • Im Bezugssystem von Teresa altert Teresa gleichmäßig um 10 Jahre (und Serena gleichmäßig um 6 Jahre).
    • Im Bezugssystem von Serena altert Teresa ungleichmäßig um 10 Jahre.
    (In der 1. Flugphase altert sie in diesem Bezugssystem langsamer als Serena, in der 2. Flugphase =Umkehr altert sie schneller als Serena und in der 3. Flugphase altert sie wieder langsamer als Serena.)

    In dieser Formulierung des Äquivalenzprinzips kommt Homogenität aber doch nirgends vor, oder?

    Doch, sie kommt in dieser Formulierung indirekt vor, durch das Wort “nahe”:

    … durch überhaupt kein Experiment feststellen kann, ob er sich in der Schwerelosigkeit fernab von Massen befindet oder im freien Fall nahe einer Masse.

    Das ist die Forderung, dass eine Abweichung von der Homogenität vernachlässigbar klein ist, damit der Satz gilt (=”lokal”).

    Ergibt es Sinn, dass du in unserem Universum Effekte beim Zwillingsparadoxon ja nach Bezugssystem einmal als ART-Effekt und einmal als SRT-Effekt ansiehst?

    Ja. Zur vollständigen Aufklärung des Zwillingsparadoxons ist das inhaltlich sinnvoll, wobei die Namensgebung “ART-Effekt” für die gravitative Zeitdilatation bei Teresa im homogenen (Pseudo?-)Gravitationsfeld im beschleunigten Serena-Bezugssystem unglücklich ist.

  359. #359 MartinB
    21. Dezember 2018

    @Selina
    “Was bedeutet hier das Wegbeschleunigen entlang einer Geodäte? War es nicht so, dass die Massen entlang einer Geodäte kräftefrei unterwegs sind?”
    Richtig, deswegen hatte ich ja auch Wegbeschleunigen von der Geodäte geschrieben – kräftefrei bleibt man auf der Geodäten, mit Kräften wird man von ihr wegbeschleunigt, siehe auch Teil 5 meiner Raumzeitkrümmungsserie (rechts bei den Artikelserien).

    @Anonym
    “Aus meiner Sicht muss man Person und Bezugssystem unterscheiden.”
    Dann entspricht deine Sicht nicht der SRT oder ART.

    “Das ist die Forderung, dass eine Abweichung von der Homogenität vernachlässigbar klein ist, damit der Satz gilt (=”lokal”).”
    Dann hast du das schlicht falsch verstanden. Das Äq-Prinzip gilt immer, egal wie gekrümmt die Raumzeit ist, man kann *immer* ein lokales Lorentz-System finden, in dem die Raumzeit flach ist (und flache Raumzeit bedeutet nicht “homogenes Gravitationsfeld”). Das steht doch sogar in dem Einsteinzitat, das du selbst gebracht hast, wenn man den Begriff “Gezeitenkräfte” hinreichend allgemein versteht. “Gezeitenkräfte” sind letztlich alle Effekte, die man nur durch Vergleich von Geodäten in einem Abstand, der gerade größer ist als das “lokale Lorentzsystem”, beobachten kann.

    Für weitere Details verweise ich jetzt doch nochmal auf das Buch,d as demnächst rauskommt, da dürfte ich insgesamt bestimmt 20 oder mehr Seiten verwendet haben, um genau diesen ganzen Kram rüberzubringen, weil es der Schlüssel zur ART ist.

  360. #360 Niels
    21. Dezember 2018

    @Anonym_2018
    .

    Zu #358:

    Aus meiner Sicht muss man Person und Bezugssystem unterscheiden.

    Wie gesagt, schau dir einfach noch mal an was ein Bezugssystem eigentlich ist.

    Doch, sie kommt in dieser Formulierung indirekt vor, durch das Wort “nahe”:

    … durch überhaupt kein Experiment feststellen kann, ob er sich in der Schwerelosigkeit fernab von Massen befindet oder im freien Fall nahe einer Masse.

    Das ist die Forderung, dass eine Abweichung von der Homogenität vernachlässigbar klein ist, damit der Satz gilt (=”lokal”).

    Nein.
    Das Gegensatzpaar ist hier “fern einer Masse” und “nahe einer Masse”.
    Verglichen wird also die Situation im Flachen (“fern einer Masse”) und im Gekrümmten (“nahe einer Masse”).
    Flach und Gekrümmt sind objektive und beobachterunabhängige Eigenschaften der Raumzeitregion.

    Wenn Homogenität deiner Meinung nach nur “nahe einer Masse” gegeben wäre, ist deiner Meinung nach die Situation “fern einer Masse” damit dann nicht homogen?
    Wäre das nicht widersprüchlich?
    .

    Davon abgesehen passt Homogenität natürlich überhaupt nicht zu “nahe einer Masse”.

    Beispiele sind die Situation auf der Oberfläche des Neutronensterns oder im Extremfall in unmittelbarer Nähe der Singularität eines schwarzen Lochs (was auch immer dort tatsächlich vorliegt). Das ist das am wenigsten homogene, das man sich überhaupt vorstellen kann.
    Ursache ist gerade, dass es “nahe einer Masse” ist.
    Je Näher, desto schlechter.

    Ergibt es Sinn, dass du in unserem Universum Effekte beim Zwillingsparadoxon ja nach Bezugssystem einmal als ART-Effekt und einmal als SRT-Effekt ansiehst?

    Ja. Zur vollständigen Aufklärung des Zwillingsparadoxons ist das inhaltlich sinnvoll

    Dazu fällt mir nichts mehr ein.

    Der Flug der Zwilling in Universum A und B ist exakt identisch.
    Wenn sie sich wieder treffen, sind sie in A und B gleich gealtert und weisen dadurch den gleichen Altersunterschied auf.
    Es kann grundsätzlich keine Möglichkeit geben, durch Durchführung des Zwillingsexperiments zu unterscheiden, ob man sich in Universum A oder B befindet.

    Du nimmst an, dass du dich in Universum A ohne Einsteinsches Äquivalenzprinzip befindest.
    Du erklärst das Experiment vollständig und widerspruchsfrei ausschließlich mit der SRT. Für alle Bezugssysteme und alle Beobachter.
    Ach für das beschleunigte Serena-Bezugssystem.
    In der Erklärung kommt mit Sicherheit nicht das Wort gravitative Zeitdilatation vor.
    Weil es gravitative Zeitdilatation im Universum A gar nicht gibt.

    Jetzt erzählt dir jemand, dass du dich geirrt hast und du dich doch in Universum B mit Einsteinsches Äquivalenzprinzip aufhältst und nicht in A.
    Dann rufst du:
    Moment. Die Erklärung war unvollständig. Wir müssen zwingend das Einsteinsche Äquivalenzprinzip, die gravitative Zeitdilatation und die ART berücksichtigen.

    Das findest du wirklich logisch?
    .

    Allgemein:
    Ich weiß eigentlich nicht, wie MartinB und ich das Ganze noch ausführlicher erklären könnten.
    Das wurde alles jetzt schon mehrmals und außerdem in verschiedenen Formulierungen ausgeführt.

    Vielleicht hilft es, wenn du alles ab #123 noch einmal durchliest?

    Naturgemäß ist die Kommentarspalte eines Blogs allerdings auch nicht das beste Mittel, um die Grundlagen der ART zu vermitteln.

    Hast du dir die zahllosen Artikel-Serien von MartinB zur ART schon angeschaut?

    Es gibt natürlich außerdem auch wahnsinnig viel weitere Literatur zur ART.
    Da können MartinB und ich dir sicher etwas passendes empfehlen, wenn du uns deinen mathematischen Kenntnisstand verrätst.

  361. #361 MartinB
    21. Dezember 2018

    @Niels
    “Ich weiß eigentlich nicht, wie MartinB und ich das Ganze noch ausführlicher erklären könnten.”
    Ich schon 😉
    https://www.springer.com/de/book/9783662572924

  362. #362 Alderamin
    21. Dezember 2018

    @MartinB

    Na ENDLICH !!! Gratulation!

  363. #363 Alderamin
    21. Dezember 2018

    @myself

    Ach nee, Erscheinungsdatum Februar 2019… 😮

  364. #364 Anonym_2018
    21. Dezember 2018

    @Niels #360

    Wie gesagt, schau dir einfach noch mal an was ein Bezugssystem eigentlich ist.

    erledigt:

    Ein Bezugssystem ist in der Physik ein gedachtes raum-zeitliches Gebilde, das erforderlich ist, um das Verhalten ortsabhängiger Größen eindeutig und vollständig zu beschreiben.

    Quelle:
    https://de.wikipedia.org/wiki/Bezugssystem

    Der Beobachter ist in der Physik derjenige, der ein Phänomen beobachtet. Es kann sich dabei um eine reale Person, um einen geeigneten Messapparat oder – in einem Gedankenexperiment – um eine gedachte Person handeln. Der Beobachter beschreibt das Phänomen in der Regel in seinem Ruhesystem. Ein Wechsel zu einem anderen Beobachter bedeutet daher im Allgemeinen auch den Wechsel zu einem anderen Bezugssystem und damit zu einer anderen Beschreibung desselben Phänomens. Größen, deren Wert nicht vom Bewegungszustand des Beobachters abhängen, nennt man invariant.

    Ist der Beobachter beschleunigt, so treten in seinem Bezugssystem Trägheitskräfte auf, die für einen „ruhenden“ Beobachter nicht existieren und daher als Scheinkräfte bezeichnet werden.

    Quelle:
    https://de.wikipedia.org/wiki/Beobachter_(Physik)

    Wenn man das Wort “Beobachter” benutzt, muss man sicherstellen, dass man die Welt im Bezugssystem beschreibt, in dem dieser Beobachter ruht, und nicht nur über eine Person als Objekt schreibt, ohne das Bezugssystem anzugeben.

    Bei dem Beispiel mit dem Ball und der Plattform ist mir ein Gedankenfehler unterlaufen. Zu dem Zeitpunkt, zu dem der Ball im Scheitelpunkt ist, wird von ihm ein Photon von der am Boden liegenden Uhr empfangen. Dieses Photon hat auch im Bezugssystem des Balls eine Rotverschiebung. Allerdings ist das in diesem Bezugssystem keine gravitative Rotverschiebung, sondern eine Rotverschiebung durch relative Geschwindigkeit. Zu dem Zeitpunkt des Empfangs ist die Relativgeschwindigkeit zwischen Erde und Ball zwar Null, aber das Photon wurde schon vorher (rotverschoben) abgestrahlt, als die Erde sich noch mit einer bestimmten Geschwindigkeit vom Ball weg bewegt hat.

    Im Bezugssystem der Plattform wird zum Zeitpunkt, zu dem der Ball sich im Scheitelpunkt befindet, von der Person auf der Platform das Photon mit ausschließlich gravitativer Rotverschiebung empfangen.

    Eine Berechnung dazu habe ich nicht durchgeführt.

    Du nimmst an, dass du dich in Universum A ohne Einsteinsches Äquivalenzprinzip befindest.
    Du erklärst das Experiment vollständig und widerspruchsfrei ausschließlich mit der SRT.

    Diese beiden Sätze widersprechen sich. Ein solches Universum kann es deswegen nicht geben.

  365. #365 MartinB
    21. Dezember 2018

    ” Zu dem Zeitpunkt des Empfangs ist die Relativgeschwindigkeit zwischen Erde und Ball zwar Null, aber das Photon wurde schon vorher (rotverschoben) abgestrahlt, als die Erde sich noch mit einer bestimmten Geschwindigkeit vom Ball weg bewegt hat.”
    Und wenn ich eine tausendstel Sekunde bevor das Photon ankommt (das aber lange vorher abgestrahlt wurde, weil ich auf einem sehr großen Planeten stehe) hochspringe und deshalb nur momentan im freien Fall bin?

    “Diese beiden Sätze widersprechen sich. Ein solches Universum kann es deswegen nicht geben.”
    Aha, die SRT war also zwischen 1905 und 1915 inkonsistent und niemand hat’s gemerkt…

  366. #366 Selina
    21. Dezember 2018

    @MartinB:

    “Richtig, deswegen hatte ich ja auch Wegbeschleunigen von der Geodäte geschrieben – kräftefrei bleibt man auf der Geodäten, mit Kräften wird man von ihr wegbeschleunigt, siehe auch Teil 5 meiner Raumzeitkrümmungsserie (rechts bei den Artikelserien).”

    Verstehe, aber in ART gibt es nur Raumzeit Krümmung, die den Weg für ein Teilchen vorgib. Da wirkt gar keine Kraft, weder in erster noch in zweiter Ordnung. Ich finde die Formulierung kräftefrei oder Wegbeschleunigen unglücklich gewählt, da man wieder Elemente aus der Newtonsche Gravitationstheorie mit ART vermischt.

  367. #367 MartinB
    21. Dezember 2018

    @Selina
    Ich glaube, du missverstehst mich – mit Kraft meine ich in dem Falll niemals die “Schwerkraft”, die gibt es ja nicht,w ie du richtig schreibst. Wenn ich z.B. fest auf der Erde stehe, ist meine Geodäte gekrümmt, weil ich im freien Fall ja nach unten beschleunigt würde – die Kraft des Bodens beschleunigt mich von der Geodäte weg und hält mich am selben Ort, ohne diese Kraft würde ich der Geodäte folgen.
    Insofern stimme ich dir in jeder Hinsicht zu
    Siehe auch hier:
    https://scienceblogs.de/hier-wohnen-drachen/2016/08/27/wie-einstein-die-newtonschen-gesetze-vertauschte/?all=1

  368. #368 Anonym_2018
    21. Dezember 2018

    Und wenn ich eine tausendstel Sekunde bevor das Photon ankommt (das aber lange vorher abgestrahlt wurde, weil ich auf einem sehr großen Planeten stehe) hochspringe und deshalb nur momentan im freien Fall bin?

    Der Einfachheit halber 🙂 im Bezugssystem der Platform berechnet eimpfange ich das Photon mit der gravitativen Rotverschiebung. Diese wird noch überlagert durch eine Geschwindigkeits-abhängige Blauverschiebung (Doppler-Effekt) , falls ich mich durch den freien Fall zum Zeitpunkt des Empfangs auf das Photon zubewegen sollte.

    Aha, die SRT war also zwischen 1905 und 1915 inkonsistent und niemand hat’s gemerkt…

    Ohne das Äquivalenzprinzip ist eine Aufklärung des Zwillingsparadoxons nicht vollständig möglich. Die bzgl. des Zwillingsparadoxons unvollständige Aufklärung klärt nur den weniger komplizierten Drei-Brüder-Ansatz vollständig.

  369. #369 Selina
    21. Dezember 2018

    @MartinB
    “die Kraft des Bodens beschleunigt mich von der Geodäte weg und hält mich am selben Ort, ohne diese Kraft würde ich der Geodäte folgen.”

    Ich glaube du meinst die elektromagnetische Kraft, die verhindert, dass man von der Erde verschluckt wird. Habe die ganze Zeit gerätselt, was dieses Wegbeschleunigen sein könnte.

    Ich habe grundsätzlich mit dem Wort Kraft in ART Probleme, weil es sie einfach nicht gibt. Kräftefrei in ART macht für mich wenig Sinn, das wirkt auf mich wie Newtonsche Theorie.

  370. #370 MartinB
    21. Dezember 2018

    @Selina
    Naja, elektromagnetisch plus quantenmechanisch wegen des Pauli-Prinzips, aber ja, die Kraft, die mich daran hindert, durch den Boden zu fallen.
    Und die gibt es auch in der ART und die ist da auch relevant, sonst würde ja immer alles der Geodäten folgen und alle Sterne würden zu schwarzen löchern implodieren. Zu sagen “Kärfte gibt es in der ART nicht” ist natürlich richtig, wenn man ausschließlich Effekte der ART anguckt – aber physikalisch macht das ja wenig Sinn. Deswegen findet man ja auch in Büchern zur ART z.B. Gleichungen über Sternmodelle, Berechnungen des Drucks in Neutronensternen und solche Sachen.

  371. #371 MartinB
    21. Dezember 2018

    @Anonym_2018
    “Ohne das Äquivalenzprinzip ist eine Aufklärung des Zwillingsparadoxons nicht vollständig möglich. Die bzgl. des Zwillingsparadoxons unvollständige Aufklärung klärt nur den weniger komplizierten Drei-Brüder-Ansatz vollständig.”
    O.k., du beharrst darauf, etwas falsches zu behaupten – kann man dann nicht ändern.

  372. #372 Alderamin
    21. Dezember 2018

    @MartinB

    Vielleicht war meine Ugly-Rechnung ja doch nicht so schlecht, da kommen nur SRT-Formeln drin vor (Zeitdilation und Geschwindigkeitsaddition) und am Ende passt es auch… 😉

  373. #373 MartinB
    21. Dezember 2018

    @Alderamin
    Aber du hast doch nicht die Beschleunigung aus Serenas Sicht berechnet, oder?
    Es geht doch darum, dass das angeblich in der SRT nicht geht und dass da auf wundersame Weise (Pseudo-)Gravitationsfelder und das Äq-Prinzip notwendig werden sollen (obwohl man alles auch im Rahmen der SRT korrekt berechnen kann), weil – keine Ahnung….

  374. #374 Alderamin
    21. Dezember 2018

    @MartinB

    Nö, Beschleunigungen habe ich ausgeklammert, aber da der Effekt ohne Beschleunigung bereits auftritt, braucht es diese oder die ART zur Erklärung offenbar nicht (im Detail bin ich Eurer Diskussion nicht gefolgt, hab’ nur sporadisch mal reingeschaut).

    Beschleunigung sollte im Endeffekt nur die Zeit verlängern, die der Reisende bei niedriger Geschwindigkeit mit weniger Dilatation verbringt (und diejenige mit hoher Dilatation verkürzen). So ähnlich als wenn er am Umkehrpunkt eine Zeit verweilt. Er altert dann etwas mehr (und nicht etwa weniger) als ohne Beschleunigung (bei fester Wegstrecke gegenüber instantanem Wechsel von v_hin -> v_rück).

  375. #375 Anonym_2018
    21. Dezember 2018

    O.k., du beharrst darauf, etwas falsches zu behaupten – kann man dann nicht ändern.

    Ich beharre nicht darauf, falls mir jemand einen Fehler in meiner Argumentation nachweist. Diese hatte ich in #368 versäumt zu meiner Behauptung mitzuliefern. Das hole ich jetzt gerne nach:

    Das scheinbare Paradoxon aufzuklären umfasst mehr, als nur den letztendlichen Altersunterschied auszurechnen. Dafür würde ein Bezugssystem für die Berechnung ausreichen. Sondern man muss nacheinander in beiden Bezugssystemen jeweils die Eigenzeiten von beiden Zwillingen berechnen und dann die Endergebnisse vergleichen.

    Das scheinbare Paradoxon besteht ja darin, dass Teresa in ihrem Bezugssystem berechnet, dass sie um 10 Jahre altert und Serena um 6 Jahre altert, während Serena in ihrem Bezugssystem berechnet, dass sie um 6 Jahre altert und Teresa um nur 3,6 Jahre altert (Widerspruch). Bei diesem Widerspruch würde es in einem Universum bleiben, in dem wie bei Newton die Scheinkraft im Bezugssystem von Serena nicht zu eine Zeitdilatation bei Teresa führt. In unserem Universum mit Äquivalenzprinzip berechnet Serena für ihr Bezugssystem die Eigenzeit von Terease korrekt als 10 Jahre (kein Widerspruch).

  376. #376 MartinB
    21. Dezember 2018

    @Anonym_2018
    Ich gebe auf.

  377. #377 MartinB
    21. Dezember 2018

    @Alderamin
    “aber da der Effekt ohne Beschleunigung bereits auftritt, braucht es diese oder die ART zur Erklärung offenbar nicht ”
    Es gibt halt Leute, die sehen das anders und lassen sich auch mit Argumenten nicht vom Gegenteil überzeugen….

  378. #378 Anonym_2018
    21. Dezember 2018

    @Alderamin #372

    Deine verlinke Beschreibung enthält Zauberei:

    Bei Alpha Centauri beamt Horst auf das entgegenkommende Raumschiff H”

    Nur für diejenigen, die nicht an Zauberei glauben, empfehle ich die Rechnung in obigem Kommentar #274.

    @Alderamin #374

    Dass die Beschleunigung von Serena am Umkehrpunkt für die Berechnung keine Auswirkung hat, gilt nur im Bezugssystem von Teresa.

  379. #379 Alderamin
    21. Dezember 2018

    @Anonym_2018

    Das scheinbare Paradoxon besteht ja darin, dass Teresa in ihrem Bezugssystem berechnet, dass sie um 10 Jahre altert und Serena um 6 Jahre altert, während Serena in ihrem Bezugssystem berechnet, dass sie um 6 Jahre altert und Teresa um nur 3,6 Jahre altert (Widerspruch).

    Dann hat Serena aber falsch gerechnet und vergessen, dass sie das Bezugssystem gewechselt hat. Deswegen habe ich die Rechnung im in #372 verlinkten Artikel mal durchgeführt, aus Sicht aller drei Bezugssysteme (das der Erde, bei mir verbleibt dort eine Angela, das der Hinreise, es reist ein Horst, und das der Rückreise, Horst wechselt am Umkehrpunkt dorthin). Wenn man jedes der Systeme beibehält, kommt immer dasselbe für die Flugzeit von Horst und die Verweilzeit von Angela raus (sind fast dieselben Zahlen wie hier, 0,8c Reisegeschwindigkeit [Erdsicht], 4,3 LJ Entfernung [Erdsicht], 10,75 J für die auf der Erde Gebliebene [alle Sichten] und 6,45 Jahre für den Gereisten [dito]).

    Man darf nur nicht die ganze Reise aus Sicht des Reisenden rechnen, weil man dann gerade nicht im gleichen Bezugssystem bleibt. Man muss stur das System beibehalten. Im System, in dem Horst (oder Serena) hinreist, verabschiedet er/sie sich im Umkehrpunkt und hetzt dann mit erhöhter Geschwindigkeit und Zeitdilatation einer bereits mit 0,8c enteilenden Erde hinterher, die er/sei erst nach 14,69 Jahren erreicht, aber mit so viel Zeitdilatation (wegen der erhöhten Geschwindigkeit), dass er/sie trotzdem nur um 3,23 Jahre altert.

    Und umgekehrt, im System, in dem Horst zurückfliegt, braucht er 14,69 Jahre, um von der Erde aus Alpha-Centauri erst einmal zu erreichen, weil er nur ein bisschen schneller als die Erde und den gleich schnellen, etwas näheren Alpha Centauri auf den im Bezugssystem ruhenden Beobachter zufliegt (mit dem er dann auf dem “Rückflug” ruhend auf die mit 0,8c herannahende Erde wartet). Aber wegen der hohen Zeitdilatation vergehen für ihn während der 14,69 Jahre Hinflug trotzdem nur 3,23 Jahre.

    Dass in jedem Bezugssystem die Zeiträume der anderen länger erscheinen, obwohl immer die gleiche Reise betrachtet wird, liegt daran, dass sie sich sehr uneinig über den Zeitpunkt sind, wann die Reise beginnt bzw. aufhört (Relativität der Gleichzeitigkeit; die 14,69 Jahre Vor- bzw. Nachlauf in den “Reise-Bezugssystemen”, die das Erdsystem viel kürzer sieht).

    Einfach mal im Kopf vergegenwärtigen, wie die Reise in den drei Bezugsystemen aussieht und dann die Zahlen nachvollziehen. Bei mir hat’s damals “Pling” gemacht, als mich mir die Sichtweisen der drei Bezugssysteme vor Augen geführt habe.

  380. #380 Alderamin
    21. Dezember 2018

    @Anonym_2018

    Deine verlinke Beschreibung enthält Zauberei:

    Bei Alpha Centauri beamt Horst auf das entgegenkommende Raumschiff H”

    Nur für diejenigen, die nicht an Zauberei glauben, empfehle ich die Rechnung in obigem Kommentar #274.

    Kommt darauf an. Es vereinfacht die Betrachtung, auch wenn es für einen massebehafteten Reisenden nur idealisiert möglich wäre (man kann die Strecke gegen unendlich gehen lassen, dann verkürzt sich die Beschleunigungsdauer gegen einen Nullanteil der Betrachtungsdauer).

    Man könnte aber durchaus real die vergangene Zeit am Treffpunkt mit Funk vom Hin- auf den Rückflug übertragen (gleicher Ort, Lauzeit 0), und sich dann am Ziel den vergangenen Zählerstand ansehen. Wenn man keine physische Person bewegt, sondern nur die Zeiträume misst und überträgt (was nach Abzug der Lichtlaufzeiten zwischen allen Systemen möglich ist), dann kommen genau die Ergebnisse heraus, die ich vorrechne.

    Alleine mit SRT kommt man auf die geringere vergangene Zeit der “gereisten Uhrzeit” gegenüber der auf der Erde verbliebenen, in Übereinstimmung der Sichten aller Bezugssyteme.

    Und mit dem in #374 Gesagten wird klar, dass mit Beschleunigung (also ohne Zauberei) nur mehr Zeit mit geringerer Dilatation für den Reisenden, also weniger Altersunterschied, herauskommt, nicht mehr. Wenn’s alleine an der Beschleunigung läge, müsste ja der ganze Altersunterschied nur von der Beschleunigung verursacht werden.

  381. #381 Max
    21. Dezember 2018

    @Alderamin:

    “Man darf nur nicht die ganze Reise aus Sicht des Reisenden rechnen, weil man dann gerade nicht im gleichen Bezugssystem bleibt.”

    Wenn man aus Sicht des Reisenden rechnet, bleibt er sowieso in Ruhe. Aus seiner Sicht kann man wohl die ganze Reise rechnen.

    “Wenn’s alleine an der Beschleunigung läge, müsste ja der ganze Altersunterschied nur von der Beschleunigung verursacht werden.”

    Das hat ja keiner gesagt, es gibt Teile, die durch SRT zustande kommen und welche wie beim Umkehrpunkt, der durch Beschleunigung wegen des Richtungswechsels.

  382. #382 Anonym_2018
    21. Dezember 2018

    @Alderamin #379

    Dann hat Serena aber falsch gerechnet und vergessen, dass sie das Bezugssystem gewechselt hat.

    Sie kann ihr Bezugssystem, in dem sie ja ruht, nicht wechseln. Das allgemeine Relativitätsprinzip erlaubt es, dass ein Bezugssystem auch ein nicht-Inertialsystem sein darf. In dem Serena-Bezugssystem muss man halt während der Umkehrphase das Äquivalenzprinzip berücksichtigen, welches bei Teresa eine “gravitative” Zeitdilatation verursacht.

    @Alderamin #380

    Wenn man keine physische Person bewegt, sondern nur die Zeiträume misst und überträgt (was nach Abzug der Lichtlaufzeiten zwischen allen Systemen möglich ist), dann kommen genau die Ergebnisse heraus, die ich vorrechne.

    Die Aufgabenstellung wird modifiziert. In der Eigenzeit von Serena wird in deren Bezugssystem ein wichtiger Abschnitt übersprungen, in dem Teresa rasant altert.

  383. #383 Niels
    21. Dezember 2018

    @Anonym_2018

    Hoppla.
    Jetzt brauchen wir für die Erklärung des Zwillingsparadoxons nicht mehr nur das Äquivalenzprinzip sondern plötzlich sogar außerdem noch zusätzlich das allgemeine Relativitätsprinzip?

    Das allgemeine Relativitätsprinzip erlaubt es, dass ein Bezugssystem auch ein nicht-Inertialsystem sein darf.

    Nein.
    Auch in der SRT und sogar schon in der newtonschen Mechanik darf man natürlich jedes denkbare Bezugssystem verwenden. Natürlich auch Nicht-Inertialsysteme.
    .

    Relativitätsprinzipen (galileisches, spezielles, allgemeines) sagen vereinfacht zusammengefasst etwas über die Gleichwertigkeit von Bezugssystemen aus.

    Sind bestimmte Bezugssysteme gegenüber anderen ausgezeichnet?
    Nehmen die physikalischen Gesetze in bestimmten Bezugssystemen eine besonders einfache Form an?
    .

    Vielleicht solltest du dir einfach noch mal ein paar Einführungstexte zur ART anschauen.
    Die Artikelserien von MartinB hast du gelesen?

    Anscheinend gibt es ja sowohl beim Äquivalenzprinzip als auch beim (speziellen und allgemeinen) Relativitätsprinzip noch sehr große Verständnisschwierigkeiten.
    Das sind aber wirklich die absoluten unumgänglichen Grundlagen. Die muss sich einfach aneignen, da führt kein Weg dran vorbei.

  384. #384 Anonym_2018
    21. Dezember 2018

    @Niels #383

    Hoppla.
    Jetzt brauchen wir … zusätzlich das allgemeine Relativitätsprinzip?

    Ich habe nicht geschrieben, dass wir das brauchen, sondern nur A. Einstein sinngemäß zitiert, dass es das erlaubt:

    Rel.: Es ist sicherlich richtig, daß wir uns vom Standpunkt der allgemeinen Relativitätstheorie aus ebensogut des Koordinatensystems K’ bedienen können als des Koordinatensystems K. Aber man sieht leicht ein, daß die Systeme K und K’ mit Bezug auf den betrachteten Vorgang keineswegs gleichwertig sind.

    Quelle:
    https://wikilivres.org/wiki/Dialog_%C3%BCber_Einw%C3%A4nde_gegen_die_Relativit%C3%A4tstheorie

  385. #385 Alderamin
    21. Dezember 2018

    @Max

    Wenn man aus Sicht des Reisenden rechnet, bleibt er sowieso in Ruhe. Aus seiner Sicht kann man wohl die ganze Reise rechnen.

    Wenn er die ganze Reise in Ruhe bleibt, wechselt er das Bezugssystem, und dann rechnet man zwei Systeme durcheinander und so löst man das Paradoxon nicht auf, warum er (oder sie) eindeutig und aus allen Perspektiven betrachtet weniger Reisezeit verbucht, als die auf der Erde verbliebene Beobachterin, obwohl die SRT doch Symmetrie voraussagt (aber eben nur für den Fall, dass man im selben Bezugssystem bleibt – egal in welchem).

    Das hat ja keiner gesagt, es gibt Teile, die durch SRT zustande kommen und welche wie beim Umkehrpunkt, der durch Beschleunigung wegen des Richtungswechsels.

    Das Beispiel mit den per Funk ausgetauschten Zeitständen zeigt, dass der gesamte Altersunterschied durch die SRT zustande kommt. Und ich habe begründet, dass eine Beschleunigungsphase bestenfalls für einen geringeren Unterschied sorgt.

  386. #386 Max
    21. Dezember 2018

    @Alderamin:

    “obwohl die SRT doch Symmetrie voraussagt (aber eben nur für den Fall, dass man im selben Bezugssystem bleibt – egal in welchem).”

    Eben, da die Symmetrie gebrochen ist, tritt doch ein Zusatz Effekt auf, der den Altersunterschied verursacht.

    “Und ich habe begründet, dass eine Beschleunigungsphase bestenfalls für einen geringeren Unterschied sorgt.”

    Ich rede gar nicht von einer BeschleunigusPHASE, sondern von einem Richtungswechsel im Bezugssystem von Serena. Dieser Knick im Minkowski-Diagramm ist ja letztlich verantwortlich für den Altersunterschied. Diesen Knick verbinde ich mit Beschleunigung.

  387. #387 Anonym_2018
    21. Dezember 2018

    @MartinB #365

    Ergänzung zu #368

    Und wenn ich eine tausendstel Sekunde bevor das Photon ankommt (das aber lange vorher abgestrahlt wurde, weil ich auf einem sehr großen Planeten stehe) hochspringe und deshalb nur momentan im freien Fall bin?

    Jetzt beschreibe ich das in meinem Bezugssystem:

    Zunächst stehe ich auf der Plattform, bis das vom Boden kommende Photon den halben Weg bis zur Plattform zurückgelegt hat. Es ist dort gravitativ “halb-rotverschoben”. Jetzt versuchen meine Beine für einen begrenzten Zeitraum, mich durch eine Kraft nach oben zu beschleunigen. Das passiert aber nicht, weil ein neues Scheingravitationsfeld entsteht, dass dieser Kraft entgegenwirkt. Daher bleibe ich (wie immer) in Ruhe. Dieses Scheingravitationsfeld beschleunigt allerdings die Plattform, die Erde und den dahinter liegenden Teil des Universums von mir weg. Es verstärkt auch das Erd-Gravitationsfeld. Dadurch wird das weiter nach oben fliegende Photon in diesem Zeitraum stärker pro Centimenter rot verschoben als vorher. Sobald ich abgehoben habe, ist kein Gravitationsfeld mehr da. Die Plattform ist jetzt auf mich zu beschleunigt. Das Photon hat die Farbe “halb-rotverschoben-plus” und bewegt sich auf dem letzten Teil der Strecke demgegenüber unverfärbt mit Geschwindigkeit c weiter. Dann sehe ich es.

  388. #388 MartinB
    21. Dezember 2018

    @Anonym
    Wie gesagt, ich bin raus, wird mir zu mühselig, immer dasselbe zu erklären.

  389. #389 MartinB
    21. Dezember 2018

    @Max
    Bist du Toni unter neuem Namen? Du verwendest genau dieselben Argumente und Begriffe…

  390. #390 Anonym_2018
    21. Dezember 2018

    @Alderamin #385

    Wenn er die ganze Reise in Ruhe bleibt, wechselt er das Bezugssystem, und dann rechnet man zwei Systeme durcheinander und so löst man das Paradoxon nicht auf

    Er wechselt das Inertialsystem aber nicht das Bezugssystem. In dem ist er per Definition in Ruhe.

    Wie man in diesem (ztw. nicht-Inertial-)Bezugssystem des Reisenden bleiben kann und das Paradoxon auflöst, hat A. Einstein 1918 in einem Paper qualitativ beschrieben.

    Wie gesagt, eine einfache Berechnung anhand eines Wikipedia-Beispiels gibt es dazu im obigen Kommentar #274.

    Es gibt auch ein Video dazu:
    Twin Paradox in General Relativity

  391. #391 Max
    21. Dezember 2018

    MartinB
    Nein, ich bin aber mit Tox und Toni mehr oder weniger einverstanden, dass die Zeitdilatation durch eben diese Beschleunigung verursacht wird.

  392. #392 Alderamin
    21. Dezember 2018

    @Anonym_2018

    Gut, er wechselt das Inertialsystem, das habe ich synonym verwendet, dann ersetze das oben in meinem Text und dann gilt das Gesagte noch immer.

    Und @Max:

    Ich habe ja auch das Beispiel mit den per Funk übertragenen Uhrzeiten (statt eines Hinüberwechselns der Person) genannt. Ein Zahlenwert für sich hat keine Masse, die man beschleunigen könnte, und keine Richtung, die er wechseln könnte. Hier kommen aber genau die von der SRT vorhergesagten Zeiten heraus, ohne Beschleunigung. Wenn das nicht als Beweis ausreicht (zusammen mit der Rechnung), dann weiß ich auch nicht, wie ich es besser erklären soll.

    Und Du weißt vermutlich keine Formel, aus der Du aus einer Richtungsänderung einen von der Strecke abhängigen Altersunterschied der Zwillinge ableiten kannst.

  393. #393 Max
    21. Dezember 2018

    @Alderamin
    “Ich habe ja auch das Beispiel mit den per Funk übertragenen Uhrzeiten (statt eines Hinüberwechselns der Person) genannt”

    Auch beim Funken hast du diesen Knick im Minkowski-Diagramm. https://www.leifiphysik.de/relativitaetstheorie/spezielle-relativitaetstheorie/ausblick/zwillingsparadoxon

    Es geht mir nicht um eine Formel, sondern darum wodurch die Zeitverzögerung zustande kommt? Ohne diesen Knick wäre Serena ja nie jünger nach der Rückkehr.

  394. #394 Anonym_2018
    21. Dezember 2018

    @Alderamin #392

    Und Du weißt vermutlich keine Formel, aus der Du aus einer Richtungsänderung einen von der Strecke abhängigen Altersunterschied der Zwillinge ableiten kannst.

    Ich habe eine solche gefunden. Link:
    ∆t = v/c² * L

    Dabei ist im Bezugssystem von Serena:
    ∆t die Differenz der Eigenzeiten, die für Teresa und Serena während der Beschleunigungsphase vergehen,
    v ist die Geschwindigkeitsdifferenz beim Umkehren (z.B. 1,6c beim Wechsel von 0,8c zu -0,8c)
    L ist die Entfernung zwischen Teresa und Serena beim Umkehren (z.B. 4 Lichtjahre).

  395. #395 Alderamin
    21. Dezember 2018

    @Anonym_2018

    Ich habe eine solche gefunden.

    Nö. L ist der Abstand vom Anfang der Beschleunigung bis zum Ende, nicht die Flugstrecke. Bei zunehmender Beschleunigung wird L immer kleiner. Wenn die Beschleunigung instantan erfolgt, ist L=0.

    In derselben Quelle wird das Zwillingsparadoxon auch mit Zahlen vorgerechnet, aber diese Rotverschiebungsformel kommt gar nicht vor, sondern nur die SRT-Formeln, die ich auch verwendet habe.

    @Max

    Auch beim Funken hast du diesen Knick im Minkowski-Diagramm

    Das ist der Wechsel des Inertialsystems, den der Zahlenwert von der einen Uhr auf die andere macht, aber wenn das eine Raumschiff am anderen vorbeifliegt und jemand dabei die außen angezeigte Flugzeit abliest und in die eigene Uhr eingibt, wird da nichts beschleunigt. Die Uhr beim entgegenkommenden Raumschiff macht genau so viele Schläge auf dem Weg zur Erde, wenn man die Zeit vom hinfliegenden eintippt, als wenn man das nicht tut und sich die Augen zu hält (sie startet dann nur mit einem anderen Wert). Wenn Du da noch irgendeine Beschleunigung findest, herzlichen Glückwunsch.

    Der Knick ist nur, dass man vom Hinflug den Zählerstand nimmt und ihn mit dem des Rückflugs addiert, und das ist eben das Addieren von Werten aus verschiedenen Inertialsystemen.

    Das ergibt die Abweichung davon, dass in jedem einzelnen Inertialsystem die Zeit aller anderen langsamer zu laufen scheint.

    Es geht mir nicht um eine Formel, sondern darum wodurch die Zeitverzögerung zustande kommt? Ohne diesen Knick wäre Serena ja nie jünger nach der Rückkehr.

    Die Zeitverzögerung kommt zustande, weil aus Sicht jedes der drei Inertialsysteme die Zeit für den bewegt Gesehenen gemäß SRT-Zeitdilatation langsamer vergeht. Der Reisende selbst sieht die Zeit auf der Erde auch langsamer gehen. Wenn er das Inertialsystem wechselt, geht ihm aber ein ganzes Stück vergangener Zeit auf der Erde verloren (das sieht man dann wieder im Minkowski-Diagramm am besten, die Linie der Gleichzeitigkeit verkippt sich, was auf dem Hinflug gleichzeitig mit dem Reisenden war, ist auf dem Rückflug eine viel spätere Zeit auf der Erde). Siehe Bild oben im Artikel.

  396. #396 Anonym_2018
    21. Dezember 2018

    @Alderamin #395

    Nö.

    Doch 🙂 Auf dieser Seite wird die Formel hergeleitet:
    https://www.relativitätsprinzip.info/gedankenexperiment/beschleunigung-srt.html
    Da ist L definiert als der Abstand zwischen Bug und Heck der Rakete (das wird auch in der Grafik gezeigt) und die Formel berechnet den Zeitunterschied zwischen Bug und Heck der Rakete:

    Das Gerät A befindet sich vor der Beschleunigung am Ortsnullpunkt (x=0) und das Gerät B befindet sich im Abstand L an der Spitze der Rakete (x=L)

    Damit ergibt sich dann für den Zeitunterschied zwischen Bug und Heck der Rakete die Formel:
    ∆t = v/c² * L

    Also ist L der Abstand zwischen zwei Potentialen des Scheingravitationsfelds mit unterschiedlich schnellem Zeitablauf. Und im beschleunigten Bezugssystem der Rakete besteht dieses äquivalente homogene Scheingravitationsfeld im ganzen Universum.

    Im Bespiel in Kommentar #274 passt auch das Ergebnis dieser Formel zum Rest.

  397. #397 Max
    21. Dezember 2018

    @Alderamin

    “Wenn er das Inertialsystem wechselt, geht ihm aber ein ganzes Stück vergangener Zeit auf der Erde verloren”

    Ja eben, und dieser Wechel der Inertialsysteme wird durch Beschleunigung induziert. Wie soll denn sonst ein Wechsel stattfinden, wenn da nicht irgendein Zusatz Effekt da ist? Das ist für mich der Knackpunkt.

    Ein Satelitt um die Erde gegenüber einem Beobachter auf der Erde wechselt auch sein Inertialsystem ständig, eben durch eine Beschleunigung, nähmlich Zentripetalkraft.

  398. #398 Anonym_2018
    21. Dezember 2018

    @Alderamin #395

    Wenn er das Inertialsystem wechselt, geht ihm aber ein ganzes Stück vergangener Zeit auf der Erde verloren

    .. und zwar genau soviel: ∆t = v/c² * L.

  399. #399 Alderamin
    22. Dezember 2018

    @Anonym

    Also ist L der Abstand zwischen zwei Potentialen des Scheingravitationsfelds mit unterschiedlich schnellem Zeitablauf.

    Mag sein. In der Rakete. Während der Beschleunigung. Zwischen Uhren, die beide an Bord des Raumschiffs sind.

    Während des überwiegenden restlichen Flugs gibt’s aber keine (Schein)Gravitation. Die Beschleunigung dauert einen Tag. Und dann?

    Wie ist es beimBesipiel mit dem ausgetauschten Funksignal? Was wird beschleunigt? Beide Raumschiffe fliegen mit konstant 0,8c. Nichts bewegt sich (außer Funksignal transversal). Wo ist die Beschleunigung?

    Wo ist der Fehler in meiner Rechnung in meinem Blog?
    Wo ist der Fehler in dieser Rechnung? Warum wird ∆t = v/c² * L dort nicht verwendet?
    Siehst Du nicht den Widerspruch zu Deiner Annahme?

    Du hast Dich dermaßen in Deinem Schützengraben verbuddelt, ich denke nicht, dass Du Argumenten noch zugänglich bist. Niels ist beim Thema RT eine Bank, wenn der sagt, irgendetwas sei soundso, kannst Du Deine Hand darauf wetten, dass das stimmt. Es hat aber jeder das Recht, das zu glauben, was er mag.

    Mehr als vorrechnen (mit Herleitung der Zeitdilation) kann ich auch nicht (siehe meinen Artikel). Ich bin dann auch raus.

  400. #400 MartinB
    22. Dezember 2018

    @Alderamin
    Das nennt man wohl Deja vu – genau diese Diskussionspuntke hatten wir oben auch schon…

  401. #401 MartinB
    22. Dezember 2018

    @alle
    So, hier nochmal abschließend meine Sicht der Dinge:
    Wenn Serena umkehrt, beschleunigt sie, während sich Teresa (und ggf. Undine) auf Geodäten bewegen. Serena kann nach dem Äq-Prinzip sagen: Meine Situation ist genau analog zu der einer Person, die auf der Oberfläche eines Planeten mit “homogenem Gravitationsfeld” steht, von der Oberfläche permanent wegbeschleunigt wird, während Teresa und Undine im freien Fall ihrer Geodäten folgen und deshalb auf mich zu beschleunigt werden. Wegen dieser Analogie kann ich auch die entsprechende Formel verwenden, die eine solche Person in der ART verwenden würde und bekomme das richtige Ergebnis.
    Soweit alles gut. Was aber nicht geht ist, zu behaupten, dass Serena tatsächlich irgendeine Form von Gravitation, gravitativer Dilatation oder sonst etwas erfährt. Die Konsistenz zwischen SRT und ART erfordert nach dem Äq-Prinzip, dass die Rechnung so funktioniert, trotzdem ist der Effekt vollständig innerhalb der SRT erklärbar, weil die Situationen zwar analog sind, aber eben nicht physikalisch identisch. (Im einen Fall ist die Raumzeit flach, im anderen gekrümmt.) Von einem Effekt der ART zu sprechen, ist deshalb irreführend und hilft dem Verständnis wirklich nicht weiter.

  402. #402 MartinB
    22. Dezember 2018

    Sorry
    “von der Oberfläche permanent wegbeschleunigt wird”
    sollte besser
    “von der Kraft, die die Oberfläche ausübt, permanent von ihrer Geodäten wegbeschleunigt wird”
    heißen.

  403. #403 Toni
    22. Dezember 2018

    @MartinB:

    Ja, ZP lässt sich vollständig innerhalb der SRT auflösen, wenn man Delta T bei der Umkehr außer Acht lässt. Genauer gesagt die Zeit:
    “Wenn er das Inertialsystem wechselt, geht ihm aber ein ganzes Stück vergangener Zeit auf der Erde verloren.”

    Ansosnten müsste man es genau so beschreiben, wie du in #401 geschieben hast. Zumindest diesbezüglich, fanden wir zum Abschluss dieser über Tage intensiv geführte Diskussion einen gemeinsamen Nenner. Ich hoffe, wir haben deine Nerven nicht allzu sehr strapaziert, falls doch tut es mir Leid.

  404. #404 MartinB
    22. Dezember 2018

    @Toni
    “Ja, ZP lässt sich vollständig innerhalb der SRT auflösen, wenn man Delta T bei der Umkehr außer Acht lässt. Genauer”
    Nein. Das ZP lässt sich vollständig innerhalb der SRT auflösen, ohne jedes Wenn und Aber. Werde ich aber nicht noch ein 10000000test mal erklären.

  405. #405 Anonym_2018
    22. Dezember 2018

    Sorry, aber die im folgenden Zitat aus diesem Artikel stehende Aussage kann meiner Ansicht nach nicht stimmen:

    The Bad

    Die SRT hat überhaupt keine Probleme damit, beispielsweise die Zeitdilatation zu beschreiben, die auf eine Beobachterin wirkt, die gerade beschleunigt – das ist sogar vergleichsweise einfach. (Wer’s genau wissen will: Der Dilatationsfaktor ist zu jedem Zeitpunkt gegeben durch die momentane Geschwindigkeit, man muss also bei einer sich ändernden Geschwindigkeit lediglich aufintegrieren.)

    Quelle:
    https://scienceblogs.de/hier-wohnen-drachen/2018/07/04/the-good-the-bad-and-the-ugly-die-erklaerungen-des-zwillingsparadoxons/

    Grund: Mit dem in der Klammer beschriebenen Vorgehen berechnet man nur einen verschwindend geringen Anteil der Zeitdilatation, die Serena bei Teresa beobachtet – nämlich den aufintegrierten Geschwindigkeits-abhängigien Anteil. Bei einem Abstand von mehreren Lichtjahren überwiegt der Anteil durch “schein-gravitative” Zeitdilatation aufgrund des homogenen Schein-Gravitationsfeldes das i.d. Regel um mehrere Größenordnungen.

  406. #406 MartinB
    22. Dezember 2018

    @Anonym
    Das ist die Dilatation, die ein anderer Beobachter momentan misst, nicht mehr.

  407. #407 MartinB
    22. Dezember 2018

    PS: Es wäre wirklich guter Stil, wenn du langsam, damit aufhören würdest, jedes mal, wenn du etwas nicht oder missverstehst, gleich zu behaupten, es läge ein Fehler vor.

  408. #408 Anonym_2018
    22. Dezember 2018

    Sorry, ich hatte unter “die auf eine Beobachterin wirkt, die gerade beschleunigt” die Serena verstanden. Dann sollte ich wohl ggf. beim nächsten Missverständnis dieses als Frage formulieren.

  409. #409 Niels
    22. Dezember 2018

    @MartinB

    Die Konsistenz zwischen SRT und ART erfordert nach dem Äq-Prinzip, dass die Rechnung so funktioniert
    […]
    Wegen dieser Analogie kann ich auch die entsprechende Formel verwenden, die eine solche Person in der ART verwenden würde

    Ich bin mir immer noch sicher, dass das so falsch ist.
    Oder zumindest unsauber formuliert.

    Die Konsistenz zwischen SRT und ART führt dazu, dass die Formeln buchstäblich identisch sind.
    Egal, ob ich ART oder SRT rechne.
    Es gibt keinen Unterschied zwischen SRT-Formel und ART-Formel.

    .

    Wenn ich mit homogenen Gravitationsfeldern und dem Äquivalenzprinzip argumentiere, ist das nicht die ART.
    Es ist eine Mischung aus SRT, netwonscher Gravitation und Äquivalenzprinzip.

    .

    Unter Annahme der Gültigkeit der SRT und des Äquivalenzprinzips kann man auch andere Gravitationstheorien als die ART konstruieren.
    Ob ART, Brans-Dicke, MOND oder sonst etwas, beim Zwillingsparadoxon kommt das selbe raus.

    Wenn ich nur mit SRT + Äquivalenzprinzip argumentiere, ist das eben auch nur eine Argumentation mit Hilfe von SRT + Äquivalenzprinzip.
    Das also noch lange nicht automatisch eine ART-Argumentation.
    Es wäre erst eine ART-Argumentation, wenn Raumzeitkrümmung (genau auf ART-Weise) eine Rolle spielen würde.
    Und wäre das so, würden sich SRT-Formel und ART-Formel unterscheiden.

    .

    Nicht zuletzt ist die Argumentation über das Äquivalenzprinzip aber vor allem grundsätzlich abzulehnen, weil sie fundamental und offensichtlich Ockhams Rasiermesser verletzt.

    Es ist völlig egal, ob das Äquivalenzprinzip gültig oder ungültig ist. Beim Zwillingsparadoxon kommt in beiden Fällen das selbe raus.

    Daher kann das Äquivalenzprinzip für mich rein logisch bei der Erklärung auch prinzipiell keine Rolle spielen.

    Doppelt kann die ART keine Rolle spielen.
    Da fließen schließlich noch mehr irrelevante Zusatzannahmen als nur das Äquivalenzprinzip ein.

    .

    Ist vielleicht nitpicking und da würde ich sonst auch nicht so ein Bohei drum machen.
    Wir beide sind uns da auch einig, da bin ich mir absolut sicher.

    Aber über diese Feinheiten diskutieren wir in diesem speziellen Fall jetzt schon seit 300 Kommentaren.
    Deswegen bestehe ich hier auf diese übermäßige Genauigkeit. 😉

  410. #410 MartinB
    22. Dezember 2018

    @Niels
    Ich glaube, ich verstehe nicht, was du schreibst:
    Ich kann rein innerhalb der SRT argumentieren und die Konsequenz der Beschleunigung berechnen, darüber sind wir uns ja einig.
    Die SRT ist ein Teil der ART, ich kann also dieselbe Rechnung innerhalb der ART durchführen, die Raumzeit ist hier eh flach und beide Theorien sagen dasselbe.

    Ich kann aber alternativ auch dank des Äq-Prinzips doch die *Analogie* der Situation zu der eines “homogenen G-Feldes” sehen (zumindest in beliebig guter Näherung, in der kann es ein homogenes G-Feld ja geben – letztlich nehme ich an, ich wäre in einer Situation, in der mein lokales Lorentzsystem so groß, dass es Serena und Teresa einschließt) und dann argumentieren, dass ich die Formal auch über diese Analogie bekommen kann. So hat es ja auch Einstein 1918 in dem link oben gemacht, und ich denke nicht, dass das falsch ist. Dass es keine besonders gute Betrachtungsweise und ziemlich irreführend ist, sehe ich ja auch so.

    In einer anderen Theorie mag es diese Analogie nicht geben – wenn in unserem Universum eine ganz andere Schwerkrafttheorie richtig wäre, würde das Argument mit der Analogie halt nicht funktionieren, das ZP wäre davon aber unbeeinflusst, da stimme ich natürlich vollkommen zu.

  411. #411 Anonym_2018
    22. Dezember 2018

    @Niels #409

    Es ist völlig egal, ob das Äquivalenzprinzip gültig oder ungültig ist. Beim Zwillingsparadoxon kommt in beiden Fällen das selbe raus.

    Lass mich mal beschreiben, wie ich das verstanden habe. Ggf. kannst du mich ja korrigieren.
    Das Äquivalenzprinzip der Physik drückt aus, dass die schwere und die träge Masse eines Körpers zwei äquivalente Größen sind.

    In einem Universum ohne Äquivalenzprinzip könnten z.B. die trägen Masse sein wie bisher und die schwere Masse könnte z.B. Null sein. Das wäre z.B. eine Minkowski-Welt ohne RZK und damit wahrscheinlich auch ohne ART.

    Wenn Serena in dieser Welt aus Sicht eines inertialen Bezugssystems beschleunigt, dann tritt in ihrem (beschleunigten) Bezugssystem immer noch ein homogenes Scheinkraftfeld incl. Rot-/Blauverschiebungen auf, das eine Kraft auf die (nur trägen, aber scheinbar auch schweren) Massen ausübt. Serena würde immer noch die durch das Scheinkraftfeld verursache Zeitdilatation bei Teresa ausrechnen (z.B. Teresa altert wärend der Umkehrphase durch das Scheinkraftfeld um 6,4 Jahre + 10 Tage, während Serena um 10 Tage altert).

    Richtig?

  412. #412 Niels
    22. Dezember 2018

    @MartinB

    Ich kann aber alternativ auch dank des Äq-Prinzips doch die *Analogie* der Situation zu der eines “homogenen G-Feldes” sehen (zumindest in beliebig guter Näherung, in der kann es ein homogenes G-Feld ja geben – letztlich nehme ich an, ich wäre in einer Situation, in der mein lokales Lorentzsystem so groß, dass es Serena und Teresa einschließt) und dann argumentieren, dass ich die Formal auch über diese Analogie bekommen kann.

    Klar. Wir sind uns fachlich in Allem völlig einig, da bin ich mir sehr sicher.
    .

    Mir ging es um die Exaktheit und die Feinheiten deiner Formulierung in #401.
    Allgemein wäre das kein Problem, eigentlich sollte dich jeder mit einem bisschen gutem Willen nicht falsch verstehen können.
    Andererseits führen wir her schon seit 300 Kommentaren eine Diskussion, die das Gegenteil belegt…
    .

    Die Kritik am exakten Wortlaut im Einzelnen:

    Wegen dieser Analogie kann ich auch die entsprechende Formel verwenden, die eine solche Person in der ART verwenden würde

    Damit habe ich Probleme.
    Aber wie gesagt nur wegen der ganzen Vorgeschichte hier in den Kommentaren.

    Was ist gemeint mit
    die entsprechende Formel […] in der ART verwenden?

    Welche Formel ist gemeint?
    Eine Formel mit Gravitations-Feld oder -Pontential kann es ja nicht sein, die verwendet niemand in der ART.

    Die SRT-Formel?
    .

    Aber die SRT-Formel kann man doch nicht wegen dieser Analogie verwenden.

    Auch die ART-Formel (die identisch zur SRT-Formel ist) kann man nicht wegen dieser Analogie anwenden.
    Oder wie ist “wegen” hier von dir gemeint bzw. definiert?

    Die Konsistenz zwischen SRT und ART erfordert nach dem Äq-Prinzip, dass die Rechnung so funktioniert

    Verstehe ich ebenfalls nicht.
    Die Konsistenz zwischen SRT und ART erfordert, dass die Rechnung identisch sein muss. Klar.

    Erfordert nach dem Äq-Prinzip ist aber doch falsch?
    Das Einsteinsche Äquivalenzprinzip ist schließlich mathematisch und logisch nicht beteiligt, es ist für das funktionieren der Rechnung völlig irrelevant.

    Darauf wollte ich oben hinaus, war aber wohl zu verklausuliert.

    Außer man stellt sich auf den Stand, dass SRT und ART konsistent sind, weil es das Einsteinsche Äquivalenzprinzip in der ART gibt.
    Kann man so sehen.
    (Ich bin mir nicht ganz sicher, aber kann es nicht zur SRT konsistente Gravitationstheorien geben, in denen das Einsteinsche Äquivalenzprinzip nicht gilt?
    Können halt keine metrischen Gravitationstheorien sein, aber es gibt ja auch andere Ansätze.)

    Deswegen finde ich deinen Satz mit dem funktionieren ein bisschen irreführend formuliert.
    Jedenfalls als Fazit für diese ganz spezielle (ein bisschen absurde) Diskussion, in der die seltsamsten Dinge in Aussagen hinein interpretiert wurden.

    .

    Für die Betrachtungsweise über den *Analogie*-Schluss der Situation zu der eines “homogenen G-Feldes” verwendet man das Äquivalenzprinzip.
    Keine Frage.

    Zum einen ist das aber gerade keine *ART-Analogie*.
    Zum anderen eben eine Interpretation einer Rechnung, keine alternative Rechnung.

    Wie gesagt, das sind aber Formulierungs-Pingeligkeiten.
    Über den physikalischen Sachverhalt sind wir uns einig, dafür lege ich die Hand ins Feuer.

    .

    @Anonym_2018

    Richtig?

    Ja.
    Der Zwillings-Flug würde exakt gleich ablaufen und das gleiche Ergebnis liefern. Alle Wahrnehmungen aller Beobachter wären identisch zu unserem Universum.

    durch das Scheinkraftfeld verursache Zeitdilatation

    Ein Scheinkraftfeld verursacht aber nichts. Egal in welchem Universum.

    In diesem speziellen Universum könnte außerdem auch niemand auf die Idee kommen, dass das doch so wäre.
    Wenn es keine Gravitation gibt, wäre die Interpretation des Effektes als der eines Scheinkraftfeldes, dass gravitative Zeitdilatation verursacht, schließlich unglaublich fernliegend.

  413. #413 MartinB
    23. Dezember 2018

    @Niels
    Irgendwie reden wir aneinander vorbei, glaube ich, wahrscheinlich, weil ich wirklich zu unpräzise bin.

    Ich sehe die Analogie so: In der ART gilt in einem “homogenen Grav-Feld” (also einem Bereich, in dem die Schwerebeschleunigung in hinreichend guter Näherung konstant ist), für die Zeitdilatation zwischen zwei Beobachterinnen mit Höhendifferenz h die Formel
    delta t’=delta t (1+ gh/c²)
    Diese Formel kann man soweit ich sehe ableiten, wenn man nur annimmt, dass frei fallende Objekte sich auf Geodäten bewegen und dabei einen Weg maximaler Eigenzeit gehen. (Ich hab’s sorum nie gemacht, aber umgekehrt kann man aus dieser Formel und der maximalen Eigenzeit ableiten, dass frei fallende Objekte konstant beschleunigt werden, also sollte es auch andersrum gehen.)

    Serena kann jetzt sagen, dass ihre Situation während der Beschleunigungsphase genau analog ist zu der von jemandem, der stationär auf der Oberfläche eines gigantischen Planeten steht und der über sich in 4 Lj Entfernung Teresa frei auf sich zufallen sieht.

    Wenn wir also annehmen, dass kräftefreie Objekte Geodäten maximaler Eigenzeit folgen und dass die Schwerkraft eigentlich Raumzeitkrümmung ist, dann folgt daraus direkt die Dilatationsformel der ART und damit kann Serena dank der Analogie diese Rechnung so durchführen wie Einstein das ja auch in dem Link oben gemacht hat.

    Dass man diese Analogie nicht braucht und die gesamte Rechnung natürlich auch rein innerhalb der SRT durchführen kann, ohne dass das Äq-Prinzip oder sonstwas eine Rolle spielt, darüber sind wir uns ja auf jeden Fall einig. Diese Rechnung geht natürlich auch innerhalb der ART genauso – die ART schließt ja die SRT mit ein.

    Wenn es eine andere Schwerkrafttheorie gibt/gäbe, die ohne Geodäten maximaler Eigenzeit funktioniert, dann würde auch diese Analogie nicht gelten. Ich behaupte also nicht, dass man allein aus dem ZP (der SRT) schon die ART oder deren Grundlagen ableiten kann, aber ZP + Äq-Prinzip + Wege max. Eigenzeit zusammen erfordern soweit ich es sehe diese Dilatationsformel.
    Wobei die natürlich in der ART immer eine Näherung ist, weil es ein echtes homogenes G-Feld, das auch keinen Einfluss auf die Raumkrümmung hat, ja nicht gibt. (Und fairerweise müsste man spätestens bei der Frage nach der Entfernung zwischen Serena und Teresa mal über die Frage der Koordinaten nachdenken, denn wenn das wirklich ein homogenes G-Feld wäre, dann gäbe es ja auch eine Raumkrümmung, die die Entfernung zwischen Serena und teresa beeinflusst.)

  414. #414 Anonym_2018
    23. Dezember 2018

    @Niels #412

    durch das Scheinkraftfeld verursache Zeitdilatation

    Ein Scheinkraftfeld verursacht aber nichts. Egal in welchem Universum.

    Wenn bestimmte Formulierungen nicht passen, interessiert mich natürlich, welche denn passen würden.

    Ich habe dazu einen Vorschlag. Ggf. kannst du mich ja korrigieren.

    Als neue Formulierung schlage ich vor:
    “durch das Schwerefeld verursachte Zeitdilatation”

    Begründungen:
    1)
    Ein Schwerefeld ist ein Kraftfeld, verursacht durch Gravitation und bestimmte Trägheitswirkungen.

    • An der Erdoberfläche in unserem Universum sind die Schwerebeschleunigung und die Gravitationsbeschleunigung leicht unterschiedlich. Die Kraft des Schwerefeldes beinhaltet neben der Gravitationskraft auch noch die Fliehkraft der rotierenden Erde.

    • In Serena’s Raumschiff setzt sich dagegen die Kraft des Schwerefeldes zusammen aus 0% Gravitationskraft und 100% Fliehkraft.

    2)
    Ich möchte das Adjekt “scheinbar” vermeiden. Nach dem Relativitätsprinzip tritt das Schwerefeld im Bezugssystems des Raumschiffs tatsächlich auf und kann auch gespürt und gemessen werden.

    Richtiger Vorschlag?

  415. #415 MartinB
    23. Dezember 2018

    @Anonym
    Fliehkraft ist keine Schwerkraft.

    “Nach dem Relativitätsprinzip tritt das Schwerefeld im Bezugssystems des Raumschiffs tatsächlich auf und kann auch gespürt und gemessen werden.
    Richtiger Vorschlag?”
    Nein, immer noch falsch – es gibt in einer flachen Raumzeit keine Schwerkraft, egal wie oft du das umformulierst. Das Einstein-Argument beruht auf dem Äq-Prinzip, also darauf, dass zwei Situationen sich nicht durch lokale Messung unterscheiden lassen. Das bedeutet aber nicht, dass die Situationen identisch sind.

  416. #416 MartinB
    23. Dezember 2018

    PS Der Wiki-Satz ist ziemlicher Müll, wenn man über die ART redet, weil er unglaublich unscharf ist (“gewisse Trägheitswirkungen”?).

  417. #417 Anonym_2018
    23. Dezember 2018

    Das Einstein-Argument beruht auf dem Äq-Prinzip

    Ich benutzte aber garnicht Einstein’s Begriff und auch nicht das Äq-Prinzip. Er redet von Gravitationsfeld, ich von Schwerefeld. Das sind laut Wikipedia unterschiedliche Sachen.

  418. #418 MartinB
    23. Dezember 2018

    @Anonym
    Das macht die Sache in meinen Augen nur unnötig verwirrend. Wer außer Wiki bezeichnet denn Scheinkräfte als “Schwerefeld”? Macht vielleicht Sinn, wenn man über Planetenoberflächen redet, aber doch nicht als allgemeiner begriff.

  419. #419 Anonym_2018
    23. Dezember 2018

    @MartinB #418

    Macht vielleicht Sinn, wenn man über Planetenoberflächen redet, aber doch nicht als allgemeiner begriff.

    Im weiteren Sinne vielleicht schon:

    Im weiteren Sinne spricht man vom Schwerefeld in beliebig beschleunigten Bezugssystemen. Im Schwerefeld einer Zentrifuge dominiert die Zentrifugalkraft. In frei fallenden Bezugssystemen (Bsp. Raumstation) herrscht Schwerelosigkeit.

    Quelle:
    https://de.wikipedia.org/wiki/Schwerefeld

  420. #420 MartinB
    23. Dezember 2018

    Ich sehe nicht, was das tut außer zu verwirren, insbesondere weil ja “Schwerkraft” die Scheinkräfte hoffentlich nciht einschlißet. Dan habe ich also Schwerkraft und Schwerefeld als zwei unterschiedliche Dinge.

  421. #421 Anonym_2018
    23. Dezember 2018

    @MartinB #420

    Dan habe ich also Schwerkraft und Schwerefeld als zwei unterschiedliche Dinge.

    • Zu dem Wort “Schwerkraft” gibt es keinen deutschen Wikipedia-Artikel.
    • Die zum Schwerefeld gehörende Kraft ist bei Wikipedia wie folgt definiert:

    Die Gewichtskraft, auch Gewicht, ist die durch die Wirkung eines Schwerefeldes verursachte Kraft auf einen Körper.

    Quelle:
    https://de.wikipedia.org/wiki/Gewichtskraft

  422. #422 MartinB
    23. Dezember 2018

    @Anonym
    Ja, toll, und was sollen diese begrifflichkeiten, wenn wir über die ART reden, wo es in der normalen geometrischen Formulierung schlicht keine Schwerkraft gibt?

    Du machst es dir wirklich nicht leicht, wenn du 100 Jahre alte Einstein-Zitate und Wiki-Artikel als Quellen nutzt, um die ein Bild der ART zu zimmern.

  423. #423 Anonym_2018
    23. Dezember 2018

    @MartinB #422

    wenn wir über die ART reden

    Durch meine Beschreibung in Kommentar #411 (bestätigt von Niels in #412) wird doch klar, dass wir beim ZP nicht über ART reden, auch nicht im Bezugssystem von Serena.

    Ältere Kommentare von mir um ZP sind diesbezüglich teilweise fehlerhaft. Ich lerne ja dazu.

  424. #424 MartinB
    23. Dezember 2018

    @Anonym_2018
    Im Rahmen der SRT macht der Begriff “Schwerefeld” oder “Gewichtskraft” natürlich viel mehr Sinn. [/sarcasm]

  425. #425 MartinB
    23. Dezember 2018

    PS und es wäre auch hilfreich, wenn du bei so einer Diskussion ab und zu mal sagen würdest, dass du Punkt X oder Argument Y jetzt akzeptierst, die du vorher für falsch erklärt hast, das würde das Erklären auch wesentlich vereinfachen. Wenn du jetzt einsiehst, dass die ART mit dem ZP nicht wirklich viel zu tun hat, worum geht es denn jetzt noch?

  426. #426 Anonym_2018
    23. Dezember 2018

    @MartinB #424

    Im Rahmen der SRT macht der Begriff “Schwerefeld” oder “Gewichtskraft” natürlich viel mehr Sinn. [/sarcasm]

    Das wurde im Prinzip schon von Niels in #409 beantwortet. Der weiss das bestimmt besser als ich:

    Wenn ich mit homogenen Gravitationsfeldern und dem Äquivalenzprinzip argumentiere, ist das nicht die ART.
    Es ist eine Mischung aus SRT, netwonscher Gravitation und Äquivalenzprinzip.

  427. #427 Anonym_2018
    23. Dezember 2018

    @MartinB #425

    Wenn du jetzt einsiehst, dass die ART mit dem ZP nicht wirklich viel zu tun hat, worum geht es denn jetzt noch?

    Das hatte sich jetzt überschnitten.

    Momentan geht es darum, dass ich die optimalen Worte finden möchte, um in Serenas Bezugssystem die durch das dortige Schwerefeld(?) verursachte(?) Zeitdilatation bei Teresa charakterisieren zu können und begrifflich von der Geschwindigkeits-abhängigen Zeitdilatation differenzieren zu können. Das sollte im Idealfall möglich sein, ohne auf Serena’s Beschleunigung referenzieren zu müssen, die es in diesem Bezugssystem per Definition ja garnicht gibt (Grund: Relativitätsprinzip).

  428. #428 Niels
    23. Dezember 2018

    @MartinB

    Ich bin jetzt leider nur für fünf Minuten am Computer und den Rest der Feiertage gar nicht mehr.
    Deswegen nur ganz kurz ein paar Gedankenfetzen, damit eine Antwort nicht erst in sechs Tagen kommt.

    delta t’=delta t (1+ gh/c²)
    Diese Formel kann man soweit ich sehe ableiten, wenn man nur annimmt, dass frei fallende Objekte sich auf Geodäten bewegen und dabei einen Weg maximaler Eigenzeit gehen.

    Tatsächlich?
    Ist mir spontan nicht klar, müsste man mal konkret porbieren.

    Ich kenne eine ART-Herleitung dieser Formel nur darüber, dass man die Schwarzschild-Metrik nimmt und dort die gravitative Zeitdilatation berechnet.

    Für schwache Krümmungem liefert die obige Formel eine gute Näherung dieser Schwarzschild-Formel.

    Mathematisch entspricht das einer Reihnentwiklung der ART-Formel, die man bei frühstmöglicher Ordnung abbricht

    ZP + Äq-Prinzip + Wege max. Eigenzeit zusammen erfordern soweit ich es sehe diese Dilatationsformel

    Hm, kommt mir gerade nicht hundertprozentig koscher vor. Müsste ich noch mal drüber nachdenken.

    Irgendwie reden wir aneinander vorbei, glaube ich, wahrscheinlich, weil ich wirklich zu unpräzise bin.

    Wie gesagt, ich bin mir sehr sicher, das wir eigentlich völlig einer Meinung sidn.

    Wir können das auch gerne ab dem 28.12. noch mal ausführlicher fortsetzen.
    Dann würden wir uns aber auch nur um Formulierungen und Definitionen umgangssprächlicher Ausdrücke herumstreiten.
    Würde mich jedenfalls sehr wundern, wenn das dich oder mich beim physikalischen Verständnis auch nur das kleinste Fitzelchen weiterbirngt.

    Deswegen hätte ich momentan auch kein Problem, das Ganze hier abzuschließen.

    .

    Ich wünsche euch allen frohe Weihnachten.
    Ganz herzlichen Dank für deinen tollen Blog, MartinB.

  429. #429 Anonym_2018
    23. Dezember 2018

    In meiner ZP-Berechnung in #274 hatte ich die von mir gefundene Formel
    ∆t = v/c² * L verwendet.

    In deren Herleitung werden einfach SRT-Berechnungen vor und nach der Beschleunigung in einem Inertialsystem – für eine beschleunigte Rakete – gemacht und dann auf das Raketen-Bezugssystem umgerechnet.

    Ich werde nun zeigen, dass ich die gleiche Formel aus den SRT-Phasen des ZP (durch zurückrechnen auf die “fehlende” Teresa-Alterung) allgemein herleiten kann:

    Dazu verwende ich:
    • L = räumliche Entferung von Teresa bis zum Umkehrpunkt (in Teresa’s Bezugssystem)
    • w = Geschwindigkeit von Serena’s Rakete
    • TTT = Alterung von Teresa in Teresa’s Bezugssystem während Serena’s Reise
    • TST = Alterung von Serena in Teresa’s Bezugssystem während Serena’s Reise
    • TTS = Alterung von Teresa in Serena’s Bezugssystem während Serena’s Reise
    • ∆t = “fehlende” Teresa-Alterung

    Berechnung:

    ∆t = TTT – TTS

    mit TTS = TST * √(1-w²/c²) = TTT * (1-w²/c²)
    … ergibt sich:

    ∆t = TTT – TTT * (1-w²/c²)

    mit TTT = 2 * L / w
    … ergibt sich:

    ∆t = 2 * L / w (1 – 1 + w²/c²)
    ∆t = 2 * L / w (w²/c²)
    ∆t = 2 * L * w/c²

    mit v = 2 * w (Geschwindigkeitsdifferenz beim Umkehren)
    … ergibt sich:

    ∆t = v/c² * L

    Das ist die gleiche Formel wie oben. Beide SRT-Herleitungen in Inertialsystemen müssen also mathematisch äquivalent sein.

  430. #430 Torq
    23. Dezember 2018

    Diskussionen finde ich gut, das ist sogar eine der Stärken von Martins Blog. Aber diese hier finde ich schon fast suspekt… Wie schade, dass die von Martin investierte Zeit nicht in einen neuen Artikel geflossen ist… My two cents…

  431. #431 Anonym_2018
    23. Dezember 2018

    Erläuterung zur Berechnung in #429:

    In der Berechnung sind nur die Zeiten aus den üblichen “unphysikalischen” Szenarien berücksichtig, wo die Umkehrzeit mit Null angesetzt wird. Wegen der Differenzbildung führt das aber zu keinem Fehler bei der Berechnung der Formel.

    In einem physikalisch möglichen Szenario muss man noch eine endliche Zeit für die Umkehr ansetzen, z.B. 10 Serena-Tage in Serena’s Bezugssystem. Teresa würden dann während der Umkehr um ∆t + 10 Tage in Serena’s Bezugssystem altern. D.h. Teresa und Serena würden beide um 10 Tage mehr altern als im Szenario der obigen Rechnung, das würde sich bei der Berechnung der Formel aber wieder wegsubtrahieren, also nicht auswirken.

  432. #432 MartinB
    24. Dezember 2018

    @Niels
    “Tatsächlich?
    Ist mir spontan nicht klar, müsste man mal konkret porbieren.”

    Ich habe es wie gesagt nie explizit so rum gerechnet.
    Aber wenn man eine konstante Fallbeschleunigugn haben will, dann geht das ja über einem mit der Höhe linear zunehmenden Faktor aus der Gleichung oben.
    Habe ich ja mal hier durchgerechnet:
    https://scienceblogs.de/hier-wohnen-drachen/2017/01/13/der-freie-fall-und-die-maximale-eigenzeit/

    Also Weg max. Eigenzeit + linear höhenabhängige Dilatation gibt konstante Schwerebeschleunigung und ne Wurfparabel.

    Vielleicht gibt es noch andere Möglichkeiten, das konsistent hinzubekommen, ich wüsste aber auf Anhieb nicht, wie.

    @Anonym
    Ich habe mir die Rechnung jetzt nicht mehr im Detail angeguckt, die tut doch nichts anderes als rückwärts zu berechnen, wie groß die Zeitdifferenz aus Serenas Sicht wäre, wenn man ignoriert, dass sie das Bezugssystem wechselt.

  433. #433 Anonym_2018
    24. Dezember 2018

    @MartinB # 432

    die tut doch nichts anderes als rückwärts zu berechnen, wie groß die Zeitdifferenz aus Serenas Sicht wäre, wenn man ignoriert, dass sie das Bezugssystem wechselt.

    Das ist richtig. Das interessante ist aber, dass bei dieser Rückrechnung die gleiche Formel hearuskommt, die für die Berechnung der gravitativen Zeitdilatation verwendet wird (dort wird mit dem Äquivalenzprinzip argumentiert). Diese Formel passt also damit nicht nur bei dem konkreten Zahlenbeispiel in Kommentar #274 sondern für beliebige Zahlenbeispiele.

  434. #434 MartinB
    24. Dezember 2018

    @Anonym
    Ja, ich dachte, das wäre klar.

  435. #435 Anonym_2018
    24. Dezember 2018

    Anmerkung zur Erläuterung in #431 (zur Berechnung in #429):

    Ich habe bei den Eigenzeit-Angaben in Erläuterung #431 den Geschwindigkeits-abhängigen Anteil der Zeitdilatation während der Umkehrphase vernachlässigt, weil diese Abweichungen i.d. Regel um Größenordnungen kleiner sind als die übrigen Zeiten.

  436. #436 Anonym_2018
    27. Dezember 2018

    @MartinB #413

    (Und fairerweise müsste man spätestens bei der Frage nach der Entfernung zwischen Serena und Teresa mal über die Frage der Koordinaten nachdenken, denn wenn das wirklich ein homogenes G-Feld wäre, dann gäbe es ja auch eine Raumkrümmung, die die Entfernung zwischen Serena und teresa beeinflusst.)

    Laut dem folgenden Text scheint bei einem 100%-tig homogenen Schwerefeld keine Raumkrümmung vorzuliegen (die Richtigkeit dieses Textes kann ich nicht beurteilen):

    Ein feldfreier Raum kann durch ein Inertialsystem beschrieben werden. Befindet sich in dem Raum dagegen ein gleichmäßiges Gravitationsfeld in eine Richtung, so muss man das Koordinatensystem so modifizieren, dass die Zeit in Richtung des Gravitationsfeldes immer langsamer vergeht. Solch ein Koordinatensystem könnte man als in der Zeit gekrümmt bezeichnen: Während die Raumkoordinaten überall den gleichen Maßstab verwenden, wird der Zeitmaßstab in das Feld hinein immer länger, die vergehende Zeit langsamer.
    Nach der üblichen Definition von Krümmung, die ich auf der nächsten Seite erklären werde, ist solch eine rein Zeitliche Verzerrung allerdings noch keine Krümmung. Ein beschleunigtes Koordinatensystem ist schließlich nur eine andere Darstellung der gewöhnlichen, flachen Raumzeit. Die allgemeine Relativitätstheorie beschreibt Gravitation über eine Krümmung der Raumzeit, die sowohl Raum als auch Zeit verzerrt.

    Quelle:
    https://www.relativitätsprinzip.info/gravitation-durch-kruemmung.html

    Die Frage der Entfernung zwischen Serena und Teresa ist, so glaube ich, im Bezugssystem von Serena trotzdem nicht trivial. Während der Beschleunigung des Bezugssystems ändert sich dieser Abstand und auch die Geschwingkeit, die für das “Gamma” bei der Längenkontraktion dieses Abstandes zuständig ist.

    Nach meiner Berechnung z.B. in #429 passt die Formel
    ∆t = v/c² * L (mit v = Geschwindigkeitsänderung)
    … wenn man für “L” den Abstand ohne Längenkontraktion einsetzt.

    Den Grund dafür habe ich noch nicht ganz verstanden. Dass kann man aber bestimmt berechnen, auch wenn das nicht ganz einfach ist.

    Dass aber der Abstand ohne Längenkontraktion eingesetzt werden muss, wird auch im deutschen Wikipedia-Artikel über das ZP bestätigt:

    In jedem dieser Inertialsysteme ergibt sich jedoch für den Zeitpunkt, der gleichzeitig auf der Erde herrscht, ein anderer Wert und zwar derart, dass der fliegende Zwilling auf eine Nachalterung des irdischen schließt. Je weiter sich die Zwillinge voneinander entfernt haben, umso größer ist dieser Effekt. ( Δ τ = Δ v ⋅ x ′ / c²
    mit x ′ = x γ „ursprüngliche“ Entfernung im unbeschleunigten System
    ).

    Quelle:
    https://de.wikipedia.org/wiki/Zwillingsparadoxon#Variante_mit_Beschleunigungsphasen

  437. #437 Ralf
    17. Juni 2019

    Guten Tag die Herren Physikerinnen,

    ich traue mich mal, auch meinen Senf an diesen langen Blog zu hängen. Ich lese ihn nun schon eine Weile und irgendwie habe ich das Gefühl, daß die Diskussion immer wieder vom Wesentlichen Punkt abgleitet in Diskussionen um Beschleunigungen. Warum ist das so? Vielleicht hilft es, wenn ich eine Zusammenfassung der Grundgedanken versuche:

    Das Diagramm, das MartinB ganz oben im Artikel zeigt, zeigt sehr schön, daß die Reise in zwei symmetrische Phasen und und ein unsymmetrisches Ereignis zwerfällt: Während der Flüge sieht jeder der beiden Zwillinge den anderen weniger altern. Das ist symmetrisch und somit nicht verantwortlich für eine Altersdifferenz nach der Rückkehr. Das Interessante geschieht im Moment des Umkehrens des einen Zwillings (“Bezugssystemwechsel”). Aus der Sicht (Gleichzeitigkeitslinien) des Umkehrenden wird bein Bezugssystemwechsel ein Stück des Lebens des ruhenden Zwillings übersprungen. (Formulierungsversuch: “Der umkehrende Zwilling bezieht sich vom neuen Bezugssystem aus auf einen späteren ruhenden Zwilling”). Das sagt das Minkowski-Diagramm und das kann man so ausrechnen. In dieser Rechnung kommen keine Beschleunigungen vor. Das heißt, daß etwaige Beschleunigungen, wenn denn welche auftreten sollten, zumindest (aus dem Modell der SRT heraus) nicht ursächlich für den Altersdifferenz-Effekt sind.

    Wo ist jetzt das Paradoxon und warum kommt immer wieder die Diskussion mit der Beschleunigung ins Spiel? Wenn ich versuche, über den Reiseablauf nachzudenken, bleibe ich immer wieder an folgenden Gedanken hängen:

    1) Wenn die Modalitäten der Richtungsumkehr des einen Zwillings nicht relevant sind für das Auftreten des Altersunterschiedes nach der Rückkehr (sonder nur die Tatsache, daß eine Umkehr stattfindet), dann bleiben nur noch die beiden Reisen übrig zum drüber Nachdenken – dort entsteht aber der Unterschied nicht. Es drängt sich immer die Überlegung auf, daß ja der Raketenzwilling stehen und die Erde sich bewegen könnte (incl. Richtungswechsel)). Symmetrisch halt, also Gedankensackgasse.
    2) Das Vorzeichen der Altersdifferenz nach der Umkehr zeigt, ob die Rakete oder die Erde auf Reise war.
    Die Punkte 1 und 2 zusammen sehen schon paradox aus.

    Das Minkowski-Diagramm zeigt, daß Ereignisreihenfolgen beobachterabhängig sind – das führt in meinen Augen direkt zum Blockuniversum. Das BU erinnert auf den ersten Blick an den “absolutem Raum”, obwohl es so nicht ist.

    Ich vermute, daß die Ursache für das Wiederkehren der Diskussion um die Beschleunigung darin liegt, daß der Begriff “Bezugssystem” hier nicht klar genug verstanden ist und deshalb auch nicht klar ist, was ein Wechsel zwischen solchen physikalisch bedeutet. Eigentlich kann nur das das Gedankenproblem liegen, denn alle anderen Phasen der Reise sind nicht für den Altersunterschied verantwortlich.
    Wodurch unterscheidet sich physikalisch die Umkehr des einen Zwillings sich von der (gedachten) Umkehr der Erde? Was ist der eigentliche physikalische Umstand, der die Symmetrie bricht, wenn es weder ein absoluter Raum (Newton), noch die Beschleunigung ist?

    Ich denke, einen absoluten Raum favorisieren die wenigsten, aber Mangels besserer Ideen bleibt man dann an der Beschleunigung hängen. Hier fehlt eine knackige Formulierung, was wirklich des Pudels Kern ist – weiß jemand eine? Mir fällt gerade nix Gescheites dazu ein…

    Die SRT beschreibt eine Kinematik im Bockuniversum – kann man diese Frage überhaupt klären ohne die Grenzen dieses Ansatzes zu verlassen? Vielleicht nicht, und das würde erklären, warum die Diskussion nicht endet. Man will halt wissen: “Wenn’s an der Beschleunigung nicht liegt, woran dann?” Die Antwort “Am Richtungswechsel” ist unbefriedigend, denn ohne ein greifbares Ereignis, daß den Raketen-Richtungswechsel vom gedachten Erd-Richtungswechsel unterscheidet, fragt man sich was daran unsymmetrisch ein soll – und das sieht paradox aus.

    Es ist spät, ich hoffe, das war nicht zu konfus – wenn doch, tut’s mir leid, ich will keinen weiteren Zyklus anstoßen.

    P.S.: Danke, Martin für Dein gutes Buch – ich lese es gerne.

  438. #438 MartinB
    https://scienceblogs.de/hier-wohnen-drachen
    18. Juni 2019

    @Ralf
    Ich verstehe deinen Punkt nicht wirklich – wen du statt “absoluten Raum” schlicht “Raumzeit” setzt, ist sofort klar, dass eine Strecke aus zwei Stücken länger (also kürzere Eigenzeit) ist als der direkte Weg. Die Raumzeit ist eben nicht relativ, Raumzeitabstände sind ja invariant.
    Und natürlich muss ein Objekt, um einer solchen Linie zu folgen, auch beschleunigen, aber die Linie aus zwei Stücken ist in der Raumzeit immer länger, egal ob ein Objekt ihr folgt oder nicht, das ist schlicht Geometrie.

  439. #439 Anonym_2019
    18. Juni 2019

    @Ralf (17. Juni 2019)

    “Es drängt sich immer die Überlegung auf, daß ja der Raketenzwilling stehen und die Erde sich bewegen könnte (incl. Richtungswechsel)).”

    Diese Überlegung ist richtig. In der SRT darf man auch ein nicht-inertiales Bezugssystem, wie das Ruhesystem des Raketenzwillings, wählen, um die Abläufe zu beschreiben. In dem Ruhesystem des Raketenzwillings existiert während des Richtungswechsels ein homogenes Schwerefeld, weil die Rakete durch die Kraft ihres Antriebes beschleunigt wird (=absolute Beschleunigung). Die Erde befindet sich in diesem Schwerefeld im freien Fall (=Koordinatenbeschleunigung) in großer “Höhe”, wo ein schnellerer Zeitablauf durch “gravitative” Zeitdilatation vorhanden ist. Der wird häufig vergessen.

    Im Inertialsystem des Erd-Zwillings oder des Minkowski-Diagramms sieht das ganze völlig anders aus. Das Endergebnis ist aber das selbe.

    Die Asymmetrie liegt in “absolute Beschleunigung” vs. “Koordinatenbeschleunigung”. Nur kinematische Betrachtungen zeigen die Asymmetrie nicht.

  440. #440 Ralf
    18. Juni 2019

    Hi, Martin,

    Die Raumzeit ist eben nicht relativ, Raumzeitabstände sind ja invariant.

    Danke für den Schubs. Ich denke, das ist der Punkt, der Verwirrung schafft (zumindest bei mir, also vielleicht auch bei anderen).

    wen du statt “absoluten Raum” schlicht “Raumzeit” setzt, ist sofort klar, dass eine Strecke aus zwei Stücken länger (also kürzere Eigenzeit) ist als der direkte Weg.

    Ist schon klar. Mein Kopf weiß das, mein Bauch aber nicht – der ist nicht intelligent genug, will aber immer mitreden 😉 Ich glaube, das nennt man “Intuition”.

    Mein Anliegen ist es, eine kurze, klare Aussage zu finden, die das Wiederkehren der Beschleunigungs-Diskussion beendet, denn deren Existenz ist ein Hinweis darauf, daß viele Leser in die gleiche Falle tappen und dadurch am Thema vorbeidenken.

    Es ist nicht intuitiv, gedanklich von einem gültigen Äq-Prinzip (relativer Raum) auf eine nicht relative Raumzeit zu kommen.
    Es ist viel verführerischer zu denken, daß weil während der Flüge Symmetrie herrscht (jeder beobachtet einen weniger alternden Zwilling), deshalb während der Flüge auch kein Altersunterschied anwachsen könne. Somit scheinen die Flüge irrelevant und man hat nur noch den Richtungswechsel übrig als potentielle Ursache für den Altersunterschied – leider falsch, aber vielleicht kommt hier die Motivation für die Beschleunigungs-Diskussion her.
    Leider lenkt auch das Minkowski-Diagramm das Auge sofort auf den Punkt der Umkehr und suggeriert dadurch die Annahme, daß es der Richtungswechsel ist, der einen Altersdifferenz-Sprung bewirkt (Umklappen der Gleichzeitigkeits-Linien) – ebenfalls nicht intuitiv.

    Es ist aber der Raumzeit-Distanz-Unterschied (nicht der Raum-Distanz-Unterschied), der während der Flüge entsteht und über die Dreiecks-Ungleichung den Altersunterschied bedingt. Ein Raum-Distanz-Unterschied entsteht natürlich auch, aber der bewirkt keine Altersdifferenz.

    Vielleicht läßt sich die Verwirrung also dadurch lösen, daß man sich erst einmal klar macht, daß der Raum relativ, die Raumzeit aber absolut ist. Dann denkt man beim ZP auch nicht mehr über Beschleunigungen nach. Als ich zum ersten mal von der SRT hörte, war diese Erkenntnis für mich der schwerste Schritt, der Rest folgt einfach daraus.

    War das klarer als mein Beitrag von gestern?

  441. #441 MartinB
    https://scienceblogs.de/hier-wohnen-drachen
    19. Juni 2019

    @Ralf
    “Vielleicht läßt sich die Verwirrung also dadurch lösen, daß man sich erst einmal klar macht, daß der Raum relativ, die Raumzeit aber absolut ist. ”
    Das ist zumindest mein Standpunkt, so versuche ich es ja auch in meinem Buch deutlich zu machen.

    @Anonym_2019
    “In der SRT darf man auch ein nicht-inertiales Bezugssystem, wie das Ruhesystem des Raketenzwillings, wählen, um die Abläufe zu beschreiben. In dem Ruhesystem des Raketenzwillings existiert während des Richtungswechsels ein homogenes Schwerefeld”
    In der SRT entsteht kein homogenes Schwerefeld, allenfalls in der ART, aber auch da ist die Vorstellung, dass eine Beschleunigung ein Schwerefeld “ist”, hoch problematisch, wie wir ja schon mehrfach diskutiert haben.

  442. #442 MartinB
    https://scienceblogs.de/hier-wohnen-drachen
    19. Juni 2019
  443. #443 Tetrapack
    3. August 2019

    Dieser Link enthält etwas interessantes:

    https://mahag.com/neufor/viewtopic.php?f=6&t=893&p=150161#p150157

  444. #445 Arrakai
    4. August 2019

    Das Ende der Speziellen Relativitätstheorie.

    “Das ganze Kartenhaus fällt nun zusammen…
    Die SRT ist…da fehlen mir die Worte”

    Mir fehlen die Worte ebenfalls. Ich erzittere.

  445. #446 The Ugly
    4. August 2019

    Wikipedia:
    “Ein Bezugssystem ist in der Physik ein gedachtes raum-zeitliches Gebilde, das erforderlich ist, um das Verhalten ortsabhängiger Größen eindeutig und vollständig zu beschreiben. ”

    Die Heilige Muttergottes Wikipedia hat gesagt, was Bezugssystem ist. (Das ist echt lustig)
    Jetzt sind wir auch in der Lage zu sagen, was ein Bezugssystem bewirken kann und was nicht:

    Ein Bezugssystem bestimmt nicht, was in Wirklichkeit passiert.

    Mein Bezugssystem bestimmt höchstens meine Perspektive auf das, was mit oder ohne mich, passiert.

    Ein Bezugssystem ist nicht im Geringsten dazu fähig, die Ausbreitungsgeometrie und die Geschwindigkeit der Lichtstrahlen zu verändern.

    Ein Bezugssystem erzeugt keine Wirkung.

    Das ist echt lustig

  446. #447 The Ugly
    4. August 2019

    Ein Bezugssystem ist nicht im Geringsten dazu fähig, die Ausbreitungsgeometrie der Lichtstrahlen [b]und die Geschwindigkeit, mit der die Zeit vergeht[/b], zu beeinflussen.

    Grund:
    Ein Bezugssystem erzeugt keine Wirkung.

    Das ganze Kartenhaus ist am Boden…

  447. #448 Anonym_2019
    4. August 2019

    The Ugly (4. August 2019) #446, #447

    “Mein Bezugssystem bestimmt höchstens meine Perspektive auf das, was mit oder ohne mich, passiert.”

    Das stimmt. Und die Gangrate der Eigenzeit geradlinig bewegter Myonen ist z.B. von meiner Perspektive abhängig. Perspektiven-unabhängig wird der Altersunterschied relativ zueinander bewegter Myonen erst gemacht, indem das Zwillingsparadoxon realisiert wird, d.h. indem sich die Zwillinge mindestens 2x zum Altersvergleich treffen. Das ZP wurde in einem Kreisbeschleuniger des CERN mit Myonen realisiert. Den Altersunterschied der Myonen im experimentellen ZP kann man alleine mit der SRT ausrechnen.

    Siehe hierzu Abbildung 4:
    https://www.desy.de/~pschmues/Relativitaetstheorie.pdf

  448. #449 The Ugly
    4. August 2019

    Die Wirklichkeit ist unabhängig davon, aus welchem BS sie betrachtet wird. Die Bezugssysteme haben keinen Einfluss auf das individuelle Altern, weil sie keine Wirkung auf die Wirklichkeit besitzen.
    Das Wirkende sind nicht Bezugssysteme. Das Wirkende ist die Wirklichkeit.

    Unsere Bezugssystem-Zugehörigkeit entscheidet unsere Perspektive, nicht wie Einstein uns einreden will, die Wirklichkeit.
    Die letztere spielt sich unabhängig von der Existenz und unabhängig vom Bewegungszustand der Bezugssysteme ab.
    So einfach ist das:

    No Paradox, no SRT…

    Die SRT postuliert, dass Wirklichkeit, ihre Abstände, ihre Ausbreitungssymmetrie des Lichtes, ihre Zeitintervalle etc. in verschiedenen Bezugssystemen unterschiedlich [b]SIND[/b].
    Bezugssysteme sollen die Wirklichkeit sein!

    Die Bezugssysteme gehören aber der Wirklichkeit gar nicht an. Sie sind GEDACHTE GEBILDE…

    Welchen Einfluss sollen sie denn auf das Licht, auf die Geometrie, auf die Zeit und den Raum haben?

  449. #450 The Ugly
    4. August 2019

    “Die Wirklichkeit ist unabhängig davon, aus welchem BS sie betrachtet wird. Die Bezugssysteme haben keinen Einfluss auf das individuelle Altern, weil sie keine Wirkung auf die Wirklichkeit besitzen.
    Das Wirkende sind nicht Bezugssysteme. Das Wirkende ist die Wirklichkeit.

    Unsere Bezugssystem-Zugehörigkeit entscheidet unsere Perspektive, nicht wie Einstein uns einreden will, die Wirklichkeit.
    Die letztere spielt sich unabhängig von der Existenz und unabhängig vom Bewegungszustand der Bezugssysteme ab.
    So einfach ist das:

    No Paradox, no SRT…

    Die SRT postuliert, dass Wirklichkeit, ihre Abstände, ihre Ausbreitungssymmetrie des Lichtes, ihre Zeitintervalle etc. in verschiedenen Bezugssystemen unterschiedlich [b]SIND[/b].
    Bezugssysteme sollen die Wirklichkeit sein!

    Die Bezugssysteme gehören aber der Wirklichkeit gar nicht an. Sie sind GEDACHTE GEBILDE…

    Welchen Einfluss sollen sie denn auf das Licht, auf die Geometrie, auf die Zeit und den Raum haben?”

  450. #451 MartinB
    4. August 2019

    @theUgly
    Hör bitte auf, hier solchen Unsinn zu posten.
    Es ist gar nicht schlimm, die SRT nicht zu vrstehen, aber man sollte die eigene Ahnungslosigkeit nicht zum Maßstab erheben.

  451. #452 The Ugly
    4. August 2019

    @Anonym_2019 #448

    Bis auf die Tatsache, dass sich laut SRT nicht sagen lässt, welcher Myon sich bewegt und welcher ruht, stimmt deine Aussage.
    Wenn allerdings die SRT wörtlich genommen wird, dann sollten Myonen wechselseitig langsamer altern.
    So oder so ein fiasko.
    Jetzt aber Schluss, der Mafia-Boss wird hier ungemütlich, wenn ich….(tödlich getroffen sinkt er zu Boden)

  452. #453 Anonym_2019
    4. August 2019

    @The Ugly (4. August 2019) #450

    “Unsere Bezugssystem-Zugehörigkeit entscheidet unsere Perspektive, nicht wie Einstein uns einreden will, die Wirklichkeit.”

    Das erste sagtest du bereits und ich habe dem zugestimmt. Das zweite halte ich für ein Strohmann-Argument, weil nach meiner Kenntnis A. Einstein nie so etwas behauptet hat.

    @The Ugly (4. August 2019) #452

    “Wenn allerdings die SRT wörtlich genommen wird, dann sollten Myonen wechselseitig langsamer altern.
    So oder so ein fiasko.”

    Das ist kein Widerspruch, weil jedes Myon die Gangrate der Eigenzeit des anderen nur anhand seiner eigenen Zeit, genannt “Eigenzeit”, beurteilt. Das Postulat von I. Newton zur “absoluten Zeit” hat A. Einstein zugunsten der beiden SRT-Postulate verworfen.

  453. #454 The Ugly
    4. August 2019

    Hast du das Argument mit vier Blitzen von ihm gelesen?
    Ziffer B, #444

    Da wird gezeigt, dass die Bezugssysteme doch gar keinen Einfluss auf die Wirklichkeit besitzen.

  454. #455 Anonym_2019
    4. August 2019

    @The Ugly (4. August 2019)

    “Hast du das Argument mit vier Blitzen von ihm gelesen?”

    Nein. Aus Sicherheitsgründen klicke ich nicht auf Links, die mir dubios erscheinen.

  455. #456 The Ugly
    4. August 2019

    @Anonym_2019
    “Unsere Bezugssystem-Zugehörigkeit entscheidet unsere Perspektive, nicht wie Einstein uns einreden will, die Wirklichkeit.”

    Du: “Das erste sagtest du bereits und ich habe dem zugestimmt. Das zweite halte ich für ein Strohmann-Argument, weil nach meiner Kenntnis A. Einstein nie so etwas behauptet hat.”

    Wikipedia dazu:
    »Die in der Relativitätstheorie gefundene Lösung geht davon aus, dass Abstände, Winkel und Zeitintervalle in verschiedenen Bezugssystemen unterschiedlich SEIN können.«

    D.h. die physikalischen Eigenschaften der Wirklichkeit (darin Naturkonstanten) SIND in verschiedenen Bezugssystemen unterschiedlich.

    adé RP

  456. #457 Herr Senf
    4. August 2019

    Naturkonstanten sind invariant, sonst sind sie keine.
    hy RP

  457. #458 Anonym_2019
    4. August 2019

    @The Ugly (4. August 2019)

    “D.h. die physikalischen Eigenschaften der Wirklichkeit (darin Naturkonstanten) SIND in verschiedenen Bezugssystemen unterschiedlich.”

    Nein. Naturkonstanten sind doch in dem Wikipedia-Zitat garnicht mit aufgelistet.

    Physikalische Theorien, wie die SRT oder Newton’s “Principia”, starten oft mit Postulaten (Annahmen) und liefern dann mathematische Voraussagen unter dem Vorbehalt, dass diese Annahmen stimmen. Durch Experimente oder Aufdecken von Inkonsistenzen in der Theorie können diese Vorassagen dann ggf. falsifiziert werden.

    Untersuchungen über die “Wirklichkeit” gehören zur Metaphysik, einem Teilgebiet der Philosophie. Ich habe allerdings den Eindruck, dass die Metaphysik bisher keinen Mehrwert geliefert hat.

    Siehe auch unter Metaphysikkritik:
    https://de.wikipedia.org/wiki/Metaphysik#Metaphysikkritik

  458. #459 The Ugly
    4. August 2019

    »Die in der Relativitätstheorie gefundene Lösung geht davon aus, dass Abstände, Winkel und Zeitintervalle in verschiedenen Bezugssystemen unterschiedlich SEIN können.«

    D.h. die FUNDAMENTALEN physikalischen Eigenschaften der Wirklichkeit (Raum und Zeit), in verschiedenen Bezugssystemen unterschiedlich SIND.

    adé RP (es verändert sich dadurch bezugssystemabhängig die Form der Naturgesetze: z.B. Massen- oder Impulserhaltung)

  459. #460 Anonym_2019
    4. August 2019

    @The Ugly (4. August 2019) #459

    “(Raum und Zeit), in verschiedenen Bezugssystemen unterschiedlich SIND.”

    Bezugssystem-unabhängig ist laut SRT der raumzeitliche Abstand zwischen zwei Ergeignissen. Dessen Aufteilung in die räumliche und zeitliche Komponente ist Bezugssystem-abhängig.

    “es verändert sich dadurch bezugssystemabhängig die Form der Naturgesetze: z.B. Massen- oder Impulserhaltung”

    Bezugssystem-unabhängig ist das Längenquadrat des Energie-Impuls-Vektors.

  460. #461 The Ugly
    4. August 2019

    “Bezugssystem-unabhängig ist laut SRT der raumzeitliche Abstand zwischen zwei Ergeignissen. Dessen Aufteilung in die räumliche und zeitliche Komponente ist Bezugssystem-abhängig.”

    Stimmt nicht: Abstände und Zeitintervale sind relativ.
    Was nicht bezugssystemabhängig ist, sind die S-Werte in S und S’-Werte in S’…

    “Bezugssystem-unabhängig ist das Längenquadrat des Energie-Impuls-Vektors.”

    Ich meine den relativistischen Impuls.

  461. #462 Anonym_2019
    4. August 2019

    @The Ugly (4. August 2019)
    Abstände und Zeitintervale sind relativ.
    Ja:
    • Der zeitliche Abstand zwischen zwei Ereignissen ist nicht Lorentz-invariant.
    • Der räumliche Abstand zwischen zwei Ereignissen ist nicht Lorentz-invariant.
    • Aber: Der raumzeitliche Abstand zwischen zwei Ereignissen ist Lorentz-invariant.

    “Ich meine den relativistischen Impuls.”

    Soviel wie ich weiss, gilt in einem Inertialsystem die Erhaltung des relativistischen Impulses.

    Ob die Massenerhaltung gilt, häng von der Definition des Wortes “Masse” ab:
    • Wenn die Ruhemasse gemeint ist, gilt die Massenerhaltung nicht.
    • Wenn die relativistische Masse gemeint ist (heute unüblich), gilt die Massenerhaltung.

    Meiner Ansicht nach könnte man die Ruhemasse auch “Ruheenergie durch c² ” nennen und den Begriff “Masse” ersatzlos streichen, weil redundant zur Energie. Die “Ruhemasse” eines Protons besteht zu 99 Prozent aus Bindungsenergie über die Gluonen und zu 1 Prozent aus Energie durch Wechselwirkung mit dem Higgsfeld.

  462. #463 The Ugly
    4. August 2019

    “Abstände und Zeitintervale sind relativ.”

    “Ja:
    • Der zeitliche Abstand zwischen zwei Ereignissen ist nicht Lorentz-invariant.
    • Der räumliche Abstand zwischen zwei Ereignissen ist nicht Lorentz-invariant.
    • Aber: Der raumzeitliche Abstand zwischen zwei Ereignissen ist Lorentz-invariant.”

    So einfach ist es doch nicht…

    Raumzeitlicher Abstand (Wikipedia):
    »Was davon als räumlicher und was als zeitlicher Abstand gemessen wird, HÄNGT AB VOM BEWEGUNGSZUSTAND DES BEOBACHTERS UND VON DER ANWESENHEIT VON MASSE BZW. ENERGIE…«
    Was nun, sind die Abstände Lorentz-invariant oder nicht?

  463. #464 Anonym_2019
    4. August 2019

    @The Ugly (4. August 2019) #463

    “Was nun, sind die Abstände Lorentz-invariant oder nicht?”

    Siehe meine letzten Kommentare #460 und #462, in denen die Antwort schon 2x steht.

    Eine im Kreis laufende Diskussion sollten wir besser beenden.

    Vielen Dank!

  464. #465 The Ugly
    5. August 2019

    So einfach ist es also doch nicht.
    .
    .
    .

    …sich im Kreis drehen: ein echtes Problem der SRT.
    Danke ebenfalls…

  465. #466 Anonym_2019
    5. August 2019

    @The Ugly (5. August 2019) #465

    “…sich im Kreis drehen: ein echtes Problem der SRT.”

    Nein, kein Problem!

    Von Leeb, W. R.; Schiffner, G.; Scheiterer, E. wurde 1979 experimentell bewiesen, dass der Sagnac-Effekt beim Faserkreisel unabhängig von der Signalgeschwindigkeit des Lichts durch Lichtleitermaterial ist.

    Link zur Experiment-Veröffentlichung “Optical fiber gyroscopes – Sagnac or Fizeau effect”:

    https://adsabs.harvard.edu/abs/1979ApOpt..18.1293L

    Die relativistisch berechnete Sagnac-Formel enthält die Naturkonstante c, aber nicht die Signalgeschwindigkeit c/N, mit N = Brechungsindex des Materials.

    Formel im mitrotierenden System auf dem Äquator (v = Bahngeschwindigkeit des Äquators):

    Δt = v/c² * 2U

    Das ist der Term für die “Relativität der Gleichzeitigkeit” in der Lorentztransformation der Zeit mit 2U = 2x Umfang für beide Lichtstrahlen zusammen.

    Alle Äther-Herleitungen liefern eine falsche Formel, falls sich das Licht nicht durch Vakuum bewegt.

    Das Äther-Kartenhaus ist zusammengebrochen 🙂

  466. #467 Anonym_2019
    5. August 2019

    @The Ugly (5. August 2019) #465

    Ein Link in #466 schein kaputt gegangen zu sein. Daher nochmal als Text für “Cut&Paste”:

    https://adsabs.harvard.edu/abs/1979ApOpt..18.1293L

  467. #468 The Ugly
    5. August 2019

    @Anonym_2019
    Überreaktion.
    So weit wollte ich gar nicht hinaus.
    Ich meinte das dialektische Imkreisdrehen:
    wie sich die raumzeitlichen Abstände, die bezugssystemunabhängig sein sollen, auf einmal doch als bezugssystemabhängig erweisen…

    (Raumzeitlicher Abstand (Wikipedia):»Was davon als räumlicher und was als zeitlicher Abstand gemessen wird, HÄNGT AB VOM BEWEGUNGSZUSTAND DES BEOBACHTERS UND VON DER ANWESENHEIT VON MASSE BZW. ENERGIE…«)

    Danke.

  468. #469 MartinB
    5. August 2019

    @alle
    Wie schon oft in meinem Blog angemerkt: dieser blog ist nicht als Tummelplatz für “altrnative Physik” Privattheorien oder ahnungslosen Unsinn wie den hiier gedacht. Diskutiet das bitte woanders, nicht hier. (Wäre ich nicht verreist, hätte ich die Diskussikn längst beendet.)
    Diese Diskussion endet hier.

  469. #470 Gebhard Greiter
    11. Dezember 2019

    Hier noch eine Frage zu Ihrer Graphik oben:

    Sie schreiben, entscheidend sei – und das mache dieses Diagramm sehr schön deutlich –, dass es einen weiten Bereich von Ereignissen bei Teresa gibt, die für Serena mit gar nichts gleichzeitig sind. Und das habe nichts mit irgendwelchen Beschleunigungen zu tun, sondern nur damit, dass Serena ihr Bezugssystem wechselt.

    Wenn wir jetzt aber annehmen, dass der stationäre Zwilling (A = Theresa) immer dann, wenn der Zeiger seiner Uhr vorrückt, ein Funksignal aussendet, das die Zahl der bei A seit Abflug von B vergangenen Sekunden nennt, so ist doch offensichtlich, dass B über seine gesamte Reise hinweg ALLE diese Signale empfangen wird (nur halt so, dass während seiner Wegreise seiner Uhr nach zwischen diesen Signalen immer länger werdene Abstände sind, wohingegen während seiner Rückreise die Signale bei ihm in immer dichter werdenden Abständen ankommen. Ankommen werden aber grundsätzlich alle im Gegensatz zu dem, was Ihr Diagramm uns suggeriert, dass nämlich B ein bei A mehr als 2 Jahre langes, zusammenhängendes Intervall dieser Signale NICHT empfangen wird.

    Ich denke deswegen, dass Ihr Diagramm irreführend ist und der Effekt, den Sie als entscheidend nennen, so gar nicht eintritt. Wie aber kann er dann entscheidend sein?

  470. #471 MartinB
    11. Dezember 2019

    Lichtsignale laufen im Diagramm unter 45 Grad, natürlich erreichen die alle B, aber das hat nichts mit der Definition der Gleichzeitigkeit zu tun.

  471. #472 Gebhard Greiter
    11. Dezember 2019

    @MartinB#471:
    Welche Definition von Gleichzeitigkeit legen Sie denn hier zugrunde?

    Was also bedeutet es, wenn Sie sagen, dass es einen weiten Bereich von Ereignissen bei Teresa gibt, die für Serena mit gar nichts gleichzeitig sind?

  472. #473 Anonym_2019
    11. Dezember 2019

    @Gebhard Greiter (11. Dezember 2019)

    Das ist eine falsche Interpretation des Diagramms:

    “Was an Bäkers Erklärung Zweifel weckt, ist Folgendes:

    Das Diagramm suggeriert, dass B ein bei A mehr als 2 Jahre langes, zusammenhängendes Intervall dieser Signale NICHT empfangen wird — was natürlich nicht sein kann, vom Modell aber behauptet wird.”

    Quelle:
    https://greiterweb.de/welt-verstehen/0-342-Das-Zwillingsparadoxon-im-Lichte-von-SRT-und-ART.htm

    Was die eingezeichneten Linien sind, steht im Artikel, in Klammern:

    “Die roten und blauen Linien zeigen, welche Ereignisse für Serena jeweils zur selben Zeit stattfinden. (Sie sind also die Raumachse, die Serena verwendet.)”

    Die haben nichts mit Lichtsignalen zu tun.

  473. #474 Gebhard Greiter
    12. Dezember 2019

    @Anonym_2019#473:

    Ich gehe davon aus, dass die dünnen roten Linien im Diagramm https://commons.wikimedia.org/wiki/File:TwinParadoxProperAcceleration2.svg den roten und blauen Linien im Diagramm https://upload.wikimedia.org/wikipedia/commons/7/7d/Zwillingsparadoxon.png entsprechen.

    Ist das nicht richtig?

  474. #475 Anonym_2019
    12. Dezember 2019

    @Gebhard Greiter (12. Dezember 2019) #474

    Doch, das ist richtig. Weil die flacher als 45 Grad verlaufen, sind es aber keine Lichtsignale. Es handelt sich um x’-Achsen, siehe auch:
    Minkowski-Diagramm

  475. #476 Gebhard Greiter
    12. Dezember 2019

    @Anonym_2019#475

    Ich verstehe diese Linien weder als Lichtsignale.noch habe ich irgend eine Vorstellung davon, was was mit “Raumachse” oder “x’-Achse” gemeint sein könnte.

    Betrachten wir das deutlich genauere der beiden Bilder: https://commons.wikimedia.org/wiki/File:TwinParadoxProperAcceleration2.svg :

    Es gibt da z.B. eine dünne rote Linie, die das blaue Ereignis 2 mit dem roten Ereignis 3 verbindet. Ich denke, sie bedeutet, dass der reisende (rote) Zwilling im roten Ereignis 3 angekommen seinen Bruder (blau) als gerade eben erst im blauen Ereignis 2 angekommen wahrnimmt.

    Interpretieren Sie das auch so?

  476. #477 MartinB
    12. Dezember 2019

    @Gebhard
    Eine Raumachse verbindet alle Ereignisse, die für die jeweilige Beobachterin gleichzeitig stattfinden.
    Es wäre wirklich sinnvoll, mal dem Link von Anonym_2019 zu folgen und zu lernen, wie Minkowski-Diagramme fnktionieren.

  477. #478 Gebhard Greiter
    12. Dezember 2019

    @MartinB#477:

    Danke für Ihre klare Aussage.

    Sie zeigt mir, dass ich das Diagramm https://commons.wikimedia.org/wiki/File:TwinParadoxProperAcceleration2.svg richtig interpretiere und es zu jedem Ereignis E(Serena) auf dem roten Weg tatsächlich ein blaues Ereignis E(Theresa) gibt, so dass aus Serenas Sicht heraus diese beiden Ereignisse gleichzeitig stattfinden.

    Wie kommen Sie dann also – nun mit diesem genaueren Diagramm – immer noch zur Ansicht, dass es einen weiten Bereich von Ereignissen bei Teresa gäbe, die für Serena mit gar nichts gleichzeitig sind? [ Es stellt sich dies doch nur im groben Diagramm (oben in Ihrem Text) so dar, da dort — in unzulässiger Vereinfachung der tatsächlichen Situation — sämtliche Beschleunigungsphasen als ohne zeitliche Ausdehnung angenommen werden. ]

  478. #479 Anonym_2019
    12. Dezember 2019

    @Gebhard Greiter (12. Dezember 2019)

    “Ich denke, sie bedeutet, dass der reisende (rote) Zwilling im roten Ereignis 3 angekommen seinen Bruder (blau) als gerade eben erst im blauen Ereignis 2 angekommen wahrnimmt.

    Interpretieren Sie das auch so?”

    Nein. Diese beiden Ereignisse sind laut SRT im Bezugssystem des reisenden, roten Zwillings gleichzeitig. Man kann aber ein räumlich entferntes Ereignis nicht gleichzeitig zu ihm wahrnehmen.

    Ich kann z.B. auch kein Ereignis, das “jetzt” auf Alpha Centaui stattfindet, “jetzt” sofort wahrnehmen.

  479. #480 Gebhard Greiter
    12. Dezember 2019

    @Anonym_2019#479:

    Mit “wahrnehmen” meine ich nätürlich, das Betrachtete so wahrzunehmen, wie Astronomen einen weit entfernten Stern wahrnehmen: Als ein Bild, das durch Licht zum Betrachter transportiert werden musste und daher das Betrachtete in genau dem Zustand zeigt, in dem es sich befand, als jenes Bild abgestrahlt wurde.

  480. #481 Anonym_2019
    12. Dezember 2019

    @Gebhard Greiter (12. Dezember 2019) #480

    “Mit “wahrnehmen” meine ich nätürlich, das Betrachtete so wahrzunehmen, wie Astronomen einen weit entfernten Stern wahrnehmen …”

    Meine Antwort in #479 gilt für genau diese Bedeutung des Wortes “wahrnehmen”.

  481. #482 Gebhard Greiter
    13. Dezember 2019

    @Anonym_2019#479:

    Wie die Minkowski-Metrik zeigt, kann Licht sich niemals durch die Zeit (sondern stets nur durch den Raum) bewegen.

    Wenn ich also einen Stern betrachte, dann sehe ich ihn (der Endlichkeit der Lichtgeschwindigkeit wegen) stets nur in einem Zustand Z, der für ihn schon Vergangenheit ist. Das Licht trägt mir dieses Bild Z zu und bettet es in meine Gegenwart ein.

    Anders ausgedrückt: Jener Zustand Z liegt auf meiner Raumachse (sie ist 3-dimensional).

    Wenn nun also die dünne rote Linie, die im Diagramm https://commons.wikimedia.org/wiki/File:TwinParadoxProperAcceleration2.svg das rote Ereignis 3 mit dem blauen Ereignis 2 verbindet, Serenas Raumachse ist (während des roten Ereignisses 3) so bedeutet das doch gerade, dass sie Teresa dann im blauen Ereignis 2 sieht, diese beiden Ereignisse dann also für B (= Serena) gleichzeitig stattfinden.

    Würden Sie mir hierin zustimmen?

  482. #483 Anonym_2019
    13. Dezember 2019

    @Gebhard Greiter (13. Dezember 2019) #482

    “Wie die Minkowski-Metrik zeigt, kann Licht sich niemals durch die Zeit (sondern stets nur durch den Raum) bewegen.”

    Ich verstehe diese Aussage nicht. Licht benötigt zum Zurücklegen der Strecke Δx die Zeit Δt = Δx/c. Daher ist die Weltline des Lichts im Minkowski-Diagramm eine Gerade, die unter 45 Grad verläuft. Sie ist die Winkelhalbierende zwischen der x-Achse und der ct-Achse.

    siehe auch das Video: Raumzeitdiagramme und Zwillingsparadoxon

    “Anders ausgedrückt: Jener Zustand Z liegt auf meiner Raumachse (sie ist 3-dimensional).”

    Nein. Was ich “jetzt” in der Entfernung sehe, kann nicht auf meiner Raumachse liegen. Was auf dem Stern und auf der Erde gleichzeitig ist, hat nichts mit Bildübertragung durch das Licht zu tun.

    “Würden Sie mir hierin zustimmen?”

    Nein, das ist falsch. Zu dem Zeitpunkt in Serenas Bezugssystem, zu dem beide genannten Ereignisse stattfinden, wird der Lichtimpuls vom Ereignis “blaue 2” erst losgesendet, aber noch nicht von Serena empfangen/gesehen. Um den Empfangszeitpunkt bei Serena zu ermitteln, kann man eine Gerade einzeichnen, die vom Ereignis “blaue 2” im 45 Grad-Winkel nach rechts/oben verläuft. Ich schätze grob, dass diese Gerade die dicke rote Linie (=Weltlinie von Serena) ca. zwischen Ereignis “rote 4” und Ereignis “rote 5” schneidet. Erst dann kann Serena das Ereignis “blaue 2” sehen.

    Ich habe den Eindruck, dass Sie sich gedanklich noch nicht ganz von Newton’s “absoluter Zeit” gelöst haben.

    P.S. Von der Messung einer endlichen Lichtgeschwindigkeit durch Ole Roemer wusste Newton, und er hat diese akzeptiert:
    https://de.wikipedia.org/wiki/Ole_R%C3%B8mer#Lichtgeschwindigkeit

  483. #484 Gebhard Greiter
    13. Dezember 2019

    @Anomym_2019#483:

    “Die roten und blauen Linien zeigen, welche Ereignisse für Serena jeweils zur selben Zeit stattfinden. (Sie sind also die Raumachse, die Serena verwendet.)”

    Das “zur selben Zeit” in dieser Aussage bezieht sich ganz offensichtlich auf einen Gleichzeitigkeitsbegriff.

    Ich dachte, es sei der, welcher sich auch definieren lässt über den Satz: Für Serena finden Ereignisse genau dann zur selben Zeit statt, wenn das als Licht vorliegende Bild dieser Ereignisse (nach ihrer eigenen Uhr) sie zur selben Zeit erreicht.

    Ist das jetzt derselbe Gleichzeitigkeitsbegriff, von dem Sie und Herr Bäker sprechen, oder doch ein anderer?

  484. #485 Anonym_2019
    13. Dezember 2019

    @Gebhard Greiter (13. Dezember 2019) #484

    “Ich dachte, es sei der, welcher sich auch definieren lässt über den Satz: Für Serena finden Ereignisse genau dann zur selben Zeit statt, wenn das als Licht vorliegende Bild dieser Ereignisse (nach ihrer eigenen Uhr) sie zur selben Zeit erreicht.”

    Ich habe den Eindruck, dass dieser Satz nur eine Erfindung von Ihnen ist. Oder haben Sie sonst eine Quellenangabe?

    “Ist das jetzt derselbe Gleichzeitigkeitsbegriff, von dem Sie und Herr Bäker sprechen, oder doch ein anderer?”

    Es ist derselbe.

  485. #486 Gebhard Greiter
    13. Dezember 2019

    @Anonym_2019#485:

    Der Satz ist tatsächlich von mir. (Aber warum “nur”?)

    Ihre Antwort zeigt mir, dass wir oben wohl doch an einander vorbeigeredet haben (und das wohl deswegen, weil Sie haupsächlich das Minkowskidiagramm vor Augen haben und alles daran festmachen wollen), ich aber ohne diese Diagrammtechnik auszukommen versuche.

    Auch Einstein hat sie zunächst ja gar nicht zur Verfügung gehabt und soll mal gesagt haben: “Seitdem die Mathematiker sich meiner Theorie bemächtigt haben, verstehe ich sie selbst nicht mehr.”

    Vielen Dank jedenfalls für Ihre Mühe mit mir. Ich werde versuchen, ob ich nicht doch einen Weg finden kann, meine Gedanken klarer auszudrücken.

    Im Moment habe ich den Verdacht, dass Sie und Herr Bäker den dünnen Linien im Diagramm https://commons.wikimedia.org/wiki/File:TwinParadoxProperAcceleration2.svg (ebenso wie den roten und blauen in seinem oben gezeigten Diagramm) andere Semantik zuordnen als ich (denn ich möchte das blaue Ereignis 2 tatsächlich als das interpretiert sehen, das Serena wahrnimmt, wenn sie selbst sich in ihrem roten Ereignis 3 befindet).

    Mich irritiert auch, dass in Bäkers (grobem) Diagramm das fehlt, was sich im genaueren Diagramm als der waagrechte dünne rote Strich darstellt, der vom Umkehrpunkt (d.h. dem roten Ereignis 6) weg führt.

  486. #487 Anonym_2019
    14. Dezember 2019

    @Gebhard Greiter (13. Dezember 2019)

    “Aber warum “nur”?”

    Weil dieser Satz eine Privat-Theorie nur von Ihnen ist, die der Gleichzeitigkeitsdefinition in der SRT widerspricht.

    “aber ohne diese Diagrammtechnik auszukommen versuche”

    Das halte ich für eine schlechte Idee. Sie könnten allerdings diese Diagrammtechnik auch durch mathematische Formeln, basierend auf der Lorentz-Transformation, ersetzen. Das wäre auch O.K.

    “Auch Einstein hat sie zunächst ja gar nicht zur Verfügung gehabt und soll mal gesagt haben: “Seitdem die Mathematiker sich meiner Theorie bemächtigt haben, verstehe ich sie selbst nicht mehr.””

    “”Das Buch der Natur ist in der Sprache der Mathematik geschrieben.” So poetisch formulierte es bereits Galileo Galilei vor fast 400 Jahren.”
    Quelle:
    https://www.welt.de/print-welt/article310901/Im-Prinzip-sind-alle-Probleme-der-Natur-mit-der-Sprache-der-Mathematik-loesbar.html

    “Im Moment habe ich den Verdacht, dass Sie und Herr Bäker den dünnen Linien im Diagramm … andere Semantik zuordnen als ich (denn ich möchte das blaue Ereignis 2 tatsächlich als das interpretiert sehen, das Serena wahrnimmt, wenn sie selbst sich in ihrem roten Ereignis 3 befindet).”

    Wir odnen den dünnen Linien die Bedeutung laut SRT zu, Sie ordnen ihnen eine physikalisch unmögliche Bedeutung zu, die eine unendliche Lichtgeschwindigkeit erfordert.

    “Mich irritiert auch, dass in Bäkers (grobem) Diagramm das fehlt, was sich im genaueren Diagramm als der waagrechte dünne rote Strich darstellt, der vom Umkehrpunkt (d.h. dem roten Ereignis 6) weg führt.”

    Dieser dünne rote Strich ist deswegen waagerecht, weil die Geschwindigkeit von Serena zum Zeitpunkt des Ereignisses “rote 6” Null ist. Der Steigungswinkel der dünnen Linien hängt von der Geschwindigkeit des jeweiligen Zwillings zum entsprechenden Zeitpunkt ab.

  487. #488 Gebhard Greiter
    14. Dezember 2019

    @Anonym_2019 #487:

    Mit Ihrem Vorwurf, ich verfolge eine “Privattheorie” widersprechen Sie sich jetzt gerade selbst, denn schließlich haben Sie mir ja kurz vorher (in #485) noch bestätigt, dass meine Definition von Gleichzeitigkeit denselben Gleichzeitigkeitsbegriff zur Folge habe wie Ihre.

    Wenn Sie dort also recht haben sollten, sprechen Sie und Ich über ein und dasselbe – nur halt mit anderen Worten (sozusagen aus unterschiedlicher gedanklicher Perspektive heraus).

    Auch Ihr Verweis auf die Mathematik überzeugt nicht, denn es gibt ja jede Menge mathematischer Wahrheiten, für die man im Lauf der Zeit zahlreiche völlig unterschiedliche Beweise fand, und in aller Regel sind die neueren die kürzeren und besseren.

    WENN Sie mit Ihrer letzten Aussage in #485 recht haben, würde ich mich in derselben Situation befinden wie Carl Friedrich Gauss, von dem wir wissen, dass er einmal sagte “Mein Ergebnis habe ich schon. Ich muss nur noch einen Weg nach dort finden.”

    Mit anderen Worten: Er hat damals etwas Richtiges erahnt, war aber noch nicht so weit, einen konkreten für andere nachvollziehbaren Beweis hinschreiben zu können.

    Es könnte ja gut sein, dass es mir jetzt ebenso ergeht.

    Ihre Aussage “Wir odnen den dünnen Linien die Bedeutung laut SRT zu, Sie ordnen ihnen eine physikalisch unmögliche Bedeutung zu, die eine unendliche Lichtgeschwindigkeit erfordert.” ist insofern völlig danaben, als ich – wenn ich den Linien andere Bedeutung zuordne – die blau nummerierten Ereignisse ja durch andere austausche.

    Ich sollte und werde jetzt also wirklich genau das tun, was Sie mir ja auch vorschlagen: Die Diagrammtechnik durch mathematische Formeln basierend auf der Lorentz-Transformation ersetzen.

  488. #489 Anonym_2019
    14. Dezember 2019

    @Gebhard Greiter (14. Dezember 2019) #488

    “Sie mir ja kurz vorher (in #485) noch bestätigt …”

    Sorry, das war nicht meine Absicht. Ich hatte die folgende Frage so verstanden, dass Sie damit nicht Ihren Glecihzeitigkeitsbegriff meinen:

    “Ist das jetzt derselbe Gleichzeitigkeitsbegriff, von dem Sie und Herr Bäker sprechen, oder doch ein anderer?”

    Damit ändere ich meine Antwort in “Nein”.

    “wenn ich den Linien andere Bedeutung zuordne”
    = Privat-Theorie

    “Ich sollte und werde jetzt also wirklich genau das tun, was Sie mir ja auch vorschlagen”
    O.K.

  489. #490 Gebhard Greiter
    14. Dezember 2019

    @Anonym_2019:

    Danke für diese Klarstellung.

    Jetzt hätte ich aber noch eine andere Frage:

    Man spricht ja in der Relativitätstheorie davon, dass der raumzeitliche Abstand zweier Ereignisse raumartig, zeitartig oder lichtartig sein kann.

    ( https://www.spektrum.de/lexikon/kartographie-geomatik/abstand-raumzeitlicher-ereignisse/35 ).

    Was genau ist denn nun die physikalische Bedeutung von Ereignissen mit lichtartigem Abstand?

    Gibt es sie nur in der Theorie (oder könnte man ihnen auch als Teil der Wirklichkeit Sinn zuordnen)?

  490. #491 MartinB
    14. Dezember 2019

    @Gebhard
    Das steht doch in dem Link drin:
    ” So bewegen sich etwa Lichtstrahlen längs Kurven der Länge Null.”

  491. #492 rolak
    14. Dezember 2019

    Frage so verstanden (..) “Nein”

    moin 2019, insbesondere bei ‘oder’-FrageKonstrukten, noch insbesonderer im Kontext ‘unverständnisvoll’ empfiehlt es sich, getreu der Schulvorgabe nur in ganzen Sätzen zu antworten.

    ‘Nein, das ist nicht derselbe..’
    ‘Ja, das ist ein anderer..’

  492. #493 MartinB
    14. Dezember 2019

    @Gebhard
    Noch ein Nachtrag: Fragen zur RT können hier gern gestellt werden, auch wenn es manchmal vielleicht einfacher wäre, die Dinge einfach mal nachzulesen, es gibt ja hinreichend viele Bücher zur SRT und ART.

    Irgendwelche Privattheorien möchte ich hier in meinem Blog aber nicht diskutieren oder diskutieren lassen, die also hier bitte nicht thematisieren.

  493. #494 Gebhard Greiter
    14. Dezember 2019

    @MartinB #491 + #493:

    Es ist nicht meine Absicht, irgendwelche Privattheorien zu diskutieren (oder auch nur zu haben).

    Mit Aussagen wie ”So bewegen sich etwa Lichtstrahlen längs Kurven der Länge Null.

    kann ich allerdings auch nicht viel anfangen (da das ja nur eine Antwort im Rahmen des Modells ist, welche keinerlei Bezug zur Wirklichkeit herstellt, die zu modellieren ein Modell ja da ist).

    Wenn ich hier Fragen stelle, dann immer in der Absicht, derart kryptische Aussagen erklärt zu bekommen in Hinsicht darauf, wie man die Aussage des Modells im Lichte beobachtbarer Wirklichkeit zu interpretieren hat.

    Es gibt ja nicht viele Blogs wie Ihren hier, in dem man solche Fragen beantwortet bekommen kann. Meine Möglichkeiten, nach anderswohin auszuweichen, sind also wirklich SEHR begrenzt. (Bitte verstehen Sie das als Kompliment, so nämlich ist es gemeint).

  494. #495 Anonym_2019
    14. Dezember 2019

    @Gebhard Greiter (14. Dezember 2019) #490

    “Was genau ist denn nun die physikalische Bedeutung von Ereignissen mit lichtartigem Abstand?”

    Wenn z.B. an einem Ort Licht ausgesendet wird, dieses sich durch Vakuum bewegt und an einem anderen Ort empfangen wird, dann befinden sich das Aussendeereignis und das Empfangsereignis in lichtartigem raumzeitlichen Abstand.

    “Gibt es sie nur in der Theorie (oder könnte man ihnen auch als Teil der Wirklichkeit Sinn zuordnen)?”

    Die RT ist zumindest noch nie experimentell widerlegt worden, die klassische, Newton’sche Theorie dagegen schon.

  495. #496 Anonym_2019
    14. Dezember 2019

    @rolak (14. Dezember 2019) #492

    “getreu der Schulvorgabe nur in ganzen Sätzen zu antworten”

    Ja, das stimmt. Sonst besteht die Gefahr, dass die Antwort die Mehrdeutigkeit von der Frage “erbt” und dadurch selbst mehrdeutig ist.

  496. #497 MartinB
    14. Dezember 2019

    @gebhard
    Die Frage lautete, “Was genau ist denn nun die physikalische Bedeutung von Ereignissen mit lichtartigem Abstand?”
    Die Antwort darauf lautet: Ereignisse mit lichtartigem Abstand können durch ein Lichtsignal verbunden sein.

    Und der Kram mit “Antwort im Rahmen eines Modells” ist einfach überflüssig – jede Aussage, die wir über die Welt machen und die über die unmittelbare Beschreibung von Sinneseindrücken hinausgeht, ist immer eine im Rahmen eines Modells, egal ob in der Physik oder im Alltag. Wenn ich sage “Ich sehe einen Baum” ist das auch eine Antwort im Rahmen eines Modells. Siehe dazu meine Artikelserie zum Thema “Gibt es Dinge, die man nicht sehen kann”

  497. #498 Gebhard Greiter
    14. Dezember 2019

    @MartinB #497:

    Wenn ich jetzt anzuwenden versuche, was Sie mir eben beigebracht haben, dann könnte ich das tun wie folgt:

    Es sei E1 ein Ereignis, das den Zusammenstoß zweier Schwarzer Löcher darstellt,
    und es sei weiter E2 das Ereignis, in dem LIGO das entsprechende Gravitationswellensignal registriert.

    Dann ist richtig, dass E1 und E2 raumzeitlichen Abstand Null haben (d.h. lichtartigen Abstand).

    Noch ein anderes Beispiel:

    Wenn ein Astronom einen Stern betrachtet (Ereignis E), dann sieht er ihn – der Endlichkeit der Lichtgeschwindigkeit wegen – in einem Zustand Z, der für den Stern schon Vergangenheit ist.

    E und das Ereignis, in dem der Stern jenes Bild Z seiner selbst abgestrahlt hat, haben raumzeitlichen Abstand 0.

    Empfinden Sie das alles nun als korrekt ausgedrückt?

  498. #499 MartinB
    14. Dezember 2019

    @Gebhard
    ZU 1: Ja, weil GW sich mit Lichtgeschwindigkeit bewegen, man muss das nicht so kompliziert ausdrücken.

    Zu 2.
    “E und das Ereignis, in dem der Stern jenes Bild Z seiner selbst abgestrahlt hat, haben raumzeitlichen Abstand 0.” Das ist richtig.

    “dann sieht er ihn – der Endlichkeit der Lichtgeschwindigkeit wegen – in einem Zustand Z, der für den Stern schon Vergangenheit ist. ”
    Das ist zwar richtig, aber natürlich ist es absolut nicht eindeutig, wie weit des Ereignis für den Stern in der Vergangenheit liegt, das hängt vom Bezugssystem ab.

  499. #500 Anonym_2019
    14. Dezember 2019

    @Gebhard Greiter (14. Dezember 2019) #494

    “Mit Aussagen wie ”So bewegen sich etwa Lichtstrahlen längs Kurven der Länge Null.”
    kann ich allerdings auch nicht viel anfangen”

    Zum Verständnis ist wichtig zu wissen, dass hier mit “Länge” keine räumliche Länge gemeint ist, sondern eine raumzeitliche Länge, die sich ergibt aus:
    ds² = – c²dt² + dx²

    Diese ist Null, wenn gilt:
    c²dt² = dx²
    Das ist die Bewegungsgleichung für Licht im Vakuum, die 45 Grad-Linien im Minkowski-Diagramm entspricht.

    Im Allgemeinen legt das Licht schon eine räumliche Strecke zurück, die größer als Null ist.

  500. #501 Tox
    14. Dezember 2019

    @Gebhard Greiter:

    Ich bin ebenfalls der Meinung, dass die Aussage aus dem Artikel

    Entscheidend ist aber […], dass es einen weiten Bereich von Ereignissen bei Teresa gibt, die für Serena mit gar nichts gleichzeitig sind.

    mindestens irreführend ist. Denn das ist nur dann der Fall, wenn Serenas Weltlinie einen Knick hat, also wenn im Umkehrpunkt eine unendlich große Beschleunigung wirkt. In jedem realistischen Szenario bleibt die Beschleunigung endlich, und dann gibt es zu jedem Ereignis bei Teresa ein Ereignis bei Serena, das aus Serenas Sicht gleichzeitig dazu ist (schon allein aus Gründen der Stetigkeit).

    Eine bessere Formulierung wäre vielleicht:

    Es gibt einen weiten Bereich von Ereignissen bei Teresa, die aus Serenas Sicht gleichzeitig zu Ereignissen bei ihr aus einem beliebig kurzen Zeitintervall stattfinden, wenn man die Beschleunigung nur groß genug werden lässt.

    Aber wir haben ja schon letztes Jahr gesehen, dass MartinB und ich was die Interpretation dieses “Paradoxons” angeht nicht ganz der selben Meinung sind.

  501. #502 Gebhard Greiter
    14. Dezember 2019

    @MartinB + @Anonym_2019:

    Herzlichen Dank für Eure Geduld mit mir. Das hat mir jetzt wirklich geholfen.

    Gehe ich richtig in der Annahme, dass auch Sie im Netz keine Version des Bildes https://commons.wikimedia.org/wiki/File:TwinParadoxProperAcceleration2.svg
    kennen, in dem jedem der blauen bzw. roten Ereignisse E das jeweils mit ihm lichtartig verbunde rote bzw. blaue Ereignis E’ auf der jeweils anderen Weltlinie zugeordnet ist?

  502. #503 MartinB
    14. Dezember 2019

    Nein kenne ich nicht, Kann man aber ja leicht malen, einfach Linien unter 45 Grad einzeichnen.

  503. #504 Anonym_2019
    14. Dezember 2019

    @Gebhard Greiter #502

    Kenne ich auch nicht.

    Übrigens: Man kann die in Kommentar #500 bzgl. des Lichts genannte Formel
    ds² = – c²dt² + dx²
    auch z.B. im inertialen Ruhesystem von Teresa auf Teresa und Serena anwenden, um zu plausibilisieren, dass Serena weniger altert als Teresa. Das ist hier im Artikel auch im Abschnitt “The Best” in Prosa beschrieben.

    Da Teresa in diesem System ruht, ist ihr dx² = Null und die Koordinatenzeit t identisch mit ihrer Eigenzeit τ, bei Serena dagegen nicht.

    Teresa: ds² = c²dτ² = – c²dt² + 0
    Serena: ds² = c²dτ² = – c²dt² + dx²

    Serena legt einen längeren räumlichen Weg zurück und, wegen der Subtraktion auf der rechten Seite der Gleichung, damit einen kürzeren raumzeitlichen Weg als Teresa, und verbraucht dadurch weniger Eigenzeit.

  504. #505 Anonym_2019
    14. Dezember 2019

    @Gebhard Greiter #502

    Korrektur zu #500 und #504

    Ich vermute einen Vorzeichenfehler in
    https://www.spektrum.de/lexikon/kartographie-geomatik/abstand-raumzeitlicher-ereignisse/35

    Dort steht das anders:

    ” ds² := … = c²dt² – dx² – dy² – dz² “
    Quelle:
    https://de.wikipedia.org/wiki/Raumzeit#Minkowski-Raum,_Vierervektoren

    Dann ergibt sich folgende Vorzeichenkorrektur:

    ds² = + c²dt² – dx²

    Teresa: ds² = c²dτ² = c²dt² – 0
    Serena: ds² = c²dτ² = c²dt² – dx²

    Vielleicht gibt es auch unterschiedliche Konventionen für den raumzeitlichen Abstand.

  505. #506 Tox
    14. Dezember 2019

    @Anonym_2019:
    Ja, es gibt an der Stelle unterschiedliche Vorzeichenkonventionen. Bei manchen Autoren ist ds² = c² dτ² und bei anderen ist ds² = – c² dτ².

    In der ART ist es noch schlimmer. Da gibt es drei (oder waren es vier?) Stellen, an denen man unterschiedliche Vorzeichen wählen kann. Das berühmte Lehrbuch “Gravitation” von Misner, Thorne, und Wheeler beginnt z.B. mit einer großen Tabelle, in der für eine Reihe anderer Bücher aufgelistet ist, welche Vorzeichen dort gewählt werden.

  506. #507 Anonym_2019
    15. Dezember 2019

    @Tox (14. Dezember 2019) #506

    Danke für die Info! Ich entscheide mich dann ab sofort für ds² = c² dτ², weil ich das schöner finde und weil das so bei Wikipedia steht.

  507. #508 Gebhard Greiter
    15. Dezember 2019

    @Anonym_2019 #505 + Tox #506:

    Es gibt sogar Namen für die beiden Konventionen: Man nennt sie die Ostküsten- bzw. die Westküstenkonvention. Details in:

    https://de.wikipedia.org/wiki/Minkowski-Raum#Reelle_Definition

  508. #509 Anonym_2019
    15. Dezember 2019

    @Gebhard Greiter (15. Dezember 2019) #508
    Vielen Dank!

    @Gebhard Greiter (14. Dezember 2019) #502

    Ich habe lichtartige Verbindungen (=45 Grad-Linien) in einem ähnlichen ZP-Diagramm gefunden. Das enthält allerdings auch keine Beschleunigung, siehe Kommentar #1, Feb 12 2013:
    ttps://www.physicsforums.com/threads/spacetime-diagram-twin-paradox.671398/

  509. #510 Anonym_2019
    15. Dezember 2019

    @Tox (14. Dezember 2019) #501

    Ich finde auch, dass eine Berücksichtigung der Beschleunigung von Serena in der Umkehrphase realistischer wäre.

    Um nur den Altersunterschied auszurechnen, reicht eine Berechnung im Inertialsystem von Teresa aus.

    Didaktisch wäre es aus meiner Sicht aber besser, zusätzlich eine Berechnung im nicht-Inertialsystem von Serena durchzuführen, um aus deren Sicht zu zeigen, dass Teresa durch die pseudo-gravitative Zeitdilatation in ihrem beschleunigten Bezugssystem schneller altert als sie selbst, siehe auch die Berechnung weiter oben im Kommentar #274.

  510. #511 MartinB
    15. Dezember 2019

    Ein schöner Überblick über die Konventionen findet sich auch hier:
    https://equatorfreq.wordpress.com/2010/08/13/signs-in-einsteins-equation/

  511. #512 Gebhard Greiter
    15. Dezember 2019

    @Tox & Anonym_2019 #510:

    Auch ich finde, dass man zur Auflösung des Paradoxons unbedingt auch den Effekt der Beschleunigung von Serena (in unterschiedlichen Phasen ihrer Reise) mit diskutieren sollte.

    Wer ihn nicht verstanden hat, wird das Szenario nie vollständig verstanden haben.

    Auf Seite https://greiterweb.de/welt-verstehen/0-342-Das-Zwillingsparadoxon-im-Lichte-von-SRT-und-ART.htm versuche ich, diese Vollständigkeit der Erklärung zu erreichen.

    Mir persönlich haben am meisten Klarheit beschert die Bilder 2 und 4 dort.

  512. #513 MartinB
    15. Dezember 2019

    @gebhard
    Hab’s nur kurz angeguckt, meiner Ansicht nach ist da einiges falsch – der Dopplereffekt hat nichts mit der Sache zu tun, und es ist zumindest irreführend, bei bild 2 darauf abzuheben, dort sei der Weg gekrümmt, ein Weg aus zwei geraden Stücken ist auch gekrümmt.
    Werde das aber aus Zeitgründen nicht im Detail diskutieren können

  513. #514 Tox
    15. Dezember 2019

    @Gebhard Greiter:

    Lasst uns annehmen, dass der stationäre Zwilling (Teresa = A) immer dann, wenn der Zeiger seiner Uhr vorrückt, ein Funksignal aussendet, das die Zahl der bei A seit Abflug von B vergangenen Sekunden nennt. Das Diagramm suggeriert, dass B ein bei A mehr als 2 Jahre langes, zusammenhängendes Intervall dieser Signale NICHT empfangen wird — was natürlich nicht sein kann, vom Modell aber behauptet wird.

    Das ist falsch. Das Modell behauptet das nicht. Die dünnen roten und blauen Linien sind nicht die Kurven, auf denen sich Lichtsignale ausbreiten. Die Linien verbinden die Ereignisse, die aus Sicht des reisenden Zwillings gleichzeitig zueinander stattfinden. Gleichzeitige Ereignisse (egal aus wessen Sicht) können nie mit Lichtsignalen verbunden werden.

    Genauer gibt es für zwei Punkte in der Raumzeit folgende drei Möglichkeiten:

    Ihr zeitlicher Abstand (multipliziert mit der Lichtgeschwindigkeit) kann größer sein als ihr räumlicher Abstand. Das ist genau dann der Fall, wenn es für ein materielles Objekt möglich ist, vom einen zum anderen Punkt zu reisen. (Und es gibt dann genau einen solchen Weg durch die Raumzeit mit konstanter Geschwindigkeit.)

    Ihr zeitlicher Abstand kann gleich ihrem räumlichen Abstand sein. Das ist genau dann der Fall, wenn es möglich ist, ein Lichtsignal vom einen zum anderen Punkt zu schicken.

    Und ihr zeitlicher Abstand kann kleiner sein als ihr räumlicher Abstand. Das ist genau dann der Fall, wenn es ein Inertialsystem gibt, in dem die beiden Punkte gleichzeitig sind.

    Gleichzeitigkeit und die Möglichkeit Signale auszutauschen sind fundamental unterschiedliche Dinge. Sind zwei Ereignisse gleichzeitig (egal für welchen Beobachter), dann kann zwischen ihnen kein Signal ausgetauscht werden. Und kann zwischen zwei Ereignissen ein Signal ausgetauscht werden, dann sind sie für keinen Beobachter gleichzeitig.

  514. #515 Gebhard Greiter
    15. Dezember 2019

    @MartinB #513:

    Natürlich würde auch ich den Weg schon dann als gekrümmt ansehen, wenn er tatsächlich nur aus zwei Seiten eines Dreiecks bestünde (also so aussähe, wie Bild 1 Serenas Weg zeigt).

    Vergleicht ich nun aber Bild 1 mit dem (deutlich genaueren) Bild 2, so scheint mir klar,
    dass Ihre Aussage, bei Teresa gäbe es einen weiten Bereich von Ereignissen, die für den reisenden Zwilling (Serena = B) mit gar nichts gleichzeitig sind, nicht richtig sein kann.

    Begründung: Der Winkel zwischen den Raumachsen von Theresa einerseits und Serena andererseits verändert sich ja stetig. Bild 1 suggeriert uns anderes – aber halt nur deswegen, weil es von einen allzu ungenauen Modell des Szenarios ausgeht:
    Nach Bild 1 besteht die Umkehrphase aus nur einem einzigen Ereignis, was so nicht stimmt. Genauer:

    Bild 1 tut so, als gäbe es für Serena nur zwei Bezugssysteme: das der Hinreise und das der Rückreise.

    Tatsächlich aber wird Serenas Bezugssystem ja ständig ausgetauscht, während der Phasen ihrer Reise, in denen sie sich beschleunigt bewegt.

    Dass der Dopplereffekt – zwar nicht für das Entstehen des Alterunterschiedes, sehr wohl aber beim Errechnen der Ankunftszeiten der Greetings in Bild 3 – mit eine Rolle spielt, erscheint mir offensichtlich.

  515. #516 Gebhard Greiter
    15. Dezember 2019

    @Tox #514:

    Im ersten Teil von #514 beziehe ich mich auf die roten und blauen Linien aus Bild 1, wohingegen Sie über die dünnen roten und blauen Linien aus Bild 2 sprechen.

    Was Sie danach über die 3 grundsätzlich gegebenen Möglichkeiten sagen, sehe ich ebenso.

  516. #517 Tox
    15. Dezember 2019

    @Gebhard Greiter:
    Leider sind die Bilder nicht nummeriert. Wenn Bild 1 das erste Bild im Artikel und Bild 2 das zweite ist, dann ist Ihre Behauptung falsch. Ich spreche über die roten und blauen Linien aus Bild 1.

  517. #518 Tox
    15. Dezember 2019

    Die roten und blauen Linien in Bild 1 haben exakt die selbe Bedeutung wie die roten Linien in Bild 2. Die blauen Linien in Bild 2 haben keine Entsprechung in Bild 1. Aber keine der Linien hat etwas mit Lichtsignalen zu tun. Die kommen erst in Bild 3 vor.

  518. #519 Jolly
    15. Dezember 2019

    Hier muss ich mich als Laie mal einmischen.

    Mir hilft das Bild wegen seiner Vereinfachung. Gerade weil nur 2 neue Inertialsysteme bei Serena eine Rolle spielen. Für beide kann ich in aller Ruhe überlegen, was in ihnen Gleichzeitigkeit bedeutet. Genau das ist dann ja auch eingezeichnet.

    Dass also bestimmte Ereignisse in Teresas System aus der Sicht von Serenas erstem Inertialsystem erst passieren nachdem sie bereits ins zweite gewechselt hat und aus Serenas zweitem Inertialsystem betrachtet aber schon passiert sind bevor sie in dieses gewechselt hat, verstehe ich, und kann das auch grafisch leicht nachvollziehen. Damit verstehe ich dann auch schon die Aussage, “dass es einen weiten Bereich von Ereignissen bei Teresa gibt, die für Serena mit gar nichts gleichzeitig sind.“ (*)

    Was Gleichzeitigkeit bedeutet, und erst recht wie das berechnet wird, in einem System das gerade beschleunigt, bei dem kontinuierlich das Inertialsystem gewechselt wird (oder besser gesagt, es sich nie in einem befindet), ist viel schwieriger zu erfassen. Es ist aus meiner Sicht schon nicht mehr trivial, wie man Gleichzeitigkeit von Ereignissen an der Spitze und am Ende des beschleunigenden Raumschiffes feststellen kann, in dessen Mitte man gerade sitzt; Stetigkeit hin oder her, Realitätsnähe hin oder her.

    (*) Dass Serena alle diese Ereignisse trotzdem auf ihrer Reise wahrnehmen kann, sollte unstrittig sein.

  519. #520 Jolly
    15. Dezember 2019

    @Gebhard Greiter

    „Der Winkel zwischen den Raumachsen von Theresa einerseits und Serena andererseits verändert sich ja stetig.“

    Nein, wir können getrost davon ausgehen, dass Serena sich senkrecht (o.B.d.A.) von Teresa entfernt und dann senkrecht zurückkommt. Der Winkel bleibt konstant, in allen Bildern 1, 2, 3 und 4. Serena fliegt weder im Dreieck noch in geschwungenen Linien.

  520. #521 Anonym_2019
    15. Dezember 2019

    @Jolly (15. Dezember 2019) #520

    “… Der Winkel bleibt konstant, in allen Bildern”

    Falsch! Die Aussage „Der Winkel zwischen den Raumachsen von Theresa einerseits und Serena andererseits verändert sich ja stetig.“ von @Gebhard Greiter stimmt, weil er keinen räumlichen Winkel, sondern einen Winkel im Raumzeit-Diagramm meint.

  521. #522 Anonym_2019
    15. Dezember 2019

    @Gebhard Greiter (15. Dezember 2019) #516

    Nehmen wir einmal an, Serena entfernt sich mit der Geschwindigkeit v von Teresa.

    Teresa sende nun mit einer Taschenlampe einen kurzen Lichtblitz zu Serena. Dieser Lichtblitz hat im Bezugssystem von Teresa die Geschwindigkeit c.

    Die klassische Newton’sche Theorie machte nun die falsche, durch Experimente widerlegte Voraussage, dass der Lichtblitz im Bezugssystem von Serena die
    Geschwindigkeit c-v hat.

    Einstein erkannte als erster, dass diese Experimente auch Newton’s Annahme einer “absoluten Zeit” widerlegten. Er ließ daher diese Annahme fallen und ersetzte sie durch die beiden Postulate der SRT. Laut dem 2. Postulat der SRT hat der Lichtblitz auch in Serena’s Bezugssystem die Geschwindigkeit c.

    Im Raumzeitdiagramm bewegt sich ein Objekt immer in Richtung seiner eigenen Zeitachse.

    • In der (überholten) Newton’schen Theorie hatten beide Bezugssysteme dieselbe Raumachse und damit dieselbe Gleichzeitigkeitslinie:
    https://de.wikipedia.org/wiki/Datei:Minkowski_diagram_-_Newtonian_physics.png
    Die Weltlinie des Lichts ist hier nicht in jedem Bezugssystem die Winkelhalbierende zwischen Zeit- und Raumachse.

    • In der SRT haben beide Bezugssysteme unterschiedliche Raumachsen und damit unterschiedliche Gleichzeitigkeitslinien:
    https://de.wikipedia.org/wiki/Datei:Minkowski_diagram_-_asymmetric.svg
    Die Weltlinie des Lichts ist hier in jedem Bezugssystem die Winkelhalbierende zwischen Zeit- und Raumachse.

  522. #523 Gebhard Greiter
    16. Dezember 2019

    @Tox #517 + 518:

    Wo ich von Bild n spreche, ist damit tatsächlich das n-te Bild in https://greiterweb.de/welt-verstehen/0-342-Das-Zwillingsparadoxon-im-Lichte-von-SRT-und-ART.htm gemeint.

    Auch ich bin der Meinung: Die roten und blauen Linien in Bild 1 entsprechen den roten Linien in Bild 2.

    Eben deswegen ist die dünne rote waagrechte Linie in Bild 2 interessant:

    Das rote Ereignis 6, von dem sie ausgeht, könnte ein kurzer Aufenthalt von Serana am Umkehrpunkt ihrer Reise sein.

    Da sie ihn unbeschleunigt durchlebt, hat sich der Winkel zwischen den Raumachsen von Teresa einerseits und Sereana andererseits dort wieder auf Null reduziert.
    Das ging aber stetig vor sich, während sie abbremsen musste.

    Sobald sie zur Rückreise aufbricht, wird sich in der entsprechenden Beschleunigungsphase wieder ein Winkel zwischen den Achsen öffnen (dieses Mal aber durch Rotation ihrer Achse im Uhrzeigersinn (wie aus Bild 4 hervorgeht), da sich ihr neues Reiseziel (die Erde) ja nun im Bild links von ihr findet).

  523. #524 Tox
    16. Dezember 2019

    @Gebhard Greiter:
    Ja was denn nun? Der Satz

    Im ersten Teil von #514 beziehe ich mich auf die roten und blauen Linien aus Bild 1, wohingegen Sie über die dünnen roten und blauen Linien aus Bild 2 sprechen.

    aus #516 und der Satz

    Auch ich bin der Meinung: Die roten und blauen Linien in Bild 1 entsprechen den roten Linien in Bild 2.

    aus #523 können ja kaum beide wahr sein.

    Und wenn der Satz aus #523 wahr ist, dann ist der Absatz aus dem Artikel, den ich in #514 zitiert habe, offensichtlich falsch.

    Der Rest entspricht dem, was ich bereits in #501 schrieb. Aber das hat überhaupt nichts mit dem Absatz, den ich in #514 zitiert habe, zu tun. Der ist einfach nur falsch.

  524. #525 Gebhard Greiter
    16. Dezember 2019

    @Tox #524:

    Die zweite Zeile des ersten Zitats war ja nur mein (ursprüngliches) Verständnis davon, auf was Sie sich beziehen (ein Missverständnis, wie sich dann herausgestellt hat, an dem ich natürlich nicht mehr festhalten will, nachdem Sie mir deutlicher gesagt haben, auf was sich Ihre Aussage in #514 bezog).

    Mit dem “Satz aus #523” (von dem Sie in #524 sprechen) meinen Sie wohl den von mir dort fett gedruckten.

    Dass die blauen Linien in Bild 2 keine Entsprechung in Bild 1 haben, halte ich für richtig. Sie verbinden Ereignisse, die Teresa als gleichzeitig wahrnimmt.

    Auch dass Linien, die etwas mit Lichtsignalen zu tun haben, nur in Bild 3 auftreten, halte ich für richtig.

    Ich bitte Sie deswegen, mir nochmals zu erklären, welche Semantik aus Ihrer Sicht den Linien aus Bild 1 und den roten Linien aus Bild 2 denn nun genau zukommt.

    Ich interpretiere die dünnen roten Linien aus Bild 2 als solche, die Ereignisse mit einenander verbinden, die aus Serenas Sicht gleichzeitig sind.

    Gleiches, so denke ich, gilt für die roten und blauen Linien aus Bild 1 — wobei man hier aber nicht ganz sicher sein kann, wie der Zeichner sie wirklich gemeint hat: Solche Zweifel werden geweckt, da sie von zweierlei Farbe sind und (zufällig nur?) im Winkel von 45 Grad verlaufen.

    Sie sehen schon: Ich halte das Bild 1 für zu ungenau (da es die Beschleunigungsphasen ignoriert) und für schwer interpretierbar (da die Linien im Winkel von 45 Grad eingezeichnet sind und parallel verlaufen).

  525. #526 Tox
    16. Dezember 2019

    @Gebhard Greiter:

    Mit dem “Satz aus #523” (von dem Sie in #524 sprechen) meinen Sie wohl den von mir dort fett gedruckten.

    Nein, ich meine den Satz (oder die Sätze) aus #523, die ich in #524 zitiert habe.

    Das hier

    Auch ich bin der Meinung: Die roten und blauen Linien in Bild 1 entsprechen den roten Linien in Bild 2.

    widerspricht doch

    Lasst uns annehmen, dass der stationäre Zwilling (Teresa = A) immer dann, wenn der Zeiger seiner Uhr vorrückt, ein Funksignal aussendet, das die Zahl der bei A seit Abflug von B vergangenen Sekunden nennt. Das Diagramm suggeriert, dass B ein bei A mehr als 2 Jahre langes, zusammenhängendes Intervall dieser Signale NICHT empfangen wird — was natürlich nicht sein kann, vom Modell aber behauptet wird.

    In dem Diagramm sind keine Funksignale eingezeichnet. Es suggeriert überhaut gar nichts über Funksignale. Es behauptet in keiner Weise, dass Signale nicht empfangen würden.

    Und wenn das

    Auch dass Linien, die etwas mit Lichtsignalen zu tun haben, nur in Bild 3 auftreten, halte ich für richtig.

    richtig sein soll, was soll dann

    Lasst uns annehmen, dass der stationäre Zwilling (Teresa = A) immer dann, wenn der Zeiger seiner Uhr vorrückt, ein Funksignal aussendet, das die Zahl der bei A seit Abflug von B vergangenen Sekunden nennt. Das Diagramm suggeriert, dass B ein bei A mehr als 2 Jahre langes, zusammenhängendes Intervall dieser Signale NICHT empfangen wird — was natürlich nicht sein kann, vom Modell aber behauptet wird.

    ? Dieser Absatz ist schlicht falsch.

    Ich bitte Sie deswegen, mir nochmals zu erklären, welche Semantik aus Ihrer Sicht den Linien aus Bild 1 und den roten Linien aus Bild 2 denn nun genau zukommt.

    Das steht doch deutlich in MartinBs Artikel:

    Die roten und blauen Linien zeigen, welche Ereignisse für Serena jeweils zur selben Zeit stattfinden.

    Gleiches, so denke ich, gilt für die roten und blauen Linien aus Bild 1 — wobei man hier aber nicht ganz sicher sein kann, wie der Zeichner sie wirklich gemeint hat:

    Naja, wenn man den Artikel gelesen hat, sollte ihre Bedeutung klar sein.

    Solche Zweifel werden geweckt, da sie von zweierlei Farbe sind und (zufällig nur?) im Winkel von 45 Grad verlaufen.

    Wo sehen Sie in dem Bild einen 45°-Winkel? Der Winkel zwischen diesen Linien und der horizontalen Achse beträgt etwa 31°.

    Ich halte das Bild 1 für zu ungenau (da es die Beschleunigungsphasen ignoriert) …

    Je nachdem, was man mit dem Bild aussagen will, stimme ich dem zu oder nicht.

    … und für schwer interpretierbar (da die Linien im Winkel von 45 Grad eingezeichnet sind und parallel verlaufen).

    Sorry, aber das ist einfach falsch.

  526. #527 Anonym_2019
    16. Dezember 2019

    @Gebhard Greiter (16. Dezember 2019) #523

    “Da sie ihn unbeschleunigt durchlebt, hat sich der Winkel zwischen den Raumachsen von Teresa einerseits und Sereana andererseits dort wieder auf Null reduziert.”

    Nein. Die Raumachse ist waagerecht, weil Serenas Geschwindigkeit zum Zeitpunkt des Ereignisses “rote 6” Null ist. Ihre Beschleunigung zu diesem Zeitpunkt hat keinen Einfluss darauf.

    Grund: Die Steigung der dicken roten Linie gibt die Geschwindigkeit von Serena an. Wenn man z.B. beim Punkt “rote 6” eine Tangente an die dicke rote Linie zeichnet (=Serenas Zeitachse) und diese an einer gedachten 45 Grad-Line (=Licht) spiegelt, kommt die dünne rote Linie heraus (Raumachse). Da taucht die Beschleunigung nicht auf.

  527. #528 Anonym_2019
    16. Dezember 2019

    Gebhard Greiter (16. Dezember 2019) #525

    “Sie sehen schon: Ich halte das Bild 1 für zu ungenau (da es die Beschleunigungsphasen ignoriert) und für schwer interpretierbar (da die Linien im Winkel von 45 Grad eingezeichnet sind und parallel verlaufen).”

    Das Bild 1 stammt ja anscheinend ursprünglich aus Wikipedia:

    https://de.wikipedia.org/wiki/Zwillingsparadoxon#Das_unterschiedliche_Altern_der_Zwillinge.

    Dort gibt es zu diesem Szenario 2 Bilder, eines mit den Raumachsen und eines mit den 45 Grad-Lichtsignalen (übrigens mit Dopplereffekt).

    • Bild mit den Raumachsen (= Gleichzeitigkeitsachsen):
    https://de.wikipedia.org/wiki/Datei:Zwillingsparadoxon.png
    Parallel verlaufen die Raumachsen hier, weil Serenas Geschwindigkeiten vor und nach dem Umkehrpunkt jeweils gleich bleiben (Inertialbewegung).

    • Bild mit den 45 Grad-Lichtsignalen (“Greetings”):
    https://de.wikipedia.org/wiki/Datei:Zwillingsparadoxon_-_Lichtsignale.png

  528. #529 Gebhard Greiter
    16. Dezember 2019

    @Tox #526:

    Lassen Sie mich damit beginnen, Ihre Antwort #526 von unten her abzuarbeiten:

    Was den Winkel von (aus meiner Sicht) 45 Grad betrifft, die – wie Sie offenbar abgemessen haben – nur 31 Grad sind, so habe ich den tatsächlich nur (wie es die meisten Leser wohl tun werden) geschätzt.

    Das macht Sinn, da man ja gar nicht wissen kann, ob nicht vielleicht auch der Zeichner gar nicht wirklich einen Winkelmesser zur Hand nahm.

    Wie dem auch sei: Lassen Sie uns jetzt davon ausgehen, er sei 31 Grad.

    Da die Linien (mir wenigstens) parallel erscheinen, können sie sich nur auf unbeschleunigte Phasen der Reise beziehen (die roten auf eine andere als die blauen).

    Die Phasen mit Beschleunigung werden dann immer noch ignoriert (der Aussage des Diagramms nach somit als unwesentlich betrachtet – was Joachom Schulz dann ja auch prompt dazu führt, allen Ernstes zu glauben “Nicht Beschleunigung mach[e] die Zeit”.

    Nun zur ersten Hälfte von #526:

    Wenn ich davon spreche, dass mir etwas suggeriert werde (durch ein Diagramm oder durch – aus meiner Sicht – nicht ausreichend klar formulierten Text, dann meine ich damit, dass man leicht missverstehen kann, was da ausgesagt wird.

    Aber natürlich nehme ich Ihre Kritik (und die Tatsache, dass mein Text Ihnen sagt, was Sie darunter verstehen (also nicht unbedingt das, was ich darunter verstehe) ernst.

    Der einzige Weg, um sicher zu sein, dass niemand die Bilder anders versteht als ich, scheint mir zu sein, auf jener Seite in einer Legende festzuhalten, welche Aussage ich ihnen entnehme.

    Diese Legende ist jetzt hinzugefügt.

    Ich hoffe, dass damit auf jener Seite nun wirklich auschließlich Wahres, in sich voll Konsistentes und durch jeden Nachvollziehbares gesagt wird.

    Für einen nochmaligen kritischen Blick darauf wäre ich dennoch dankbar.

  529. #530 Gebhard Greiter
    16. Dezember 2019

    @Anonym_2019:

    Danke für die Erinnerung #528 – und ganz besonders für Ihre Nachricht #527:

    Was Sie mir in #527 sagen, sah ich tatsächlich anders (also falsch).

    Man sieht wieder mal, wie wertvoll es für Nur-Hobby-Physiker wie mich ist, sich hier mit echten Physikern austauschen zu können.

    Nochmals herzlichen Dank für Ihre Mühe.

  530. #531 Tox
    16. Dezember 2019

    @Gebhard Greiter:

    Naja, um festzustellen, dass die Linien nicht im 45°-Winkel verlaufen, braucht man keinen Winkelmesser. Die oberste rote Linie legt von A2 nach B drei Einheiten in horizontaler Richtung und ein bisschen weniger als 2 Einheiten in vertikaler Richtung zurück. Man braucht keinen Arcustangens, um zu sehen dass das nie und nimmer 45° sein können.

    Und ganz ehrlich: Wenn jemand eine solche Behauptung aufstellt (“die Linien verlaufen im Winkel von 45°”), die einerseits trivial überprüfbar ist, und die andererseits dem zum Bild gehörenden Text fundamental widerspricht, dann gehe ich davon aus, dass er wenigstens versucht hat, sie zu überprüfen. Das ist anscheinend nicht passiert (noch nicht einmal, nachdem man Sie auf diesen Fehler hingewiesen hat). Für mich spricht das nicht dafür, dass Sie mit allzu großer Sorgfalt an diese Dinge herangehen.

    Da die Linien (mir wenigstens) parallel erscheinen, können sie sich nur auf unbeschleunigte Phasen der Reise beziehen (die roten auf eine andere als die blauen).

    Natürlich tun sie das. Die Weltlinie besteht ja aus Geradenstücken. Bis auf den einen Umkehrpunkt besteht die im Diagramm abgebildete Reise ausschließlich aus unbeschleunigten Phasen.

    … nicht ausreichend klar formulierten Text …

    Was ist an “Die roten und blauen Linien zeigen, welche Ereignisse für Serena jeweils zur selben Zeit stattfinden.” nicht ausreichend klar formuliert?

    Aber natürlich nehme ich Ihre Kritik (und die Tatsache, dass mein Text Ihnen sagt, was Sie darunter verstehen (also nicht unbedingt das, was ich darunter verstehe) ernst.

    Was genau verstehen Sie denn unter der Behauptung, das Diagramm suggeriere oder behaupte, Lichtsignale würden nicht empfangen? Ich verstehe darunter, dass das Diagramm suggeriere oder behaupte, dass Lichtsignale nicht empfangen werden. Das Diagramm tut das offensichtlich nicht.

    Ich hoffe, dass damit auf jener Seite nun wirklich auschließlich Wahres, in sich voll Konsistentes und durch jeden Nachvollziehbares gesagt wird.

    Das ist nicht der Fall. Ich habe bereits mehrmals auf einen vollkommen falschen Absatz hingewiesen. Soweit ich sehe, ist an dem nichts geändert worden.

  531. #532 Anonym_2019
    16. Dezember 2019

    @Tox (16. Dezember 2019) #531
    CC: @Gebhard Greiter

    “Was ist an “Die roten und blauen Linien zeigen, welche Ereignisse für Serena jeweils zur selben Zeit stattfinden.” nicht ausreichend klar formuliert?”

    Interessant ist auch ein Vergleich des Zitats von @Gebhard Greiter unter dem Bild 1 in dem blauen Kasten:

    Die roten und blauen Linien zeigen, welche Ereignisse für B jeweils zur selben Zeit stattfinden.

    Wenn B (Serena) von A wegfliegt

    … mit dem Original bei @MartinB:

    Die roten und blauen Linien zeigen, welche Ereignisse für Serena jeweils zur selben Zeit stattfinden. (Sie sind also die Raumachse, die Serena verwendet.) Wenn Sie von Teresa (A) wegfliegt

    Quelle:
    https://scienceblogs.de/hier-wohnen-drachen/2018/07/04/the-good-the-bad-and-the-ugly-die-erklaerungen-des-zwillingsparadoxons/2/

  532. #533 Tox
    17. Dezember 2019

    @Anonym_2019:
    Das ist interessant. Meine erste Idee war, dass Gebhard Greiter vielleicht eine ältere Version von MartinBs Text kopiert hat. Aber die einzige im Internet Archive verfügbare Version (https://web.archive.org/web/20181129100054/https://scienceblogs.de/hier-wohnen-drachen/2018/07/04/the-good-the-bad-and-the-ugly-die-erklaerungen-des-zwillingsparadoxons/2/ vom 29.11.2018) enthält den aktuellen Text.

    Es ist mir komplett schleierhaft, wie man auf Gebhard Greiters Schlussfolgerung kommen kann. Selbst wenn man nur das Diagramm betrachtet und den Text ignoriert. Und selbst wenn man meint, dass die roten und blauen Linien im 45°-Winkel verlaufen. Denn dann wären die blauen Linien ja Signale, die Serena an Teresa schickt (Signale laufen in die Zukunft, nicht in die Vergangenheit). D.h. nach dieser (falschen) Interpretation des Diagramms würde zunächst Teresa Signale an Serena schicken, und zwar genau so lange, dass das letzte Signal Serena im Umkehrpunkt erreicht. Und auf der Rückreise würde dann Serena Signale an Teresa schicken. Und sämtliche dieser Signale kommen bei der jeweiligen Empfängerin an.

    Ich verstehe zwar nicht, was der Zweck eines solchen seltsamen Signalaustauschs sein könnte; möglich ist das natürlich schon. Aber wie man auf die Idee kommen kann, das Diagramm würde suggerieren, dass irgend welche Signale ihr Ziel nicht erreichen würden, ist mir rätselhaft.

  533. #534 Tox
    17. Dezember 2019

    Und selbst wenn man ignoriert, dass Signale in die Zukunft geschickt werden und nicht in die Vergangenheit, man also die blauen Linien ebenfalls als Signale von Teresa an Serena interpretiert, dann würde man immer noch nicht auf die Schlussfolgerung kommen. Denn dann würde das Diagramm ja sagen, dass Teresa während eines längeren Intervalls keine Signale sendet, und Serena trotzdem einen gleichmäßigen Strom von Signalen erhält. Dass irgend welche von Teresas Signalen verloren gingen, sagt das Diagramm selbst in dieser extremen Missinterpretation nicht. Dafür müsste es Linien geben, die an der vertikalen Achse beginnen, im 45°-Winkel zu dieser verlaufen, und Serenas Weltlinie nicht treffen.

  534. #535 Jolly
    17. Dezember 2019

    @Anonym_2019

    „Raumachsen (= Gleichzeitigkeitsachsen)“

    „Sie sind also die Raumachse, die Serena verwendet.“

    Hab ich erst jetzt begriffen. Obwohl das tatsächlich schon immer dastand. Eigentlich glasklar, wenn ich so darüber nachdenke.

    Danke erstmal, werde mich zu Weihnachten dann wohl mit dem Buch von Martin Bäker beschenken müssen.

  535. #536 Gebhard Greiter
    17. Dezember 2019

    @Tox #531:

    Tatsache ist, dass – als ich Bild 1 zum ersten Mal sah – mir noch einige Zeit lang gar nicht klar war, welch besondere Rolle dort ein Winkel von genau 45 Grad spielen würde.

    Und als von Ihnen die Frage kam, was ich denn mit meinem Wort “suggeriert” meine, habe ich Sie Ihnen beantwortet in einer Form, wie sie unabhängig von irgend einem festen Beispiel – sei es nun Bild oder Text – Gültigkeit haben kann. Ich schrieb:

    Wenn ich davon spreche, dass mir etwas suggeriert werde (durch ein Diagramm oder durch – aus meiner Sicht – nicht ausreichend klar formulierten Text), dann meine ich damit, dass man leicht missverstehen kann, was da ausgesagt wird.

    Tatsache ist doch: Wenn X eine Nachricht an Y sendet (mit der er Y was mitteilen möchte), dann wird dennoch immer erst der Empfänger Y ihr Bedeutung geben. Natürlich immer im Glauben, es sei die Bedeutung, die der Sender X ihr geben wollte.

    Eine Garantie dafür, dass X ihr nun aber genau diese Bedeutung geben wollte, gibt es nicht, denn transportiert werden ja nur Zeichen, Worte, allgemeiner: was physisch Existentes.

    Eben das ist die Ursache dafür, dass es in jeder Diskussion zwischen Lebewesen immer und immer wieder Missverständnisse gibt.

    Betrachten wir ein Beispiel:

    Sie schreiben mir in #531:

    Die Weltlinie besteht ja aus Geradenstücken. Bis auf den einen Umkehrpunkt besteht die im Diagramm abgebildete Reise ausschließlich aus unbeschleunigten Phasen.

    Rein formal gesehen steht die Aussage des ersten Satzes im Widerspruch zur Aussage des zweiten Satzes. Dennoch erkenne ich, was Sie wirklich sagen wollten (aber eben so nicht gesagt haben).

    Ich gehen deswegen davon aus, dass Sie mir sagen wollten, was der zweite Satz sagt. und ich den ersten einfach ignorieren sollte, da er eine allzu grobe, genau so gar nicht intendierte Aussage darstellt. Tatsächlich glauben sollte ich den zweiten Satz.

    Dann aber sehe ich, dass, was er sagt, ja auch nicht stimmt, denn Beschleunigung gibt es nicht nur bei der Umkehr, sondern notwendigerweise auch zu Beginn und Ende der Reise – überall dort nämlich, wo sich Geschwindigkeit ändern muss.

    Wenn ich also ein Wortklauber sein wollte, müsste ich jetzt feststellen, dass nichts von dem, was Sie mir da mitteilen, Sinn ergibt. Da ich nun aber weder Jurist noch Wortklauber sein möchte, versuche ich, zu erraten, was Sie mir eigentlich sagen wollten (um so aus Ihrer nicht ganz richtigen Aussage eine voll richtige zu machen).

    Solches Erraten aber hat seine Tücken: Man kann sehr leicht daneben raten. Wenn man daneben rät, ist das Missverständnis passiert (wir erleben im Zuge fast all unserer Diskussionen ständig solche Beispiele des Danebenratens).

    Es macht daher keinen Sinn, sich gegenseitig vorzuwerfen, wie inkonsistent denn eigentlich sei, was der jeweils andere – ich oder Sie – gesagt hat.

    Viel produktiver ist, den Gesprächspartner zu bitten, etwas nochmas in anderen Worten zu sagen.

    Genau das tue ich jetzt, indem ich Sie bitte, mir Ihre abschließende Aussage “Ich habe bereits mehrmals auf einen vollkommen falschen Absatz hingewiesen. Soweit ich sehe, ist an dem nichts geändert worden” nochmals zu erläutern, indem Sie mir die Frage beantworten, was genau Sie (im Text auf meiner Seite) denn nun eigentlich als einen vollkommen falschen Ansatz erkennen (und warum). Ich weiß es einfach noch nicht.

  536. #537 Tox
    17. Dezember 2019

    @Gebhard Greiter:

    Rein formal gesehen steht die Aussage des ersten Satzes im Widerspruch zur Aussage des zweiten Satzes.

    Nein, das ist nicht der Fall. Da Sie diese Behauptung nicht begründen, kann ich leider nicht sagen, wo Ihr Fehler liegt.

    Dann aber sehe ich, dass, was er sagt, ja auch nicht stimmt, denn Beschleunigung gibt es nicht nur bei der Umkehr, sondern notwendigerweise auch zu Beginn und Ende der Reise …

    Auch das ist falsch. Zu “Beginn” und “Ende” der Reise (d.h. bei den Ereignissen, bei denen sich Teresas und Serenas Weltlinien begegnen) ist keine Geschwindigkeitsänderung notwendig. Der Effekt ist problemlos beobachtbar, wenn Serena ohne zu stoppen an Teresa vorbeifliegt.

    Es macht daher keinen Sinn, sich gegenseitig vorzuwerfen, wie inkonsistent denn eigentlich sei, was der jeweils andere – ich oder Sie – gesagt hat.

    Ich könnte jetzt etwas über den Untertitel ihres Artikels schreiben, …

    Genau das tue ich jetzt, indem ich Sie bitte, mir Ihre abschließende Aussage “Ich habe bereits mehrmals auf einen vollkommen falschen Absatz hingewiesen. Soweit ich sehe, ist an dem nichts geändert worden” nochmals zu erläutern, indem Sie mir die Frage beantworten, was genau Sie (im Text auf meiner Seite) denn nun eigentlich als einen vollkommen falschen Ansatz erkennen (und warum). Ich weiß es einfach noch nicht.

    Den hier:

    Lasst uns annehmen, dass der stationäre Zwilling (Teresa = A) immer dann, wenn der Zeiger seiner Uhr vorrückt, ein Funksignal aussendet, das die Zahl der bei A seit Abflug von B vergangenen Sekunden nennt. Das Diagramm suggeriert, dass B ein bei A mehr als 2 Jahre langes, zusammenhängendes Intervall dieser Signale NICHT empfangen wird — was natürlich nicht sein kann, vom Modell aber behauptet wird.

    Und natürlich auch alles was aus dieser Überlegung folgen soll.

  537. #538 MartinB
    17. Dezember 2019

    @Gebhard
    Nehmen Sie doch einfach mal Ihr Bild 1 oder Ihr Bild 2 und zeichnen Sie Linien unter 45 Grad ein, dann sehen Sie genau, was mit Lichtsignalen wie passiert. Das ist ja wirklich nicht schwer. Dann werden Sie auch sehen, dass alle Lichtsignale ankommen. (Das Bild, das Anonym_2019 in #528 verlinkt hat, zeigt das zwar auch, aber anscheinend ist die Nachricht ja nicht angekommen.)

    PS: Anzunehmen, ich würde hier in meine Diagramme vollkommen falsche Winkel einzeichnen, ohne das dazuzusagen, nur weil es einer vorgefassten Meinung entspricht, zeugt schon von einem Mangel an Skeptizismus gegenüber dem eigenen Denken. Manchmal zeichne ich hier auf dem Blog tatsächlich Bilder, die nur qualitativ korrekt sind, dann sage ich das aber auch explizit.

    @Jolly
    “Hab ich erst jetzt begriffen. Obwohl das tatsächlich schon immer dastand. Eigentlich glasklar, wenn ich so darüber nachdenke.”
    Danke, solche Rückmeldungen sind extrem hilfreich, um zu sehen, wo man seine Erklärungen verbessern könnte.

  538. #539 Gebhard Greiter
    17. Dezember 2019

    @Tox #537:

    Herzlichen Dank, nun ist mir klar, an welcher Stelle ich auf meiner Seite falsch argumentiert habe (da ich die Semantik der schrägen Linien in Bild 1 ja zunächst missverstand, da mir der Winkel als nicht genau 45 Grad vorkam und mir zudem die Bedeutung der 45 Grad nicht präsent war).

    Ihre Aussage, es gäbe – in der Version des Szenarios, die wir betrachten – nur eine Beschleunigungsphase ist aber dennoch falsch, wie doch ganz klar aus dem genaueren Bild 2 hervorgeht (man betrachte dort die sich stetig verändernde Krümmung von Serenas Weltlinie in der Umgebung der beiden Ereignisse, die Abreise von der Erde und Rückkehr zu ihr darstellen).

    Zusammen mit Ihrem Argument in #537 wird klar, dass es gleich vier Szenarien gibt, die mit dem allzu ungenauen Bild 1 verträglich scheinen:

    Man kann sich raussuchen, ob man am Anfang der Reise Beschleunigung haben will oder nicht, man kann sich auch raussuchen, ob man am Ende der Reise Beschleunigung haben will oder nicht, am Umkehrpunkt aber wird man in JEDEM Fall Beschleunigung zu berücksichtigen haben (so dass mindestens hier das Bild 1 wegen nicht tolerierbarer Ungenauigkeit als falsch einzustufen ist).

    Wie Bild 2 zeigt (auf das Herr Bäker in seinem Text ja hinwies), diskutieren wird aber wirklich das Szenario, bei dem es 3 beschleunigte Phasen gibt.
    |
    Den Argumentationsfehler in meinem Text, muss ich aber natürlich korrigieren, und das gelingt so:

    Da – nach Bild 2 – Serena zu Beginn ebenso wie am Ende ihrer Reise relativ zu Teresa Geschwindigkeit Null hat, verändert sich der Winkel zwischen der Raumachse von Teresa einerseits und der von Serena andererseits so, dass er ausgehend von Null stetig steigt und fällt, um dann am Ende der Reise wieder Null zu sein.

    Ich errinnere an etwas, auf das ich in #515 schon einmal hinwies:

    Bild 1 suggeriert uns anderes – aber halt nur deswegen, weil es von einen allzu ungenauen Modell des Szenarios ausgeht: Nach Bild 1 besteht die Umkehrphase aus nur einem einzigen Ereignis, was so nicht sein kann [, da die Geschwindigkeit sich ja stetig verändern wird]. Genauer:

    Bild 1 tut so, als gäbe es für Serena nur zwei Bezugssysteme: das der Hinreise und das der Rückreise.

    Tatsächlich aber wird Serenas Bezugssystem ja ständig ausgetauscht, während aller Phasen ihrer Reise, in denen sie sich beschleunigt bewegt.

    Mein Begründungsfehler im Text auf Seite https://greiterweb.de/welt-verstehen/0-342-Das-Zwillingsparadoxon-im-Lichte-von-SRT-und-ART.htm sollte jetzt behoben sein.

    Mein Gesamtergebnis dort scheint mir weiterhin richtig zu sen.

  539. #540 MartinB
    17. Dezember 2019

    @Gebhard
    “Die Dehnung der Eigenzeit des reisenden Zwillings modifiziert sich (einschließlich Vorzeichen) proportional zu seiner Beschleunigung”
    Nein. Das ist nach wie vor falsch. Die Zeitdilatation hängt zu jedem Zeitpunkt nur von der aktuellen Relativgeschwindigkeit ab, die Beschleunigung ist dafür irrelevant. Und ich verweise auch nochmal auf das oben beschriebene 3-Brüder-Szenario, um deutlich zu machen, dass die Beschleunigung selbst irrelevant ist.

    Dass die Dilatation nicht proportional zur Beschleunigung sein kann, lässt sich auch daran sehen, dass Serena ja genauso gut doppelt so lang oder nur 1 Jahr oder 10000 Jahre lang in eine und dann in die andere Richtung fliegen kann. Die Beschleunigung beim Umkehren wäre immer dieselbe (von +0,8c auf -0,8c), wenn wir annehmen, dass sie immer in der gleichen Weise ihre Düsen zündet, aber der Altersunterschied hängt von der Strecke ab.

    Ich weiß wirklich nicht, wieso Sie sich da in einen falschen Ansatz verrennen wollen. Ich kann nur empfehlen, noch einmal zu versuchen, komplett bei Null anzufangen und alles zu vergessen, was Sie gelernt zu haben glauben, denn irgendwo machen Sie einen fundamentalen Gedankenfehler.

  540. #541 Gebhard Greiter
    17. Dezember 2019

    @MartinB #540:

    Sie haben recht: Von “proportional” zu sprechen ist nicht richtig. Es muss heißen “Die Dehnung der Eigenzeit des reisenden Zwillings modifiziert sich (einschließlich Vorzeichen) entsprechend seiner Beschleunigung.

    Natürlich kann man jetzt auf die Idee kommen, zu argumentieren, Geschwindigkeits­änderung sei ja ganz grundsätzlich äquivalent zu Beschleunigung.

    Leider ist das aber nur aus mathematischer Sicht richtig. Aus physikalischer Sicht heraus ergibt sich Beschleunigung immer dann, wenn sie sich als Folge einer sich auf- oder abbauenden Kraft ergibt.

    Da nun aber unterschiedliche Kräfte denkbar sind, deren Auf- oder Abbau in spezifischer Situation in Summe dieselbe Beschleunigung zur Folge haben könnten, wird es schwer bis unmöglich, hier noch von irgend einer Äquivalenz zu sprechen.

    Und deswegen würde ich nie — wie Joachim Schulz es tut — davon sprechen, dass erst Geschwindigkeit die Zeit mache.

    So zu sprechen käme mir vor, als würde man als Arzt dem Patienten sagen, dass seine Krankheit durch die Symptome verursacht sei (bzw. dass aus ihrem Abklingen mit Sicherheit darauf zu schließen sei, dass auch die Krankheit abklinge).

    Meine Meinung: Ein Szenario durch ein weniger komplexes zu ersetzen (wie Bild 1 das tut), macht erst Sinn, wenn man sich klar darüber ist, welche Vereinfachung man damit vor­nimmt und wie sie sich (in entsprechenden Rechenergebnissen) quantitativ auswirkt. Solche Fehlerabschätzung habe ich bisher nirgendwo angetroffen, wo man das allzu einfache Build 1 verwendet (und damit für eine ganze Klasse leicht unterschiedlicher realer Szenarien zu ein und demselben Rechenergebnis kommt).

  541. #542 MartinB
    17. Dezember 2019

    @Gebhard
    Wenn der Beschleunigungsvorgang exakt identisch ist (was ja machbar ist), die Dilatation am Ende aber nicht, dann kann die Beschleunigung nicht der Dilatation “entsprechen”. In meinen Workshops zum wissenschaftlichen Schreiben sage ich übrigens immer, dass man das Wort “entsprechen” meiden soll, weil überhaupt nicht klar ist, was gemeint ist – korreliert, analog, oder snst etwas.

    “Da nun aber unterschiedliche Kräfte denkbar sind, deren Auf- oder Abbau in spezifischer Situation in Summe dieselbe Beschleunigung zur Folge haben könnten, wird es schwer bis unmöglich, hier noch von irgend einer Äquivalenz zu sprechen.”
    Und erstaunlicherweise spielt es für den unterschiedlichen Zeitableuf zwischen S und T kaum eine Rolle, wie genau S denn nun bei Alpha Centauri bremst und umdreht. Trotzdem zu sagen, dass die “Dehnung” der Bechleunigung entspricht, obwohl identische Beschleunigungen unterschiedliche Dehnungen und unterschiedliche Beschleunigungen identische Dehungen haben können, ist einfach nur falsch. Da gibt es auch keine “Meinung”.

    (Die Fehlerabschätzung ist übrigens sehr einfach, deswegen muss man das auch nicht machen. Man könnte einfach die Länge einer gekrümmten Beschleunigungskurve mit Minkowski-Metrik aufintegrieren und sieht dann sehr schnell, dass man den Effekt beliebig klein machen kann. Das ist übrigens exakt analog zur Situation in der Geometrie, wo die größere Länge einer Kurve mit Knick auch nicht durch den Knick zu Stande kommt und ziemlich unabhängig davon ist, wie genau man nun den Knick zeichnet. Wenn ich von Braunschweig nach Berlin und dann von da nach München fahre, ist der Weg länger als direkt von Bs nach München, aber um wie viel er länger ist hängt nur sehr unwesentlich davon ab, wie genau ich in Berlin die Autobahn wechsle.)

    “Und deswegen würde ich nie — wie Joachim Schulz es tut — davon sprechen, dass erst Geschwindigkeit die Zeit mache.”
    Würde ich auch nicht, weil das eine Formulierung ist, die so unscharf ist, dass man alles rauslesen kann.

    So, und damit ist für mich nun wirklich alles gesagt.

  542. #543 Tox
    17. Dezember 2019

    @Gebhard Greiter:

    … in der Version des Szenarios, die wir betrachten …

    Vielleicht in der Version, die Sie betrachten. Aber sicher nicht notwendigerweise in den Versionen, die ich betrachte.

    … so dass mindestens hier das Bild 1 wegen nicht tolerierbarer Ungenauigkeit als falsch einzustufen ist …

    Konsequenz daraus: Das uns durch das Bild 1 nahegelegte Rechenmodell ist — während der gesamten Umkehrphase — allzu unrealistisch — so dass man seinem Ergebnis nicht trauen kann.

    Diese beiden Behauptungen sind meiner Ansicht nach falsch (s.u.).

    Ein Szenario durch ein weniger komplexes zu ersetzen (wie Bild 1 das tut), macht erst Sinn, wenn man sich klar darüber ist, welche Vereinfachung man damit vor­nimmt und wie sie sich (in entsprechenden Rechenergebnissen) quantitativ auswirkt. Solche Fehlerabschätzung habe ich bisher nirgendwo angetroffen, …

    Vermutlich, weil die Gültigkeit dieser Näherung für Physiker offensichtlich ist, und der Beweis (mit gewissem Vorwissen) mathematisch nicht besonders schwierig ist. Wenn ich das richtig sehe, braucht man nur, dass die glatten Weltlinien im Raum der stückweise glatten Weltlinien dicht sind (z.B. bezüglich einer Topologie, die von einer Metrik ähnlich der eines Sobolev-Raums mit Ableitungen bis einschließlich der ersten Ordnung erzeugt wird), und dass die Eigenzeit auf diesem Raum ein stetiges Funktional ist.

    Mein Gesamtergebnis dort scheint mir weiterhin richtig zu sen.

    Naja. Wenn Sie meinen.

  543. #544 Tox
    17. Dezember 2019

    @Gebhard Greiter:
    Ich habe gerade erst den “Nicht die Beschleunigung macht die Zeit”-Artikel von Joachim Schulz gelesen. Dort geht es ja überhaupt nicht um das Zwillingsparadoxon. Und die Beschleunigung, von der im Titel die Rede ist, hat mit der Beschleunigung des reisenden Zwillings rein gar nichts zu tun.

  544. #545 Gebhard Greiter
    17. Dezember 2019

    @Tox #544:

    Ich sehe es halt so:

    Über mehr oder weniger Beschleunigung wählt man den Weg durch die Raumzeit.

    Ihn dann gegeben, bestimmt die Geschwindigkeit, mit der wir uns räumlich bewegen, die Menge an Eigenzeit, die wir benötigen, um ans Ziel zu kommen.

    Wenn man, wie ich das von Joachims Beispielen kenne, immer nur mit Szenarien argumentiert, in denen der Weg durch die Raumzeit schon gewählt ist, wird Beschleunigung natürlich keine Rolle mehr spielen.

  545. #546 Gebhard Greiter
    17. Dezember 2019

    @MartinB #11:

    Vielleicht kann man Folgendes als Beweis von Greene’s Behauptung sehen:

    Unser Bild 2 in Kombination mit der Erkenntnis, dass der schließlich für die Zwillinge entstehende Altersunterschied prozentual umso deutlicher ausfallen wird, je höher die Beschleunigung ist, mit der B die Reise durchführt, zeigt:

    Extrem hohe Beschleunigung gleich zu Beginn der Reise – ebenso wie zu Beginn der Rückreise – führt schnell zu hoher Geschwindigkeit und damit zu schnellerem Erreichen des Ziels, so dass A weniger lange auf die Rückkehr von B zu warten hat.

    Die Bilder 1 und 2 werden dadurch gestaucht (sie werden niedriger bei gleicher Breite), so dass der Winkel zwischen den Raumachsen der beiden Zwillinge sich zunemend der 90-Grad-Marke nähert (immer vorausgesetzt, B hält so hohe Beschleunigung aus, was aber gut sein könnte, wenn B keine Person, sondern nur eine Uhr ist).

    Je entschiedener also beschleunigt wird, desto mehr führt die Weltlinie von B dann fast nur noch durch den Raum, also fast gar nicht mehr durch die Zeit.

    Die am Umkehrpunkt dann zunehmend stärker werdende Krümmung von Serenas Weltline B korresponiert mit dem gegen Null strebenden Verhältnis Eigenzeit(B)/Eigenzeit(A).

    Grenzübergang zeigt: Alles, was sich mit Lichtgeschwindigkeit durch den Raum bewegt (Licht etwa), altert gar nicht.

  546. #547 Tox
    17. Dezember 2019

    @Gebhard Greiter, #545:
    Ich verstehe nicht, was das mit meinem Kommentar zu tun haben soll. Oder auch nur mit der bisherigen Diskussion.

    Außerdem: Eine gegebene (zweimal differenzierbare) Weltlinie bestimmt die Beschleunigung in jedem Punkt eindeutig. Und eine gegebene Funktion, die zu jeder Eigenzeit die Beschleunigung zurückgibt, bestimmt die Weltlinie eindeutig (bis auf Poincaré-Transformationen, die aber eine Symmetrie der Theorie sind). Ob man die Weltlinie oder die Beschleunigung vorgibt, ist also vollkommen gleichwertig.

    Vielleicht wäre es sinnvoll gewesen, zunächst einmal klar darzulegen, was eigentlich Ihr Ziel ist.

  547. #548 Anonym_2019
    17. Dezember 2019

    @ alle:

    Dort kann jeder ein ZP-Szenario einstellen, was seine These am besten “beweist” (z.B. große Distanz oder große Beschleunigung):
    https://www.mpia.de/homes/tmueller/projects/bmwbook/web/twinparadox.html

    Dort kann man etwas über das Minkowski-Diagramm lernen, durch Verstellen der Geschwindigkeit β:
    https://www.mpia.de/homes/tmueller/projects/bmwbook/web/minkowskiDiagram.html

  548. #549 Gebhard Greiter
    17. Dezember 2019

    @Tox #547:

    Mein Ziel ist schlicht und einfach, in der Lage zu sein, das Zwillingspradoxon und andere Szenarien, bei denen es um zwei Objekte geht, die aus Sicht eines dritten Objekts unterschiedlich schnell altern, in Wort, Schrift und Graphik so erklären zu können, dass ich selbst (und dann hoffentlich auch andere) diese Erklärung als lückenlos und überzeugend empfinden.

    Von all den Beschreibungen, die ich in Büchern und im Netz fand, hat solche Qualität in meinen Augen einzig und allein die Vorlesung von John Denker ( https://www.av8n.com/physics/twins.htm ) erreicht.

  549. #550 Tox
    18. Dezember 2019

    @Gebhard Greiter
    Es ist gut, dass Sie eine Erklärung gefunden haben, die Ihnen zusagt. Ich habe gerade nicht die Zeit (oder die Lust) sie genau durchzulesen. Aber nach kurzem überfliegen des Texts scheint es mir, als würde dort auch die “Weltlinie mit Knick” verwendet, d.h. Moes Hin- und Rückweg sind unbeschleunigt, und nur im Umkehrpunkt herrscht eine (dann notwendigerweise unendlich große) Beschleunigung.

    Ich finde es ein bisschen seltsam, dass Sie die Verwendung dieser Näherung bei manchen Texten ausführlich kritisieren, bei anderen aber nicht.

  550. #551 Anonym_2019
    18. Dezember 2019

    @Tox (18. Dezember 2019) #550

  551. #552 Anonym_2019
    18. Dezember 2019

    @Tox (18. Dezember 2019) #550
    :+1:

  552. #553 Gebhard Greiter
    18. Dezember 2019

    @Tox #550:
    Ich kritisiere sie ja auch nur dann, wenn der Autor einer Erklärung explizit behauptet, dass nicht Beschleunigung die Zeit mache (und mir so suggeriert, an sie müsse ich gar nicht denken, um wirklich zu verstehen, was da passiert).

    Tatsache ist doch:

    Reisegeschwindigkeit sowie Lage und Krümmung der Weltlinie des reisenden Zwillings werden in vollem Umfang durch seine Beschleunigung bestimmt. Damit gilt gleiches für seinen Verbrauch an Eigenzeit während der Reise, denn der entspricht der raumzeitlichen Länge der Weltlinie.

  553. #554 Jolly
    18. Dezember 2019

    @Gebhard Greiter

    „Reisegeschwindigkeit [wird] in vollem Umfang durch seine Beschleunigung bestimmt.“

    Ganze Schulbücher müssen wohl umgeschrieben werden, weil noch bei vielen Aufgaben, wenn eine Dauer abhängig von der Geschwindigkeit ermittelt werden soll – z.B. die Strecke ist 60 km, hin 20 km/h, zurück 30 km/h – die Beschleunigungsphasen unterschlagen werden.

    Eventuell darf ich jetzt auch nicht mehr sagen, wenn ich mich wegen zähflüssigen Verkehrs wieder einmal verspätet habe, ich konnte nicht immer so schnell fahren, wie geplant. Statt dessen muss ich physikalisch begründet sagen, die vorzeichenrichtige zeitliche Abfolge von Beschleunigungen bei meiner Anreise hat mein pünktliches Erscheinen verhindert.

    Wie steht es mit Ruck und Knall, sind das nicht die eigentlich relevanten Größen?

  554. #555 MartinB
    19. Dezember 2019

    @Gebhard
    Wieso? Der Zwilling kann doch auf dem Hinweg die Geschwindigkeit bereits seit dem Urknall haben – soll die Dilatation wirklich davon bestimmt werden?
    Ich kann nur noch ein letztes Mal sagen: Wenn Sie das ZP verstehen wollen, dann lösen Sie sich von der zwanghaften Vorstellung, es müsse etwas mit Beschleunigungen zu tun haben. Ansonsten ist die Mühe vergeblich.

  555. #556 Gebhard Greiter
    19. Dezember 2019

    @MartinB:
    Da wir beim Betrachten des Szenarios nicht explizit gefordert haben, dass wir davon ausgehen, dass der reisende “Zwilling” nur vorbeireist, muss unsere Argumentation so gestaltet werden, dass sie auch noch ohne diese Vorbedingung gilt.

  556. #557 Gebhard Greiter
    19. Dezember 2019

    @Jolly #554:

    Wer sich irgendwo im Stau bewegt und sämtliche Autos, die vor ihm die Autobahn verstopfen, langsamer fahren als er, dem wird nichts anderes übrig bleiben als zu bremsen (d.h. negativ zu beschleunigen), um sich so für eine andere Geschwindigkeit zu entscheiden: Sie ergibt sich nicht von selbst.

  557. #558 MartinB
    19. Dezember 2019

    @Gebhard
    Umgekehrt ist es logisch: Da es für das Ergebnis egal ist, ob der Zwilling vorbeireist oder am Anfang beschleunigt, kann man seine Lösung nicht so formulieren, dass eine Beschleunigung relevant ist.
    Wenn B passiert, egal ob A oder nicht A, dann ergibt es wenig Sinn zu sagen, dass A B verursacht.

  558. #559 Gebhard Greiter
    19. Dezember 2019

    @MartinB +558:

    An sich feststellen können die Zwillinge entstandenen Altersunterschied erst, wenn sie sich wieder treffen. Damit sie sich aber wieder treffen können (und sei es nur im Vorbeiflug) muss Serena am Umkehrpunkt wenden — und das gelingt nur mit Hilfe der Abänderung ihrer Geschwindigkeit von v auf -v, d.h. per Beschleunigung.

    Ohne Beschleunigung gäbe es keine Krümmung von Serenas Weltlinie.

  559. #560 MartinB
    19. Dezember 2019

    @Gebhard
    Darum geht es mir nicht, es geht um die angebliche Wichtigkeit der anfänglichen Beschleunigung in #556.
    Zum Umkehrpunkt habe ich oben bereits genügend Dinge gesagt…

  560. #561 Anonym_2019
    19. Dezember 2019

    Gebhard Greiter (19. Dezember 2019) #559

    An sich feststellen können die Zwillinge entstandenen Altersunterschied erst, wenn sie sich wieder treffen.

    In der Mitte zwischen der Erde und Alpha Centauri hält sich Undine auf. Sie ruht relativ zu Teresa und besitzt eine Uhr, die mit der von Teresa im gemeinsamen Ruhesystem synchron läuft. Diese Uhren wurden vor langer Zeit mal nach der Vorschrift synchronisiert, die Einstein 1905 beschrieben hat. Sobald Serena bei Undine vorbeifliegt, kann Undine Serenas Uhr mit ihrer eigenen vergleichen. Undine stellt fest, dass Serena seit deren Abflug bereits weniger gealtert ist, als Undine und Teresa.

  561. #562 Tox
    19. Dezember 2019

    @Gebhard Greiter:

    Ich kritisiere sie ja auch nur dann, wenn der Autor einer Erklärung explizit behauptet, dass nicht Beschleunigung die Zeit mache …

    Wer bitteschön tut das? Wie ich bereits schrieb, geht es in dem Artikel von Joachim Schulz überhaupt nicht um das Zwillingsparadoxon. Von den Autoren die Sie kritisieren, hat soweit ich das sehe niemand diese Behauptung explizit aufgestellt.

    Und nur so nebenbei: Selbstverständlich macht die Beschleunigung nicht die Zeit. “Die Zeit” wird nicht “gemacht”. Ich finde es sehr seltsam, dass Sie sich so extrem an einer notwendigerweise vereinfachten Formulierung aus der Überschrift eines Artikels (zu einem anderen Thema) festbeißen.

    Wenn zwei Weltlinien zwei (unterschiedliche) Ereignisse in der Raumzeit verbinden, dann vergeht auf diesen im Allgemeinen unterschiedlich viel Eigenzeit. Und wenn sie zweimal differenzierbar sind, dann kann es einen Unterschied nur dann geben, wenn auf mindestens einer die Beschleunigung nicht überall verschwindet (denn andernfalls wären die Weltlinien identisch; jedenfalls so lange die Raumzeit ein R^n ist). Darauf können wir uns doch alle einigen.

    Weitergehende Aussagen der Art “die Beschleunigung verursacht den Unterschied in der Eigenzeit” halte ich nicht für sinnvoll. Das kommt mir so vor, als würde man behaupten, in 1 + 2 = 3 würde die 3 durch die 2 verursacht werden.

    … (und mir so suggeriert, an sie müsse ich gar nicht denken, um wirklich zu verstehen, was da passiert).

    Vielleicht sollten Sie sich eher fragen, warum Sie meinen, Ihnen würden all diese Dinge suggeriert.

    Reisegeschwindigkeit sowie Lage und Krümmung der Weltlinie des reisenden Zwillings werden in vollem Umfang durch seine Beschleunigung bestimmt.

    Ja, bis auf Poincaré-Transformationen. Aber das gleiche gilt auch umgekehrt. Die Weltlinie bestimmt die Beschleunigung eindeutig. Also warum sollte die Beschleunigung das wichtigere sein?

    Und wenn wir schon dabei sind, warum gerade die Beschleunigung, also die zweite Ableitung? Die dritte Ableitung bestimmt die Weltlinie ebenfalls eindeutig (bis auf drei Anfangsbedingungen), die vierte Ableitung bestimmt die Weltlinie eindeutig (bis auf vier Anfangsbedingungen), etc.

  562. #563 Jolly
    19. Dezember 2019

    @Gebhard Greiter

    „die dritte Ableitung“ (@Tox) = “Ruck“ (@Jolly)
    “die vierte Ableitung“ (@Tox) = “Knall“ (@Jolly)

  563. #564 Anonym_2019
    19. Dezember 2019

    @Gebhard Greiter (19. Dezember 2019) #559

    Ergänzung zu #561

    Das in Kommentar #561 war die Beurteilung im Ruhesystem von Undine und Teresa.

    In Serenas Beurteilung ist bis dahin nätürlich noch Teresa langsamer gealtert (aus Symmetriegründen). Die Uhren von Undine und Teresa gehen in Serenas Bezugssystem natürlich nicht synchron, wegen dessen schiefer Raumachse (=Gleichzeitigkeitslinie). Serenas Beurteilung ändert sich während des Umkehrvorgangs, bei dem sie eine Eigenbeschleunigung und damit pseudo-Gravitation spührt. Teresa altert nun schneller.

  564. #565 Anonym_2019
    20. Dezember 2019

    @Gebhard Greiter

    Die folgende Animation zeigt das ZP mit Beschleunigung:

    https://www.geogebra.org/m/M3YVE6eT

    Bedient wird das über die rote Zeitskala rechts oben.

    An dem Punkt, der sich dann bewegt (=Serena), kreuzen sich folgende Linien:
    • ct’-Zeitachse (= Tangente an Serenas Weltlinie, grün gestrichelt)
    • x’-Raumachse (= Gleichzeitigkeitstlinie von Serena, grün gestrichelt)
    • 45 Grad-Linien (= Licht & Winkelhalbierende zwischen ct’-Achse und x’-Achse)

  565. #566 Gebhard Greiter
    21. Dezember 2019

    @Tox #562:

    Die Weltlinie bestimmt die Beschleunigung eindeutig. Also warum sollte die Beschleunigung das wichtigere sein?

    Weil sich die Lage der Weltlinie ja erst als Folge der Beschleunigung ergibt (bzw. durchquerter Gravitationsfelder).

    Man sollte nicht Ursache mit Wirkung verwechseln.

    Wie ich bereits schrieb, geht es in dem Artikel von Joachim Schulz überhaupt nicht um das Zwillingsparadoxon.

    Wo es zu solcher Verwechslung von Ursache und Wirkung kommt, ist das auch dann ein Fehler, wenn der Artikel, in dem er auftritt gar nicht explizit vom Zwillingsparadoxon spricht.

  566. #567 Gebhard Greiter
    21. Dezember 2019

    @ Anonym_2019 #561:

    Diese Uhren wurden vor langer Zeit mal nach der Vorschrift synchronisiert, die Einstein 1905 beschrieben hat.

    Da ich nun schon 40 Jahre lang in keiner Umgebung mehr arbeite, aus der heraus man sofortigen Zugriff auf physikalische Fachliteratur hätte (arXiv.org ausgenommen), kann ich nicht nachsehen, wie Einstein 1905 Uhren synchronisiert hat.

    Meine Vermutung wäre, es wird wohl darauf hinauslaufen, dass er vorschlug, Ereignisse dann als gleichzeitig einzustufen, wenn sie sich im selben Photo zeigen könnten, das ein Astronom macht.

    Mir ist durchaus klar, dass das was anders bedeutet als Gleichzeitigkeit im Sinne des Minkowski-Diagramms.

  567. #568 Anonym_2019
    21. Dezember 2019

    @Gebhard Greiter (21. Dezember 2019) #567

    “kann ich nicht nachsehen, wie Einstein 1905 Uhren synchronisiert hat.”

    so:

    Der Inhalt des letzten Teils des zweiten Absatzes kann etwa so wiedergegeben werden:

    Eine für A und B verbindliche “Zeit” kann dadurch gewonnen werden, dass man die Uhren in A und B (die beide im Bezugssystem des Beobachters ruhen) synchronisiert. Der synchrone Gang der beiden Uhren kann wie folgt kontrolliert werden: Man sendet einen Lichtstrahl zur “A-Zeit” t_A von A nach B, der dort zur “B-Zeit” t_B ankomme, reflektiert werde und nach A zurücklaufe, wo er zur “A-Zeit” t’_A eintreffe. Wegen der Konstanz der Lichtgeschwindigkeit ist die Laufzeit des Lichtstrahls auf dem Hin- und Rückweg gleich. Die beiden Uhren gehen dann synchron, wenn stets (d. h. auch bei jeder Wiederholung des Versuchs)

    t_B − t_A = t’_A − t_B

    ist. (Bei nur einmaliger Durchführung würde man nämlich auch dann den Synchronlauf der Uhren konstatieren, wenn die Uhr in B den Mittelwert von t_A und t’_A anzeigte – aber stillstünde!)

    Wenn der Synchronlauf der beiden Uhren sichergestellt ist, dann sind zwei Ereignisse, die in A bzw. in B stattfinden, genau dann gleichzeitig, wenn die Anzeigen der beiden Uhren im Moment des Eintreffens der Ereignisse E_A bzw. E_B gleich sind, wenn also

    t_A ( E_A ) = t_B ( E_B )

    ist.

    Quelle:
    https://de.wikibooks.org/wiki/A._Einstein:_Kommentare_und_Erl%C3%A4uterungen:_Zur_Elektrodynamik_bewegter_K%C3%B6rper:_Kinematischer_Teil#.C2.A0_.C2.A7_1._Definition_der_Gleichzeitigkeit

    Das stimmt mit der Gleichzeitigkeit im Sinne des Minkowski-Diagramms überein.

  568. #569 Tox
    21. Dezember 2019

    @Gebhard Greiter:

    Man sollte nicht Ursache mit Wirkung verwechseln.

    Sollte man einen einzelnen Satz aus einem längeren Kommentar herausgreifen, und alles andere ignorieren?

  569. #570 Anonym_2019
    21. Dezember 2019

    @Gebhard Greiter (21. Dezember 2019) #567

    “Meine Vermutung wäre, es wird wohl darauf hinauslaufen, dass er vorschlug, Ereignisse dann als gleichzeitig einzustufen, wenn sie sich im selben Photo zeigen könnten, das ein Astronom macht.”

    Einstein hat schon vermutet, dass Sie so etwas vermuten würden. Er hat wie folgt darauf reagiert:

    Die im ersten Absatz nur versuchsweise vorgeschlagene Definition des Zeitpunkts eines vom Beobachter entfernten Ereignisses bedeutet: Ein Geschehen ereignet sich dann, wenn der Beobachter durch ein Lichtsignal davon erfährt. Natürlich verwirft Einstein diese absurde Definition sofort; er hätte sie uns auch gleich ersparen können.

    Quelle:
    https://de.wikibooks.org/wiki/A._Einstein:_Kommentare_und_Erl%C3%A4uterungen:_Zur_Elektrodynamik_bewegter_K%C3%B6rper:_Kinematischer_Teil#.C2.A0_.C2.A7_1._Definition_der_Gleichzeitigkeit

  570. #571 MartinB
    21. Dezember 2019

    @gebhard
    Diese Definition wäre ja vollkommen sinnlos, dann wäre ein Ereignis, das vor 2 Jahren in 2LJ Entfernung stattfand gleichzeitig mit einem, das vor einem Jahr in 1LJ Entfernung war und gleichzeitig mit der Ankunft des Signals.
    Lichtlaufzeiten muss man in der SRT immer herausrechnen, alles andere führt zu Verwirrung.

  571. #572 Gebhard Greiter
    21. Dezember 2019

    @MartinB #571:

    Da gebe ich Ihnen völlig recht.

    Es macht halt einen Unterschied, wie man etwas wahrnimmt: Nur mit den Augen oder ausgehend von dieser (groben) Wahrnehmung dann auch korrigiert durch unseren Verstand. Zu unterschiedlichen Zeitbegriffen kommt es nur deswegen, weil eben nicht jeder in tatsächlich jeder Situation solche Korrektur vornimmt (bzw. scheinbare Gleichzeitigkeit zu wenig von wirklicher unterscheidet).

    Deswegen habe ich Sie ja in #472 gefragt, auf welchen Zeitbegriff Sie sich in #471 beziehen.

    Und deswegen finde ich Aussagen wie “Bewegte Uhren gehen langsamer” oder “Nach Einsteins Relativitätstheorie läuft die Zeit langsamer, je schneller man sich bewegt.” ja auch so irreführend:

    https://greiterweb.de/spw/Relativitaetstheorie.htm

  572. #573 Gebhard Greiter
    21. Dezember 2019

    @Anonym_2019 #570:

    Danke für den Link.

    In Lee Smolins Buch Time reborn wird (wenn ich mich richtig erinnere) von dieser Möglichkeit, einen global Sinn machenden Gleichzeitigkeitsbegriff zu definieren, auch gesprochen.

  573. #574 Gebhard Greiter
    21. Dezember 2019

    @Tox #569:

    Den Kontext, auf den er seine Aussage bezogen wissen möchte, hat Joachim gleich im zweiten Satz seines Artikels klar gemacht:

    Eine bewegte Uhr geht aufgrund der Raumzeit-Struktur unabhängig von ihrem Mechanismus langsamer als eine ruhende Uhr. Eine Uhr auf dem Berg oder im Flugzeug schneller als eine am Boden im Tal. Dass dieser Effekt nur von Geschwindigkeit, nicht aber von Beschleunigung abhängt, hat letztens in einer Diskussion zu Unverständnis geführt.

    https://scilogs.spektrum.de/quantenwelt/nicht-die-beschleunigung-macht-die-zeit/

  574. #575 MartinB
    21. Dezember 2019

    @Genhard
    Das steht doch auch oben in meinem Text: Zu jedem zeitpunkt ist die Größe der Dilatation durch die momentane Geschwindigkeit gegeben, die Beschleunigung ist für die momentane Dilatation irrelevant. Das ist auch intuitiv einsichtig, wenn man sich überlegt, dass die Tangente an die Weltlinie im Minkowski-Diagramm die Zeitachse anzeigt; die Krümmung der Weltlinie ist dabei irrelevant.

  575. #576 Gebhard Greiter
    21. Dezember 2019

    @MartinB #575:

    Ihre Aussage finde ich nur richtig, sofern sie sich auf den von Teresa bei Serena in jeweils infinitesimal kleinen Zeitabschnitten beobachteten Fluss der Zeit bezieht.

    Für Eigenzeit aber kann Joachims Aussage nicht als richtig anerkannt werden, denn:

    Unterm Strich nämlich, zu Ende des Szenarios, wenn die Zwillinge relativ zu einander ruhen, werden sie dennoch feststellen, dass sie jetzt unterschiedlich alt sind und das auch bleiben.

    Was die Ursache der Dehnung von Serenas Eigenzeit angeht, so gilt:

    Dass die Eigenzeit von Serena gegenüber der von Teresa gedehnt wurde, ist der Beschleunigung wegen passiert, ohne sie nämlich wäre es nie zu einer Drehung von Serenas Raumachse gekommen.

    Zeit aber, die keine Eigenzeit ist, ist immer nur beobachteter Fluss dessen, was sich dem Beobachter als Zeit des beobachteten Objekts darstellt. Mehr nicht.

  576. #577 Gebhard Greiter
    21. Dezember 2019

    @MartinB #575:

    Am wenigsten misszuverstehen ist wohl folgende Aussage:

    Wenn Zwillinge von einem gemeinsam erlebten Ereignis E1 (an dem sie gleich alt sind) auf unterschiedlichen Wegen durch die Raumzeit hin zu einem gemeinsam erlebten Ereignis E2 kommen, können sie dort nur dann unterschiedlich alt sein, wenn sie während ihrer Trennung – zeitweise wenigstens – unterschiedlich beschleunigt waren.

    Sobald ein Altersunterschied entstanden, aber noch nicht wieder abgebaut ist, wird er sich in Abhängigkeit ihrer Relativgeschwindigkeit verändern.

  577. #578 Jolly
    22. Dezember 2019

    @Gebhard Greiter

    Wo es Ihnen doch um Genauigkeit geht, wie trennen Sie eigentlich die Zwillinge, ohne sie – zeitweise wenigstens – unterschiedlich beschleunigt zu haben?

    Sie müssen mir nicht antworten, ich möchte nur (erneut) darauf aufmerksam machen, dass wir, inklusive Ihnen, nicht so sprechen, wie sie das unterstellen. Wenn zur Dauer einer Reise die Geschwindigkeit proportional (oder über- oder unterproportional) ist, nicht aber die Beschleunigung, dann ist es üblich und gerechtfertigt, zu sagen, die Dauer wird durch die Geschwindigkeit bestimmt; der Stromfluß vom Leitungsquerschnitt; (weil Advent ist:) die Farbe und Leuchtstärke der Flamme von der Temperatur.

    Hinter den Zusammenhängen können sich ganz unterschiedliche oder auch keine Kausalitäten verstecken.

    Sie haben noch nicht auf die Frage geantwortet, warum Sie alles auf Beschleunigungen beziehen, wobei doch Beschleunigungen von Beschleunigungsänderungen (3. Ableitung, bzw. Ruck) abhängig sind. (Nicht nur) Sie sagen, ohne Beschleunigung keine Geschwindigkeitsänderung; (nicht nur) ich sage, ohne Beschleunigungsänderung keine Beschleunigung.

  578. #579 Jolly
    22. Dezember 2019

    Kleine Korrektur zu #578, damit der letzte Satz Sinn ergibt muss es darin heißen, ohne Beschleunigung keine Geschwindigkeit; ohne Ruck keine Beschleunigung.

  579. #580 Gebhard Greiter
    22. Dezember 2019

    @Tox #578:

    Ich gebe Ihnen recht: Es ist üblich, dass sich Physiker so ausdrücken, wie Sie sagen.

    Sie verstehen sich und die Sache, über die sie sprechen, dennoch: aber halt nur, weil sie all das schon verstanden haben (und ihre allzu ungenaue Ausdrucksweise sie nicht mehr verwirrt).

    In Sachbüchern oder anderen Texten, die sich vor allem an Laien wenden, sollte man sich aber schon einer genaueren Ausdrucksweise bedienen, da sonst ja die Gefahr besteht, dass sach-unkundige Leser die Aussage des Sachbuchs missverstehen (wie ich
    aus lange zurückliegender eigener Erfahrung weiß).

    Auf Seite https://greiterweb.de/spw/Relativitaetstheorie.htm finden sich mehrere Beispiele.

    Und es gibt ja Physiker, die es schaffen, sich deutlicher auszudrücken (Kip Thorne etwa, aber z.B. auch John Denker, der auf Seite https://www.av8n.com/physics/twins.htm in der Überschrift zu seinem Kapitel 2 dort dasselbe oft angetroffene Missverständnis auszuräumen sucht wie ich auf Seite https://greiterweb.de/spw/Relativitaetstheorie.htm.

    Denker korrigiert es mit dem zweiten Teil seiner Überschrift, die lautet: “Do Moving Clocks Run More Slowly? No!“.

    Und natürlich können Zwillinge sich nur dadurch trennen, dass sie sich unterschiedlich beschleunigen. Das ändert aber nichts daran, dass einzig und allein unterschiedliche Beschleunigung die Ursache dafür sein kann, dass ihre Raumachsen zeitweise in unter­schiedliche Richtung zeigen (was schließlich und endlich – wenn sie sich wieder treffen – in ihrem dann i.A. unterschiedlichen Alter resultiert).

    Und wenn es Sie denken, dass ich Beschleunigung als Ursache sehe, wo sie Änderung von Beschleunigung als Ursache sehen wollen, dann erinnere ich Sie daran, dass ich nicht einfach nur Beschleunigung, sondern unterschiedlich starke Beschleunigung als Ursache sehe (s. z.B. #577):

    Bei stets gleicher Beschleunigung würden ihre Raumachsen niemals in unterschiedliche Richtung zeigen. Von einander weg bewegen könnten sich die Zwillinge dennoch, wie sich zeigt, wenn man sich ihren Weg als eine symmetrische Schleife in Form einer 8 vorstellt, die von beiden in umgekehrter Richtung durchreist wird, so dass Start und Ende der Reise beider am selben Ort stattfinden. [ Man könnte sich vorstellen, dass die beiden Raumschiffe, welche die Zwillinge transportieren, durch ein und dasselbe Funksignal gesteuert werden, das eine Kontrollstation aussendet, die sich im Kreuzungspunkt der Schleife befindet: dort also, wo die Reise beider beginnt und endet. Aus Symmetriegründen ist klar, dass in diesem Szenario am Ende der Reise die beiden gleich alt sein werden, sofern sie es zu Beginn der Reise waren. ]

  580. #581 Gebhard Greiter
    22. Dezember 2019

    Korrektur der ersten Zeile von #580:

    Es muss dort @Jolly #578 (statt @Tox) heißen. Sorry.

  581. #582 Jolly
    22. Dezember 2019

    @Gebhard Greiter

    Was Sie auf Ihrer Internetseite schreiben, „dass die Physik den Zeitpunkt, zu dem ein Ereignis E aus Sicht eines Beobachters B eintritt, definiert als den Zeitpunkt, zu dem B frühest möglich von E erfahren haben kann“, ist nicht ungenau, sondern falsch. Daraus entwickeln sich dann offensichtlich weitere Missverständnisse, bzw. diese Definition als Grundlage verwendet macht es Ihnen unmöglich, korrekte Beschreibungen als solche zu erkennen und zu verstehen.

    Um bei dem Bild hier zu bleiben.

    „Das Diagramm suggeriert, dass B ein bei A mehr als 2 Jahre langes, zusammenhängendes Intervall dieser Signale NICHT empfangen wird“

    Der Zeitraum, für den keine gleichzeitigen Ereignisse bei Serena existieren, ist deutlich mehr als 3, fast 4 Jahre; auf der Zeitachse Teresas abzulesen: knapp über 3 beginnend, knapp unter 7 endend.

    Dass auch Signale aus diesem Zeitraum von Serena empfangen werden, dass von allen Ereignissen daher also auch Photos von ihr gemacht werden können, und wann genau, kann man dem Bild 3 bei Ihnen, linke Seite, entnehmen.

    Stimmen Sie mir mittlerweile soweit zu?

    Auf ihrer Seite schreiben sie weiter, „Sofern Bewegung den Abstand des Beobachters vom beobachteten Objekt verkleinert, hat man als Beobachter den Eindruck, die Zeit des Objekts verginge schneller als die eigene“

    Wie Sie anhand der eingezeichneten Gleichzeitigkeitslinien im Bild leicht nachrechnen können, ist das ebenfalls falsch. Die 4 Jahre Rückflug entsprechen aus Serenas Sicht nur etwas mehr als 3 Jahre bei Teresa, die Zeit vergeht auch dann dort langsamer, während sie aufeinanderzufliegen. Die Zeiger der Uhren in Teresas Nähe, die von Serena beobachtet werden, scheinen aufgrund des Dopplerereffekts zwar schneller zu drehen, das sind aber keinesfalls Uhren mit denen eine Zeitmessung von Serena durchgeführt werden sollte (sie sind nicht ruhend und nicht synchronisiert).

  582. #583 Anonym_2019
    22. Dezember 2019

    @Gebhard Greiter (22. Dezember 2019) #580

    “Denker korrigiert es mit dem zweiten Teil seiner Überschrift, die lautet: “Do Moving Clocks Run More Slowly? No!“.”

    Diese Überschift von Denker halte ich für irreführend bis falsch, weil er Bezugssysteme durcheinanderwürfelt. Zumindest ist es ihm gelungen, Sie damit in die Irre zu führen, wie die detaillierte Analyse von Jolly im Kommentar #582 zeigt.

    • Im Ruhesystem von Teresa läuft die Uhr von Serena ständig um den Faktor “Gamma” verlangsamt.
    • Im Ruhesystem von Serena läuft die Uhr von Serena nicht verlangsamt. Das ist aber ein Bezugssystem, in dem Serenas Uhr gerade nicht “moving” ist.

  583. #584 Anonym_2019
    22. Dezember 2019

    @Gebhard Greiter (21. Dezember 2019) #573

    “In Lee Smolins Buch Time reborn wird (wenn ich mich richtig erinnere) von dieser Möglichkeit, einen global Sinn machenden Gleichzeitigkeitsbegriff zu definieren, auch gesprochen.”

    Jeder andere Gleichzeitigkeitsbegriff als dieser von Einstein macht in der SRT keinen Sinn:

    We can certainly define simultaneity based on the isotropy of light speed, just as we can define simultaneity based on the isotropy of mechanical inertia. Each of these is merely a definition, but the proposition that these two definitions of simultaneity based on two apparently distinct classes of physical phenomena yield the same simultaneity is a testable hypothesis with physical content.

    Needless to say, it remains true that the use of inertia-based coordinates to define secondary quantities such as “speed” is a convention – but it is by no means an arbitrary convention. The principle of inertia serves as an organizing principle, much like the conservation of energy. These are useful concepts because we find, empirically, that the descriptions of phenomena and their inter-relationships take their simplest and most perspicacious form when expressed in terms that maintain those symmetries.

    Quelle:
    https://www.mathpages.com/home/kmath684/kmath684.htm

  584. #585 Gebhard Greiter
    24. Dezember 2019

    @Jolly #582:

    Danke für den Hinweis. Ich habe ich mich dort tatsächlich nicht richtig ausgedrückt.

    Die durch Sie beanstandete Stelle – und auch der Text https://greiterweb.de/welt-verstehen/0-433-Was-uns-oft-nicht-bewusst-ist.htm#msgnr0-433 (auf den dort verwiesen wird) – sollten jetzt korrigiert sein.

    Der Rest dessen, was Sie in #582 sagen, scheint sich auf eine schon überholte Version meiner Seite https://greiterweb.de/welt-verstehen/0-342-Das-Zwillingsparadoxon-im-Lichte-von-SRT-und-ART.htm zu beziehen.

    Was schließlich meine Aussage “Sofern Bewegung den Abstand des Beobachters vom beobachteten Objekt verkleinert, hat man als Beobachter den Eindruck, die Zeit des Objekts verginge schneller als die eigene” betrifft, so wird sie unter Berücksichtigung des Dopplereffekts in den meisten Fällen durchaus richtig sein ( nach der Formel auf Seite https://greiterweb.de/spw/Zu-Zeitdilatation-und-Dopplereffekt.htm ).

  585. #586 Gebhard Greiter
    24. Dezember 2019

    @Anonym_2019 #583:

    John Denkers Aussage “Do Moving Clocks Run More Slowly? No!.” kann allein schon deswegen nicht falsch sein, da sich jede Uhr aus Sicht unterschiedlicher Beobachter ja ganz unterschiedlich schnell bewegen kann.

    “Moving” sind Uhren eben nur aus Sicht der relativ zu ihnen bewegten Beobachter.

    Klar sein sollte:

    Was wir Zeit nennen, ist gemessene, von der Uhr abgelesene Zeit, d.h. durch stetige Abstandsveränderung und die Endlichkeit der Lichtgeschwindigkeit relativ gewordenes Ergebnis einer Beobachtung der Eigenzeit der Uhr.

    Relativ zu einer Uhr unterschiedlich schnell bewegte Beobachter lesen unterschiedliche Zeit von ihr ab. Ihre Eigenzeit wird dadurch nicht verändert.

    Den Fluss von Eigenzeit kann nur Beschleunigung verändern.

  586. #587 Anonym_2019
    24. Dezember 2019

    @Gebhard Greiter (24. Dezember 2019) #586

    “Moving” sind Uhren eben nur aus Sicht der relativ zu ihnen bewegten Beobachter.

    Genau. Und in den Ruhe-/Bezugssystemen dieser Beobachter laufen diese “moving” Uhren slowly. Yes!

    Was wir Zeit nennen, ist gemessene, von der Uhr abgelesene Zeit, d.h. durch stetige Abstandsveränderung und die Endlichkeit der Lichtgeschwindigkeit relativ gewordenes Ergebnis einer Beobachtung der Eigenzeit der Uhr.

    Nein. Zeit ist, was die Uhr anzeigt, nicht das, was wie aus der Entfernung ablesen.

  587. #588 Gebhard Greiter
    24. Dezember 2019

    @Anonym_2019:

    Was die Uhr anzeigt, ist ihre Eigenzeit – nichts sonst.

    Es ist nicht die Zeit, die ein relativ zu ihr bewegter Beobachter von ihr abliest.

    Auf die Frage, was Zeit denn eigentlich sei, soll Einstein geantwortet haben: “Zeit ist, was man von der Uhr abliest.”

    Was man nun aber von ihr abliest, hängt davon ab, wie schnell man sich relativ zu ihr bewegt.

    Und genau das macht John Denker in seinem Artikel sehr schön klar.

    Aussagen wie “Bewegte Uhren gehen langsamer” sagen jedem, der sie wörtlich nimmt, was ganz anderes. Sie sind Physiker-Kauderwelsch.

  588. #589 Gebhard Greiter
    24. Dezember 2019

    @Anonym_2019 #587+588:

    Bitte bedenken Sie auch:

    Relativ ist nicht die Zeit, die eine Uhr anzeigt, sondern immer nur die, welche man von ihr abliest.

  589. #590 Karl-Heinz
    24. Dezember 2019

    @Gebhard Greiter

    Ich habe euer Streit Gespräch jetzt nicht ganz mitverfolgt. Aber Anonym_2019 hat soweit ich das sehe durchaus Recht. Zum Beispiel könnte ich rund um mich Beobachtungsposten aufbauen, dessen Uhr natürlich mit meiner synchronisiert ist. Dieser Posten liest dann die Zeit von der bewegten Uhr genau dann ab, wenn diese bewegte Uhr genau bei ihm vorbeikommt. Die Beobachtungen werden anschließend eingesammelt und ausgewertet. Nach der Auswertung wirst du feststellen, dass eine bewegte Uhr wirklich langsamer geht. 😉

  590. #591 Anonym_2019
    25. Dezember 2019

    @Gebhard Greiter (24. Dezember 2019) #588 & #589

    Auf die Frage, was Zeit denn eigentlich sei, soll Einstein geantwortet haben: “Zeit ist, was man von der Uhr abliest.”

    Es gibt da noch ein anderes Gerücht über die Antwort von Einstein:

    Auf die eingangs gestellte Frage nach dem Wesen der Zeit hat Einstein geantwortet: „Zeit ist, was die Uhr anzeigt.“

    Quelle (dort 2. Seite):
    https://www.uni-muenster.de/Physik.TP/~munsteg/10Zeit.pdf

    Was man nun aber von ihr abliest, hängt davon ab, wie schnell man sich relativ zu ihr bewegt.

    Solche Ablesefehler sollte man halt vermeiden (z.B. Ablesen in transversaler Richtung aus unmittelbarer Nähe) oder die Störeffekte herausrechnen.

    “Und genau das macht John Denker in seinem Artikel sehr schön klar.”

    John Denker schreibt jede Menge Unsinn, z.B.:

    This is a paradox. We cannot have each guy’s clock be slower than the other.

    Quelle:
    https://www.av8n.com/physics/twins.htm

    Dass das kein Paradoxon ist, sieht man gut an dem Raumzeit-Diagramm mit Teresa und Serena.

    “Aussagen wie “Bewegte Uhren gehen langsamer” sagen jedem, der sie wörtlich nimmt, was ganz anderes. Sie sind Physiker-Kauderwelsch.”

    Aussagen wie “Do Moving Clocks Run More Slowly? No!.” sagen jedem, der sie wörtlich nimmt, was ganz anderes. Sie sind Physiker-Kauderwelsch. Man sollte immer das mit dem Bezugssystem hinzuschreiben.

    “Relativ ist nicht die Zeit, die eine Uhr anzeigt, sondern immer nur die, welche man von ihr abliest.”

    Nein. Was invariant ist, ist die raumzeitliche Weg-Länge zwischen zwei Ereignissen. Die ist identisch mit dem entsprechenden mit der Eigenzeit-Uhr ermittelten Zeitintervall. Ablesefehler, weil z.B. Laufzeiteffekte nicht herausgerechnet wurden, sind hier irrelevant. Wenn die Zeit nicht relativ wäre, bräuchte man das Konzept der eigenen Zeit “Eigenzeit” nicht.

  591. #592 Karl-Heinz
    25. Dezember 2019

    @Gebhard Greiter

    Den Fluss von Eigenzeit kann nur Beschleunigung verändern.

    Das musst du mir anhand eines konkreten Beispiel erläutern. Da Myonen als bewegte Uhren aufgefasst werden können, denke ich da an das Myonen-Experiment in Cern, also im Speicherring kreisenden Myonen. 😉

  592. #593 Gebhard Greiter
    25. Dezember 2019

    @Anonym_2019:

    In #587 schreiben Sie:

    Nein. Zeit ist, was die Uhr anzeigt, nicht das, was wie aus der Entfernung ablesen.

    Stellen Sie sich jetzt zwei Beobacher der Uhr vor: B1 bewege sich auf sie zu, B2 bewege sich mit doppelter Geschwindigkeit von ihr weg. Es wird deswegen B1 von der Uhr andere Zeit ablesen als B2.

    Dennoch wird die Uhr nur eine Zeit anzeigen – es kümmert sie gar nicht, dass sie beobachtet wird.

    Das zeigt doch ganz klar, dass wir hier nur von 3 unterschiedlichen Zeitbegriffen reden. Einerseits vom Zeitbegriff der Uhr und andererseits von den auch noch unterschiedlichen Zeitbegriffen ihrer Beobachter B1 und B2.

    Und genau das will John Denker mit seinen Ausführungen uns sagen. Was er sagt, ist absolut richtig und sollte jedem, der sich für Relativitätstheorie interessiert als Allererstes beigebracht werden, denn:

    Wenn Sie einem Schüler, der zum ersten Mal was von Relativitätstheorie hört, sagen “Bewegte Uhren gehen langsamer“, dann wird der nicht gleich daran denken, dass es Bewegung im absoluten Sinn ja gar nicht gibt, sondern stets nur Relativbewegung.

    Es wird ihm also nicht klar werden, dass dieser Satz nur wahr ist, wenn man den durch die Relativgeschwindigkeit zwischen Uhr und Beobachter definierten Zeitbegriff zugrunde legt und ein zweiter Beobachter, der sich relativ zur Uhr anders bewegt, derselben Aussage einen ganz anderen Zeitbegriff zugrunde legen wird (und das sogar noch, wenn er sich gerade nur wenige Meter weit vom ersten entfernt befände).

    Ich bleibe also dabei:

    Wer sagt “Bewegte Uhren gehen langsamer“, der spricht von der Zeit, die man als Beobachter der Uhr von ihr abliest, nicht von der, welche sie anzeigt.

  593. #594 Gebhard Greiter
    25. Dezember 2019

    @Tox #544:

    Ich vermute jetzt, dass Joachim mit seiner von mir als allzu irreführend kritisierten Aussage, nicht Beschleunigung mache die Zeit ( https://scilogs.spektrum.de/quantenwelt/nicht-die-beschleunigung-macht-die-zeit/ ), wohl einfach nur aussagen wollte, was der zweite Teil folgender Feststellung ist:

    Beschleunigung bestimmt Eigenzeit (= das, was eine mehr oder weniger beschleunigte Uhr anzeigt),

    Geschwindigkeit (unsere Relativgeschwindigkeit zu ihr) bestimmt, welche Zeit wir von der Uhr ablesen.

  594. #595 Karl-Heinz
    25. Dezember 2019

    @Gebhard Greiter

    Ich vermute, dass dir wesentliche Grundlagen fehlen. So weißt du offensichtlich nicht genau, wie ein Ereignis definiert ist. Ist nicht böse gemeint. 😉

  595. #596 Gebhard Greiter
    25. Dezember 2019

    @ Karl-Heinz #595:

    Wenn ich von einem Ereignis spreche, meine ich damit einen Punkt der 4-dimensionalen Raumzeit.

  596. #597 Karl-Heinz
    25. Dezember 2019

    @Gebhard Greiter

    Ja, und wenn die bewegte Uhr zu diesem Ereignis E(t_0,x_0,y_0,z_0) vorbeischaut, dann wird die Zeit der bewegten Uhr abgelesen. Man liest die Uhr nicht im Bereich E(t,0,0,0) ab, sondern bei E(t_0,x_0,y_0,z_0). Das ist ein feiner Unterschied.

  597. #598 Gebhard Greiter
    25. Dezember 2019

    @ Karl-Heinz #597:

    Deine Nachricht erreicht mich in einer Formulierung, der ich zu wenig klar definierte Semantik entnehmen kann, als dass ich ihr einen Wahrheitswert zuordnen könnte.

    Halten wir doch zunächst mal fest: Messergebnis (= Beobachtung) ist immer ein dem Messinstrument zugegangener, vom beobachteten Objekt erzeugter Snapshot seiner selbst: ein Bild, das dem Messinstrument mit Lichtgeschwindigkeit zugestellt wird.

    Dieses Bild wird vom Messinstrument interpretiert im Lichte und der Terminologie seines eigenen Zeitbegriffs (der i.A. eben nicht der Zeitbegriff des beobachteten Objekts ist).

  598. #599 Karl-Heinz
    25. Dezember 2019

    @Gebhard Greiter

    Geschwindigkeit (unsere Relativgeschwindigkeit zu ihr) bestimmt, welche Zeit wir von der Uhr ablesen.

    Nein und nochmals nein. Man liest von der bewegten Uhr jene Zeit ab, die sie anzeigt. Natürlich wir diese angezeigte Zeit mit der eigenen Zeit verglichen. Und da ergeben sich schon Unterschiede, die von der Relativgeschwindigkeit zur bewegten Uhr abhängig ist.

  599. #600 Karl-Heinz
    25. Dezember 2019

    @Gebhard Greiter

    Das Messinstrument ist riesengroß. Man breitet über des gesamten Raum ein Gitter mit Uhren aus, die natürlich alle miteinander synchronisiert sind. Da brauch ich nichts mit Lichtgeschwindigkeit zustellen. Man kann sich ruhig zeit lassen. Natürlich müssen für die Auswertung alle relevanten Daten zur Verfügung stehen. Klingt doch logisch oder?

  600. #601 MartinB
    25. Dezember 2019

    @Gebhard
    Vielleicht mal die Kapitel 7-10 meines Buches lesen, da steht auch genau drin, wie man das mit den Uhren und den bezugssystemen richtig macht – wie Karl-heinz schon sagt, denkt man sich ein unendliches Gitter aus Uhren bzw. Kameras, die alles aufzeichnen, ausgewertet wird dann hinterher in Ruhe.

  601. #602 Gebhard Greiter
    25. Dezember 2019

    @ Karl-Heinz #600:

    Wenn die Uhren schon synchronisiert sind, hast du natürlich recht. Sie geeignet zu synchronisieren, muss man sich zuvor aber eben doch überlegt haben, wie ein Messergebnis denn überhaupt zustande kommt und wie lange die Information, die ihm innewohnt, unterwegs war, bis sie das Messinstrument erreicht hat.

  602. #603 Karl-Heinz
    25. Dezember 2019

    @Gebhard Greiter

    Am Besten das Buch von MartinB kaufen. Kostet im Gegensatz zu Fachbüchern nicht viel. 🙂

    Das mit der Zeit ist nicht so einfach. Wenn man fragt, was der Unterschied zwischen zwei Ereignissen ist, die zur gleichen Zeit aber an unterschiedlichen Orten statt finden, so lautet die gefühlte Antwort „die Entfernung“. Ein anderer Beobachter, der sich relativ zu uns bewegt, stellt in seinem System fest, dass die zwei Ereignisse nicht gleichzeitig sind.

    Die Idee mit den unendliches Gitter aus Uhren bzw. Kameras hat schon seinen Sinn. 😉

  603. #604 Gebhard Greiter
    25. Dezember 2019

    @ Karl-Heinz & MartinB #602-603:

    Wie die ganze Diskussion hier zeigt, scheint es unglaublich schwierig, eine gemeinsame Sprechweise zu haben, mit deren Hilfe an jeder Stelle der Diskussion glasklar ist, ob man das noch uninterpretierte oder das schon geeignet interpretierte Beobachtungsergebnis meint (raw vs cooked würde man als Informatiker sagen).

    Ich nehme mir jetzt vor, wirklich zu tun, was Du und MartinB vorschlagen: Sein Buch zu lesen.

  604. #605 MartinB
    25. Dezember 2019

    @gebhard
    Die e-book-Version gibt es übrigens gerade für nur 9,99.

  605. #606 Gebhard Greiter
    25. Dezember 2019

    @ Karl-Heinz #592:

    Wo Myonen sich im Kreis mit fast Lichtgeschwindigkeit bewegen, sind sie beschleunigt.

    Die Beobachtung also, dass sie dann im Vergleich zu anderen Myonen, die man direkt neben dem Speicherring in einer Penning-Falle festhält, länger existieren, kann dann meiner Ansicht nach durchaus als Bestätigung der Aussage “Beschleunigung bestimmt Eigenzeit ” gesehen werden.

    Wer anderes glaubt, sollte es schon auch quantitativ durch Zahlen und genaue Messung belegen können.

    Interessanter scheint mir das Beispiel der zur Erde fallenden Myonen.

    |
    Vorweg:

    Als paradox erscheint uns eine real gegebene Situation immer nur, solange wir sie nicht genau genug verstanden haben.

    Was also kann man am Beispiel der zur Erde fallenden Myonen missverstehen?

    Dem nachzugehen, sollte man sicherstellen, zunächst mal genau verstanden zu haben, von welch grundsätzlicher Qualität physikalische Modelle sind:

    Ein in der Sprache der Mathematik formuliertes physikalisches Modell ist ein mathema­tischer Gegenstand, dessen Verhalten durch mathematische Zusammenhänge eindeutig bestimmt ist: ein präzise definierter Mechanismus.

    Wert bekommt dieses Modell erst, wenn Experimentalphysik feststellt, dass sein Verhalten dem Verhalten der Natur entspricht (in dem Sinne, wie z.B. in der Gruppentheorie das Verhalten einer Gruppe G1 dem Verhalten einer Gruppe G2 entspricht, wenn es sich um zueinander isomorphe Gruppen handelt).

    Die Raumzeit ist ein Menge unterscheidbarer Objekte, denen sich – anhand der Minkowski-Metrik – paarweise Abstand zuordnen lässt.

    Einstein fand zwei Modelle für sie: das einfache der SRT und das deutlich kompliziertere, aussagekräftigere der ART.

    Man sieht: Unterschiedliche Modelle können unterschiedlich weit führen, entsprechen der Wirklichkeit i.A. aber immer nur teilweise. Sie stellen Analogien zur Wirklichkeit dar. Genáuer: Ihr Verhalten ist – jeweils immer nur hinsichtlich einer gewissen
    Menge von Aspekten – analog dem der Natur).

    Wie Niels Bohr immer wieder betont hat, sollten wir dabei nie vergessen, dass selbst Physik die Natur nur ihrem Verhalten nach kennen kann, niemals aber ihrer wahren Natur nach. Die Natur – als der Mechanismus, der uns eigentlich interessiert – präsentiert sich uns nur als Black Box: Wir haben kein modellunabhängiges Verständnis von ihr [schrieb Hawking].

    |
    Betrachten wir jetzt also zur Erde fallende Myonen im Lichte des Raumzeitmodells der SRT:

    Die SRT spricht nicht davon, dass die Raumzeit irgendwie gekrümmt sei oder auch nur so gesehen werden könne.

    Die SRT gibt einzig und allein Antworten auf die Frage, wie relativ zu einander bewegte Objekte sich gegenseitig wahrnehmen.

    Anders als in der Quantenphysik ist in der klassischen Physik – und dazu zählen SRT und ART – ein Objekt X zu beobachten keineswegs gleichbedeutend damit, seinen Zustand Z(X) zu verändern.

    In der klassischen Physik ist jede Beobachtung (d.h. jedes Messergebnis) Beschreibung eines Teilzustandes Z(X) von X zu einem Zeitpunkt, die man zu kennen glaubt im Rahmen mindestens eines wohldefinierten Zeitbegriffs.

    Daraus folgt: Wenn unterschiedlich schnell gegen eine Uhr bewegte Beobachter jene Uhr als unterschiedlich schnell gehend wahrnehmen, oder den Abstand zweier Uhren als unterschiedlich groß einstufen, dann tangiert das diese Uhren selbst überhaupt nicht:

    Es bedeutet einfach nur, dass jene Beobachter die Raumzeit durch unterschiedliche Koordinatengitter betrachten (so wie John Denker das mit insbesondere Bild 5 auf seiner Seite https://www.av8n.com/physics/twins.htm ja sehr schön klar macht).

    Wie also, so frägt man sich, kann es im Szenario der auf die Erde fallenden Myonen dann aber dazu kommen, dass sie trotz ihrer so kurzen mittleren Zerfallszeit von nur 2.2 μs dennoch einen Höhenunterschied von etwa 10 km durchreisen können?

    |
    Meine Antwort darauf:

    Jede Quantifizierung zeitlicher und räumlicher Abstände liefert ein Ergebnis, dessen Wert abhängig ist vom Koordinatengitter, welches wir zugrundelegen.

    Was wir quantifizieren, ist stets nur Wahrnehmung beschrieben aus der – zudem noch stets relativen – Perspektive des Wahrnehmenden.

  606. #607 Anonym_2019
    26. Dezember 2019

    @Gebhard Greiter (25. Dezember 2019) #606

    “in der klassischen Physik – und dazu zählen SRT und ART”

    Das ist in dieser Allgemeinheit falsch. Der Begriff “klassischen Physik” wird uneinheitlich verwendet:

    Die klassische Physik umfasst die Teilgebiete der Physik, die ohne die Konzepte der Quantisierung und der vierdimensionalen Raumzeit auskommen.
    ..
    Bisweilen wird auch die spezielle Relativitätstheorie dazugezählt

    Quelle:
    https://de.wikipedia.org/wiki/Klassische_Physik

    Das folgende ist, glaube ich, eine richtige Beschreibung von Joachim Schulz:

    Ein Theorie-Professor von mir meinte mal Sinngemäß: “Klassische Physik ist immer all das, was wir gerade nicht behandeln.”

    Die SRT ist in dem Sinne eine klassische Theorie, dass die auf das Relativitätsprinzip aufbaut und vollständig realistisch und lokal ist. Die ART ist eine klassische Feldtheorie. Sie enthält keine Quanteneffekte. Trotzdem wird man, wenn man relativistische Effekte behandelt, die Newtonsche Physik in der regel als klassische Physik in Abgrenzung zur neueren Relativitätstheorie bezeichnen.

    Quelle:
    https://scilogs.spektrum.de/quantenwelt/die-symmetrie-der-bewegung/#comment-37386

    Wenn Sie über die SRT schreiben, halte ich es für verwirrend, wenn sie diese in diesem Kontext zur “klassichen Physik” zählen, und nicht die Unterschiede zwischen der SRT und der Newton’sche Physik herausstellen.

  607. #608 Karl-Heinz
    26. Dezember 2019

    @Gebhard Greiter

    Beschleunigung bestimmt Eigenzeit

    Ich habe ganz bewusst das Beispiel gewählt, wo sich Myonen mit fast Lichtgeschwindigkeit auf einer Kreibahn bewegen. Der Betrag der Beschleunigung ist a = ω² · r.
    Der Betrag der Bahngeschwindigkeit ist v = ω · r.
    Ich könnte jetz die Winkelgeschwindigkeit ω halbieren und den Radius vervierfachen um die gleiche Beschleunigung a zu erhalten. Nach deiner Behauptung müsste jetzt auch die Zeitdilatation bzw. für ein gegebenes dt die Eigenzeit in beiden Fällen gleich groß sein, was aber definitiv nicht der Fall ist. Ich hoffe du kannst das nachvollziehen. Ich sehe gerade, dass auf andere Unstimmigkeiten bereits Anonym_2019 hingewiesen hat. 😉

  608. #609 Gebhard Greiter
    26. Dezember 2019

    @ Karl-Heinz #608:

    Dein Argument zeigt lediglich, dass sich der Altersunterschied, der sich für die Zwillinge Tereasa und Serena ergab, nicht proportional zur Beschleunigung zustande kam.

    Tatsächlich ergibt er sich infolge der Drehung des Koordinatengitters. Sie aber kann nur durch Beschleunigung zustande kommen, und ihr Effekt kann nur durch negative Beschleunigung wieder abgebaut werden kann.

    Ich habe das oben erklärt in den letzten 13 Zeilen von #580.

  609. #610 Gebhard Greiter
    26. Dezember 2019

    @Anonym_2019 #607:

    Nachdem Sie Ihre Meinung durch 2 Beispiele belegen, ist es wohl wirklich so, dass nicht alle Physiker den Begriff “klassische” Physik gleich interpretieren.

    Dass sie ihn im umgangssprachlichen Sinne nutzen, scheint mir spätestens nach der Jahrtausendwende aber doch recht selten geworden zu sein.

    Und dass jemand, der Newtons Gravitationstheorie meint (im Gegensatz zur ART) einfach nur von “klassischer Physik” spricht, statt explizit von “Newtons Physik”, ist mir noch gar nicht untergekommen.

    In allen Büchern, die ich während der letzten 10 Jahre las – zu Physik oder Philosophie – wird unter klassischer Physik die Physik verstanden, die davon ausgeht, dass die Welt streng deterministisch funktioniere.

    Im 20. Jahrhundert hat sich am deutlichsten und nachdrücklichsten Einstein selbst für diesen Standpunkt ausgesprochen: Man denke an seine Meinung “Der Alte [Gott also] würfelt nicht”. Soweit ich weiß, hat er an dieser Meinung bis zu seinem Tod ganz unerschütterlich festgehalten.

    Spätestens seit Entdeckung der Bellschen Ungleichung – deren Nützlichkeit 20 Jahre später zunächst Experimente zweier US-amerikanischer Physiker, dann aber noch überzeugender Alan Aspects Experimente bestätigt haben – weiß man, dass Einstein unrecht hatte und stattdessen die Quantenphysiker in die richtige Richtung dachten.

    Deswegen, und auch wegen der großen Bedeutung dieser Entdeckung für die Frage, ob der Mensch freien Willen habe, wird (nach meiner Beobachtung) unter klassischer Physik heute die Physik verstanden, welche die Ergebnisse der Quantenphysik komplett ignoriert.

  610. #611 Anonym_2019
    26. Dezember 2019

    @Gebhard Greiter (26. Dezember 2019)

    “Und dass jemand, der Newtons Gravitationstheorie meint (im Gegensatz zur ART) einfach nur von “klassischer Physik” spricht, statt explizit von “Newtons Physik”, ist mir noch gar nicht untergekommen.”

    Es geht dabei nicht nur um Gravitationstheorie, sondern vor allem auch von Newton’s Theorie der Mechanik (im Gegensatz zur SRT).

    Die klassische Mechanik oder Newtonsche Mechanik ist das Teilgebiet der Physik, das die Bewegung von festen, flüssigen oder gasförmigen Körpern unter dem Einfluss von Kräften beschreibt.

    Die physikalischen Theorien wie Relativitätstheorie und Quantenmechanik, mit denen diese Einschränkungen im 20. Jahrhundert überwunden wurden, fußen einerseits auf der klassischen Mechanik, beruhen aber auch wesentlich auf Konzepten, die mit der klassischen Mechanik nicht mehr vereinbar sind.

    Quelle:
    https://de.wikipedia.org/wiki/Klassische_Mechanik

  611. #612 Karl-Heinz
    26. Dezember 2019

    @Gebhard Greiter

    nicht proportional zur Beschleunigung zustande kam

    Ich hoffe du bist nicht sowas wie ein Philosoph. Die würden auch argumentieren, dass ein Altersunterschied dadurch zustande kam, dass jemand auf den Startknopf gedrückt hat, was ja gar nicht so falsch ist. Skurril wird das Ganze, wenn man gezwungen wird damit eine Berechnung durchzuführen. 😉

  612. #613 Gebhard Greiter
    26. Dezember 2019

    @ Karl-Heinz #612:

    Wenn ich mir John Denkers Argumentation in Abschnitt 1.4 auf seiner Seite https://www.av8n.com/physics/twins.htm ansehe, kann ich nicht nachvollziehen, dass Du den dort doch eigentlich glasklar gemachten Zusammenhang dann noch als skurill empfindest.

  613. #614 Jolly
    26. Dezember 2019

    @Gebhard Greiter

    „Note that we do not need to worry about what Gee forces Moe is subjected to during the turnaround. The forces don’t matter.“ (John Denker, im verlinkten Artikel, Abschnitt 1.4)

    Meinten Sie diesen dort glasklar gemachten (nicht vorhandenen) Zusammenhang?

  614. #615 Gebhard Greiter
    26. Dezember 2019

    @Anonym_2019 #611:

    Es liegt mir fern, bestreiten zu wollen, dass die klassische Physik auch nach Newton noch jede Menge Fortschritte gemacht hat. Sie tat das auch noch mit Einstein.

    Ihre zentrale Grundposition aber, die Welt funktioniere auf jeder Skala voll deterministisch, hat sich als falsch erwiesen.

    Und vor allem das (aber nicht nur das) meint heute fast jeder, der von klassischer Physik im Gegensatz zu Quantenphysik spricht.

  615. #616 Gebhard Greiter
    26. Dezember 2019

    @ Jolly #614:

    Ich beziehe mich auf das, was John Denkers Artikel mir klarer gemacht hat, als es mir vorher war.

    Wenn er sich irgendwo unglücklich ausdrückt, nehme ich ihm das nicht übel, denn ich glaube ihm ja nicht blind und sehe auch, dass zahlreiche andere Physiker es ebenfalls schaffen, sich äußerst unglücklich auszudrücken.

  616. #617 Karl-Heinz
    26. Dezember 2019

    @Gebhard Greiter

    Bist du dir sicher, dass du den Artikel von John Denkers richtig verstanden hast. Du weißt ja, dass Lesen und Verstehen zwei unterschiedliche Dinge sind.

  617. #618 Anonym_2019
    26. Dezember 2019

    @Gebhard Greiter (26. Dezember 2019) #615

    “jede Menge Fortschritte gemacht hat. Sie tat das auch noch mit Einstein.”

    Die SRT war keine evolutionäre Weiterentwicklung der Klassischen Mechanik, sondern eine Revolution. Das haben Sie, glaube ich, noch nicht verstanden.

    Die SRT widerspricht teilweise der Klassischen Mechanik.

    Beispiel 1: Die klassische Theorie der Mechanik sagt voraus, dass, wenn ich eine Kraft beliebig lange auf eine Masse einwirken lasse, diese auf Überlichtgeschwindigkeit beschleunigt. Die SRT sagt voraus, dass dies unmöglich ist. Bei derselben Krafteinwirkung sagen also die beiden Theorien eine unterschiedliche Bewegung voraus.

    Beispiel 2: Die klassische Theorie der Mechanik sagt voraus, dass Teresa und Serena gleich alt bleiben.

  618. #619 Karl-Heinz
    26. Dezember 2019

    @Gebhard Greiter

    Ich mache dir einen Vorschlag. Du fast den Artikel von John Denker in wenigen Sätzen zusammen. Anschließend mache ich das. 😉

  619. #620 Anonym_2019
    26. Dezember 2019

    @Gebhard Greiter (26. Dezember 2019) #615

    Ergänzung zu #618

    Einstein hat – im Gegensatz zu Newton – erkannt, dass die Geschwindigkeit eines Objekts nicht nur dessen Raumkoordinate transformiert, sondern auch dessen Zeitkoordinate.

    Einstein hat mit der SRT gezeigt, dass das Fundament der Newton’schen Theorie, das Postulat einer absoluten Zeit, der Physik widerspricht.

    Die Newton’schen Theorie ist demnach nur als Näherungslösung für den Grenzfall kleiner Geschwindigkeiten verwendbar.

  620. #621 Jolly
    26. Dezember 2019

    @Gebhard Greiter (zu #616)

    Chapeau!

    Betreiben Sie Autosuggestion? Der sich hier mit deutlichem Abstand am „unglücklichsten“ ausdrückt, das ist – aus meiner Beobachter-Perspektive – ein Hobby-Physiker.

    Sie hatten noch nicht auf meine Frage in #582 geantwortet, ob Sie den dort von mir geäußerten Interpretationen des Bildes mittlerweile zustimmen.

  621. #622 Gebhard Greiter
    26. Dezember 2019

    @ Jolly #621:

    Antwort auf Ihre Frage aus #582:

    Dass auch Signale aus diesem Zeitraum von Serena empfangen werden, dass von allen Ereignissen daher also auch Photos von ihr gemacht werden können, und wann genau, kann man dem Bild 3 bei Ihnen, linke Seite, entnehmen.

    Stimmen Sie mir mittlerweile soweit zu?

    Ja, dem stimme ich zu, das aber nur deswegen, da ich das zweite Bild auf Seite https://greiterweb.de/welt-verstehen/0-342-Das-Zwillingsparadoxon-im-Lichte-von-SRT-und-ART.htm als Präzisierung des ersten und dritten Bildes dort sehe (welche – allein für sich betrachtet – mir allzu missverständlich erscheinen, da sie so tun, als bestünden notwendige Beschleunigungsphasen aus jeweils nur einem einzigen Ereignis).

    Zu Ihrer nächsten Aussage, die da lautet:

    Auf ihrer Seite schreiben sie weiter, „Sofern Bewegung den Abstand des Beobachters vom beobachteten Objekt verkleinert, hat man als Beobachter den Eindruck, die Zeit des Objekts verginge schneller als die eigene.“

    Wie Sie anhand der eingezeichneten Gleichzeitigkeitslinien im Bild leicht nachrechnen können, ist das ebenfalls falsch.

    Dass das falsch sein soll, verstehe ich nicht, denn meine Aussage sehe ich als über den Dopplereffekt begründet. Sie scheinen mir hier zu übersehen, dass zum Errechnen effektiv beobachteter Zeitdilatation immer auch der Dopplereffekt mit berücksichtigt werden muss, und der i.A. übersteuert, was allein der relativistische Zeitdilatationsfaktor Gamma bewirkt (siehe https://greiterweb.de/spw/Zu-Zeitdilatation-und-Dopplereffekt.htm ).

    Komplett unverständlich ist für mich Ihr letzter Satz:

    … das sind aber keinesfalls Uhren mit denen eine Zeitmessung von Serena durchgeführt werden sollte (sie sind nicht ruhend und nicht synchronisiert).

  622. #623 Gebhard Greiter
    26. Dezember 2019

    @ Anonym_2019 #620:

    Dass die Newton’sche Theorie nur als Näherungslösung für den Grenzfall kleiner Geschwindigkeiten verwendbar ist, sehe ich ebenso.

    Was Newton als Lichtgeschwindigkeit sah, weiß ich nicht.

    |
    Zu #618:

    Es war nicht meine Absicht zu diskutieren, welche Revolution die SRT im Vergleich zu Newtons Theorie war. Es ging mir allein darum, klar zu machen, an was ich denke, wenn jemand heute von klassischer Physik im Vergleich zu Quantenphysik spricht.

    Ihre Aussage

    Die SRT widerspricht teilweise der Klassischen Mechanik.

    würde ich, weniger missverständlich, formulieren als:

    Die SRT widerspricht teilweise der Newton’schen Mechanik.

  623. #624 Jolly
    26. Dezember 2019

    @Gebhard Greiter

    „Komplett unverständlich ist für mich Ihr letzter Satz“

    Martin Bäker sagte ihnen weiter oben ja schon, dass Sie besser noch einmal bei Null anfangen sollten. Ich meine, soweit müssen Sie gar nicht zurück, wir könnten eine Stelle gefunden haben, wo sie aber dringend nachbessern sollten, wenn Sie die Physik der SRT besser verstehen wollen. Ich würde seine Aussage nun also dahingehend präzisieren, informieren Sie sich noch einmal über Gleichzeitigkeit. Wie wird die in einem Inertialsystem definiert, wie verhält es sich dann in zueinander bewegten Inertialsystemen. Zunächst klassisch newtonisch, dann im Vergleich dazu klassisch einsteinisch (also unter den bekannten Voraussetzungen bezüglich der Lichtgeschwindigkeit). Immer mit einem Auge darauf, was Uhren sind und welche (einsteinisch) überhaupt miteinander synchronisiert werden können, so dass sie für Zeitmessungen geeignet wären.

    Da Gleichzeitigkeit nicht das Thema dieses Blogartikels ist, würde ich lieber mit der Kurvendiskussion vom darin enthaltenen Bild weitermachen. Also, Mut zur Lücke. Mal sehen, ob wir nicht auch so noch ein gutes Stück vorankommen.

    „Sie scheinen mir hier zu übersehen, dass zum Errechnen effektiv beobachteter Zeitdilatation immer auch der Dopplereffekt mit berücksichtigt werden muss“

    Muss er dass? Das hängt wohl davon ab, was man unter “effektiv beobachtet“ und unter „Zeitdilatation“ versteht. Wir scheinen uns in beidem nicht einig zu sein.

    „hat man als Beobachter den Eindruck, die Zeit des Objekts verginge schneller als die eigene“

    Woraus bezieht der Beobachter seinen Eindruck, aus den nacheinander bei ihm ankommenden Signalen oder aus Gleichzeitigeistsbetrachtungen?

    Lassen Sie uns zunächst den Eindruck, den ein Beobachter hat, aus Gleichzeitigkeitsbedingungen kalkulieren:

    Sie haben ja bereits zugestimmt, dass die Raumachsen (Gleichzeitigkeitslinien) für den Rückflug korrekt eingezeichnet sind. Ich setze desweiteren voraus, dass Sie auch an den Zeitachsen und deren Skalierung für Teresa und Serena nichts auszusetzen haben. Dann lässt sich der Eindruck den Serena hat, wieviel Zeit für den Rückflug vergeht, anhand ihrer Gleichzeitigkeitslinien (Raumachsen) im Diagramm ablesen. Ein Uhr in ihrer Nähe zeigt beim Beginn des Rückflugs 4 Jahre an (Punkt B). Für Serena gleichzeitig – wir verfolgen die blaue Linie von Punkt B nach schräg oben – zeigt eine Uhr, die immer in Teresas Nähe war, ca. 6,8 Jahre an.

    Sind Sie soweit bei mir?

  624. #625 Anonym_2019
    26. Dezember 2019

    @Gebhard Greiter (26. Dezember 2019) #623

    “Was Newton als Lichtgeschwindigkeit sah, weiß ich nicht.”

    Er akzeptierte die im Jahr 1678 von Ole Rømer & Christiaan Huygens astronomisch ermittelten 212.000 km/s als LG.

  625. #626 Anonym_2019
    27. Dezember 2019

    @Gebhard Greiter (26. Dezember 2019) #623

    Ergänzung zu #625:

    Newton hat die LG in seinem Buch Opticks genauer angegeben als der Astronom Ole Rømer:

    Isaac Newton (1643–1727) also accepted Rømer’s idea; in his 1704 book Opticks giving a value of “seven or eight minutes” for light to travel from the Sun to Earth, closer to the true value (8 minutes 19 seconds) than Rømer’s initial estimate of 11 minutes.

    Quelle:
    https://en.wikipedia.org/wiki/R%C3%B8mer%27s_determination_of_the_speed_of_light

    Newton ging von der Emissionstheorie aus:

    Qu. 29. Are not the Rays of Light very small Bodies emitted from shining Substances?

    Quelle:
    https://scienceblogs.de/astrodicticum-simplex/2017/09/20/newtons-opticks-ein-bahnbrechendes-werk-der-physik/

  626. #627 Karl-Heinz
    27. Dezember 2019

    @Gebhard Greiter

    Sie scheinen mir hier zu übersehen, dass zum Errechnen effektiv beobachteter Zeitdilatation immer auch der Dopplereffekt mit berücksichtigt werden muss.

    1)Ich bitte Sie, führens keine Begriffe wie “effektiv“ ein ohne diesen Begriff zu erläutern.
    2) Sie meinten wohl, dass für den relativistischen Doppler-Effekt auch die Zeitdilatation berücksichtigt werden muss und nicht umgekehrt, oder?!
    3)Erläutern Sie bitte den Begriff “Beschleunigung“. Beschleunigung zu welchem Bezugsystem? Meinen Sie nur die räumliche Beschleunigung im jeweiligen Bezugsystem oder meinen sie doch was ganz anderes?

  627. #628 Karl-Heinz
    27. Dezember 2019

    @Gebhard Greiter

    Die Bilder 1 und 2 werden dadurch niedriger (bei gleicher Breite) und der Winkel zwischen den Raumachsen der beiden Zwillinge könnte so gut auf bis fast 90 Grad wachsen (immer vorausgesetzt, B hält so hohe Beschleunigung aus, was aber gut sein könnte, wenn B keine Person, sondern nur eine Uhr ist).

    Und ich dachte bei fast Lichtgeschwindigkeit sprich 45° sei Schluss. Bist du dir wirklich sicher alles richtig verstanden zu haben? :-);

  628. #629 Karl-Heinz
    27. Dezember 2019

    Richtig ist stattdessen:
    Beschleunigung bestimmt Eigenzeit, Geschwindigkeit bestimmt das Messergebnis (sprich: die Zeit als beobachteten Fluss der Zeit). Was wir Zeit nennen, ist gemessene Zeit, d.h. durch stetige Abstandsänderung und die Endlichkeit der Lichtgeschwindigkeit relativ gewordenes Ergebnis einer Beobachtung des Flusses der Eigenzeit des beobachteten Objekts.

    In meinen Augen ist das vollkommener Unsinn. Obwohl ich selbst Laie auf dem Gebiet der Relativitätstheorie bin, fallen mir die Patzer von Gebhard Greiter auf und die tun einem so was von Weh. Ich habe nicht das geringste gegen Gebhard. Es würde mich wirklich interessieren, warum er sich zutraut als Laie einen fehlerfreien Parkour hinzulegen.

  629. #630 Herr Senf
    27. Dezember 2019

    … warum er sich zutraut als Laie einen fehlerfreien? Parkour hinzulegen.

    … warum er sich zutraut als Laie einen fehlerhaften! Parkour hinzulegen.

  630. #631 Gebhard Greiter
    27. Dezember 2019

    @ Karl-Heinz #629:

    In meinen Augen ist das vollkommener Unsinn.

    Es sollte hier eigentlich nicht darum gehen, was Sie für Unsinn halten, sondern wie Sie mein Argument entkräften wollen, dass Zwillinge nur dann irgendwann (in einem gemeinsam erlebten Ereignis) nicht mehr gleich alt sein können, wenn sie zuvor mindestens zeitweise unterschiedlich beschleunigt waren.

  631. #632 Gebhard Greiter
    27. Dezember 2019

    @ Karl-Heinz #628:

    Und ich dachte bei fast Lichtgeschwindigkeit sprich 45° sei Schluss. Bist du dir wirklich sicher alles richtig verstanden zu haben? :-);

    Schau dir mal auf Seite https://www.av8n.com/physics/twins.htm John Denkers Figure 5 an: Der Winkel zwischen den Raumachsen des roten und des blauen Gitters ist dort größer als 45 Grad.

  632. #633 Jolly
    27. Dezember 2019

    @Gebhard Greiter

    Bei Markus Pössels Blog ‚Relativ Einfach‘ findet sich ein Artikel, nach dessen Lektüre Sie meinen Satz mit den Uhren besser verstehen könnten.

    https://scilogs.spektrum.de/relativ-einfach/gleichzeitigkeit-ist-relativ/

    Lassen Sie sich nicht von den über 2000 Kommentaren abschrecken, dort findet man viele haarsträubende Fehler und Fehlinterpretationen. Der eigentliche Artikel ist verständlich geschrieben und das darin Beschriebene sollte unstrittig sein.

    @Karl-Heinz, @Herr Senf

    Machen wir alle nicht gelegentlich Fehler?

    Fehlerfrei: einen Parcours hinzulegen
    Fehlerhaft: einen Parkour hinzulegen

    Parkour ist eine Sportart.

  633. #634 Karl-Heinz
    27. Dezember 2019

    @Gebhard Greiter

    Da steht aber bei dir “Raumachsen der beiden Zwillinge könnte so gut auf bis fast 90 Grad wachsen“. Rot und blau gehören aber zu jenem Zwilling der unterwegs ist. Also zu dem Zwilling der wegfliegt umkehrt (Richtungsumkehr) und den gleichen Weg zurückfliegt.

  634. #635 Karl-Heinz
    27. Dezember 2019

    @Jolly

    Sorry, ich meinte natürlich, wie du richtig festgestellt hast, Parcours. Im Gegensatz zu Gebhard Greiter stehe ich zu meinen Fehlern. 🙂

  635. #636 rolak
    27. Dezember 2019

    Parkour ist eine Sportart

    Deswegen fand ich es eigentlich ziemlich passend, Jolly, gehts doch dabei darum, auftauchende Hindernisse geschickt zu umgehen. Der Haken ist halt, daß dies beim Sport unter ‘elegant’ fällt, in der Wissenschaft jedoch unter ‘unangemessen’.

  636. #637 Karl-Heinz
    27. Dezember 2019

    @rolak
    Sehr gute Analyse. 😉

    @Gebhard Greiter
    Wie ich bemerkt habe, stellst du sehr gerne fremde Beiträge auf deine Homepage. Es wäre nicht schlecht wenn du ab und zu diese auch lesen würdest wie die unten stehende. 🙂

    https://www.manus-zeitforum.de/file/1/file=477/filename=Trillingsparadoxon.jpg
    Diese Antwort wiederum hat dazu geführt, dass oftmals die Beschleunigungsphasen von B für die Verjüngung verantwortlich gemacht werden. Aber auch das ist nicht richtig, wie sich leicht zeigen lässt, indem man aus dem Zwillingsparadoxon ein Drillingsparadoxon macht. In der nachstehenden Skizze sind die Weltlinien der Drillinge A, B und C dargestellt, wobei C identische Beschleunigungsphasen (gekrümmte Kurvenstücke, durchgezogen gezeichnet) durchläuft, allerdings ist seine Reise kürzer. Nach Zusammenkunft aller drei ist A älter als C und C älter als B. Die Beschleunigungsphasen können also nicht für den Altersunterschied verantwortlich gemacht werden. Es ist die physikalische Geometrie der relativistischen Raumzeit: Der Weg von B ist kürzer als der von C, und dieser wiederum ist kürzer als der Weg von A.

  637. #638 Jolly
    27. Dezember 2019

    @Gebhard Greiter

    Es ist völlig unstrittig, dass in der Realität durch einen der Zwillinge ein Ruck gehen muss, dieser daraufhin beschleunigt und somit Geschwindigkeit aufnimmt, und er anschließend erst nach mehreren weiteren Rucks wieder auf seinen anderen Zwilling treffen kann.

    Die Frage ist doch, warum altert der eine mehr, der andere weniger in der Zwischenzeit, genauer: den Zwischenzeiten? Der (Ur-) Grund, so könnte man auch sicher noch einvernehmlich sagen, liegt in der Struktur der Raumzeit verborgen. Eigentlich auch wieder nicht verborgen, wir bemerken das ja und können den Unterschied sogar exakt berechnen. Müssen wir zur genauen Berechnung der Zeitdilatation die Rucks, die Beschleunigungen oder die Geschwindigkeiten kennen?

    @Karl-Heinz

    Gebhard Greiter steht ebenfalls zu seinen Fehlern. Er hat schon einiges auf seiner Homepage korrigiert.

    Dass es etwas dauern kann, bis man seine eigenen Fehler erkennt, und damit andere leicht nerven kann, das hast ja auch Du gerade wieder erst beim Adventsrätsel anschaulich demonstriert.

  638. #639 Karl-Heinz
    27. Dezember 2019

    @Jolly

    Ich habe mich aber anschließend in Demut geübt und bei der Interpretation der Hinweise nicht teilgenommen. Das Lösungswort ist doch Petit-Prince (Mond), oder?

  639. #640 Karl-Heinz
    28. Dezember 2019

    @Gebhard Greiter

    Will natürlich deine Bemühungen nicht schmälern und auch nicht als Spielverderber dastehen. Ich hoffe, dass der Funke überspringt und es zu einem Aha-Erlebnis kommt.
    https://de.m.wikipedia.org/wiki/Aha-Erlebnis

  640. #641 Gebhard Greiter
    28. Dezember 2019

    @Jolly #638:

    Müssen wir zur genauen Berechnung der Zeitdilatation die Rucks, die Beschleunigungen oder die Geschwindigkeiten kennen?

    Rein rechnerisch betrachtet reicht es, die Geschwindigkeit (in aller Genauigkeit) zu kennen.

    Das ist aber nur deswegen so, weil sich, mathematisch gesehen, eines aus dem anderen ergibt – natürlich nur bis auf eine Integrationskonstante für den, der von Beschleunigung ausgeht, aber relevant ist ja stets nur Relativgeschwindigkeit (absolute Geschwindigkeit gibt es ja gar nicht).

    Daher bin ich der Meinung, wer sagt “Nicht Beschleunigung macht Eigenzeit “, der sagt damit implizit auch “Nicht Geschwindigkeit macht Eigenzeit “.

    Wirklich ursächlich ist, dass die Zwillinge sich entscheiden, getrennte – genauer: hinreichend unterschiedliche – Wege durch die Raumzeit zu nehmen. Dies zu tun müssen sie für unterschiedliche Beschleunigung sorgen.

    Warum also soll es dann falsch sein, zu sagen erst unterschiedliche Beschleunigung führe zu unterschiedlicher Eigenzeit?

  641. #642 Karl-Heinz
    28. Dezember 2019

    @Gebhard Greiter

    Weil es den Begriff (relative) Momentangeschwindigkeit gibt. Da steckt auch die Beschleunigung, Ruck usw. drin.
    Es ist wichtig zu verstehen, dass die Uhren auch dann unterschiedlich gehen, wenn nicht beschleunigt wird.

  642. #643 Karl-Heinz
    28. Dezember 2019

    @Gebhard Greiter

    Wer hat den jemals behauptet, dass bis auf ein ausgezeichnetes Bezugsystem die Geschwindigkeit der zu untersuchenden Objekte konstant sein muss. Ich liebe Geschwindigkeit. 🙂

  643. #644 Gebhard Greiter
    28. Dezember 2019

    @ Karl-Heinz #642:

    Dass nicht beschleunigte Uhren gleicher Bauart unterschiedlich schnell, gehen ist falsch.

    Sie werden nur – aus unterschiedlichen Koordinatensystemen heraus – als unterschiedlich schnell gehend wahrgenommen.

  644. #645 Gebhard Greiter
    28. Dezember 2019

    @ Karl-Heinz #643:

    Du versuchst mir eine Aussage zu unterstellen, die ich nie gemacht habe.

  645. #646 Anonym_2019
    28. Dezember 2019

    @Gebhard Greiter (28. Dezember 2019)

    “Sie werden nur – aus unterschiedlichen Koordinatensystemen heraus – als unterschiedlich schnell gehend wahrgenommen.”

    Das ist falsch. Die Zeitdilatation gibt es auch, wenn kein Beobachter vorhanden ist, der etwas wahrnehmen könnte. Das kann man mit der Lorentz-Transformation berechnen. Die Lichtuhr geht in dem Bezugssystem, in dem sie bewegt ist, langsamer als in ihrem Ruhesystem, weil das Licht mit der Geschwindigkeit c einen schrägen und damit nach Pythagoras verlängerten Weg zurücklegt.

  646. #647 Gebhard Greiter
    28. Dezember 2019

    @ Anonym_2019 #646:

    Zeitdilatation ist relativ:

    Bei gleichförmiger Relativgeschwindigkeit der beiden Uhren wird jede der jeweils anderen gegenüber verzögert erscheinen. Das bedeutet aber nicht, dass man sagen kann, jede der beiden ginge langsamer als die jeweils andere.

    Es dennoch zu sagen hätte nicht nur eine in sich widersprüchliche Aussage zur Folge, sondern wäre auch absolut unehrlich, da man dann ja verschweigt, dass es sich um zwei unterschiedliche Zeitbegriffe handelt.

    Du solltest wirklich mal das Buch von Kip S. Thorne lesen (statt immer nur schlecht durchdachte Formulierungen aus Sachbüchern zu nutzen, deren Autoren nicht selten auch noch selbst wörtlich nehmen, was sie da schreiben).

    Als Physiker sollte man solch vereinfachte Sprechweise nur nutzen, wo man sicher sein kann, dass der Gesprächspartner genau weiß, was damit gemeint ist (sie also nicht allzu wörtlich nimmt – wie jemand es tun muss, der noch nie was von Relativität im Sinne Einsteins gehört hat).

  647. #648 Anonym_2019
    28. Dezember 2019

    @Gebhard Greiter (28. Dezember 2019) #647

    “Das bedeutet aber nicht, dass man sagen kann, jede der beiden ginge langsamer als die jeweils andere.”

    Das habe ich in Kommentar #646 nicht gesagt.

    “Es dennoch zu sagen”

    Das ist die Strohmann-Methode.

    “da man dann ja verschweigt, dass es sich um zwei unterschiedliche Zeitbegriffe handelt.”

    Was habe ich in Kommentar #646 verschwiegen? Ich habe die jeweiligen Bezugssysteme genannt.

    Wie definieren Sie “Zeitbegriff”?

    “statt immer nur schlecht durchdachte Formulierungen aus Sachbüchern zu nutzen

    Sorry – Im Gegensatz zu Ihnen schreibe ich nicht ab, sondern denke selbst nach.

    “sie also nicht allzu wörtlich nimmt”

    Ich möchte, dass meine Äußerungen in Kommentar #646 wörtlich genommen werden.

  648. #649 Gebhard Greiter
    28. Dezember 2019

    @ Anonym_2019:

    Sehen Sie: Auch ich denke nach und werde mich deswegen – ganz bewusst – keinem Sprachgebrauch anschließen, der zu inkonsistenter Aussage führt.

  649. #650 Anonym_2019
    28. Dezember 2019

    @Gebhard Greiter (28. Dezember 2019) #649

    “werde mich deswegen – ganz bewusst – keinem Sprachgebrauch anschließen, der zu inkonsistenter Aussage führt.”

    Sie behaupten damit indirekt, dass mein Sprachgebrauch zu inkonsistenten Aussagen führt. Was ich vermisse ist eine Begründung mit Bezug auf meine Aussagen in Kommentar #646 (incl. Bezugssystem-Nennungen), nicht mit Bezug auf mir falsch unterstellte Strohmann-Aussagen.

  650. #651 Gebhard Greiter
    28. Dezember 2019

    @ Anonym_2019:

    Ich will nicht nur andeuten, sondern behaupte ganz explizit, dass Ihr Sprachgebrauch zu inhaltlich inkonsistenter Aussage führt:

    Zur Aussage nämlich “Jede der beiden Uhren geht langsamer als die andere”.

  651. #652 Karl-Heinz
    28. Dezember 2019

    Die Entscheidung 🙂
    Gegeben sei ein wirklich sehr großer Kreis mit dem Radius R auf dem sich Gebhard mit konstanter Geschwindigkeit bewegt. Ich komme mit der konstanten Geschwindigkeit v von unten nach oben, wobei ich den Mittelpunkt des Kreises durchquere. Wie es der Teufel so haben will treffen wir uns ganz unten am gleichem Ort, synchronisieren unsere Uhren und auch zum Schrecken von Gebhard ganz oben ebenfalls am gleichem Ort, wo wir unsere Uhren vergleichen.
    Frage: wie groß ist der Gangunterschied, wenn der Radius R und meine Geschwindigkeit v gegeben sind.
    Lieber, lieber Gebhard zeig uns mal wie du diese Aufgabe löst. Bitte verstecke dich jetzt nicht hinter einem philosophischen Argument.

  652. #653 Anonym_2019
    28. Dezember 2019

    @Gebhard Greiter (28. Dezember 2019) #651

    Ihr Zitat: “Jede der beiden Uhren geht langsamer als die andere”.

    Meine Aussage im #646: “Die Lichtuhr geht in dem Bezugssystem, in dem sie bewegt ist, langsamer als in ihrem Ruhesystem …”

    Wen haben Sie zitiert und warum gehen nicht auf meine Begründung in #646 ein?

  653. #654 Jolly
    28. Dezember 2019

    @Gebhard Greiter (#641)

    „Warum also soll es dann falsch sein, zu sagen erst unterschiedliche Beschleunigung führe zu unterschiedlicher Eigenzeit?“

    Vielleicht ist das ja gar nicht so falsch.

    Zunächst würde ich mir allerdings Gedanken machen, ob der Begriff Eigenzeit in der Frage passt. Eigenzeit, so liest man ja gelegentlich, wäre invariant, also gerade nicht unterschiedlich von Bezugssystem zu Bezugssystem. Wie wäre es z.B. mit „unterschiedlicher Dauer, gemessen in jeweiliger Eigenzeit“?

    Nachdem ich die Diskussionen rund um die Uhren verfolgt habe, insbesondere den Streit, ob bewegte Uhren nun langsamer laufen oder nicht – die, wie ich gerade mit Erschrecken bemerke, auch hier wieder fortgeführt werden – komme ich zu dem Schluß, es kommt darauf an, was man damit meint. Bei den für mich interessanten Diskussionen – es gibt andere – waren sich die Beteiligten über die dahinterliegende Physik einig. Ich sage nun also: Beides ist richtig! Also scheinbar Gegensätzliches ist richtig. Auch dieses Paradoxon lässt sich auflösen, wenn auch auf andere Art als das hier diskutierte Zwillingsparadoxon.

    Bei ihrer Aussage kommt es wohl ebenfalls darauf an, was Sie damit meinen – und was andere meinen könnten, was Sie meinen. Problematisch wird die Aussage dann, wenn damit Assoziationen verknüpft werden, die zu falschen Vorstellungen führen, zu falscher Physik.

    Ein naheliegender Fehler ist es, zu schließen, die Zeitdilatation (der sich ergebende Altersunterschied der Zwillinge) würde ausschließlich oder wesentlich in den Phasen der Beschleunigung bestimmt.

    Eine weitere irrige Deduktion wäre: „wer sagt “Nicht Beschleunigung macht Eigenzeit “, der sagt damit implizit auch “Nicht Geschwindigkeit macht Eigenzeit “.“

    Falls Sie sich dabei auf Joachim Schulz beziehen, der sagt, „Nicht die Beschleunigung macht die Zeit“, so würde ich in der von ihm gewählten Überschrift Zeit nicht als Eigenzeit interpretieren. Ich würde das eher paraphrasieren mit, „Nicht die Beschleunigung macht die Zeitdilatation“, oder ausführlicher, „Die Beschleunigung geht nicht direkt in die Berechnung der Zeitdilatation ein“. Es wäre offensichtlich falsch, zu folgern, er würde damit auch sagen, „Die Geschwindigkeit geht nicht direkt in die Berechnung der Zeitdilatation ein“.

    ________

    Betrachten wir einmal ganz analog Weglängen. Ja, ohne Geschwindigkeit wird die Wegeslänge wohl immer bei 0 bleiben. Wie könnte eine Strecke zurückgelegt werden, wie ein Weg entstehen, ohne Geschwindigkeit? Ohne Geschwindigkeit, also ohne Beschleunigung, wenn kein Ruck durch einen der Zwillinge gegangen ist, er möglicherweise den Knall nicht gehört hat, bleibt die Differenz der Zwillinge stets bei 0.

    Bestehen Sie auf die Aussage (halten Sie die für geeignet, Physik verständlich zu machen): Erst unterschiedliche Geschwindigkeit führt zu unterschiedlicher Weglänge.

    Ganz falsch erscheint mir die Aussage zwar nicht, aber …

    Würden Sie jemanden, der behauptet, “Nicht die Geschwindigkeit macht die Länge“, unterstellen, er hätte zu kurz, nicht bis ans Ende gedacht?

  654. #655 Karl-Heinz
    28. Dezember 2019

    Wer löst Beispiel #652? 🙂

  655. #656 Anonym_2019
    28. Dezember 2019

    @Gebhard Greiter (28. Dezember 2019) #651

    Korrektur zu “Anonym_2019 28. Dezember 2019 #653”

    Ich hatte dis Aussage in Anführungszeichen am Schluss fälschlicherweise als Zitat verstanden.

    “Ich will nicht nur andeuten, sondern behaupte ganz explizit, dass Ihr Sprachgebrauch zu inhaltlich inkonsistenter Aussage führt: Zur Aussage nämlich “Jede der beiden Uhren geht langsamer als die andere”.”

    Mein Sprachgebrauch führt aus meiner Sicht zu:

    Wenn sich zwei Uhren relativ zueinander geradlinig-gleichförmig bewegen, dann geht jede der Uhren im Ruhesystem der jeweils anderen Uhr langsamer als in ihrem eigenen Ruhesystem.

    Warum ist das widerspüchlich?

  656. #657 Gebhard Greiter
    28. Dezember 2019

    @ Anonym_2019 #646:

    Sie schrieben:

    Die Lichtuhr geht in dem Bezugssystem, in dem sie bewegt ist, langsamer als in ihrem Ruhesystem, weil das Licht mit der Geschwindigkeit c einen schrägen und damit nach Pythagoras verlängerten Weg zurücklegt.

    Dieser Aussage stimme ich zu.

    Warum aber behaupten Sie dann direkt davor, meine Aussage “Sie werden nur – aus unterschiedlichen Koordinatensystemen heraus – als unterschiedlich schnell gehend wahrgenommen.” sei falsch?

    In einem Bezugssystem zu gehen, in dem die Uhr bewegt ist, bedeutet doch nichts anderes, als dass man sie aus jenem Bezugssystem heraus beobachtet (und ihre Ganggeschwindigkeit an dem durch dieses Bezugssystem definierten Zeitbegriff misst).

    |
    Mein Vorschlag:

    Warum einigen wir uns nicht darauf, zu sagen,

    Meine Zeit ist, was ich von meiner Uhr ablese (= was sie mir anzeigt),
    Deine Zeit ist, was Du von deiner Uhr abliest (= was deine Uhr dir anzeigt), und ob diese beiden Zeitbegriffe derselbe sind, ergibt sich aus folgender Regel:

    Jede Uhr U hat ihren eigenen Zeitbegriff Z(U).

    Sind U1 und U2 zwei Uhren identischer Bauart, so wird Z(U1) = Z(U2) genau dann gelten, wenn sie zu einander höchstens gleichförmig bewegt sind.

    Das kann nur der Fall sein, solange sie nicht unterschiedlich beschleunigt sind.

    Als Uhr eines Objekts X gilt jede Uhr, die sich relativ zu X nicht bewegt, Du und ich sind Objekte in diesem Sinne.

  657. #658 Karl-Heinz
    28. Dezember 2019

    @Anonym_2019

    Warum ist das widerspüchlich?

    Das erinnert mich an zwei Scheinwerfer die sich gegenüberstehen und beide etwas gegen oben geneigt sind. Jeder sieht vom anderen nur die Projektion der Leuchtfläche des anderen. Ob das für Gebhard Greiter ein Widerspruch ist? 😉

  658. #659 Karl-Heinz
    28. Dezember 2019

    @Gebhard Greiter

    Meine Zeit ist, was ich von meiner Uhr ablese (= was sie mir anzeigt),
    Deine Zeit ist, was Du von deiner Uhr abliest (= was deine Uhr dir anzeigt), und ob diese beiden Zeitbegriffe derselbe sind, ergibt sich aus folgender Regel:

    Jede Uhr U hat ihren eigenen Zeitbegriff Z(U).

    Sind U1 und U2 zwei Uhren identischer Bauart, so wird Z(U1) = Z(U2) genau dann gelten, wenn sie zu einander höchstens gleichförmig bewegt sind.

    Das kann nur der Fall sein, solange sie nicht unterschiedlich beschleunigt sind.

    Totaler Quatsch der nichts aussagt.
    In einem Inertialsystem befinden sich zwei Weltlinien die vollkommene gleich sind, bis auf die Tatsache, dass sie um die t-Achse des Inertialsystem gespiegelt sind. Ups, beide haben bezüglich t die gleiche Eigenzeit τ

  659. #660 Anonym_2019
    28. Dezember 2019

    @Gebhard Greiter (28. Dezember 2019) #657

    “Warum aber behaupten Sie dann direkt davor, meine Aussage “Sie werden nur – aus unterschiedlichen Koordinatensystemen heraus – als unterschiedlich schnell gehend wahrgenommen.” sei falsch?”

    Grund: Die Zeitdilatation gibt es auch, wenn kein Beobachter vorhanden ist, der etwas wahrnehmen könnte. Das kann man mit der Lorentz-Transformation berechnen.

    “Meine Zeit ist, was ich von meiner Uhr ablese (= was sie mir anzeigt)”

    Warum so kompliziert? Der Text in der Klammer sollte wegen des “=”-Zeichens ausreichen.

    “Jede Uhr U hat ihren eigenen Zeitbegriff Z(U).”

    Wie definieren Sie das Wort “Zeitbegriff”? Bei Wikipedia kann ich keine Definition finden.

    “Sind U1 und U2 zwei Uhren identischer Bauart, so wird Z(U1) = Z(U2) genau dann gelten, wenn sie zu einander höchstens gleichförmig bewegt sind.”

    Falls Sie mit “Zeitbegriff” die angezeigte Uhrzeit meinen und Sie nicht verschiedene Bezugssysteme vermischen, stimmt das wegen der Zeitdilatation nicht.

    “Als Uhr eines Objekts X gilt jede Uhr, die sich relativ zu X nicht bewegt, Du und ich sind Objekte in diesem Sinne.”

    O.K. Aber was ist mit Ihrem Argument in #567 mit der verzögerten Information über die angezeigte Uhrzeit wegen der endlichen Lichtlaufzeit?

  660. #661 Karl-Heinz
    28. Dezember 2019

    @Gebhard Greiter

    Man bekommt nur dann ein Gefühl für die RT, wenn man ein paar Beispiele eigenständig rechnet. Ansonsten wird man zum Elefanten, so wie du. 😉

  661. #662 Gebhard Greiter
    28. Dezember 2019

    @ Anonym_2019 #656:

    Mit Sicherheit widersprüchlich – und zudem noch missverstanden von jedem, der ihr ohne genaue Kenntnis der SRT begegnet –, ist die oft gehörte Aussage “Bewegte Uhren gehen langsamer.”

    Ihre Aussage

    Wenn sich zwei Uhren relativ zueinander geradlinig-gleichförmig bewegen, dann geht jede der Uhren im Ruhesystem der jeweils anderen Uhr langsamer als in ihrem eigenen Ruhesystem.

    ist nicht widerspüchlich, wenn man die Wendung “in Ruhesystem R gehen” interpretiert als “aus Koordinatensystem R heraus betrachtet”.

    Es geht die Uhr ja schließlich ganz unabhängig davon, aus welchen Koordinatensystemen heraus man sie betrachtet: Wirklichkeit ist eindeutig.

    Relativ, kann – möglicher unterschied­licher Betrachtungssperspektiven wegen – immer nur ein Betrachtungsergebnis sein.

  662. #663 Jolly
    28. Dezember 2019

    @Gebhard Greiter

    „Sind U1 und U2 zwei Uhren identischer Bauart, so wird Z(U1) = Z(U2) genau dann gelten, wenn sie zu einander höchstens gleichförmig bewegt sind.“

    Das entspricht nicht den Aussagen der SRT. U1 und U2 müssen zueinander ruhen, damit sie die „gleiche Zeit“ anzeigen. Schon gleichförmige Bewegung führt zu „anderer Zeit“.

    Ich vermute, Sie sind noch in dem Glauben verfangen, dass „Zeit“ bei gleichförmiger Bewegung nur anders wahrgenommen wird, u.a wegen des Dopplereffekts. Dass das nichts stimmt, kann man dem hier diskutierten Diagramm sehr schön entnehmen. Z(Serenas Uhr) ist nicht Z(Teresas Uhr).

  663. #664 Anonym_2019
    28. Dezember 2019

    @Gebhard Greiter (28. Dezember 2019) #662

    “Es geht die Uhr ja schließlich ganz unabhängig davon, aus welchen Koordinatensystemen heraus man sie betrachtet: Wirklichkeit ist eindeutig.”

    Was genau bedeutet “Es geht die Uhr”? Die Gangrate dτ/dt der Uhr jedenfalls hängt von ihrer Geschwindigkeit im Bezugssystem ab.

    “Wirklichkeit ist eindeutig”

    Relative Größen, wie z.B. Zeitintervalle, sind nicht weniger “wirklich” als invariante Größen.

  664. #665 MartinB
    28. Dezember 2019

    @Karl-Heinz #658
    Ich vergleiche das ja gern mit der Perspektive: Von zwei entfernten Beobachterinnen sieht jede die andere verkleinert.

    @gebhard
    So ziemlich jeder Versuch, physikalische Aussagen rein sprachlich zu fassen, ist immer unscharf – deswegen verwenden wir ja mathematikl als Sprache.

    Was den kern hier angeht – die Frage, ob es die Beschleunigungen sind, die die Dilatation beim ZP verursachen – wiederhole ich nochmal meine Frage von vor einiger Zeit: Ist es der Knick in der Strecke Braunschweig-Berlin-München, der die größere länge dieser Strecke gegenüber dem geraden Weg verursacht?

    Ich kann nur noch ein weiteres mal sagen, dass die Situation im ZP so genau analog zur Dreiecksungleichung in der Geometrie ist, dass man am besten fährt, wenn man jede Frage, die man sich zum ZP stellt, erstmal in der ebenen Geometrie für die entsprechende Situation beantwortet.

  665. #666 Anonym_2019
    28. Dezember 2019

    @Gebhard Greiter (28. Dezember 2019) #662

    In der SRT-Literatur wird häufig “Aus Sicht von Beobachter A” geschrieben, wenn “Im Bezugssystem A” gemeint ist.

    Wenn ich “Im Bezugssystem A” schreibe, dann meine ich z.B., dass die Gangrate der bewegten Uhr anhand der Koordinatenzeit des Bezugssystems A bewertet wird.

    Die unglückliche Formulierung “Aus Sicht von Beobachter A” in der SRT-Literatur hat bei Ihnen vielleicht den falschen Eindruck erweckt, dass die Gangrate der bewegten Uhr “aus Sicht von Beobachter A” auch vom longitudinalen Dopplereffekt beeinflusst würde.

  666. #667 Karl-Heinz
    28. Dezember 2019

    Beispiel #652 habe ich mit der Methode Schütteln aus dem Ärmel durchgerechnet. Habe keine Beschleunigung für die Berechnung benötigt. Ergebnis ist unabhängig vom Radius. Hätte ich auf gebhard gehört müsste ich schon beim Ansatz verzweifeln. 😉

  667. #668 Gebhard Greiter
    28. Dezember 2019

    @ Jolly #654:

    Würden Sie jemanden, der behauptet, “Nicht die Geschwindigkeit macht die Länge“, unterstellen, er hätte zu kurz, nicht bis ans Ende gedacht?

    Ich würde nicht wissen, was er sich dabei gedacht hat. Ich selbst würde schreiben:

    Die beobachtete Länge bewegter Objekte ergibt sich in Abhängigkeit der Geschwindigkeit des Objekts relativ zum Beobachter.

    Martin Carrier erklärt Längenkontraktion wie folgt:

    Jede Längenmessung erfordert, dass Anfang und Ende eines Objekts gleichzeitig markiert werden. Markiert man aber Anfang und Ende eines bewegten Objektes zu unterschiedlichen Zeiten, dann ist es nicht erstaunlich, dass sich abweichende Werte ergeben. Wegen der Relativität der Gleichzeitigkeit unter­scheiden sich die Urteile beider Beobachter über die Gleichzeitigkeit der Markierung der beiden Enden — mit der Folge unterschiedlicher Messergebnisse.

    Deutlich wird, dass die relativistische Längenkontraktion eine Folge des Verfahrens der Längenmessung ist.

    |
    Nun weiter zu Ihren Bemerkungen:

    Zunächst würde ich mir allerdings Gedanken machen, ob der Begriff Eigenzeit in der Frage passt. Eigenzeit, so liest man ja gelegentlich, wäre invariant, also gerade nicht unterschiedlich von Bezugssystem zu Bezugssystem.
    Wie wäre es z.B. mit „unterschiedlicher Dauer, gemessen in jeweiliger Eigenzeit“?

    Da gebe ich Ihnen recht. Und tatsächlich spreche ich auf meiner Seite ja explizit vom “Verbrauch an Eigenzeit während der Reise”.

    Das Wort Eigenzeit steht für “Zeitverbrauch eines Objekts gemessen in einem Koordinatensystem, in dem das Objekt ruht”. Sie kann, so definiert, gar nicht relativ sein.

    Ein naheliegender Fehler ist es, zu schließen, die Zeitdilatation (der sich ergebende Altersunterschied der Zwillinge) würde ausschließlich oder wesentlich in den Phasen der Beschleunigung bestimmt.

    Das sehe ich ebenso, denn Tatsache ist:

    Der (am Ende bleibende) Altersunterschied wird deutlich beeinflusst durch die Relativgeschwindigkeit der Zwillinge. Die Stärke des Effekts aber steht und fällt korrespondierend zur jeweils gegebenen Größe des Winkels zwischen der stets an gleicher Stelle liegenden Raumachse von Teresa einerseits und der infolge von Beschleunigungsstärke mehr oder weniger stark gedrehten Raumachse des Koordinatengitters von Serena. [John Denkers Zeichnung, in der die Koordinatengitter beschleunigungsfreier Phasen in unterschiedlicher Farbe dargestellt sind, zeigt das recht schön. War mir vorher in dieser Klarheit nicht bewusst.]

    Eine weitere irrige Deduktion wäre: „wer sagt “Nicht Beschleunigung macht Eigenzeit “, der sagt damit implizit auch “Nicht Geschwindigkeit macht Eigenzeit “.“

    Falls Sie sich dabei auf Joachim Schulz beziehen, der sagt, „Nicht die Beschleunigung macht die Zeit“, so würde ich in der von ihm gewählten Überschrift Zeit nicht als Eigenzeit interpretieren.
    Ich würde das eher paraphrasieren mit, „Nicht die Beschleunigung macht die Zeitdilatation“, oder ausführlicher, „Die Beschleunigung geht nicht direkt in die Berechnung der Zeitdilatation ein“.
    Es wäre offensichtlich falsch, zu folgern, er würde damit auch sagen, „Die Geschwindigkeit geht nicht direkt in die Berechnung der Zeitdilatation ein“.

    Ja, so sehe ich das unterm Strich ja auch.

    Optimale Formulierung wäre wohl:

    Die Beschleunigung geht nicht direkt in die Berechnung der Zeitdilatation ein.

    Sie bestimmt jedoch die Krümmung der Weltlinie des reisenden Zwillings und damit seinen Verbrauch an Eigenzeit bis zur Rückkehr.

    Unter Joachims Rechnung auf Seite https://www.relativitätsprinzip.info/gedankenexperiment/zwillingsparadoxon.html findet sich inzwischen auch eine korrekte Bemerkung. Er sagt dort:

    Der reisende Zwilling ist der, der die Beschleunigungen durchführt. Dadurch ist er stets weniger gealtert als sein zuhause gebliebener Zwilling.

    Ich war zunächst mal nicht so recht davon überzeugt, dass man das Zwillingsparadoxon beliebig genau auch nur per SRT durchrechnen könne.
    [ Unter welcher Nebenbedingung das wirklich klappt, habe ich mir erst später selbst zurechtgelegt: https://greiterweb.de/welt-verstehen/0-469-Objekt-vs-Modell–in-der-Physik.htm ]

    Habe deswegen versucht, das mit Joachim Schulz (anhand seines Beispiels) zu diskutieren.

    Leider ging er auf meine Argumente nie ein. Es kam immer nur ein Hinweis auf seine Rechnung, so dass ein wirkliches Argumentieren zwischen uns eigentlich gar nicht stattfand.

    Die letzte Mail, auf die er mir noch geantwortet hat, war ähnlich kurz wie alle vorher.
    Sie bestand nur aus folgendem Hinweis, mit dem ich aber bis heute nicht wirklich was anzufangen weiß. Er schrieb mir,

    Ihr Fehler liegt in der Annahme, aus relativen Geschwindigkeiten könne keine vom Beobachter unabhängige Konsequenz entstehen.
    Hier gibt es eine zugrunde liegend Symmetrie, die dafür sorgt, dass die vergangene Eigenzeit eines bewegten Objektes zwischen zwei Ereignissen unabhängig vom Bezugssystem ist.

    Erst Monate später las ich Claus Kiefers Buch und fand dessen Erklärung für die Ursache des entstehenden Altersunterschiedes verständlich und überaus einleuchtend.

    Daher also meine Gegenüberstellung der Aussage beider.

    Schulz bekam von mir einen Link auf diese Notiz – der Fairness wegen, aber auch in der Hoffnung, er würde mir dann vielleicht noch was dazu sagen. Es kam nichts, und so vermute ich, dass er mich für irgend einen Crackpot hält. Sei’s drum …

    Natürlich bin ich davon überzeugt, dass Joachim richtig denkt, es war und ist halt nur so, dass sein Spruch mir unakzeptabel erscheint bzw. nicht verstehbar.

    Wo Noethers Theorem (auf das sich der Hinweis in seiner letzten Mail an mich wohl bezieht) im Originaltext zu finden ist, weiß ich nicht.
    Sollte er im Internet stehen (frei zugänglich) wäre ich dankbar für den Link nach dort.

    |
    Nun noch zur Schwierigkeit, sich in einer Diskussion zu verstehen. Sie schreiben:

    Bei ihrer Aussage kommt es wohl ebenfalls darauf an, was Sie damit meinen – und was andere meinen könnten, was Sie meinen.
    Problematisch wird die Aussage dann, wenn damit Assoziationen verknüpft werden, die zu falschen Vorstellungen führen, zu falscher Physik.

    Das ist ganz sicher richtig, und die Gefahr besteht durchaus, wenn man (wie ich) selbst nie Physik studiert hat.

    Aber gerade weil mir die Gefahr bewusst ist, kommt mir eine Diskussion wie diese hier gerade recht, anderen zu sagen, was ich denke.
    Dabei versuche ich stereotype sprachliche Wendungen zu vermeiden, die – wenn man sie wörtlich nimmt – nicht aussagen, was sie aussagen sollen:

    Es hat ja schließlich jede Nachricht zwei Bedeutungen: eine, die ihr der Empfänger zuordnet, und eine, die der Absender dachte ihr mitgegeben zu haben.

    Sie sollten gleich sein, sind es aber keineswegs immer.

  668. #669 Karl-Heinz
    28. Dezember 2019

    Wenn man mit Minkowski-Diagrammen arbeit gibt’s einige Stolpersteine. Eine davon ist die unterschiedliche Skalierung von x auf x’ sowie t auf t’. 😉

    https://de.wikipedia.org/wiki/Minkowski-Diagramm#/media/Datei:MinkScale.svg

  669. #670 Karl-Heinz
    28. Dezember 2019

    Ihr Fehler liegt in der Annahme, aus relativen Geschwindigkeiten könne keine vom Beobachter unabhängige Konsequenz entstehen.
    Hier gibt es eine zugrunde liegend Symmetrie, die dafür sorgt, dass die vergangene Eigenzeit eines bewegten Objektes zwischen zwei Ereignissen unabhängig vom Bezugssystem ist.

    Beobachter A misst die Länge der Weltlinie von Susi zwischen Ereignis E1 und E2 in seinem Bezugsystem. Beobachter B misst die Länge der Weltlinie von Susi zwischen Ereignis E1 und E2 in seinem Bezugsystem.
    Oh Wunder, die Länger der Weltlinie von Beobachter A entspricht die Länge der Weltline von Beobachter B. Es sieht so aus als sei die Länge der Weltlinie invariant gegenüber dem Bezugsystem.
    Die Länge der Weltlinie, die auf Grund der geometrischen Struktur der Raumzeit koordinatenfrei definiert ist, ist gleich der Eigenzeit eines hypothetischen Beobachters, der sich entlang dieser Weltlinie bewegt.

  670. #671 Anonym_2019
    29. Dezember 2019

    @Karl-Heinz (28. Dezember 2019) #669

    Wenn man in der folgenden Animation die Geschwindigkeit β verstellt, sieht man die Änderung der Skalierung der x’- und ct’-Achsen:

    https://www.mpia.de/homes/tmueller/projects/bmwbook/web/minkowskiDiagram.html

    Man kann auch den Punkt E verschieben.

  671. #672 Karl-Heinz
    29. Dezember 2019

    @Anonym_2019

    Danke für Link.
    Wenn ich Punkt E am Handy verschiebe, dann muss ich den Schieberegler ß ein wenig betätigen, damit’s wieder stimmt. Wahr im ersten Moment soo richtig erschrocken. 😉

  672. #673 Anonym_2019
    29. Dezember 2019

    @Gebhard Greiter (28. Dezember 2019) #668

    “Wo Noethers Theorem (auf das sich der Hinweis in seiner letzten Mail an mich wohl bezieht) im Originaltext zu finden ist, weiß ich nicht.
    Sollte er im Internet stehen (frei zugänglich) wäre ich dankbar für den Link nach dort.”

    Ich glaube nicht, dass er Noethers Theorem meint. Aber der Link ist:

    https://de.wikisource.org/wiki/Invariante_Variationsprobleme

    via:
    https://scienceblogs.de/mathlog/2018/09/14/100-jahre-noether-theorem/

  673. #674 Gebhard Greiter
    29. Dezember 2019

    @Anonym_2019

    Sie schreiben in #656:

    Mein Sprachgebrauch führt aus meiner Sicht zu:

    Wenn sich zwei Uhren relativ zueinander geradlinig-gleichförmig bewegen, dann geht jede der Uhren im Ruhesystem der jeweils anderen Uhr langsamer als in ihrem eigenen Ruhesystem.

    Warum ist das widerspüchlich?

    Was Sie in diesen Worten über Uhren sagen, ist natürlich richtig (und ganz und gar nicht widersprüchlich).

    Wenn ich von “Ihrem Sprachgebrauch” spreche (und den inkonsistent finde), dann beziehe ich mich dabei auf Ihre Aussage “Das ist falsch” aus #646:

    Was bedeutet es denn, dass Uhren U1 und U2 “in Bezugssystem R gehen”?

    Es bedeutet nichts anderes, dass wir sie (gedanklich jedenfalls) durch das Koordinaten­gitter von R betrachten.

    Da relativ zu einander bewegte Bezugssysteme R1 und R2 unterschiedliche Koordinaten­gitter haben, sehen wir durch eines dieser Gitter betrachtet U1 schneller als U2 gehen, durchs andere Gitter betrachtet aber U2 schneller als U1 gehen.

    Den wirklichen Gang der Uhren tangiert das überhaupt nicht.

    Warum also beurteilen Sie dann in #646 meine Aussage als falsch?

  674. #675 Gebhard Greiter
    29. Dezember 2019

    @MartinB #665:

    In #665 schreiben Sie:

    Was den Kern hier angeht – die Frage, ob es die Beschleunigungen sind, die die Dilatation beim ZP verursachen – wiederhole ich nochmal meine Frage von vor einiger Zeit: Ist es der Knick in der Strecke Braunschweig-Berlin-München, der die größere länge dieser Strecke gegenüber dem geraden Weg verursacht?

    Mein Argument: Es können nur die Beschleunigungen sein (genauer: unterschiedliche Beschleunigung der Zwillinge).

    Bei stets gleicher (oder gar keiner) Beschleunigung würden sie sich ja auf derselben Weltlinie bewegen — höchstens vielleicht, räumlich gesehen, in umgekehrter Richtung, wie mein Beispiel im letzten Absatz von #580 zeigt.

    |
    Ihren Hinweis auf die Dreiecksungleichung für Metriken kann ich durchaus verstehen.

    Beachten allerdings muss man, dass die Dreiecksgleichung für die Minkoswki-Metrik nicht d(x,y) ≤ d(x,z) + d(z,y) lautet, sondern stattdessen d(x,z) + d(z,y) ≤ d(x,y).

    Ein Knick an Stelle z spielt eine Rolle (in dem Sinne, dass er aus ≤ ein < macht).

  675. #676 Gebhard Greiter
    29. Dezember 2019

    @ Karl-Heinz #669:

    Auch John Denkers Figure 5 auf Seite https://www.av8n.com/physics/twins.htm zeigt diese unterschiedliche Skalierung ganz deutlich.

  676. #677 MartinB
    29. Dezember 2019

    @gebhard
    Du hast meine Frage falsch verstanden.
    Mir geht es um die ebene Geometrie nach Euklid, nicht um Weltlinien. (Und ja, natürlich ist das Vorzeichen in der Dreiecksungleichung bei der SRT andersherum.)
    Ist die Strecke BS-B-M länger (in km gemessen, nicht als Weltlinie in der SRT, sondern direkt auf dem Globus oder der Deutschlandkarte gemessen), weil sie einen Knick hat? Ist der Knick die Ursache dafür, dass die Strecke länger ist? Wäre das in deinen Augen eine sinnvolle Sprechweise?

  677. #678 Karl-Heinz
    29. Dezember 2019

    @Gebhard Greiter

    Mir geht’s um was anderes. Man malt im Minkowski-Diagramm den Sachverhalt auf. Vergisst natürlich auf die unterschiedliche Skalierung und wundert sich, dass rein optisch die eine Strecke länger als die andere ist, obwohl es umgekehrt sein müsste. Darum habe ich auch von Stolpersteine geschrieben.

  678. #679 Anonym_2019
    29. Dezember 2019

    @Gebhard Greiter (29. Dezember 2019)

    “Den wirklichen Gang der Uhren tangiert das überhaupt nicht.”

    Die Gangrate einer Uhr in ihrem Ruhesystem halte ich nicht für “wirklicher” als ihre Gangrate in einem Bezugssystem, in dem sie bewegt ist. Zeitintervalle sind relativ.

    “Warum also beurteilen Sie dann in #646 meine Aussage als falsch?”

    Den Grund habe ich bereits 2x, in #646 und in #660 angegeben.

  679. #680 Gebhard Greiter
    29. Dezember 2019

    @ Anonym_2019 #679:

    Sie schrieben in #646:

    “Die Lichtuhr geht in dem Bezugssystem, in dem sie bewegt ist, langsamer als in ihrem Ruhesystem, weil das Licht mit der Geschwindigkeit c einen schrägen und damit nach Pythagoras verlängerten Weg zurücklegt.”

    Das ist richtig, ist aber einfach nur Folge der Tatsache, dass die beiden Koordinaten­systeme nicht denselben Zeitbegriff haben.

    Bitte beachten Sie:

    Wirklichkeit kann nicht relativ sein. Relativ ist immer nur ein durch Beobachtung entstehendes Bild jener Wirklichkeit, hier also der Zeitbegriff des Koordination­systms, aus dem heraus man beobachtet.

    Insbesondere geht keine Uhr in einem bestimmten Koordinatensystem. Sie kann nur aus der Perspektive unterschiedlicher Koordinatensysteme heraus betrachtet werden (d.h. aus der Perspektive von Beobachtern, die man sich im Ursprung jener Koordinatensysteme denkt).

    In Koordinatensystem K zu “gehen”, bedeutet für die Uhr nur, aus K heraus – und deswegen unter dem durch K gegebenen Zeitbegriff – betrachtet zu werden.

    Zeitdilatation infolge gleichförmiger Relativbewegung existiert nur als Ergebnis eines Vergleichs der Zeitbegriffe relativ zueinander bewegter Beobachter (bzw. Koordinaten­systeme).

  680. #681 Jolly
    29. Dezember 2019

    @Gebhard Greiter (#668)

    Meine Analogie aus #654 haben Sie leider auch nicht so interpretiert, wie es von mir gedacht war. Lassen Sie die Raumzeit außer acht, malen Sie den Weg, den ein Objekt in der Realität zurückgelegt hat, auf eine Landkarte. Ist die Länge, die Form, oder sind die auf dem Weg überwundenen Höhenmeter von der Geschwindigkeit abhängig, die das Objekt gehabt hat?

    Oder generell, sind niedrigere Ableitungen eines Weg-Zeit-Diagramms von höheren bestimmt, Beschleunigung von Ruck, Geschwindigkeit von Beschleunigung und Weg von Geschwindigkeit? Sind damit alle „korrespondierenden Eigenschaften“ der niedrigeren Ableitung damit auch von der höheren abhängig?

    Lassen Sie bitte ART, Noether-Theorem und weiteren unnötigen sprachlichen Ballast bei Seite. Bitte bleiben Sie in der SRT, bei deren Axiomen, und halten sich möglichst eng an das hier vorgestellte und diskutierte Diagramm.

  681. #682 Gebhard Greiter
    29. Dezember 2019

    @ Anonym_2019:

    In #660 fragen Sie mich:

    Wie definieren Sie das Wort “Zeitbegriff”? Bei Wikipedia kann ich keine Definition finden.

    Unter dem Zeitbegriff Z(K) der sich einem Koordinatensystem K zuordnet, verstehe ich das, was ein in seinem Ursprung sitzender Beobachter — seiner eigenen Uhr nach — als zeitlichen Abstand von ihm (paarweise) beobachteter Ereignisse misst.

  682. #683 Gebhard Greiter
    29. Dezember 2019

    @ Jolly #681:

    In #654 schreiben Sie:

    Betrachten wir einmal ganz analog Weglängen. Ja, ohne Geschwindigkeit wird die Wegeslänge wohl immer bei 0 bleiben. Wie könnte eine Strecke zurückgelegt werden, wie ein Weg entstehen, ohne Geschwindigkeit? Ohne Geschwindigkeit, also ohne Beschleunigung, wenn kein Ruck durch einen der Zwillinge gegangen ist, er möglicherweise den Knall nicht gehört hat, bleibt die Differenz der Zwillinge stets bei 0.

    Bestehen Sie auf die Aussage (halten Sie die für geeignet, Physik verständlich zu machen): Erst unterschiedliche Geschwindigkeit führt zu unterschiedlicher Weglänge.

    Ganz falsch erscheint mir die Aussage zwar nicht, aber …

    Die Aussage “Erst unterschiedliche Geschwindigkeit führt zu unterschiedlicher Weglänge” ergibt keinen Sinn, da es ja auch darauf ankommt, wie lange man sich mit welcher Geschwindigkeit bewegt hat.

    Zudem ist mit Weglänge ja wohl zurückgelegte Strecke gemeint. Ihr Satz lautet dann:

    Erst unterschiedliche Geschwindigkeit führt zu unterschiedlich langer zurückgelegter Strecke, was natürlich ganz sicher nicht stimmt.

    |
    Mein Vorschlag: Verlassen wir dieses Thema. Warum mich Joachims Aussage stört, habe ich ja schon in #668 erläutert, als ich dort schrieb:

    Erst Monate später las ich Claus Kiefers Buch und fand dessen Erklärung für die Ursache des entstehenden Altersunterschiedes verständlich und überaus einleuchtend.

    Daher also meine Gegenüberstellung der Aussage beider.

    Natürlich bin ich davon überzeugt, dass Joachim richtig denkt, es war und ist halt nur so, dass sein Spruch mir unakzeptabel erscheint bzw. nicht verstehbar.

  683. #684 Anonym_2019
    29. Dezember 2019

    @Gebhard Greiter (29. Dezember 2019) #682

    “ein in seinem Ursprung sitzender Beobachter — seiner eigenen Uhr nach — als zeitlichen Abstand von ihm (paarweise) beobachteter Ereignisse misst.”

    Geht in den “Zeitbegriff” aus Ihrer Sicht noch der Dopplereffekt ein, wie auf Ihrer Seite beschrieben?

    https://greiterweb.de/spw/Zu-Zeitdilatation-und-Dopplereffekt.htm

    Warum haben Sie in der Doppler-Formel die (auf der dort verlinkten Wikipedia-Seite verwendete) Größe “Freqenz” durch “Zeitbegriff” ersetzt?

  684. #685 Gebhard Greiter
    29. Dezember 2019

    @ MartinB #677:

    Ist die Strecke BS-B-M länger (in km gemessen, nicht als Weltlinie in der SRT, sondern direkt auf dem Globus oder der Deutschlandkarte gemessen), weil sie einen Knick hat?

    Ist der Knick die Ursache dafür, dass die Strecke länger ist? Wäre das in deinen Augen eine sinnvolle Sprechweise?

    Ich würde mich nie so ausdrücken.

    Ein geknickter Weg zwischen zwei Orten A und B wird dennoch länger sein als die gerade Verbindung zwischen A und B. Das genau ist ja die Aussage der Dreiecksgleichung.

    Was mit “Strecke BS-B-M” gemeint ist, verstehe ich nicht.

  685. #686 Jolly
    29. Dezember 2019

    @Gebhard Greiter

    „Die Aussage “Erst unterschiedliche Geschwindigkeit führt zu unterschiedlicher Weglänge” ergibt keinen Sinn“

    Ganz anlog ergäbe dann aber auch, „Erst unterschiedliche Beschleunigung führt zu unterschiedlicher [Reisegeschwindigkeit ! Zeitdilatation ! Weltlinie ].“ keinen Sinn.

    Keine Ahnung, warum Sie trotzdem darauf beharren wollen.

  686. #687 Gebhard Greiter
    30. Dezember 2019

    @ Jolly #686:

    Ich wüsste nicht, wie Teresa und Serena zu unterschiedlicher Weltlinie kommen könnten, wenn sie das ganze Szenario über gleich (also gar nicht) beschleunigt wären.

  687. #688 Anonym_2019
    30. Dezember 2019

    @Gebhard Greiter (29. Dezember 2019) #680

    Vielleich ist für Sie die Relativität der GleichORTigkeit, die es auch schon in der Newton’schen Theorie gab, leichter zu begreifen als die in der SRT zusätzliche Relativität der Gleichzeitigkeit.

    Halten Sie folgenden Aussagen für richtig?

    1) Im Bezugssystem A legt der Zug B den Weg ∆x zurück.
    2) Im Bezugssystem B legt der Zug B den Weg Null Meter zurück.

  688. #689 Gebhard Greiter
    30. Dezember 2019

    @ Anonym #684:

    Geht in den “Zeitbegriff” aus Ihrer Sicht noch der Dopplereffekt ein, wie auf Ihrer Seite beschrieben?

    Ein Beispiel: Versteht man unter U(B) die Uhr eines Beobachters B, und definiert man für je zwei Beobachter X und Y

    Z(n,X:Y) = Stand der Uhr U(Y), wenn Y das Ereignis U(X) = n eintreten sieht,

    so wird zur Berechnung von Z(n,X:Y) der Dopplereffekt mit berücksichtigt werden müssen.

    |
    Die auf Seite https://greiterweb.de/spw/Zu-Zeitdilatation-und-Dopplereffekt.htm gewählte Notation schien mir verständlicher als Anderes. Mehr Gedanken habe ich mir darüber nicht gemacht.

  689. #690 Gebhard Greiter
    30. Dezember 2019

    @ Anonym_2019 #688:

    Aussage 2) ist stets richtig.

    Ohne Bezug auf irgend ein gegebenes Szenario kann man Aussage 1) keinen Wahrheitswert zuordnen. Wollten Sie vielleicht wissen, ob 2) wahr ist, unter der Voraussetzung, dass 1) wahr sein sollte?

  690. #691 Anonym_2019
    30. Dezember 2019

    @Gebhard Greiter (30. Dezember 2019) #690

    “Wollten Sie vielleicht wissen, ob 2) wahr ist, unter der Voraussetzung, dass 1) wahr sein sollte?”

    Nein. Es ging mit nicht um den inhaltlichen Wahrheitswert. Es kann vorausgesetzt werden, dass das Szenario passt. Es ging mir darum, ob die formale Beschreibung in (1) O.K. ist, ohne einen Beobachter hinzuzufügen.

  691. #692 Jolly
    30. Dezember 2019

    @Gebhard Greiter

    „Ich wüsste nicht, wie Teresa und Serena zu unterschiedlicher Weltlinie kommen könnten, wenn sie das ganze Szenario über gleich (also gar nicht) beschleunigt wären.“

    Das sagten Sie bereits. Und ich sagte ja auch bereits, dass ich das nicht für komplett falsch halte. Aber das ist ja nur der erste Schritt. Was würde also daraus folgen? Was sagt das über das Verhältnis von Beschleunigung und Zeitdilatation und das Verhältnis von Geschwindigkeit und Zeitdilatation aus? Da scheiden sich doch die Geister.

    Nachdem Sie das jetzt wiederholt haben, müssen Sie auch zustimmen, dass sie nicht wissen können, wie Teresa und Serena zu unterschiedlichen Wegen kommen könnten, wenn sie das ganze Szenario über sich mit gleicher (also ohne) Geschwindigkeit bewegt hätten. Was sagt das über das Verhältnis von Geschwindigkeit und Höhenmetern und das Verhältnis von Weg und Höhenmetern?

    Es liegt an Ihnen, ob Sie die Ihnen gebotenen Anregungen, viele und sehr unterschiedliche, versuchen zu nutzen, um die Problem in Ihren Formulierungen und Gedanken zu erkennen. Diese Arbeit kann ihnen keiner abnehmen.

    Wenn Sie zustimmen, dass der Weg (nicht die Geschwindigkeit) entscheidend ist für die gesamte Höhendifferenz, dann sollten Sie auch zustimmen, dass die Geschwindigkeit (nicht die Beschleunigung) entscheidend ist für die gesamte Zeitdilatation.

  692. #693 Anonym_2019
    30. Dezember 2019

    @Gebhard Greiter (30. Dezember 2019) #689

    “der Dopplereffekt mit berücksichtigt werden müssen”

    Das ist das Problem. Um die Zeit von einem Inertialsystem in ein andere zu transformieren, muss die Lorentz-Transformation benutzt werden, nicht die Doppler-Formel. Das erfundene Wort “Zeitbegriff” sorgt darüber hinaus nur für Konfusion. Wie wäre es mit “Koordinatenzeit”?

  693. #694 Gebhard Greiter
    30. Dezember 2019

    @ Anonym_2019 #691:

    Bezugssysteme (= Koordinatengitter) existieren auch, ohne dass in ihrem Ursprung jemand sitzt, der tatsächlich beobachtet. Jedes ist so eine Art “Brille”, durch die man die Raumzeit betrachten kann (aber natürlich nicht betrachten muss).

  694. #695 Gebhard Greiter
    30. Dezember 2019

    @ Anonym_2019 #693:

    Ich habe ja gar nicht behauptet, dass es reichen würde, nur den Dopplereffekt zu berücksichtigen. Der Lorentz-Faktor muss natürlich dazukommen.

    |
    Kip Thorne schrieb: “Wenn Sie und ich uns relativ zueinander bewegen, muss das, was ich als Raum bezeichne, eine Mischung aus Ihrem Raum und Ihrer Zeit sein, und das, was Sie Raum nennen, eine Mischung aus meinem Raum und meiner Zeit.

    Es spricht also auch er von unterschiedlichen Zeitbegriffen. Warum nicht seinem Vorbild folgen?

  695. #696 Jolly
    30. Dezember 2019

    @MartinB (#665)

    Ich vergleiche das ja gern mit der Perspektive: Von zwei entfernten Beobachterinnen sieht jede die andere verkleinert.

    Wobei uns die SRT lehrt, angewendet auf die Perspektive: Wenn die eine Beobachterin weggegangen ist und anschließend zurückkehrt, dann ist diese wirklich verkleinert. – Oder nicht?

    Jede Analogie hat ihre Grenzen.

  696. #697 Anonym_2019
    30. Dezember 2019

    @Gebhard Greiter (30. Dezember 2019) #695

    “Ich habe ja gar nicht behauptet, dass es reichen würde, nur den Dopplereffekt zu berücksichtigen. Der Lorentz-Faktor muss natürlich dazukommen.”

    Falsch. Der Dopplereffekt darf nicht berücksichtigt werden. Der Lorenz-Faktor allein ist manchmal falsch. Statt Doppler-Effekt & Lorenz-Faktor sollte nur die Lorentz-Transformation verwendet werden:
    x’ = γ (x – v/c * ct)
    ct’ = γ (ct – v/c * x)

    “Es spricht also auch er von unterschiedlichen Zeitbegriffen.”

    In dem Zitat sehe ich das Wort “Zeitbegriff” aber nicht. Er hat die Lorentz-Transformation (s.o.) in Worte gefasst.

  697. #698 Anonym_2019
    30. Dezember 2019

    @Gebhard Greiter (29. Dezember 2019) #680

    “Insbesondere geht keine Uhr in einem bestimmten Koordinatensystem.”

    Dann dürfte ja auch kein Zug in einem bestimmten Koordinatensystem einen Weg zurücklegen.

    Sie haben aber meiner Aussage in #688:
    “1) Im Bezugssystem A legt der Zug B den Weg ∆x zurück.”
    zugestimmt in #694.

    Sie habe das nicht abgeleht mit einer Begündung wie “Insbesondere legt kein Zug in einem bestimmten Koordinatensystem einen Weg ∆x zurück (nur weil ihm ein Weg ∆x entgegenkommt).”

  698. #699 Gebhard Greiter
    30. Dezember 2019

    @ Anonym_2019: #682 + #684:

    Wie definieren Sie das Wort “Zeitbegriff”? Bei Wikipedia kann ich keine Definition finden.

    Unter dem Zeitbegriff Z(K) der sich einem Koordinatensystem K zuordnet, verstehe ich das, was ein in seinem Ursprung sitzender Beobachter — seiner eigenen Uhr nach — als zeitlichen Abstand von ihm (paarweise) beobachteter Ereignisse misst.

    Sie können meine Notation auf Seite https://greiterweb.de/spw/Zu-Zeitdilatation-und-Dopplereffekt.htm aber auch lesen wie folgt:

    Z(B:B) = Z(B) ist die Frequenz der Uhr U(B) von B wie B selbst sie beobachtet,
    Z(B:A) ist die Frequenz von U(B) wie A sie beobachtet.

    Damit ist Z(A:B) die durch B beobachtete Frequenz von U(A).

  699. #700 Karl-Heinz
    30. Dezember 2019

    @Jolly

    Wobei uns die SRT lehrt, angewendet auf die Perspektive: Wenn die eine Beobachterin weggegangen ist und anschließend zurückkehrt, dann ist diese wirklich verkleinert. – Oder nicht?
    Jede Analogie hat ihre Grenzen

    Nein, damit ist nicht gemeint, dass man die absolute Zeit der Uhr vergleicht, sondern ihr Gangunterschied im Zeitintervall. Wenn die Relativgeschwindigkeit zwischen den beiden Uhren null ist, dann zeigen sie jetzt natürlich unterschiedliche Zeiten an, aber die Zeit vergeht auf beiden wieder gleich schnell. Man bräuchte die beiden Uhren jetzt nur zu synchronisieren und sie wären anschließend wieder identische Uhren, die korrekt die Zeit anzeigen.

  700. #701 Gebhard Greiter
    30. Dezember 2019

    @ Jolly:

    Ihre Aussage aus #696 ist falsch.

    Dass B die Länge eines relativ zu ihm bewegten Objekts in Bewegungsrichtung verkürzt sieht, ist ein rein nur beobachungstechnisch begründeter Effekt.

    Mehr dazu in #668 (s. Carriers Aussage dort).

  701. #702 MartinB
    30. Dezember 2019

    @gebhard #685
    BS-B-M = Braunschweig-Berlin-München.
    “Ich würde mich nie so ausdrücken.”
    Warum willst du dann in der Raumzeit plötzlich den Knick (d.h. die Beschleunigung) verantwortlich machen, obwohl es (abgesehen von Minkowski- statt Euklidischer Metrik) exakt dieselbe Situation ist? In beiden Fällen gibt es eine Dreiecksungleichung, in beiden Fällen betrachten wir gerade Linien bzw. Weltlinien, aber nur in einem der beiden Fälle ist deiner Ansicht nach der Knick fundamental dafür verantwortlich, dass die Streckenlängen (entsprechend Eigenzeiten) sich unterscheiden.

    Das erschließt sich mir absolut nicht.

    Auch das Thorne-Zitat lässt sich am besten in der ebenen Geometrie veranschaulichen: Wenn du deiner x- und y-Achse zu meinen geneigt legst, dann hat das, was für mich eine reine Strecke in x-Richtung ist, für dich auch eine y-Komponente. Auch da ist es unnötig, da mehr hineinzugeheimnissen, als drin ist – Koordinatentransformationen mischen Koordinaten, und in der SRT gehören eben Zeit und Raum zusammen.

    Und auch wenn es die anderen schon mehrfach gesagt haben, sage ich es auch nochmal: Um die Lorentz-Transformationen richtig anzuwenden und die Ereignisse in unterschiedlichen Bezugssystemen zu beschreiben, ist der Dopplereffekt irrelevant und die Formel dafür hat in solchen Rechnungen nichts zu suchen.

  702. #703 Gebhard Greiter
    30. Dezember 2019

    @ Anonym #697:

    In dem Zitat sehe ich das Wort “Zeitbegriff” aber nicht.

    Es steckt implizit drin, denn er spricht ja wörtlich von “Ihrer Zeit” und “meiner Zeit”.

  703. #704 Gebhard Greiter
    30. Dezember 2019

    @ MartinB #702:

    Warum willst du dann in der Raumzeit plötzlich den Knick (d.h. die Beschleunigung) verantwortlich machen, obwohl es (abgesehen von Minkowski- statt Euklidischer Metrik) exakt dieselbe Situation ist? In beiden Fällen gibt es eine Dreiecks­ungleichung, in beiden Fällen betrachten wir gerade Linien bzw. Weltlinien, aber nur in einem der beiden Fälle ist deiner Ansicht nach der Knick fundamental dafür verantwortlich, dass die Streckenlängen (entsprechend Eigenzeiten) sich unterscheiden.

    Die Dreiecksungleichung beschreibt ja nur eine Folge des Knicks.

    Ursache des Knicks aber ist, dass er wegen einer kurzen Beschleunigungsphase zustande kam, die zur Drehung der Raumachse des Koordinatengitters geführt hat.

    Genau genommen ist der Knick sogar ein Bogen, da die Beschleunigungsphase ja niemals aus nur einem einzigen Zeitpunkt besteht.

  704. #705 Karl-Heinz
    30. Dezember 2019

    Man kann sich auch vorstellen, daß der raumfahrende Zwilling zu Beginn der Reise schon mit voller Geschwindigkeit an seinem Bruder vorbeifliegt und im Moment des Vorbeifliegens als Startzeit die auf dessen Uhr abgelesene Zeit übernimmt. Am vorgesehenen Umkehrpunkt seiner Reise übergibt er dann seine Zeit an einen Klon seiner Person, der mit einer identischen Kopie seiner Uhr ausgestattet ist, mit der gleichen Geschwindigkeit wie er in umgekehrter Richtung fliegt und ihm am Umkehrpunkt begegnet. Der Klon übergibt seine Zeit schließlich beim Erreichen der Erde im Vorbeiflug an den zurückgebliebenen Zwilling. Bei dieser (gleichwertigen) Betrachtung sind alle Beschleunigungen vermieden.
    All das nimmt Gebhard Greiter nicht zur Kenntnis, obwohl von MartinB ausführlich in seinem Artikel beschrieben. Typisch Crackpot halt, der niemals von seiner vorgefertigten Meinung abweichen wird.
    @Jolly: Ein Elefanten im Porzellanladen bleibt ein Elefant. Ein Crank bleibt ein Crank. Da wirst auch du nichts ausrichten können!

  705. #706 Gebhard Greiter
    30. Dezember 2019

    @ MartinB #704:

    Sie schreiben:

    Auch das Thorne-Zitat lässt sich am besten in der ebenen Geometrie veranschaulichen: Wenn du deiner x- und y-Achse zu meinen geneigt legst, dann hat das, was für mich eine reine Strecke in x-Richtung ist, für dich auch eine y-Komponente.

    Völlig richtig – auch ich sehe so.

    … ist es unnötig, da mehr hineinzugeheimnissen, als drin ist – Koordinaten­transformationen mischen Koordinaten, und in der SRT gehören eben Zeit und Raum zusammen.

    Ja, so ist es. Ich wüsste jetzt wirklich nicht, was ich da zusätzlich noch “reingeheimse”.

    Was soll denn – deinem Eindruck nach – dieses Zusätzliche sein?

  706. #707 Gebhard Greiter
    30. Dezember 2019

    @ Karl-Heinz #705:

    Eine brauchbare Lösung des ZP darf solche Vorstellung aber nicht voraussetzen. Sie muss schließlich auch noch für den Fall überzeugen, dass die Zwillinge vor Serenas Abreise ruhig neben einander sitzen.

  707. #708 Karl-Heinz
    30. Dezember 2019

    @Gebhard Greiter #707
    Nein!
    Gleiches Ereignis genügt. Mit anderen Worten heißt das, am gleichen Ort zur gleichen Zeit.

  708. #709 Gebhard Greiter
    30. Dezember 2019

    @ MartinB #702:

    Deine Aussage:

    Um die Lorentz-Transformationen richtig anzuwenden und die Ereignisse in unterschiedlichen Bezugssystemen zu beschreiben, ist der Dopplereffekt irrelevant und die Formel dafür hat in solchen Rechnungen nichts zu suchen.

    Mir scheint, man sollte es genauer so ausdrücken:

    Die Lorentz-Transformationen beschreibt, wie sich aus Raum und Zeit im Sinne des Beobachters Raum und Zeit im Sinne des beobachteten Objekts ergeben.

    Letzteres ist jedoch nicht genau dasselbe, was der Beobachter als Messergebnis erhält:

    Was er als Gang der Uhr des beobachteten Objekts wahrzunehmen glaubt – sein Beobachtungsergebnis also –, ist ein durch den Dopplereffekt verfälschtes Bild dieses Uhrengangs.

    Wie sonst könnte es sein, dass jener Uhrengang sich ihm unterschiedlich darstellt, je nachdem ob die fremde Uhr auf ihn zukommt oder sich von ihm entfernt?

    Am Lorentzfaktor kann es nicht liegen, denn der ist ja nur abhängig vom Betrag der Geschwindigkeit.

  709. #710 Karl-Heinz
    30. Dezember 2019

    @Gebhard

    Mir scheint, du hast als Kind nie Kohlkopfrennen gespielt.

  710. #711 Jolly
    30. Dezember 2019

    @Karl-Heinz (#700)

    Nein, damit ist nicht gemeint, […]

    Ach! (vrgl. Loriot)

    „Elefant bleibt Elefant, Crank bleibt Crank.“ (#705; von mir paraphrasiert, bzw. gekürzt)

    Hoffen wir also, dass es sich in beiden Fällen eigentlich doch um verständige Menschen handelt.

    Die Antwort von @Gebhard Greiter auf meinen ursprünglich nur scherzhaft gemeinten Kommentar hat allerdings noch einmal sehr deutlich gemacht, woran die Diskussion hier tatsächlich scheitert. Dazu später mehr.

    The best material model of a cat is another, or preferably the same, cat.“ (Norbert Wiener)

    Seien Serena und Teresa zwei junge Zwillinge. Die Kinder befinden sich in einer Phase gleichmäßigen Wachstums. …
    Nach ihrer Rückkehr ist Serena kleiner als Teresa.

    So besser?

  711. #712 Karl-Heinz
    30. Dezember 2019

    Seien Serena und Teresa zwei junge Zwillinge. Die Kinder befinden sich in einer Phase gleichmäßigen Wachstums. …
    Nach ihrer Rückkehr ist Serena kleiner als Teresa.

    So besser?

    Oh, … Elefant versteht. 😉

  712. #713 Anonym_2019
    30. Dezember 2019

    @Gebhard Greiter (30. Dezember 2019) #703

    “Es steckt implizit drin, denn er spricht ja wörtlich von “Ihrer Zeit” und “meiner Zeit”.”

    Das hat Kip Thorne unglücklich formuliert / abgekürzt. Er meint: “Koordinatenzeit Ihres Ruhesystems” / “Koordinatenzeit meines Ruhesystems”.

  713. #714 MartinB
    30. Dezember 2019

    @Gebhard
    Das Zusätliche ist, dass du irgendwie die Beschleunigung oder den dopplereffekt oder sonst etwas bemühen willst, obwohl all das gar nicht notwendig ist.

    Was den Knick angeht – was “verursacht” denn den Knick bei einer aus zwei Segmenten bestehenden Linie in der Geometrie?


    Was er als Gang der Uhr des beobachteten Objekts wahrzunehmen glaubt – sein Beobachtungsergebnis also –, ist ein durch den Dopplereffekt verfälschtes Bild dieses Uhrengangs.”
    Das war falsch, ist falsch und bleibt falsch. Der Dopplereffekt spielt keine Rolle, wie man leicht sieht, wenn man sich vorstellt, jeder Beobachter würde sein Koordinatensystem mit einer Vielzahl an Kameras ausstatten, so wie in meinem Buch ausführlichst beschrieben.

  714. #715 Karl-Heinz
    30. Dezember 2019

    @Gebhard Greiter #709

    Schon vergessen?
    Ein unendliches Gitter aus Uhren bzw. Kameras, die alles aufzeichnen.

    Wie was in der Praxis gemessen wird, das würde ich den Profis überlassen, die was davon verstehen.

  715. #716 Gebhard Greiter
    30. Dezember 2019

    @ MartinB #714:

    Aha, jetzt also ist klar, worin wir unterschiedlicher Meinung sind.

    Die vielen Kameras, von denen Du sprichst, finden sich im Koordinatengitter, wie es sich im Minkowski-Diagramm ergibt (s. z.B. John Denkers Figure 5 auf Seite https://www.av8n.com/physics/twins.htm ). Man kann sie sich vorstellen an den Schnittpunkten der Linien gleicher Farbe).

    Wenn dem so sein sollte, zeigt das Bild, das sie liefern, den Zeitbegriff des beobachteten Objekts.

    Die Messinstrumente des Beobachters in der Wirklichkeit aber zeigen das durch den Dopplereffekt zusätzlich noch relativierte Bild der von Ferne beobachteten Uhr (auf Seite https://greiterweb.de/spw/Zu-Zeitdilatation-und-Dopplereffekt.htm nenne ich es » den A von B unterstellten Zeitbegriff « – der Dopplereffekt bewirkt, dass er sich vom Zeitbegriff, den die SRT für A per Lorentz-Transformation errechnet, unterscheidet).

    Hier also ist der Punkt, an dem unser beider Überzeugungen sich trennen.

    Vielen Dank für die Diskussion. Sie hat mir zusätzliche Klarheit beschert.

  716. #717 MartinB
    30. Dezember 2019

    @gebhard
    Es gibt hier keine Meinungen oder Überzeugungen, es gibt nur eine korrekte Physik.
    Du kannst dir natürlich irgendwelche Zeibegriffe definieren, aber physikalisch sinnvoll ist das nicht.

  717. #718 Gebhard Greiter
    30. Dezember 2019

    @ MartinB #717 (zu #716):

    Es ist einfach nur eine unterschiedlich detaillierte Darstellung gegebener Physik.

    Vor etwas, das nur einen zusätzlichen Begriff darstellt (um genauer sein zu können), sollte man sich nicht fürchten.

  718. #719 Karl-Heinz
    30. Dezember 2019

    @Gebhard Greiter

    Hast du mit deiner Konvention jemals einmal ein Beispiel gerechnet? Wenn nein, dann entschuldige dich doch bei uns!

  719. #720 Karl-Heinz
    30. Dezember 2019

    Besonders einleuchtend ist die folgende Variante der eben durchgeführten Betrachtung. 2 Die Zwillinge werden statt mit Fernrohren mit baugleichen Lampen ausgestattet, die monochromatisches Licht abstrahlen. Während der Reise leuchten sie sich gegenseitig an und beobachten das vom Bruder kommende Licht. Jeder der Zwillinge sieht während des ersten Teils der Reise rotverschobenes Licht und während des zweiten blauverschobenes. Der Zwilling im Raumschiff kehrt im Wellenfeld des von der Erde kommenden Lichts um und erlebt den Übergang von der Rot- zur Blauverschiebung sofort beim Umkehren, exakt nach der Hälfte seiner Reisezeit. Der auf der Erde zurückgebliebene Zwilling muß dagegen nach der Umkehr des Raumschiffes noch warten, bis das erste nach der Umkehr abgesandte Licht den Weg bis zur Erde zurückgelegt hat, und sieht erst dann, also weit nach der Hälfte der von seiner Uhr angezeigten Reisezeit, den Umschlag zur Blauverschiebung. Da das Ausmaß der Rot- bzw. Blauverschiebung aber aus Symmetriegründen für beide Brüder gleich ist, entsteht ein Unterschied in der Anzahl der beobachteten Wellenmaxima, der einem Unterschied in der vom Bruder erlebten Zahl von Herzschlägen entspricht. Die hier geschilderten gegenseitigen Beobachtungen enthalten Laufzeiteffekte des Lichts.

  720. #721 Jolly
    31. Dezember 2019

    @Gebhard Greiter

    Danke auch Ihnen für die Diskussion. Sicher ein guter Zeitpunkt die heute zu beenden.

    Deshalb auch nur noch kurz, und nur weil ich es schon angekündigt hatte:

    Ihre Aussage aus #696 ist falsch

    Ja, sicher. Aber ihre Begründung warum die falsch ist (#701), die ist wieder sowas von daneben, wie es typisch für viele ihrer Kommentare ist. Darin ist ja häufig etwas schief, missverstanden oder falsch zusammengerührt – wie auch auf ihrer Webseite.

    Hier ging es nie um Längenkontraktion, weder im Blogartikel noch in meinem Kommentar. Im Artikel kann man das bei der Betrachtung des Diagramms erahnen, da dort auf keiner von Serenas Raumachsen Markierungen angebracht sind, die ansonsten ja schon hilfreich oder sogar nötig gewesen wären. In der von Martin Bäker ins Spiel gebrachten und von mir verwendeten Analogie kann man das auch erkennen. In #696 steht „entfernten“. Die Beobachterinnen entfernen sich nicht mehr voneinander. Man darf sogar davon ausgehen, die stehen da entfernt, ruhend zueinander. Deswegen sehen die sich auch gegenseitig “verkleinert“, nicht verkürzt. Es muss also um was anderes gehen.

    Allen einen Guten Rutsch.

  721. #722 Karl-Heinz
    31. Dezember 2019

    Auch ich wünsche allen einen guten Rutsch.
    Diskussionen werden nun mal hart und emotional geführt. Ist aber kein Grund sich den Rutsch in das neue Jahr vermiesen zu lassen.

    Ihr Alle und damit meine ich wirklich alle, Guten Rutsch ins neue Jahr. 🙂

  722. #723 Anonym_2019
    31. Dezember 2019

    @Gebhard Greiter (28. Dezember 2019) #558

    Sie bestand nur aus folgendem Hinweis, mit dem ich aber bis heute nicht wirklich was anzufangen weiß. Er schrieb mir,

    Ihr Fehler liegt in der Annahme, aus relativen Geschwindigkeiten könne keine vom Beobachter unabhängige Konsequenz entstehen.
    Hier gibt es eine zugrunde liegend Symmetrie, die dafür sorgt, dass die vergangene Eigenzeit eines bewegten Objektes zwischen zwei Ereignissen unabhängig vom Bezugssystem ist.

    Vermutlich meint Herr Schulz mit dem ersten Satz richtigerweise, dass Serena in Teresas Inertialsystem zu jedem Zeitpunkt um den Faktor γ(v) verlangsamt altert und dass die Konsequenz daraus der Altersunterschied beim Zusammentreffen ist.

    Mit der Symmetrie im zweiten Satz meint er warscheinlich die Isotropie des Raumes, mit deren Annahme man in Verbindung mit Gruppenpostulaten und dem Michelson-Morley-Experiment (absolute LG) begründen kann, dass als Transformationsgleichung zwischen Inertialsystemen nur die Lorentz-Transformation infrage kommt. Mit dieser kann man wiederum zeigen, dass die vergangene Eigenzeit (=raumzeitlicher Weg) eines bewegten Objektes zwischen zwei Ereignissen unabhängig vom Bezugssystem ist.

    Siehe Abschnitt “From group postulates”:
    https://en.wikipedia.org/wiki/Derivations_of_the_Lorentz_transformations#From_group_postulates

  723. #724 Karl-Heinz
    1. Januar 2020

    @Jolly
    🙂

    Gebhard Greiter steht ebenfalls zu seinen Fehlern. Er hat schon einiges auf seiner Homepage korrigiert.

    Nein hat er nicht. Guck dir mal dem letzteren Teil von Gebhard’s Artikel an.
    https://greiterweb.de/zfo/Zwillingsparadoxon-im-Lichte-von-ART-SRT.htm

    Sollen wir Gebhard sich selbst überlassen?
    Mein Rat: Da Zeit ein kostbares Gut ist, JA!

  724. #725 Anonym_2020
    1. Januar 2020

    @Gebhard Greiter (27. Dezember 2019) #632

    Und ich dachte bei fast Lichtgeschwindigkeit sprich 45° sei Schluss. Bist du dir wirklich sicher alles richtig verstanden zu haben? :-);

    Schau dir mal auf Seite https://www.av8n.com/physics/twins.htm John Denkers Figure 5 an: Der Winkel zwischen den Raumachsen des roten und des blauen Gitters ist dort größer als 45 Grad.

    In einem Minkowski-Diagramm ist nur dasjenige Koordinatengitter das Bezugssystem, das waagerechte und senkrechte Achsen hat, also in Denkers Figure 5 das grüne Koordinatengitter. Nur darauf beziehen sich die Geschwindigkeiten und Winkel mit tan α = v/c.

    Die Geschwindigkeitsdifferenz zwischen dem blauen und grünen Gitter darf größer als c sein.

    Wenn z.B. das blaue Gitter Bezugssystem werden soll, muss das Diagramm neu gezeichnet werden.

  725. #726 Anonym_2020
    1. Januar 2020

    @Gebhard Greiter (27. Dezember 2019) #632

    Korrektur zu #725:
    Die Geschwindigkeitsdifferenz zwischen dem blauen und roten Gitter darf größer als c sein.

  726. #727 Karl-Heinz
    1. Januar 2020

    @Anonym_2020

    Anonym_2019 –> Anonym_2020
    Du alterst wohl nie. 😉

  727. #728 Karl-Heinz
    2. Januar 2020

    Man kann ruhig mal versuchen die Anzahl der beobachteten Wellenmaxima, die jeder vom anderen empfängt, über die Doppler-Verschiebung zu berechnen. Bei Gebhard, obwohl er Mathematik studiert hat, gehe ich nicht davon aus, dass er das schaft.

  728. #729 Karl-Heinz
    2. Januar 2020

    Zu #728

    Jeder der Zwillinge strahlt monochromatisches Licht der Frequenz f ab.

  729. #730 Anonym_2020
    5. Januar 2020

    @Gebhard Greiter

    Die Seite …
    https://greiterweb.de/spw/Zu-Zeitdilatation-und-Dopplereffekt.htm

    … wurde verändert, aber da stimmt vieles immer noch nicht, z.B.:

    Wie Zeitdilatation und Dopplereffekt die Zeit relativ machen

    Falsch. Der Dopplereffekt macht nicht die Zeit relativ.

    In voller Allgemeinheit — auch, wenn nicht konstante Beschleunigung mit zu berücksichtigen ist — wird der Zeitdilatationseffekt durch die Lorentz-Transformation beschrieben. Der Dopplereffekt ist dann schon mit berücksichtigt.

    Falsch. Der Dopplereffekt steckt nicht in der LT drin.

    Nochmals sei wiederholt: Die in Sachbüchern oft genannten einfachen Formeln
    Δx_A:B = γ(v)⁻¹ Δx_B:B
    Δt_A:B = γ(v) Δt_B:B
    gelten nur für Bewegung, die in Blickrichtung weg vom Beobachter führt.

    Falsch. Die gelten auch für Bewegung, die in Blickrichtung hin zum Beobachter führt.

  730. #731 Karl-Heinz
    6. Januar 2020

    @Anonym_2020

    Willst den Gebhard missionieren? 🙂

  731. #732 Peter Strohmayer
    Wien
    11. Januar 2020

    Martin #555, #575, #665
    Das Zwillingsparadoxon wird insofern durch Beschleunigung verursacht, als diese die zwischen den Zwillingen bei gleichförmiger Bewegung an sich bestehende Symmetrie des Zurückbleibens der Eigenzeit hinter der Zeit des gegenbeteiligten Zwillings durchbricht.
    Allerdings haben das Maß der Geschwindigkeit und die Dauer der Reise einen bedeutenderen Einfluss auf den Altersunterschied als das Maß der Beschleunigung.
    Wenn man aber fragt, was entscheidend (was “wichtiger”) ist, die Beschleunigung oder die Geschwindigkeit, so ist es die Beschleunigung, weil diese definitionsgemäß mit einer Geschwindigkeit verbunden ist, die einen Altersunterschied bewirkt, während die Dauer einer Reise bei gleichförmiger Relativgeschwindigkeit ohne Beschleunigung keine Effekt hätte.

    Martin # 560
    Eine anfängliche Beschleunigung wäre nicht wichtig, wenn zueinander bewegte Zwillinge bei einer Erstbegegnung ihre Alters-Uhren auf Null stellen könnten. Da es sich aber in Anbetracht ihrer gemeinsamen Geburt nicht um eine bewegte Erstbegegnung handeln kann, bedarf es, um nicht aus dem Bild zu fallen, einer anfänglichen Beschleunigung. Die ist lebensnäher und macht den Vorgang eindrucksvoller.

    Karl-Heinz #705
    Die Eigenzeit eines Materiepunkts, um die es beim Zwillingsparadoxon geht, ist keine Handelsware, die man auf andere Materiepunkte übertragen kann. Ein Materiepunkt behält sein Zeitguthaben bei Wechsel in ein anderes Bezugssystem, aber er muss es selber tun. Der von Dir vorgeschlagene Zeitübertrag ist nach seinem physikalischen Gehalt erst wieder eine Beschleunigung.

  732. #733 Anonym_2020
    11. Januar 2020

    @Peter Strohmayer Wien (11. Januar 2020) #732

    “während die Dauer einer Reise bei gleichförmiger Relativgeschwindigkeit ohne Beschleunigung keine Effekt hätte.”

    Doch sie hat auch dann einen Effekt, siehe Kommentar #561.

  733. #734 MartinB
    12. Januar 2020

    @Peter
    “Allerdings haben das Maß der Geschwindigkeit und die Dauer der Reise einen bedeutenderen Einfluss auf den Altersunterschied als das Maß der Beschleunigung.”
    Genauer gesagt hat das Maß der Beschleunigung (bei instantaner Umkehr) gar keinen Einfluss!

    “während die Dauer einer Reise bei gleichförmiger Relativgeschwindigkeit ohne Beschleunigung keine Effekt hätte. ”
    Doch, natürlich hat sie einen Einfluss (Zeitdilatation wirkt immer), der ist nur für beide symmetrisch und nicht direkt beobachtbar.

    Auch für dich die Frage: Würdest du sagen, dass die Strecke Braunschweig-Berlin-München länger ist als die Braunschweig-München, weil sie einen Knick hat oder weil sie aus zwei nicht auf einer Geraden liegenden Teilen besteht? Ist es der Knick, der entscheidet, oder die Strecken selbst?

    Die Anfangsbeschleunigung ist irrelevant (wie auch die genaue Beschleunigug am Umkehrpunkt). Klar braucht man sie, damit es “Zwillinge” sein können, aber notwendig ist sie nicht. Es könnte auch Serena erst in die Gegenrichtung fliegen, dort umkehren und dann an der Erde vorbeifliegen, wobei beide ihre Uhren nullen. Wenn man anfängt, über dieses Detail zu diskutieren, verwirrt man die Sache in meinen Augen nur weiter.

  734. #735 Peter Strohmayer
    Wien
    12. Januar 2020

    Martin #734
    Es gibt keine (und schon gar keine “instantane”) Umkehr ohne Beschleunigung.

    Die Zeitdilatation wirkt zwar auch bei gleichförmiger Geschwindigkeit, aber nur symmetrisch. Das ist beobachtbar, wenngleich von Anonym 2020 in # 561 unrichtig interpretiert: Undine ist eine Uhr im Gitternetz von Teresa, und symmetrisch dazu gibt es eine Undine im Gitternetz von Serena. Beide machen die Beobachtung, dass die Anzeige der Uhr (der jeweils vorbeikommenden Undinen) weiter vorangeschritten ist. Diese Symmetrie kann nur durch Beschleunigung beseitigt werden.

    Es hilft nichts: der Effekt, von dem beim Zwillingsparadoxon die Rede ist, ein langsameres Altern eines der Zwillinge, tritt beim gleichförmigen Reisen nicht ein.

    Braunschweig-Berlin-München: dass zwei Seiten eines Dreiecks länger sind als die dritte, hat Hilbert, Grundlagen der Geometrie, axiomatisch bewiesen. Der Gedanke, dass der Knick oder die zwei Strecken die “Ursache” dafür sein könnten bzw. dass das eine “entscheidender” ist als das andere, ist ihm nicht gekommen. Mir auch nicht.

    Fehlt noch etwas? Ach ja, das Myon! Bei gleichförmiger Eigengeschwindigkeit legt ein Materiepunkt in Eigenzeit eine relativistisch verkürzte Reisestrecke zurück, so wie der entgegenkommende Materiepunkt (das Ziel) in gleicher Eigenzeit eine relativistisch gleich verkürzte Reisestrecke zurücklegt (auch hier herrscht Symmetrie). Die Eigengeschwindigkeit (ein Rechenwert) kann die Lichtgeschwindigkeit überschreiten, deshalb gelangt das Myon auf die Erde, es altert aber – Gravitation ist ausgeblendet – nicht anders als diese.

  735. #736 Anonym_2020
    12. Januar 2020

    @Peter Strohmayer Wien (12. Januar 2020) #735

    “wenngleich von Anonym 2020 in # 561 unrichtig interpretiert”

    Ich habe nichts unrichtig interpretiert. Vielleicht war das ein Formulierungsfehler auf Ihrer Seite. Um Ihre Aussage aus #732
    (“während die Dauer einer Reise bei gleichförmiger Relativgeschwindigkeit ohne Beschleunigung keine Effekt hätte”) zu widerlegen, reicht die Betrachtung in genau einem Inertialsystem aus.

    “Es hilft nichts: der Effekt, von dem beim Zwillingsparadoxon die Rede ist, ein langsameres Altern eines der Zwillinge, tritt beim gleichförmigen Reisen nicht ein.”

    Jetz schränken Sie gegenüber #732 ein, was Sie mit “Effekt” meinen, nämlich einen Lorentz-invarianten Effekt. Ein nicht Lorentz-invariantes langsameres Altern eines der Zwillinge trittt aber trotzdem bei gleichförmigem Reisen ein, wie in #561 beschrieben.

  736. #737 Karl-Heinz
    Graz
    12. Januar 2020

    @Peter Strohmayer

    Wenn du die Eigenzeit eines Teilchen aus seiner Weltlinie berechnest, so wirst du feststellen, dass die Beschleunigung nicht das Mass für die Eigenzeit ist. Ist das so schwer zu verstehen?
    Natürlich treten Beschleunigungen auf, wenn die Geschwindigkeit nicht konstant ist. Gerade beim Zwillingsparadoxon kann man sehr schön zeigen, dass das abrupte Umdrehen gar nicht in die Eigenzeit eingeht. Capisci cosa voglio dire?

  737. #738 MartinB
    13. Januar 2020

    @PeterStrohmeyer
    “Braunschweig-Berlin-München: dass zwei Seiten eines Dreiecks länger sind als die dritte, hat Hilbert, Grundlagen der Geometrie, axiomatisch bewiesen. Der Gedanke, dass der Knick oder die zwei Strecken die “Ursache” dafür sein könnten bzw. dass das eine “entscheidender” ist als das andere, ist ihm nicht gekommen. Mir auch nicht.”
    Warum soll das dann in der Raumzeit anders sein? Eine Weltlinie aus zwei Stücken hat eine kürzere Eigenzeit als eine aus einem Stück. Ist matehmatisch bis auf ein Vorzeichen exakt dieselbe Situation wie in der Geometrie, warum soll man also im einen Fall den Knick (die Beschleunigung) für wichtig erachten, im anderen nicht?

    Doc Brown würde sagen “Du musst lernen, vierdimensional zu denken!” 😉

  738. #739 Peter Strohmayer
    Wien
    13. Januar 2020

    Der mathematischen Formalismus (Aufaddieren der Eigenzeiten einer Weltlinie) ist das eine, die vom mathematischen Formalismus abgebildete Wirklichkeit das andere. Im Ersten herrscht (im Sinn der Verursachung irgendeiner erfundenen Weltlinie) die Phantasie, im Zweiten die Mechanik.
    Braunschweig-Berlin-München wird durch die geographischen Verhältnisse in Deutschland verursacht, die gekrümmte Weltlinie durch Beschleunigung (auch wenn sie, wenn es eine instantane Beschleunigung gäbe, nicht in die Eigenzeit einginge).

  739. #740 Karl-Heinz
    13. Januar 2020

    Peter Strohmayer

    Bist das du?. Also der Einsteinversteher mit dem Link
    https://www.strohmayerwien.de/

  740. #741 MartinB
    13. Januar 2020

    @Peter
    Ich verstehe das Argument nicht.
    Wir haben einmal eine euklidische Geometrie in 2D, einmal eine Minkwoski-Geometrie in 4D. In beiden gibt es ne Dreiecksungleichung. Wo da die Fantasie ne Rolle spielen soll, versstehe ich nicht.

  741. #742 Anonym_2020
    13. Januar 2020

    @Peter Strohmayer Wien (13. Januar 2020) #739

    “die gekrümmte Weltlinie durch Beschleunigung”

    Selbst wenn sich Teresa und Serena “wegen” der Beschleunigung wieder treffen, wird der Altersunterschied nur in der Theorie Bezugssystem-unabhängig. Wenn Serena z.B. am Ende der Reise abbremst und dann einen konstanten Abstand von 20 cm von Teresa hält, dann ist der Altersunterschied immer noch Bezugssystem-abhängig. Ein vorbeifliegendes Neutrino, welches in seinem Ruhesystem ein Gitternetz von synchronisierten Uhren angebracht hat, wird z.B. einen anderen Altersunterschied zwischen Teresa und Serena feststellen, als denjenigen, über den sich Teresa und Serena einig sind.

  742. #743 Tox
    13. Januar 2020

    @MartinB #734 MartinB:

    Ich weiß nicht, ob ich das Folgende hier schon einmal geschrieben habe. Aber bei einem so langen Thread ist ein bisschen Wiederholung vielleicht nicht so schlimm.

    Es gibt jede Menge Wege von Berlin nach München die nicht über Braunschweig gehen und die länger sind als jeder Weg von Berlin über Braunschweig nach München.

    Oder anders ausgedrückt: Du scheinst Geraden bzw. Geradenstücke und deren Eigenschaften als gegeben vorauszusetzen.

    Wenn man Geradenstücke nicht als “Ur-Objekte” des Modells ansieht die nicht zu definieren sind, dann stellt sich die Frage, wie man sie definiert. Ich sehe da im Wesentlichen zwei Möglichkeiten: Entweder darüber, dass auf ihnen die Beschleunigung gleich Null ist, oder darüber, dass sie bei festem Anfangs- und Endpunkt extremale Länge haben.

    Im zweiten Fall ist die Aussage in der Tat ziemlich trivial (das Infimum einer Menge reeller Zahlen ist kleiner oder gleich dem Infimum einer Teilmenge). Aber dann ist meiner Ansicht nach aus der Beschreibung der Situation der beiden Zwillinge nicht direkt klar, dass die beiden sich auf solchen Geradenstücken bewegen (also vor allem Teresa; Serena kann sich ja auch anders bewegen).

    Es heißt ja nicht: “Teresa und Serena bewegen sich so durch die Raumzeit, dass sie sich zwei Mal treffen. Und zwischen diesen Treffen bewegt sich Teresa auf einem Weg maximaler Eigenzeit, während Serena einen anderen Weg wählt.” Bei dieser Beschreibung wäre sofort klar, dass für Teresa mehr (oder jedenfalls nicht weniger) Eigenzeit vergeht. (Allerdings ist nicht unmittelbar klar, dass solche Wege tatsächlich möglich sind, und wie sie ggf. physikalisch zu realisieren sind.)

    Stattdessen heißt es “Während Teresa auf der Erde bleibt, macht sich Serena auf eine Reise nach Alpha Centauri …”. Und dass das “auf der Erde bleiben” einem Geradenstück (im Sinne extremaler Eigenzeit) in der Raumzeit entspricht, ist meiner Meinung nach nicht trivial.

  743. #744 MartinB
    14. Januar 2020

    @Tox
    Physik ist ja keine axiomatische Wissenschaft. Die Aussagen
    Die Eigenzeit entlang der Weltlinie ist maximal
    Die Weltlinie ist gerade
    Die Weltlinie entspricht einer unbeschleunigten Bewegung
    sind alle äquivalent und es ist letztlich vollkommen egal, welche davon du als fundamental ansiehst. (Genau das habe ich in meinem Buch in Kapitel 11 ausgenutzt, damit Isaac die ART ohne den Begriff Schwerkraft entdecken kann.)

    “dass das “auf der Erde bleiben” einem Geradenstück (im Sinne extremaler Eigenzeit) in der Raumzeit entspricht, ist meiner Meinung nach nicht trivial.”
    Nein, das ist nicht von vornherein trivial – aber wenn man erstmal das Bild der Raumzeit hat, dann ist es ja zwingend so , und genau das versuche ich ja hier zu sagen.

    Aber vielleicht hast Du recht und all die “Die Beschleunigung ist entscheidend”-Leute wollen eigentlich sagen: “Entscheidend ist, dass in der Raumzeit eine Linie maximaler Eigenzeit unbeschleunigt ist.”

  744. #745 Gebhard Greiter
    14. März 2020

    @Anonym_2019 #570:
    Wie wenig Sinn machend Einsteins Definition der Gleichzeitigkeit ist, wird sehr schön klar aus Fußnote 64 in Carlo Rovelli: Die Ordnung der Zeit (2018).

    Man liest dort:

    Das klassische Argument für die Idee des Blockuniversums liefert der Philosoph Hilary Putman in seinem viel zitierten Aufsatz “Time and Physical Geometry” in the Journal for Philosophy 64 (1967), S. 240-247. Putman nutzt Einsteins Definition der Gleichzeitigkeit [aus der sich ergibt]:

    Wenn sich die Erde und Proxima B einander nähern, ist ein Ereignis A auf der Erde (für Erdbewohner) gleichzeitig mit einem Ereignis B auf Proxima, das seinerseits (für Beobachter auf Proxima) gleichzeitig mit einem Ereignis C auf der Erde ist, das in der Zukunft von A liegt.

    Putman vermutet, das “gleichzeitig sein” “jetzt real sein impliziert” und schließt daraus, dass die zukünftigen Ereignisse (wie C) jetzt schon real seien.

    Der Fehler solcher Überlegung, schreibt Rovelli, liege darin, anzunehmen, dass Einsteins Definition der Gleichzeitigkeit ontologisch bedeutsam sei, während sie aber nur eine Behelfsdefinition darstellt. Sie dient dazu, einen relativistischen Begriff zu erklären, der sich in einer Näherung auf den nicht relativistischen reduziert. Die nicht relativistische Gleichzeitigkeit ist ein reflexiver und transitiver Begriff, die Einsteins dagegen nicht. Es macht daher keinen Sinn, anzunehmen, dass beide (außerhalb der nicht-relativistischen Näherung) dieselbe ontologische Bedeutung haben.

    Diese Einsicht wird in Fußnote 65 noch ergänzt. Man liest dort:

    Das Argument, wonach die physikalische Entdeckung, dass der Präsentismus unmöglich sei, beinhaltet, dass die Zeit eine Illusion ist. […] Der Irrtum besteht nach wie vor darin, die Zeit als einen einheitlichen konzeptuellen Block zu definieren, den es entweder vollständig oder überhaupt nicht gibt Eine klare Darstellung hierzu gibt Mauro Dorato in: Che cos’è il tempo? Einstein, Gödel e l’esperienza comune (Italiano, 2013, S. 77).

    Es bestätigt sich hier mein (Gebhard Greiters) Eindruck, dass Einsteins Definition der Gleichzeitigkeit wertlos ist. Es gibt einfach nur Gleichzeitigkeit im Sinne der stets relativen Zeit des Beobachters.

  745. #746 Anonym_2020
    14. März 2020

    @Gebhard Greiter (14. März 2020)

    Wenn sich die Erde und Proxima B einander nähern, ist ein Ereignis A auf der Erde (für Erdbewohner) gleichzeitig mit einem Ereignis B auf Proxima, das seinerseits (für Beobachter auf Proxima) gleichzeitig mit einem Ereignis C auf der Erde ist, das in der Zukunft von A liegt.

    Das ist falsch. Ob sich Erde und Proxima B annähern, ist für die Gleichzeitigkeit irrelevant. Eine Geschwindigkeit wird dargestellt durch die Steigung einer Weltlinie im Minkowski-Diagramm. Es gibt weder “ruhende” noch “bewegte” Ereignisse, weil diese durch Punkte im Minkowski-Diagramm dargestellt werden, nicht durch Weltlinien.

    Die nicht relativistische Gleichzeitigkeit ist ein reflexiver und transitiver Begriff, die Einsteins dagegen nicht.

    Das ist auch falsch. Einsteins Gleichzeitigkeit ist reflexiv und transitiv. Die nicht-relativistische Gleichzeitigkeit dagegen wurde experimentell widerlegt.

    Es bestätigt sich hier mein (Gebhard Greiters) Eindruck, dass Einsteins Definition der Gleichzeitigkeit wertlos ist.

    Ich empfehle folgendens Büchlein zum besseren Verständnis der RT und für den Zeitbegriff in der Physik insbesondere den § 8:
    https://archive.org/details/berdiespezielle00unkngoog/page/n22/mode/2up

  746. #747 Anonym_2020
    14. März 2020

    @Gebhard Greiter (14. März 2020)

    Korrektur zu Anonym_2020 (14. März 2020)

    Wenn sich die Erde und Proxima B einander nähern, ist ein Ereignis A auf der Erde (für Erdbewohner) gleichzeitig mit einem Ereignis B auf Proxima, das seinerseits (für Beobachter auf Proxima) gleichzeitig mit einem Ereignis C auf der Erde ist, das in der Zukunft von A liegt.

    Das ist doch richtig. Das liegt an der Relativität der Gleichzeitigkeit.Trotzdem gilt: Einsteins Gleichzeitigkeit ist in jedem Bezugssystem reflexiv und transitiv.

  747. #748 Jolly
    14. März 2020

    Wenn sich die Erde und Proxima B einander nähern, ist ein Ereignis A auf der Erde (für Erdbewohner) gleichzeitig mit einem Ereignis B auf Proxima, das seinerseits (für Beobachter auf Proxima) gleichzeitig mit einem Ereignis C auf der Erde ist, das in der Zukunft von A liegt.

    Das ist falsch.

    Äh, nein, Putnam hat recht.

    Für eine große Geschwindigkeit lässt sich das leicht an der Grafik oben nachvollziehen. Man ziehe z.B. eine waagrechte Linie vom Punkt ‘5’ zu dem Punkt neben ‘B’, was der Gleichzeitigkeit der Erdbewohner entspräche, und folge dann der bereits eingezeichneten blauen Gleichzeitigkeitslinie von dort zurück zur Zeitachse von A, die die Gleichzeitigkeit der Beobachter auf Proxima veranschaulicht.

  748. #749 Jolly
    Proxima B
    14. März 2020

    Ein Beweis für die Relativität der Gleichzeitigeit. – bei Kommentaren
    (bitte den Ort beachten)

  749. #750 Jolly
    14. März 2020

    @Gebhard Greiter

    Putman

    Sie verwechseln oft etwas, und das dann auch noch sehr hartnäckig, hier m und n. Der Gemeinte heißt Putnam.

    Es bestätigt sich hier mein (Gebhard Greiters) Eindruck, dass Einsteins Definition der Gleichzeitigkeit wertlos ist.

    Hier verwechseln Sie offensichtlich wertlos mit wertvoll. Die Definition der Gleichzeitigkeit von Einstein hat in vielerlei Hinsicht wertvolle Dienste geleistet. Insbesondere wertvoll ist sie z.B. für die korrekte Auslegung ihres folgenden Satzes:

    Es gibt einfach nur Gleichzeitigkeit im Sinne der stets relativen Zeit des Beobachters.

    So ist es.

    Diese Einsicht hilft wiederum, das Zwillingsparadoxon zu verstehen und aufzulösen.

    Die nicht relativistische Gleichzeitigkeit ist ein reflexiver und transitiver Begriff, die Einsteins dagegen nicht.

    Die philosophische Frage, ob “nicht relativistische Gleichzeitigkeit” nicht ein ontologisch leerer Begriff ist, scheint mir eigentlich auch geklärt. Nicht nur mir. In der Verlagsbeschreibung zur ‘Die Ordnung der Zeit’ findet sich: “warum es im Universum keine allgemeine Gegenwart gibt”.

  750. #751 Anonym_2020
    14. März 2020

    @Gebhard Greiter (14. März 2020)

    siehe auch:

    According to Stein, Putnam presupposes notions that are simply not available in special relativity. He has failed to take note of the changed situation in that context, that “defniteness to the present” has to be replaced by “defniteness at a given space-time point”.

    Quelle:
    https://users.ox.ac.uk/~lina0174/kansas.pdf

  751. #752 Gebhard Greiter
    15. März 2020

    @Jolly #750:
    Danke für den Hinweis auf meinen Lese- und Tippfehler: In Rovellis Fußnote 64 steht tatsächlich “… liefert der Philosoph Hilary Putnam (1926-2016) in …“.

  752. #753 Gebhard Greiter
    15. März 2020

    @Jolly #750:

    Die Definition der Gleichzeitigkeit von Einstein hat in vielerlei Hinsicht wertvolle Dienste geleistet. Insbesondere wertvoll ist sie z.B. für die korrekte Auslegung ihres folgenden Satzes …

    Diese Ihre Aussage verstehe ich nicht. Gibt es noch ein zweites Beispiel? Anders gefragt: Worin genau sehen Sie denn den Wert?

  753. #754 Jolly
    16. März 2020

    @Gebhard Greiter

    Worin genau sehen Sie denn den Wert [der Definition der Gleichzeitigkeit von Einstein]?

    Diese ihre Frage verstehe ich nicht. Ich nehme doch an, Sie wissen um die Bedeutung der Gleichzeitigkeit für die Spezielle Relativitätstheorie und um deren wiederum in vielen Bereichen.

    Warum wollen Sie z.B. das Zwillingsparadoxon verstehen, wenn das keinen Wert für sie hat? Ohne die Definition der Gleichzeitigkeit zu verstehen, werden Sie bei der Erklärung über Bad nicht hinauskommen, nicht zu Ugly vorstoßen und sicher nie bei Good landen.

  754. #755 Peter Strohmayer
    16. März 2020

    In Anbetracht der maximalen Wirkungsausbreitung c=1 (repräsentiert zB in der Ausbreitung einer Signalfront von Licht) stellt sich die Frage, wie man auf die Idee kommen kann, zwei Ereignisse wären absolut (für alle wie immer bewegte Beobachter) gleichzeitig.

  755. #756 Gebhard Greiter
    16. März 2020

    @Jolly #754:
    Wie ich (und auch Sie) schon festgestellt haben:

    Es gibt einfach nur Gleichzeitigkeit im Sinne der stets relativen Zeit des Beobachters.

    Jeder Versuch, Gleichzeitigkeit irgendwie globaler zu definieren, kann nur unter ganz bestimmten Nebenbedingungen Sinn machen (und damit nur auf ganz spezifische Szenarien anwendbar sein). Das reduziert seinen Wert natürlich ganz gewaltig und ist nur etwas, das als Gleichzeitigkeit definiert wird.

  756. #757 Jolly
    16. März 2020

    @Gebhard Greiter

    Das [jeder Versuch, Gleichzeitigkeit irgendwie globaler zu definieren (als Einstein das gemacht hat)*] reduziert seinen Wert natürlich ganz gewaltig und ist nur etwas, das als Gleichzeitigkeit definiert wird.

    Sehe ich auch so.

    Wobei das Philosophieren über die Zeit, mit abweichenden Gleichzeitigkeitsvorstellungen, sicher noch einen Wert inne hat. Es soll dafür sogar schon mal einen Nobelpreis gegeben haben **. Das sollte man dann aber besser nicht hier diskutieren, eventuell dort.

    * Die Definition der Gleichzeitigkeit laut Einstein ist eigentlich schon global *** (und reflexiv ist sie sowieso).
    ** Henri Bergson, Nobelpreis für Literatur 1927
    *** wiktionary kennt tatsächlich Steigerungsformen: globaler; am globalsten.

  757. #758 Anonym_2020
    17. März 2020

    Der Philosoph Henri Bergson und der Physiker A. Einstein hatten 1922 eine öffentliche Diskussion über die Bedeutung von “Zeit”. Sie konnten sich nicht einigen.

    Philosophy, he modestly argued, still had a place. Einstein disagreed. He fought against giving philosophy (and by inference Bergson) any role in matters of time.

    Bergson temporarily had the last word during their meeting at Société française de philosophie. His intervention negatively affected Einstein’s Nobel Prize, which was given “for his services to theoretical physics, and especially for his discovery of the law of the photoelectric effect” and not for relativity.”

    Quelle:
    https://dash.harvard.edu/bitstream/handle/1/3210598/canales-einstein,bergsonandtheexperimentthatfailed%282%29.pdf?sequence=2

  758. #759 Peter Strohmayer
    17. März 2020

    Gebhard G: “Es gibt einfach nur Gleichzeitigkeit im Sinne der stets relativen Zeit des Beobachters.”

    Wenn man das Maß des Vergehens von Zeit bei verschiedenen Beobachtern (“relativ”?) und den (fehlenden) zeitlichen Abstand zweier entfernter Ereignisse aus der Sicht eines dieser Beobachter nicht unterscheidet bzw. mit dem Begriff “gleichzeitig” in einen Topf wirft, muss das Zwillingsparadoxon Mysterium bleiben.

  759. #760 Peter Strohmayer
    15. Dezember 2021

    Beschleunigt werden kann mE nur, was Masse besitzt. Eine sprunghafte („unendlich hohe“) Beschleunigung ist undenkbar (die Beschleunigung ist mathematisch eine Abfolge infinitesimaler endlicher Beschleunigungsschritte).

    Der Begriff der Beschleunigung kann mE nicht auf einen „Bewußtseinstransfer“ bezogen werden, weil der Inhalt einer Information keine Masse (und natürlich auch kein Alter) hat. Das Bekanntwerden oder Mitteilen einer Information ist keine „Geburt“, ab der sie altern könnte.

    Es ist mE verfehlt, die Mitteilung einer Information als deren instantane Beschleunigung (instantanen Übertritt in ein anderes Inertialsystem) aufzufassen bzw. den Inhalt einer Information in einem Bezugssystem “ruhen” oder in ein anderes Bezugssystem “wechseln” (und dort ruhen) zu lassen. Die angebliche Irrelevanz der Beschleunigung wird also mit einem Vorgang erklärt, den es weder physikalisch noch nach den Denkgesetzen geben kann. Mit dieser Methode kann man jede beliebige These verteidigen, “einen Unsinn mit einem anderen untermauern”.

    Der Effekt der Zeitdilatation bei Rückkehr zum Zwillingsbruder wäre ohne Beschleunigung einer Masse unverständlich. Das langsamere Altern eines beschleunigten Massepunktes im Vergleich zu einem nicht beschleunigten ist dadurch zu erklären, dass der erwähnte infinitesimale Beschleunigungssprung in ein anderes (schnelleres) Bezugssystem immer bei einer Uhr eintrifft, deren Stand gegenüber der Uhr, die der Beschleunigte mit sich führt („Eigenzeit“) nach den Grundsätzen der L-T immer vorgehen muss (obwohl die Uhren aller Systeme bei Ursprungsdeckung auf Null gestellt wurden).

    Das Ausmaß des jünger Bleibens (des Zurückbleibens der Eigenzeit) ergibt sich, indem die einzelnen Zeitvorsprünge im Zuge der Beschleunigung über die Beschleunigungszeit aufintegriert werden. Ist tA die Lebenszeit des daheim gebliebenen Zwillings und tB die eines gleichförmig beschleunigten Zwillings, so ergibt sich tA=sinh(a*tB)/a (a ist die Beschleunigung in c/s2; c=1).

    Bei einer Beschleunigung von 0,000.000.032.72 c/s2 (das entspricht der Erdbeschleunigung g von 9,81 m/s2) bliebe der Beschleunigte – hoffentlich habe ich mich nicht verrechnet – pro Tag (tB=86400 Sekunden) um 0,1 Sekunden jünger als sein unbeschleunigter Bruder.

    Merkwürdig ist, dass ein Bewohner auf der Erdoberfläche nicht auch pro Tag um 0,1 sec jünger bleibt als ein außerhalb des Gravitationsfeldes lebender. Immerhin soll es sich nach dem Äquivalenzprinzip um eine vergleichbare Situation handeln. Der Effekt der langsameren Alterung im Gravitationsfeld ist kleiner; er soll auch nicht von der Beschleunigung, sondern vom Energiepotential abhängen. Das muss also ein ganz anderer Vorgang sein als der oben nach SRT beschriebene. Hier (bei der ART) ist mir alles unverständlich, aber ich habe ja das Buch noch nicht fertig gelesen.

  760. #761 MartinB
    18. Dezember 2021

    @Peter
    Hat etwas gedauert, sorry.

    Der Bewusstseinstransfer ist nur eine Metapher, meinetwegen können die Raumschiffe auch Fackeln dabeihaben und wenn sie sich begegnen, auch vom einen das Olympische Feuer aufs andere übertragen.
    Esgeht um folgendes: Wenn in einem solchen Fall exakt dasselbe herauskommt wie bei einer echten Umkehr eines Objekts, dann macht das sehr stark deutlich, dass die Beschleunigung nicht der entscheidende Punkt ist.
    Mathematisch sieht man das ohnehin sofort, wenn man einfach Distanzen in der Raumzeit anguckt, es geht ja nur darum, das auch intuitiv besser einsehbar zu machen.

    Ich habe das mit der Beschleunigung jetzt nicht nachgerechnet, insofern kann ich nicht bestätigen, dass du richtig gerechnet hast.

  761. #762 MartinB
    18. Dezember 2021

    Nachtrag: Ich bin mir ziemlich sicher, dass man das mit der Beschleunigung nach Äq-Prinzip so nicht machen kann für den vergleich Erdoberfläche/Weltall, das sollte sich nicht in der Weise aufintegrieren lassen.

  762. #763 Karl-Heinz
    Graz
    20. Dezember 2021

    @Peter Strohmayer

    Gravitative Zeitdilatation
    Zeitdilatation zwischen weit entferntem
    Beobachter und einer Uhr in einem Gravitationsfeld von einem g. Man beachte, dass der weit entfernte Beobachter immer in Ruhe bezüglich der Uhr im Gravitationsfeld ist.

    Zeitdilatation durch Beschleunigung
    Hier nehme ich an, dass die Uhr gegenüber dem ruhenden Beobachter mit genau 1 g beschleunigt wird. Man beachte, dass die Uhr gegenüber dem ruhenden Beobachter an Geschwindigkeit mit der Zeit zunimmt.

    Vergleich der Zeitdilatation der beiden Effekte
    Die Zeitdilatation durch relative Geschwindigkeit bei der beschleunigten Bewegung vermeide ich, indem ich die Anfangsgeschwindigkeit 0 setze, mit anderen Worten die Zeitdilatation durch Beschleunigung nur zu Beginn betrachte.

    Vergleiche ich jetzt die Gravitative Zeitdilatation mit der Zeitdilatation durch Beschleunigung für sehr kleine Zeitintetvalle kommt natürlich das gleiche raus. 🙂

  763. #764 Karl-Heinz
    Graz
    20. Dezember 2021

    @763

    Upss …
    Doch ein Blödsinn von mir.
    Bitte vergessen, sorry.

  764. #765 Karl-Heinz
    Graz
    20. Dezember 2021

    Aber einen Zusammenhang habe ich gefunden.
    Zeitdilatation von einem Bezugsystem zum anderen bzw. von einem Potential zum anderen.
    Muss das mathematisch mal gescheit ausformulieren. 🙂

  765. #766 MartinB
    20. Dezember 2021

    @Karl-Heinz
    Ich denke schon, dass das auf die Weise gehen sollte, habe es gestern aber nur sehr kurz versucht und dabei fehlte mir immer irgendwo ein Quadrat in meiner Taylor-Reihe.
    Ich bin mir ziemlich sicher, wir hatten hier ne ähnliche Diskussion schon mal, in der jemand sogar Einsteins Originalarbeiten herangezogen hat, vielleicht sogar hier in diesem Kommentarthread.

  766. #767 Anonym_2021
    21. Dezember 2021

    @MartinB (20. Dezember 2021) #766
    CC: @Karl-Heinz

    “Ich bin mir ziemlich sicher, wir hatten hier ne ähnliche Diskussion schon mal, in der jemand sogar Einsteins Originalarbeiten herangezogen hat, vielleicht sogar hier in diesem Kommentarthread.”

    Der Link steht im Kommentar #240. Die deutsche Version ist allerdings verschwunden (wegen Urheberrecht?). Hier steht die englische Übersetzung:

    https://en.wikisource.org/wiki/Translation:Dialog_about_Objections_against_the_Theory_of_Relativity

  767. #768 MartinB
    22. Dezember 2021

    @Anonym
    Danke.

  768. #769 Peter Strohmayer
    Wien
    22. Dezember 2021

    Vielen Dank für den Hinweis. A.E. bejaht die Frage nach der Äquivalenz von (Rückstoß)Beschleunigung (träge Masse) und Gravitationsbeschleunigung (Gewicht). Er setzt das Zurückbleiben der Uhr 2 aus Sicht des Systems K (Beschleunigung) gleich mit dem Zurückbleiben der Uhr 2 aus Sicht des Systems K (Gravitation). Auch Martin betont die lokale Geltung des Äquivalenzprinzips (#276 und #294).

    Das Folgende ist wahrscheinlich naiv, aber weiter bin ich bisher nicht gekommen: Ich sehe keinen Ansatzpunkt für diese Äquivalenz bzw. die Gleichheit der Zeitdilatation bei Beschleunigung und bei Ruhe im Gravitationsfeld. Ein einzelnes Lichtquant gewinnt beim Aufstieg von der Erdoberfläche potentielle Energie auf Kosten seiner Frequenz (Rotverschiebung). Dies – wie beim transversalen Dopplereffekt – als „Verlangsamung“ der Uhr auf der Erdoberfläche zu deuten, ist ohnehin problematisch (niemand würde auf die Idee kommen, bei einem sich entfernenden Folgetonhorn ein langsameres Vergehen von Zeit zu vermuten). Dazu kommt: Wenn der Beobachter auf der Erdoberfläche pro Sekunde ein Photon (einen Lichtpuls) aussendet, dann kommt oben bei einem Beobachter auf einer Geodäte, der praktisch außerhalb des Gravitationsfeldes situiert ist, ebenfalls jede Sekunde ein Photon (nur eben rotverschoben) an, weil die Bedingungen der Ausbreitung für jedes Photon gleich bleiben. Die Signalfrequenz ist aus der Sicht beider Beobachter gleich. Die Dauer der Aussendung und des Empfangs einer Signalfolge sind bei den Beobachtern gleich. Ihre Eigenzeiten vergehen gleich. Der Beobachter auf der Erdoberfläche bleibt nicht jünger, der Beobachter oben altert nicht schneller, nur weil die von ihnen ausgesendeten Photonen ihre Energiebilanz ändern. Ein (zB durch Rückstoß) beschleunigter träger Massepunkt altert langsamer, ein äquivalent schwerer Massepunkt nicht. (Natürlich habe ich etwas nicht bedacht, aber was?).

  769. #770 Anonym_2021
    23. Dezember 2021

    @Peter Strohmayer (22. Dezember 2021) #769

    “Wenn der Beobachter auf der Erdoberfläche pro Sekunde ein Photon (einen Lichtpuls) aussendet, dann kommt oben bei einem Beobachter auf einer Geodäte, der praktisch außerhalb des Gravitationsfeldes situiert ist, ebenfalls jede Sekunde ein Photon (nur eben rotverschoben) an, weil die Bedingungen der Ausbreitung für jedes Photon gleich bleiben.”

    Nein. Dass Uhren schneller gehen, wenn sie höher im Gravitationspotential sind, ist experimentell nachgewiesen worden, z.B.durch Vergleich mit Bodenuhren beim das Hafele-Keating-Experiment.

    Quelle:
    https://www.cosmos-indirekt.de/Physik-Schule/Hafele-Keating-Experiment

    Das hat nichts mit der unterschiedlichen Stärke des Gravitationsfeldes bei den beiden unterschiedlich hohen Uhren zu tun. Entscheidend ist der Unterschied im Gravitationspotential:

    https://www.spektrum.de/lexikon/physik/gravitationspotential/6096

  770. #771 MartinB
    23. Dezember 2021

    @Peter
    “Wenn der Beobachter auf der Erdoberfläche pro Sekunde ein Photon (einen Lichtpuls) aussendet, dann kommt oben bei einem Beobachter auf einer Geodäte, der praktisch außerhalb des Gravitationsfeldes situiert ist, ebenfalls jede Sekunde ein Photon (nur eben rotverschoben) an, weil die Bedingungen der Ausbreitung für jedes Photon gleich bleiben. Die Signalfrequenz ist aus der Sicht beider Beobachter gleich”

    Nein. Wenn die Photonen rotverschoben werden, dann auch die Signalabstände. Stell dir vor, du würdest eine em-Welle mit einer Frequenz von 1 Hz aussenden, und außerdem genau dann, wenn die Welle ihr Maximum hat, zusätzlich einen kurzen Puls.
    Welle und Puls laufen auf derselben Weltlinie, der Puls muss sich also genau so bewegen wie das Maximum der Welle. Wenn die Maxima der Welle außerhelb des Schwerefelds rotverschoben sind, also größere Abstände haben, dann muss das logischerweise auch für die Pulse gelten.
    MaW: Wenn Photonen rotverschoben sind, dann muss auch die Zeit insgesamt langsamer vergehen, sonst ergibt sich eine Inkonsistenz.

  771. #772 Peter Strohmayer
    26. Januar 2022

    @Martin
    “Wenn Photonen rotverschoben sind, dann muss auch die Zeit insgesamt langsamer vergehen, sonst ergibt sich eine Inkonsistenz.”

    Könnte nicht umgekehrt die Deutung der gravitativen Rotverschiebung als Folge einer Verlangsamung des Zeitablaufs inkonsistent sein?

    In #760 wird langsameres Altern bei (Rückstoß)beschleunigten bejaht, bei Erdbeschleunigten auf der Erdoberfläche eher verneint.

    Wer eine vom Gravitationspotential abhängige Rotverschiebung als “langsameres Altern” bei der Lichtquelle deutet, muss in Anbetracht der (bei Erdbeschleunigung = Rückstoßbeschleunigung) um Potenzen anderen Werte zugestehen, dass es sich nicht um den gleichen physikalischen Vorgang handeln kann. Gleichwohl ist die Behauptung, die gravitative Rotverschiebung sei eine Folge langsamer gehender Uhren auf der Erdoberfläche und dieses langsamer Gehen sei eine Folge der Gravitation (aber eben eine andere als die des Wechsels inerter Bezugssysteme bei einer Rückstoßbeschleunigung), isoliert betrachtet kaum zu widerlegen.

    Nun soll aber die Situation in einem Gravitationsfeld zumindest lokal einer Rückstoßbeschleunigung äquivalent sein. (Zwar könnte man an dieser Äquivalenz in Anbetracht der unterschiedlichen Folgen für die Alterung Zweifel anmelden, aber halten wir einstweilen an ihr fest.) Stellen wir uns zwei Massepunkte in bestimmter Anfangsentfernung vor, die zugleich in jeweils gleichem Ausmaß beschleunigt werden. Aus der Warte des in Fahrtrichtung vorderen Massepunktes sind dann die vom hinteren Massepunkt ausgesendeten Photonen rotverschoben. Ist deshalb die Zeit beim hinteren Massepunkt langsamer vergangen?

    Ich meine, nein (dass der Faden zwischen ihnen reisst ist eine andere Sache). Die Zeit ist bei beiden Massepunkten gegenüber dem ursprünglichen Ruhesystem im gleichen Ausmaß langsamer vergangen (“Zwillingsparadoxon”), daher kann auch im direkten Vergleich der beiden Massepunkte kein unterschiedliches Vergehen von Zeit behauptet werden. Es wäre auch eigenartig, wenn man bei einem allein beschleunigten Massepunkt ein langsameres Altern auf Grund nicht extremaler Eigenzeit errechnet hat, diese Rechnung aber in Frage stellen müsste, nur weil vor oder hinter dem Massepunkt andere Massepunkte in bell’scher Weise beschleunigt werden.

    “Nein. Wenn die Photonen rotverschoben werden, dann auch die Signalabstände.” Auch dies ist bei der synchronen Rückstoßbeschleunigung zweier entfernter Massepunkte nicht leicht einzusehen. Jedes der hintereinander ausgesendeten Photonen findet (bis auf den langsam sich vergrößerten Abstand zwischen den Massepunkten, der aber vernachlässigt werden kann) die gleichen Verhältnisse und Laufzeiten vor. Die Rotverschiebung bei den einzelnen Lichtquanten hat offenbar rein energetische Ursachen. Sie ist offenbar nicht die Folge eines mit Signalverschiebungen einhergehenden Dopplereffekts.

  772. #773 MartinB
    26. Januar 2022

    @Peter
    “Die Zeit ist bei beiden Massepunkten gegenüber dem ursprünglichen Ruhesystem im gleichen Ausmaß langsamer vergangen (“Zwillingsparadoxon”), daher kann auch im direkten Vergleich der beiden Massepunkte kein unterschiedliches Vergehen von Zeit behauptet werden.”
    Das hängt davon ab, wie du welche Zeitpunkte miteinander vergleichst. Ich empfehle, ein Raumzeitdiagramm zu zeichnen, dann siehst du das Problem – für den einen Beobachter scheinen beide immer genau identisch langsamer zu werden, für den anderen nicht, weil der ein anderes Bezugssystem hat und in dem der eine schneller altert als der andere.

    Unterschied zur Gravitation: Dort ist eindeutig, wessen Zeit langsamer vergeht.

    Im übrigen habe ich nach wie vor keine wirkliche Ahnung, was du hier eigentlich zu argumentieren versuchst. Dass die Interpretation eines langsameren Zeitverlaufs im Gravitationsfeld falsch ist? Wie deutest du es dann, dass aus der Warte des Senders, der z.B. auf der Oberfläche eines Neutronensterns sitzt, nach seiner Uhr Signale im Sekundentakt ausgesandt werden, nach der Uhr eines weiter entfernten Beobachters aber langsamer? Ist das hier mehr als ein Diskutieren über Begrifflichkeiten?

  773. #774 Peter Str
    26. Januar 2022

    @Martin “Ist das hier mehr als ein Diskutieren über Begrifflichkeiten?”
    Ich vermute, dass die Interpretation der Rotverschiebung als langsamerer Zeitverlauf im Gravitationsfeld falsch sein könnte und dass sich im Gravitationsfeld – gleich wie im Bereich zwischen zwei beschleunigten Massepunkten – aus der Sicht verschiedener Potentiale nur die Energie (die Frequenz) eines Lichtquants h verändert.

    @Martin: “Unterschied (der rückstoßbeschleunigten Massepunkte) zur Gravitation: Dort ist eindeutig, wessen Zeit langsamer vergeht.”

    Auch bei den bell’schen Raumschiffen wäre es für beide Beobachter in den Raumschiffen auf Grund der so gedeuteten Rot- bzw. Blauverschiebung eindeutig, dass der hintere langsamer altert als der Vordere.

    Es fragt sich aber, ob diese Deutung in Anbetracht des gleichen Alterns zweier gleich beschleunigter Drillinge gegenüber dem daheimgebliebenen Drilling sinnvoll ist. Wenn sich die beiden Drillinge während ihrer Reise so positionieren, dass immer derselbe in Bezug auf die Beschleunigungsrichtung den Abschluss bildet, dann sind sie gegenüber dem Daheimgebliebenen genau im selben Ausmaß jünger. Dann kann aber beim hinteren die Zeit nicht langsamer vergangen sein als beim vorderen.

    Aber ich würde gerne Deinem Rat folgen:
    “Das hängt davon ab, wie du welche Zeitpunkte miteinander vergleichst. Ich empfehle, ein Raumzeitdiagramm zu zeichnen, dann siehst du das Problem – für den einen Beobachter scheinen beide immer genau identisch langsamer zu werden, für den anderen nicht, weil der ein anderes Bezugssystem hat und in dem der eine schneller altert als der andere.”

    Alle drei Beobachter befinden sich zu Beginn und am Ende der Reise im selben Bezugssystem. Ergibt sich aus den dazwischen angefertigten Raumzeitdiagrammen wirklich, dass die heimgekehrten zwei Drillinge untereinander nicht mehr gleich alt sind? Könntest Du bitte präzisieren, wer wann wen oder was beobachten soll (oder meinst Du die Diagramme der beschleunigten Beobachter aus Deinem Artikel “Reisst der Faden”)?

  774. #775 MartinB
    26. Januar 2022

    “oder meinst Du die Diagramme der beschleunigten Beobachter aus Deinem Artikel “Reisst der Faden”
    Ja, das meinte ich.
    Und ich finde die ganze Debatte ehrlich gesagt etwas müßig. Wir wissen, wer was beobachtet und sind uns darüber hoffentlich einig. Ob nun der, der auf dem Neutronenstern “wirklich” langsamer altert oder nicht – wen kümmert’s?
    Wenn er vom Stern aus startet und seinen Zwilling im All trifft, ist er jünger, also ist er – nach meiner Sprechweise – langsamer gealtert, mehr gibt es da eigentlich in meinen Augen nicht zu sagen.

  775. #776 Peter Strohmayer
    26. Januar 2022

    @Martin
    “Wenn er vom Stern aus startet und seinen Zwilling im All trifft, ist er jünger” – da machst Du aber ein neues Fass auf – von Reisenden war nie die Rede.
    Eins aber bleibt: wir werden alle älter.
    Liebe Grüße!

  776. #777 MartinB
    27. Januar 2022

    @Peter
    Nein, aber das ist doch die eindeutige Möglichkeit, nachuweisen, dass die zeit im Gravitationsfeld langsamer vergeht, wenn dir die anderen Argumente (warum auch immer) nicht überzeugend genug sind.
    Wir fliegen zum Neutronenstern, ich lande auf der Oberfläche, du nicht, ich starte wieder, wir vergleichen unsere Uhren.
    Das machen wir mehrfach, wobei ich darauf achte, dass Lande- und Startvorgang immer exakt gleich ablaufen, aber jedes Mal verweile ich eine andere zeit auf dem Stern.
    Dann sehen wir, dass der Unterschied zwischen unseren Uhren proportional zu meiner Verweildauer auf dem Stern ist; die Reise kann (da jedesmal identisch) nicht verantwortlich sein, also vergeht die Zeit auf dem Stern langsamer.

  777. #778 Anonym_2022
    27. Januar 2022

    @Peter Strohmayer (26. Januar 2022) #772

    “Stellen wir uns zwei Massepunkte in bestimmter Anfangsentfernung vor, die zugleich in jeweils gleichem Ausmaß beschleunigt werden. Aus der Warte des in Fahrtrichtung vorderen Massepunktes sind dann die vom hinteren Massepunkt ausgesendeten Photonen rotverschoben. Ist deshalb die Zeit beim hinteren Massepunkt langsamer vergangen?”

    Langsamer als was? Im Vergleich zur Koordinatenzeit des beschleunigten Bezugssystems, in dem der vordere Massenpunkt ruht, ja!

    In diesem beschleunigten Bezugssystem gibt es Pseudogravitation mit einer gravitativen Zeitdilatation.

    Wenn man denselben Vorgang in einem Inertialsystem beschreibt, liegt die Rotverschiebung dagegen am Dopplereffekt. Während das Photon unterwegs ist, ändert sich die Geschwindigkeit des vorderen Massenpunktes.

    Man muss differenzieren zwischen Zeitdilatation (Vergleich einer Eigenzeit mit einer Koordinatenzeit) und unterschiedlicher Alterung (Vergleich zweier Eigenzeiten beim Treffen der Uhren – Zwillings”paradoxon”).

  778. #779 Peter Strohmayer
    28. Januar 2022

    @Martin
    Beim vorgeschlagenen Experiment sollen Beschleunigungsvorgänge, die beim “Zwillingsparadoxon” eine Rolle spielen, in einem Gravitationsfeld stattfinden. Ich muss zugeben, dass ich bei dieser zusätzlichen Voraussetzung nicht in der Lage bin, die Konsequenzen auf Zeitabläufe zu überblicken.

    Vielleicht ist es aber legitim, zunächst nicht einen Massepunkt, sondern nur einen Lichtpuls zu beurteilen, der im Gravitationsfeld des Neutronensterns aufsteigt und nach Reflexion an einem Spiegel (dessen Bewegungszustand keine Rolle spielt) zu seiner Oberfläche zurückkehrt. Ich frage mich, ob in jedem Fall die “Halbzeitreflexion” gilt, ob also aus der Sicht des Beobachters auf dem Stern Hinweg bzw. -zeit des Lichtpulses gleich sind wie Rückweg bzw. -zeit. Bei der Laser-Messung der Entfernung Erde – Mond wird mE von dieser Gleichheit ausgegangen. Zu Recht? Immerhin kann bei den bell’schen Raumschiffen im Verhältnis der jeweiligen Lichtpulse der beiden beschleunigten Beobachter offenbar keine “Halbzeitreflexion” gelten.

    @Anonym
    Die Koordinatenzeit aus der Sicht des vorderen beschleunigten Beobachters ist dessen Eigenzeit, die Koordinatenzeit aus der Sicht des hinteren beschleunigten Beobachters ist dessen Eigenzeit. Meine Frage bezog sich auf einen Vergleich dieser beiden Eigenzeiten (der angeblich unterschiedliche “Alterung” der Beobachter).

    Beim Zwillingsparadoxon werden auch zwei Eigenzeiten miteinander verglichen, nämlich die des umbeschleunigten Daheimgebliebenen mit der des beschleunigten Reisenden.

    Der Vergleich zweier Zeiten ist immer dasselbe, gleich ob man sie aus eigener Warte als eigene Koordinatenzeit oder fremde Eigenzeit bezeichnet. Aber natürlich ist bloße Zeitdilatation (bei unbeschleunigten Reisenden) und unterschiedliche Alterung (beim Zwillingsparadoxon) nicht das gleiche (obwohl die Rolle der Beschleunigung bei letzterem manchmal in Abrede gestellt wird, #735ff).

  779. #780 Peter Strohmayer
    28. Januar 2022

    Nachtrag @Anonym: Wenn man will, kann man die Eigenzeit als die im Ursprung des eigenen Koordinatensystems gegebene Zeit von Fällen eigener Koordinatenzeiten unterscheiden, die an vom eigenen Ursprung entfernten Orten gegeben sind.

  780. #781 Anonym_2022
    28. Januar 2022

    @Peter Strohmayer (28. Januar 2022) #780

    “Wenn man will, kann man die Eigenzeit als die im Ursprung des eigenen Koordinatensystems gegebene Zeit von Fällen eigener Koordinatenzeiten unterscheiden, die an vom eigenen Ursprung entfernten Orten gegeben sind.”

    Die Zeitdilatationsformel – siehe Kottler-Møller coordinates (2b) – für den hinteren Massenpunkt lautet, wenn der vordere Massenpunkt bei x=0 ruht und die x-Achse in Beschleunigungsrichtung zeigt:

    dτ = dt √{ (1+gx/c²)² – v²/c² }

    Die x-Koordinate des hinteren Massenpunkts ist dann negativ.

  781. #782 Anonym_2022
    28. Januar 2022

    @Peter Strohmayer (26. Januar 2022) #772

    Ergänzung zu #778 und #781

    “Ist deshalb die Zeit beim hinteren Massepunkt langsamer vergangen?”

    Die Zeitdilatationsformel in #781 kann man vereinfacht aus dem Dopplereffekt herleiten. Diese entspricht daher der Rotverschiebung.

    Mein Bezugssystem ist ein Inertialsystem, in dem die beschleunigte Rakete ruhte, als der hinter Massenpunkt das Photon nach vorne abgesendet hat.

    Das Photon ist für die Zeitdauer
    Δt = h/c
    unterwegs, wenn “-h” die (negative) x-Koordinate des hinteren Massenpunktes ist.

    In dieser Zeit wird der vordere Massenpunkt auf die Geschwindigkeit
    v = g * Δt = (g*h)/c
    beschleunigt.

    Wenn diese Geschwindigkeit klein ist, kann man die klassische Dopplerformel anwenden:

    f/f₀ = 1-v/c = 1- (g*h)/c².

    Das ist gleich der Formel in #781 für x=-h und v=0.

    Der hintere Massenpunkt ruht ja im beschleunigten Bezugssystem. Er ruhte beim Aussenden des Photons auch im Inertialsystem.

    Für den vorderen Massenpunkt befindet der hintere Massenpunkt in einem um g*h niedrigeren Gravitationspotential.

  782. #783 MartinB
    28. Januar 2022

    @Peter
    “Beim vorgeschlagenen Experiment sollen Beschleunigungsvorgänge, die beim “Zwillingsparadoxon” eine Rolle spielen, in einem Gravitationsfeld stattfinden. Ich muss zugeben, dass ich bei dieser zusätzlichen Voraussetzung nicht in der Lage bin, die Konsequenzen auf Zeitabläufe zu überblicken.”
    Halte die Beschleunigung einfach klein, variiere sie nach Belieben, aber so, dass die relativistischen Effekte durch die Geschwindigkeiten klein bleiben, und dann lässt sich das problemlos trennen.
    Man kann das sicher auch durchrechnen, wenn man Spaß hat…

    Und ja, bei der Reflektion muss Gleichheit gelten, mMn schon allein deshalb, weil alle involvierten Gleichungen Zeitumkehrinvariant sind…

    Nach wie vor weiß ich nicht, was eigentlich dein Problem ist….

  783. #784 Peter Strohmayer
    28. Januar 2022

    @ Martin
    Es ist kein Problem, sondern nur die Folge der Angewohnheit, dass ich mir, um zu verstehen, meiner Klugheit entsprechende Fragen stelle, und hoffe, dass jemand, der klüger ist als ich, sich ihrer annimmt, was Du glücklicherweise machst.

    “Halte die Beschleunigung klein …”:
    Ok, Maryland-Experiment. Das gilt offenbar auch im Verhältnis zwischen hyperbolisch beschleunigten Beobachtern: der hintere altert weniger schnell als der vordere, die zwei Drillinge kehren jünger zum dritten Drilling zurück, derjenige, der in der Rakete hinten saß, wäre aber noch ein wenig jünger. Das will ich einstweilen zur Kenntnis nehmen.

    “Bei der Reflexion muss Gleichheit gelten”: Diese Frage bewegt mich seit Langem. Ich hoffe, dass es so ist, wie Du sagst, denn es würde die Verhältnisse vereinfachen. Mich irritiert aber die gebogene Bahn eines horizontal ausgesendeten Lichtquants im beschleunigten Lift/im homogenen Gravitationsfeld, denn es kann, ganz gleich welchen Winkel ein Spiegel einnimmt, offenbar nicht auf gleicher Bahn gleich lange zu seinem Ausgangspunkt zurückkehren. Wenn es hier keine Gleichheit gäbe, kann es sie auch bei vertikaler Aussendung nicht geben.

    @Anonym
    Danke für das Aufzeigen der wissenschaftlichen Rahmenbedingungen. Sie entsprechen dem, was Martin in #773 vorgeschlagen hat. Auch aus Vergleich der Koordinatensysteme des vorderen und des hinteren Beobachters ergibt sich also eine Rotverschiebung. Die Frage, ob dieser Rotverschiebung eine unterschiedliche Alterung entspricht, wird damit nicht beantwortet. Es kann ja auch bei einem Dopplereffekt, der zwischen zwei zueinander gleichförmig bewegter Beobachter auftritt, nicht behauptet werden, dass einer anders altern würde als der andere. Aber freilich, das Experiment von Martin spricht eindeutig für eine unterschiedliche Alterung. Damit muss es einstweilen sein Bewenden haben.

  784. #785 MartinB
    29. Januar 2022

    @Peter
    Achtung: Das mit dem Lift darf man nie zu weit treiben, weil das Äquivalenzprinzip nur lokal gilt, d.h. für kleine Umgebungen eines Raumzeitpunkts. Ich weiß zwar nicht genau, wie du dir das Bild denkst, aber ich vermute, da steckt ein Problem.
    Das Argument mit der zeitumkehrinvarianz ist da mMn einfacher.

    Und nach wie vor verstehe ich nicht, was dein Ziel/Problem ist. Ich frage mal umgekehrt: Welche Eigenschaften müssen zwei Beobachter A und B haben, damit du akzeptierst, dass sie “unterschiedlich altern”, denn das scheint ja dein Problem zu sein.?

  785. #786 Anonym_2022
    29. Januar 2022

    @Peter Strohmayer (28. Januar 2022) #784

    “Es kann ja auch bei einem Dopplereffekt, der zwischen zwei zueinander gleichförmig bewegter Beobachter auftritt, nicht behauptet werden, dass einer anders altern würde als der andere.”

    1)
    Eine unterschiedliche Alterung kann nur beim Zwillings-Szenario vorliegen. Eine Zeitdilatation ist dagegen Bezugssystem-abhängig.

    2)
    Beim relativistischen transversalen Doppler-Effekt bei gleichförmiger Bewegung kann die Frequenzveränderung nur durch eine Zeitdilatation erklärt werden, weil es keine longitudinale Geschwindigkeit-Komponente gibt.

    Analog kann in dem beschleunigten Bezugssystem die Frequenzveränderung nur durch eine Zeitdilatation erklärt werden, weil es keine longitudinale Geschwindigkeit-Komponente zwischen den beiden Massenpunkten gibt.

    Die Erklärung ist allerdings eine andere im Inertialsystem (logitudinaler Dopplereffekt).

  786. #787 Peter Strohmayer
    29. Januar 2022

    @Anonym
    zu 1): Stimmt (sofern ich Dich richtig verstehe).
    zu 2): Du meinst, vom Sender orthogonal zu den parallelen Bewegungsachsen der Beobachter ausgesendete Lichtpulse würden beim Empfänger zu einem der Zeitdilatation entsprechenden Dopplereffekt führen (Rotverschiebung), da sich der Abstand zwischen Sender und Empfänger nicht ändert? Ist es aber nicht vielmehr so, dass sich die Frequenz erhöht (Blauverschiebung)? Demnach müsste die Zeit beim Sender schneller vergehen, was natürlich auch nicht stimmt. Was ist also mit der Zeitdilatation als angeblich alternativlose Erklärung für diesen Effekt? Diese Sprachgewohnheit mit dem anderen Vergehen von Zeiten ist jedenfalls in diesem Bereich unnütz und kontraproduktiv. – Die Frequenzänderung zwischen zwei gleich beschleunigten Beobachtern kann mM nach nicht mit dem transversalen Dopplereffekt verglichen werden.

    @Martin
    “Welche Eigenschaften müssen zwei Beobachter A und B haben, damit du akzeptierst, dass sie “unterschiedlich altern”, denn das scheint ja dein Problem zu sein?”

    Von zwei Drillingen, die nach einer Beschleunigungen mit bell’schen Raumschiffen von einer Reise zum dritten Drilling zurückkehren, müsste der hinten reisende noch etwas jünger sein als der vorne Reisende.

    Das kann mAn nicht sein. Ein Zwilling unterliegt einer exakten, seiner krummen Weltlinie entsprechenden anderen Alterung. Ob noch ein anderer in bell’scher Weise mit ihm beschleunigt wird oder nicht, und ob der vor ihm reist oder hinter ihm, kann den genannten exakten Wert nicht beeinflussen.

  787. #788 Anonym_2022
    29. Januar 2022

    @Peter Strohmayer (28. Januar 2022) #784

    Ergänzung:

    “Aber freilich, das Experiment von Martin spricht eindeutig für eine unterschiedliche Alterung.

    Ein weiteres Experiment, das gravitative Zeitdilatation mit dem Zwillingparadoxon verbindet, ist ein Handballspieler, der einen Ball mit hoher Anfangsgeschwindigkeit senkrecht nach oben wirft und ihn wieder auffängt.

    Der Ball muss stärker gealtert sei als der Handballspieler, da sich der Ball ohne Eigenbeschleunigung (=inertial) bewegt hat.

    Im (inertialen) Bezugssystem des Balls hat sich der Handballspieler langsamer werdend wegbewegt und dann schneller werdend zurück bewegt, mit einer SRT-Zeitdilatation.

  788. #789 Anonym_2022
    29. Januar 2022

    @Peter Strohmayer (29. Januar 2022) #787

    “zu 2): Du meinst, vom Sender orthogonal zu den parallelen Bewegungsachsen der Beobachter ausgesendete Lichtpulse würden beim Empfänger zu einem der Zeitdilatation entsprechenden Dopplereffekt führen (Rotverschiebung), da sich der Abstand zwischen Sender und Empfänger nicht ändert? Ist es aber nicht vielmehr so, dass sich die Frequenz erhöht (Blauverschiebung)?”

    Beim transversalen Doppler-Effekt wird das Licht im Bezugssystem des Empfängers orthogonal zur Bewegungsrichtung des Senders empfangen (Rotverschiebung, Faktor 1/γ).

    Im Bezugssystem des Senders beträgt der Winkel zwischen Licht-Richtung und Bewegungsrichtung des Empfängers dann nicht 90 Grad.

    In einem anderen Szenario, in dem der Winkel zwischen Licht-Richtung und Bewegungsrichtung des Empfängers im Bezugssystem des Senders 90 Grad beträgt, würde eine Blauverschiebung empfangen werden (Faktor γ).

    Dass der Winkel zwischen Lichtrichtung und x-Achse in den beiden Bezugssystem unterschiedlich ist (Aberration), sieht man anschaulich bei der bewegten Lichtuhr.

  789. #790 Peter Strohmayer
    31. Januar 2022

    @Anonym
    (Hommage an Martin’s Verfahren:)
    “SAN: Das bedeutet, dass über alle möglichen Einfallsrichtungen der von einem bewegten Sender stammenden Lichtpulse betrachtet der auf die Longitudinalkomponenten der Geschwindigkeit zurückzuführende Dopplereffekt durch die “Zeitdilatation” so überlagert wird, dass die Fälle, in denen Blauverschiebung eintritt, die Fälle, in denen Rotverschiebung eintritt, um den Lorentz-Faktor überwiegen (Frequenzerhöhungen aus Sicht des Empfängers, die dem Maß der ‘Massezunahme’ bewegter Körper entspricht).
    ISAAC: Das ist korrekt.
    SAN: Dann wird also die Dauer der Prozesse zwischen den Signalen beim Sender, die – neben den longitudinalen Geschwindigkeitskomponenten – den jeweiligen Dopplereffekt bestimmt, aus meiner Sicht durch die Relativbewegung verkürzt. An sich müssten aber die Prozesse, die sich beim Sender abspielen, verlangsamt erscheinen, also aus meiner Sicht länger dauern.
    ISAAC: Das ist allerdings richtig. Lassen Sie mich einen Moment überlegen.
    (Es vergehen einige Minuten.)
    Die überwiegend eintretenden Blauverschiebungen haben mit den unterschiedlichen Winkeln zu tun, mit denen Lichtpulse ausgesendet bzw. empfangen werden. Eines dieser Winkelverhältnisse ‘sieht man anschaulich bei der bewegten Lichtuhr’.
    SAN: Aber die übliche Erklärung zur bewegten Lichtuhr läuft doch darauf hinaus, dass der ruhende Beobachter die Bahn des Photons verlängert ‘sieht’, also muss die Frequenz aus seiner Sicht kleiner werden, wie man das zB in der Animation von Markus Pössel sehen kann (Rotverschiebung). Das mit den Winkeln hat mich verwirrt, jetzt verliere ich auch noch die Lichtuhr als festen Boden unter den Füßen.
    ISAAC: Ein Tor kann mehr fragen, als zehn Weise beantworten können.
    SAN: Sollte man nicht die verunglückten Redeweisen vom “Beobachten eines Photons” oder vom “langsameren Vergehen der Zeit aus Sicht des …” hinter sich lassen, um endlich Klarheit zu schaffen?
    ISAAC: Dafür bin ich nicht programmiert. Außerdem bin ich Eigentum der Universität.”
    [Aus den gelöschten Dateien Isaacs rekonstruiert und geleakt.]

  790. #791 Anonym_2022
    31. Januar 2022

    @Peter Strohmayer (31. Januar 2022) #790

    Sowohl der Winkel zwischen der Licht-Richtung und der Bewegungsrichtung Sender/Empfänger als auch die Zeitdilatation sind Bezugssystem-abhängig. Daher ist bei beiden immer das Bezugssystem mit anzugeben.

    Im Empfänger-Bezugssystem gilt:

    Der Sender ist bewegt und unterliegt einer Zeitdilatation. Dazu passend führt ein orthogonaler Lichtstrahl zu einer Rotverschiebung.

    Im Sender-Bezugssystem gilt:

    Der Empfänger ist bewegt und unterliegt einer Zeitdilatation. Dazu passend führt ein orthogonaler Lichtstrahl zu einer Blauverschiebung.

  791. #792 Peter Strohmayer
    31. Januar 2022

    @Anonym

    “Im Empfänger-Bezugssystem gilt:
    Der Sender ist bewegt und unterliegt einer Zeitdilatation. Dazu passend führt ein orthogonaler Lichtstrahl zu einer Rotverschiebung.”

    Richtig. (Nun musst Du Aussage “weil es keine longitudinale Geschwindigkeit-Komponente gibt” (#786) korrigieren, womit die dort gegebene Erklärung der Ursache der Rotverschiebung notleidend wird.)

    “Im Sender-Bezugssystem gilt:
    Der Empfänger ist bewegt und unterliegt einer Zeitdilatation. Dazu passend führt ein orthogonaler Lichtstrahl zu einer Blauverschiebung.”

    ME im zweiten Teil des letzten Satzes der Aussage inkonsequent (orthogonal aus Sicht des Senders stimmt noch), weil Du hier, um das gewünschte Ergebnis zu erhalten, ohne Berechtigung vom Sender-Bezugssystem in das Empfänger-Bezugssystem wechselst.

    Im Grunde hast Du aber mit diesen Sätzen ohnehin nur die Effekte noch einmal beschrieben, bist aber auf den Widerspruch, dass einmal bei gleichförmiger Bewegung eine (überwiegende) Blauverschiebung und einmal die gravitative Rotverschiebung auf eine “Verlangsamung der Zeit beim Sender” zurückzuführen sein soll, ist, nicht eingegangen.

  792. #793 Anonym_2022
    31. Januar 2022

    @Peter Strohmayer (31. Januar 2022) #792

    “(Nun musst Du Aussage “weil es keine longitudinale Geschwindigkeit-Komponente gibt” (#786) korrigieren, womit die dort gegebene Erklärung der Ursache der Rotverschiebung notleidend wird.)”

    Dazu sehe ich keinen Grund. Im Empfänger-Bezugssystem gibt es bei dem Szenario die longitudinale Geschwindigkeit-Komponente nicht.

    “ME im zweiten Teil des letzten Satzes der Aussage inkonsequent (…), weil Du hier, um das gewünschte Ergebnis zu erhalten, ohne Berechtigung vom Sender-Bezugssystem in das Empfänger-Bezugssystem wechselst.”

    Das Messergebnis des Empfängers (Empfangsfrequenz) kann dokumentiert und später dem Sender mitgeteilt werden. Messergebnisse sind grundsätzlich Bezugssystem-unabhängig.

    “Im Grunde hast Du aber mit diesen Sätzen ohnehin nur die Effekte noch einmal beschrieben, bist aber auf den Widerspruch, dass einmal bei gleichförmiger Bewegung eine (überwiegende) Blauverschiebung und einmal die gravitative Rotverschiebung auf eine “Verlangsamung der Zeit beim Sender” zurückzuführen sein soll, ist, nicht eingegangen.”

    Im ersten Szenario in #791, beschrieben im Empfänger-Bezugssystem, gibt es keine (überwiegende) Blauverschiebung.

  793. #794 Anonym_2022
    31. Januar 2022

    @Peter Strohmayer (31. Januar 2022) #792

    Ergänzung:

    Siehe die Doppler-Formel in Abhängigkeit des Winkel
    • im Empfängersystem (Eq. 6)
    • im Sendersystem (Eq. 7)

    https://en.wikipedia.org/wiki/Relativistic_Doppler_effect#Motion_in_an_arbitrary_direction

  794. #795 Peter Strohmayer
    31. Januar 2022

    @Anonym
    Danke für die Offenlegung Deiner Argumentationsquelle.

    “Im Empfänger-Bezugssystem gibt es bei dem Szenario [Rotverschiebung] die longitudinale Geschwindigkeit-Komponente nicht.”

    Man nimmt beim transversalen Dopplereffekt eine Lichtquelle an, die in großer Entfernung vom Empfänger gleichförmig bewegt vorüberzieht, sodass – über kleine Zeiträume betrachtet – die einzelnen analysierten Signalfolgen auch aus der Sicht des Empfängers konstante zeitliche Entfernungen voneinander aufweisen.

    Eine longitudinale Geschwindigkeitskomponente iS einer Annäherung der Beobachter gibt es aus Sicht des Empfängers bei der Blauverschiebung (orthogonale Ausrichtung des Lichtpulses aus der Sicht des Senders). Hier ist der Abstand zwischen Sender und Empfänger bei einem ersten Signal größer als beim nachfolgenden zweiten Signal, was den Dopplereffekt beeinflusst. Dass diese Entfernungsänderungen in Bezug auf die Gesamtentfernung nicht erkennbar sind, spielt für den Dopplereffekt keine Rolle.

    Der von Dir genannte Artikel stellt auf die “einfachere” Sicht des Senders ab, was beim Dopplereffekt, bei dem es – bei gleichen Senderverhältnissen – nur auf die Situation des Empfängers ankommt, eigenartig anmutet. Es wird behauptet, aus Sicht des Senders gäbe es keine longitudinale Geschwindigkeitskomponente. Aber über eine (über die genannten kurzen Zeiträume) gleichförmige Relativgeschwindigkeit sind zwei Beobachter immer gleicher Meinung. Gibt es eine aus Sicht des Empfängers, dann auch aus Sicht des Senders. Der Artikel stellt überdies immer nur auf die Verhältnisse eines ersten Signals ab, “friert” zu diesem Zeitpunkt die Entfernung ein und tut so, als ob es für den Dopplereffekt eines zweiten Signals für seine Beweisführung nicht bedürfte.

    Dann passiert die Lichtquelle den Bereich des geringsten Abstandes zwischen Sender und Empfänger. Diese Entfernung quer zur Bewegungsrichtung ist aus der Sicht von Sender und Empfänger gleich groß. Aussendewinkel und Empfangswinkel stehen scherenförmig zueinander und sind gleich groß. Hier tritt (in dem genannten kleinen Zeitabschnitt) keine Longitudinalgeschwindigkeit und kein Dopplereffekt auf (siehe Deine Belegstelle), und das trotz angeblicher Zeitdilatation beim Sender aus Sicht des Empfängers.

    Wenn der Sender diesen Punkt überschritten hat, nimmt die Entfernung zwischen Sender und Empfänger aus beider Sicht wieder zu. Es gibt eine longitudinale Geschwindigkeitskomponente iS einer Entfernung der Beobachter voneinander und eine Rotverschiebung (orthogonale Ausrichtung des Lichtpulses aus der Sicht des Empfängers). Hier ist der Abstand zwischen Sender und Empfänger bei einem ersten Signal kleiner als beim nachfolgenden zweiten Signal, was den Dopplereffekt beeinflusst.

    “Im ersten Szenario in #791, beschrieben im Empfänger-Bezugssystem, gibt es keine (überwiegende) Blauverschiebung.”

    Das “Überwiegen” der Blauverschiebung aus Sicht des Empfängers ist auf eine Summe von gleichen Signalfolgen bzw. Signalpaketen zu beziehen, die im Zuge eines Transits des Senders in gleichen Abständen ausgesendet werden. Hier zeitigt eine Gesamtbilanz der eintreffenden Signalpakete eine Blauverschiebung als dominierenden physikalischen Sachverhalt; dies trotz angeblicher “Zeitdilatation” beim Sender aus Sicht des Empfängers. Wenn man nur auf die Ausrichtung der Sichtachse des Empfängers abstellt, so findet im größeren Teil des Schwenkbereichs eine Rotverschiebung statt. Die Ausrichtung des Sehwerkzeugs ist zwar interessant, aber für die Gesamtheit der (energetischen) Gesamtwirkung des Dopplereffekts im Sinne einer Blauverschiebung nicht relevant. (In Bezug auf die Ausrichtung der gesendeten Strahlen aus der Sicht des Senders wird im überwiegenden Schwenkbereich beim Empfänger eine Blauverschiebung ausgelöst.)

  795. #796 Anonym_2022
    31. Januar 2022

    @Peter Strohmayer (31. Januar 2022) #795

    “Wenn man nur auf die Ausrichtung der Sichtachse des Empfängers abstellt, so findet im größeren Teil des Schwenkbereichs eine Rotverschiebung statt.”

    Die Messung der Zeitdilatation des bewegten Senders im Empfängersystem wird zu einem Empfangszeitpunkt durchgeführt, wie im Abschnitt Receiver sees the source as being at its closest point beschrieben.

    Eq. 4: f_r = f_s / γ

    Grund: Der bewegte Sender hatte zum Zeitpunkt des Aussende-Ereignisses des Photons keine longitudinale Geschwindigkeitskomponente (bzgl. der Licht-Richtung Sender -> Empfänger) im Empfänger-Bezugssystem.

  796. #797 Peter Strohmayer
    31. Januar 2022

    Lieber Anonym,
    ich bedanke mich für Deine Diskussionsbereitschaft, die hier das Wertvollste ist. Jetzt ist ein wenig Erschöpfung eingetreten und ich fürchte, weitere Argumente werden sich im Kreis drehen. Daher werde ich erst einmal Pause machen.

  797. #798 Anonym_2022
    31. Januar 2022

    @Peter Strohmayer (31. Januar 2022) #797

    Danke auch!

  798. #799 Peter Strohmayer
    1. Februar 2022

    @Anonym
    Es ist nur auf das Ereignis der Aussendung und das Ereignis des Eintreffens eines Lichtpulses aus der Sicht der jeweiligen Beobachter abzustellen. Du hast Recht: Aus der Sicht des Empfängers weisen diese beiden Ereignisse bei orthogonaler Ausrichtung die geringste räumliche Entfernung auf. Da bei der Aussendung des nächsten Signals (wegen der Beobachtung eines weit entfernten Senders) gleiche Verhältnisse unterstellt werden, haben die besagten Ereignisse des nächsten Signals die gleiche räumliche Entfernung voneinander. Dies veranlasste Dich davon zu sprechen, dass aus der Sicht des Empfängers keine “longitudinale Geschwindigkeitskomponente” vorliegen würde. Das liegt aber an der unterstellten Voraussetzung der gleichen Verhältnisse beim nächsten Signal und gilt für die Analysen der Lichtsignale aller Richtungen. Natürlich gibt es aus der Sicht beider Beobachter nur eine unendlich kurze Zeitspanne, in der die longitudinale Geschwindigkeitskomponente Null wird.

    Aber ich verstehe, dass Du auf die besondere Konstellation der (perpetuierten) kürzesten räumlichen Entfernung aus Sicht des Empfängers hinweisen willst, sodass sich hier die Zeitdilatation in reiner Form auswirkt. Du hast hier auf das Beispiel der Lichtuhr verwiesen. Fällt Dir aber nicht auf, dass bei der bewegten Lichtuhr der Lichtpuls aus Sicht des Senders orthogonal ausgesendet wird, also jene Konstellation vorliegt, in der der Empfänger seine Blickrichtung schräg ausrichten muss und in der keine Rotverschiebung, sondern eine Blauverschiebung auftritt?

  799. #800 Anonym_2022
    1. Februar 2022

    @Peter Strohmayer (1. Februar 2022)

    “Natürlich gibt es aus der Sicht beider Beobachter nur eine unendlich kurze Zeitspanne, in der die longitudinale Geschwindigkeitskomponente Null wird.”

    Im Empfänger-Bezugssystem ist die Zeitspanne unendlich kurz, um das orthogonale Photon zu empfangen, ja.

    Im Sender-Bezugssystem empfängt der Empfänger das selbe Photon dagegen auch mit einer longitudinalen Geschwindigkeitskomponente. Grund: Im Sender-Bezugssystem wird dasselbe Photon nach “schräg hinten” gesendet.

    Das Vorhandensein einer longitudinalen Geschwindigkeitskomponente ist Bezugsssystem-abhängig. Nur das Endergebnis des Dopplereffekts, das Verhältnis der vom Empfänger gemessenen Frequenz zur vom Sender gemessenen Frequenz ist nicht vom Bezugssystem abhängig.

    “Du hast hier auf das Beispiel der Lichtuhr verwiesen. Fällt Dir aber nicht auf …”

    Ja, vielleicht war mein Verweis auf die Lichtuhr in #789 zur Veranschaulichung der Aberration in diesem Zusammenhang verwirrend.

  800. #801 Peter Strohmayer
    1. Februar 2022

    @Anonym
    Also ist der Begriff “longitudinale Geschwindigkeitskomponente” eine Umschreibung dafür, dass ein Lichtpuls aus der Sicht eines der Beobachter nicht orthogonal zur Bewegungsachse ausgesendet bzw. empfangen wird.

    Unterscheidet man zwischen positiver und negativer “longitudinaler Geschwindigkeitskomponente” verstehe ich Deine Argumentation jetzt so, dass, wenn aus Sicht des Empfängers die Sichtachse zum Ausgangspunkt der Bewegung des Senders geneigt ist, wegen der positiven “longitudinalen Geschwindigkeitskomponente” des Senders eine Blauverschiebung auftritt, die aber mit zunehmender Vergrößerung des Winkels wegen der “Zeitdilatation” beim Sender früher, als bei klassischem DE zu erwarten wäre, in eine Rotverschiebung übergeht, die bei orthogonaler Ausrichtung das Maß des Lorentz-Faktors erreicht und mit der dann negativen “longitudinalen Geschwindigkeitskomponente” weiter zunimmt. – Der Schönheitsfehler bei dieser Instrumentalisierung einer Geschwindigkeitskomponente ist, dass sich aus Sicht des Empfängers bei einem Lichtstrahl, der bei ihm eintrifft, keine “longitudinalen Geschwindigkeitskomponente” hinzuaddieren kann, weil die “Lichtgeschwindigkeit” die “Lichtgeschwindigkeit” ist.

    Aus Sicht des Senders folgt aus der Neigung des Aussendewinkels in die Richtung des sich annähernden Empfängers eine positive “longitudinale Geschwindigkeitskomponente”, die beim Empfänger zu einer Blauverschiebung führt. Länger als nach klassischem DE zu erwarten wäre, bleibt aber die Blauverschiebung wegen der Zeitdilatation beim Empfänger über die orthogonale Ausrichtung hinaus erhalten und geht je nach Relativgeschwindigkeit erst spät in eine Rotverschiebung über. Hier bleibt die Geschwindigkeit des Photons aus Sicht des Senders die “Lichtgeschwindigkeit” und die “longitudinale Geschwindigkeitskomponente” des Empfängers wirkt sich auf das Eintreffen des Photons aus (Fahrplanaufgabe). – Für diese Art der Erklärung des relativistischen Dopplereffekts – sie ist mir nicht sympathisch, ja eigentlich ein Gräuel, weil aus der Sicht des Senders Überlicht- und Unterlichtgeschwindigkeiten imaginiert werden -, scheint daher nur die Sicht des Senders ein einigermaßen schlüssiges Ergebnis zu zeitigen. Die Photonen treffen mit “unterschiedlichen Geschwindigkeiten” beim Empfänger ein, wodurch sich schon einmal ein klassischer Dopplereffekt ergibt, und das alles spielt sich bei einem verlangsamten Vergehen von Zeit beim Empfänger ab, was für ihn die Frequenzen entsprechend höher erscheinen lässt.

    Dass (aus der bei dieser Geschichte maßgeblichen Sicht des Senders) die Blauverschiebung überwiegt, zeigt sich sogar bei diesem Gedankenkonstrukt. Es kann ja energetisch auch gar nicht anders sein: die beim Empfänger eintreffenden Strahlen haben in Summe eine höhere Energie als die ausgesendeten. Das ist ein Faktum, an dem keine “unterschiedliche Sichtweise der Beobachter ” etwas ändern kann. Die “reine” Zeitdilatation im Ausmaß des Lorentz-Faktors tritt erst auf, wenn der Lichtpuls aus Sicht des Empfängers orthogonal verläuft, und zwar im Sinne einer Blauverschiebung, weshalb ja auch die Aussage richtig war, dass eine überwiegende Blauverschiebung auftritt und die Zeit (bei dieser Sicht der Dinge) beim Empfänger langsamer vergeht.

    Was aber die Hauptsache ist: Bei dem Ganzen handelt es sich insbesondere wegen der Additionen von “Geschwindgkeitskomponenten” zur Lichtgeschwindigkeit und der kongenial hinzugefügten “Zeitdilatation” um ein widersprüchliches und untaugliches Erklärungsmuster ohne tieferen Wahrheitsgehalt. Das “Beobachten eines Photons” (in einer Lichtuhr) und das “langsamere Vergehen der Zeit aus Sicht des …” sind verunglückte Redeweisen, die man sich abgewöhnen sollte (#790).

  801. #802 Anonym_2022
    1. Februar 2022

    @Peter Strohmayer (1. Februar 2022) #801

    “Also ist der Begriff “longitudinale Geschwindigkeitskomponente” eine Umschreibung dafür, dass ein Lichtpuls aus der Sicht eines der Beobachter nicht orthogonal zur Bewegungsachse ausgesendet bzw. empfangen wird.”

    So ungefähr. Ich vermeide wegen Mehrdeutigkeit (optisches Sehen, zeitverzögert?) “aus der Sicht eines der Beobachter” und rede lieber von Bezugssystemen im Sinne von Koordinatensystemen, die sich relativ zueinander bewegen. Im Ursprung des Bezugssystems S ruht z.B. der Empfänger und im Ursprung des Bezugssystems S’ der Sender.

    “verstehe ich Deine Argumentation jetzt so, dass, wenn aus Sicht des Empfängers die Sichtachse zum Ausgangspunkt der Bewegung des Senders geneigt ist, wegen der positiven “longitudinalen Geschwindigkeitskomponente” des Senders eine Blauverschiebung auftritt …”

    So etwas habe ich nicht geschrieben.

    Ich habe ein sehr einfaches Szenario im Empfänger-Bezugssystem beschrieben, bei dem der Empfänger sein Zielfernrohr senkrecht hält und dadurch genau ein Photon (rotverschoben) empfängt, welches senkrecht über ihm ausgesendet wurde und sich durch das Zielfernrohr bewegt, während der Sender sich weiter nach rechts bewegt.

    Im Senderbezugssystem lautet die Beschreibung wie folgt (dazu fehlt bei Wikipedia leider ein Bildchen): Der Sender ist in Ruhe. Der Empfänger bewegt sich mit seinem ständig senkrecht gehaltenen Zielfernrohr nach links. Zum Aussendezeitpunkt des Photons ist der Empfänger genau unter dem Ort des Aussendeereignisses und bewegt sich weiter nach links. Das Photon bewegt sich schräg nach links unten und trifft den Empfänger zu einem Zeitpunkt, zu dem die Geschwindigkeitsrichtungen von Photon und Empfänger einen Winkel von ungleich 90 Grad einschließen. Wegen der Aberration hält der Empfänger sein Zielfernrohr nach dem Zeitpunkt des Aussendeereignisses nicht in Richtung des Ortes des Aussendeereignisses, sondern senkrecht nach oben. Das Photon bewegt sich trotzdem durch das Zielfernrohr.

    Was aber die Hauptsache ist: Bei dem Ganzen handelt es sich insbesondere wegen der Additionen von “Geschwindgkeitskomponenten” zur Lichtgeschwindigkeit und der kongenial hinzugefügten “Zeitdilatation” um ein widersprüchliches und untaugliches Erklärungsmuster ohne tieferen Wahrheitsgehalt. Das “Beobachten eines Photons” (in einer Lichtuhr) und das “langsamere Vergehen der Zeit aus Sicht des …” sind verunglückte Redeweisen, die man sich abgewöhnen sollte (#790).

    Ich habe nichts zur Lichtgeschwindigkeit addiert. Die Zeitdilatation in einem Bezugssystem gibt es, kann durch transversalen Dopplereffekt oder lokalen Vergleich mit im Bezugssystem synchronisierten Uhren gemessen und kann aufintegriert werden zur Alterung eines Zwillings zwischen zwei Meeting-Ereignissen. Das “Beobachten eines Photons” (in einer Lichtuhr) verwende ich nicht und “aus Sicht des …” verwendet ich, wie bereits geschrieben, auch nicht.

  802. #803 Anonym_2022
    1. Februar 2022

    @Peter Str (26. Januar 2022) #774

    Ich vermute, dass die Interpretation der Rotverschiebung als langsamerer Zeitverlauf im Gravitationsfeld falsch sein könnte und dass sich im Gravitationsfeld – gleich wie im Bereich zwischen zwei beschleunigten Massepunkten – aus der Sicht verschiedener Potentiale nur die Energie (die Frequenz) eines Lichtquants h verändert.

    Albert Einstein hat ohne Rückgriff auf die Quantentheorie (W=h*f) mit der SRT berechnet, dass sich bei Transformation von einem Inertialsystem in ein anderes die Energie eines Lichtimpulses aus elektromagnetischen Wellen proportional zu dessen Frequenz ändert. Das fand er im Jahr 1905 “bemerkenswert”:

    § 8. Transformation der Energie der Lichtstrahlen. Theorie des Strahlungsdrucks

    Es ist bemerkenswert, daß die Energie und die Frequenz eines Lichtkomplexes sich nach demselben Gesetze mit dem Bewegungszustande des Beobachters ändern.

    Quelle (S. 914):
    https://de.wikibooks.org/wiki/A._Einstein:_Kommentare_und_Erl%C3%A4uterungen:_Zur_Elektrodynamik_bewegter_K%C3%B6rper:_Elektrodynamischer_Teil:_%C2%A78

    Für ein homogenes Gravitationsfeld hat er das im Jahr 1011 berechnet. Nur der Schluss mit der Lichtablenkung an der Sonne ist falsch und wurde später von ihm korrigiert.

    Quelle (s. “§ 3. Zeit und Lichtgeschwindigkeit im Schwerefelde.”):
    https://alpha.physics.uoi.gr/foudas_public/ModernPhys-I-UoI-2010/Uber-den-EInfluss-der-Schwerkaft-auf-die-Ausbreitung-des-Lichtes.pdf

  803. #804 Peter Strohmayer
    1. Februar 2022

    @Anonym
    “Ich vermeide … ‘aus der Sicht eines der Beobachter’”:
    “Im Senderbezugssystem” ist auch in Ordnung, ich vermeide wiederum deine Formulierung, weil “im Bezugssystem” als geistigem Konstrukt nichts stattfindet, sondern nur etwas aus der Warte desselben verzeichnet wird.

    Es lag nicht in meiner Absicht, Dir fremde Gedanken unterzuschieben. Deiner Beschreibung der Ausbreitung des Photons “im Senderbezugssystem” pflichte ich mit der Bemerkung bei, dass der Prozess der Ausbreitung dieses Photons “im Senderbezugssystem” länger dauert als “im Empfängerbezugssystem”.

    “Ich habe nichts zur Lichtgeschwindigkeit addiert”: Dann habe ich die Rolle der “longitudinalen Geschwindigkeitskomponenten” bei Deiner Erklärung des Zustandekommens des relativistischen Dopplereffekts missverstanden. Das Erklärungsthema beschränkt sich sohin auf die Konstellation der orthogonalen Ausrichtung “im Empfängerbezugssystem”: er ist eine Folge der Zeitdilatation.

    Demgegenüber möchte ich an einem Beispiel das Zustandekommen dieses Effekts wie folgt beschreiben. Ich argumentiere nur mit der L-T, dh nur mit unterschiedlichen zeitlichen und räumlichen Entfernungen zwischen Ereignissen (Prozessdauern und Strecken). Die Deutung als “Zeitdilatation” will ich mir bei gleichförmigen Bewegungen wegen ihrer Missverständlichkeit nicht zu eigen machen, man braucht sie nicht, sie stört nur:

    Beim longitudinalen Dopplereffekt, bei dem keine Aberration stattfindet, werden Signalfolgen von Lichtpulsen entlang der gemeinsamen Bewegungsachse von Sender 1 und dem sich annähernden Empfänger 2 ausgesendet.

    Aus Sicht des Senders 1 dauert der Prozess zwischen zwei gesendeten Signalen (zwei Ereignissen am selben Ort) die Zeit ts. Der Sender nähert sich während dieser Zeit dem Empfänger um die am ruhenden Sender „vorbeistreichende“ Strecke von v*ts (diese räumliche Entfernung zwischen den zwei Ereignissen wird im Bezugssystem des Empfängers um den Faktor 1/sqr(1-v2) länger verzeichnet).

    Aus Sicht des Empfängers 2 finden die beiden Ereignisse an verschiedenen Orten statt. Die zeitliche Entfernung zwischen den beiden Ereignissen (die Dauer des genannten Prozesses) und die räumliche Entfernung zwischen den beiden Ereignissen (die Strecke der Annäherung des Senders) verlängern sich gegenüber den Werten ts bzw. v*ts zeitlich bzw. räumlich um den Faktor 1/sqr(1-v2). Da der zweite Lichtpuls nun einen kürzeren Weg zum Empfänger zurückleget, verkürzt sich die Zeit, die er für den Weg bis zum Empfänger benötigt, gegenüber der Lichtlaufzeit des ersten Signals um die Zeitspanne von v*ts/sqr(1-v2). Der Effekt der Verkürzung der Signallaufzeit überwiegt den Effekt der Verlängerung der zeitlichen Entfernung zwischen den Ereignissen der Aussendung der Signale. Der Prozess verkürzt sich insgesamt auf te = ts/sqr(1-v2) – v*ts/sqr(1-v2) = ts*(1-v)/sqr(1-v2) = ts*sqr((1-v)/(1+v)). Das Verhältnis von Sendefrequenz zu Empfangsfrequenz ist der Kehrwert von te/ts. fe = fs*sqr((1+v)/(1-v)).

    Das klingt zwar komplizierter als die Wikipedia Erklärung, ist aber ehrlicher und berücksichtigt vor allem sowohl die zeitlichen als auch die räumlichen Komponenten der ineinandergreifenden Veränderungen.

  804. #805 Anonym_2022
    1. Februar 2022

    Die Viererfrequenz eines Photons ist N = (N_t, N_x, N_y, N_z) = f (1, ñ).

    Dabei ist ñ der Einheitsvektor in Ausbreitungsrichtung und f die Frequenz. Wenn sich das Photon in x-Richtung ausbreitet, gilt
    N_t = f und N_x = f.

    Lorentz-Transformation (Empfänger entfernt sich vom Sender):
    N’_t = γ (N_t – ß N_x)
    =>
    f’ = γf(1-ß) = γf (1-v/c) = f √{(1-v/c)/(1+v/c)}.

  805. #806 Anonym_2022
    1. Februar 2022

    Berechnung der Zeitdilatation:

    Wenn eine Uhr im gestrichenen Koordinatensystem S’ ruht, gilt für den räumlichen Abstand zweier aufeinanderfolgender Tick-Ereignisse:
    ∆x’ = 0.

    Eingesetzt in die inverse LT für die Zeit:
    ∆t = γ (∆t’ + v/c² ∆x’) = γ ∆t’.

    Im ungestrichenen Koordinatensystem S dauert das zeitliche Intervall zwischen den Ticks länger.

  806. #807 Peter Strohmayer
    1. Februar 2022

    @Anonym
    Sehr elegant. Fehlende Anschaulichkeit korrigiert immerhin verfehlte Anschaulichkeit (“Zeitdilatation”).
    Liebe Grüße

  807. #808 Peter Strohmayer
    1. Februar 2022

    Nachtrag zu #807:

    Alles richtig. Unnötig ist nur, eine größere zeitliche Entfernung zwischen Ereignissen im Verhältnis zweier gleichförmig bewegter Beobachter auf eine mystische Veränderung des Ablaufs einer newtonisch aufgefassten Zeit zurückzuführen.

  808. #809 MartinB
    2. Februar 2022

    Verfolge hier zwar nicht alles, aber eine Anmerkung dazu:

    “Nur der Schluss mit der Lichtablenkung an der Sonne ist falsch und wurde später von ihm korrigiert”
    Was daran liegt, dass Einstein 1911 eine Theorie hatte, bei der nur die Zeitdilatation berücksichtigt wurde, aber nicht die Raumkrümmung (er hat quasi ohne es zu wissen mit der Newtonschen Näherung der ART gearbeitet).

  809. #810 Peter Strohmayer
    3. Februar 2022

    @Anonym #791 und #806
    Das Verwirrungspotential des Begriffs “Zeitdilatation” ist beeindruckend. Man glaubt als Lateiner, damit sei gemeint, dass sich aus “die Zeit dehnt”, also die Dauer eines Prozesses länger wird, als im eigenen Bezugssystem gemessen. Tatsächlich soll aber mit diesem Begriff ausgedrückt werden, dass die Zeit aus der Sicht des anderen Bezugssystems “langsamer vergeht”. (Wikipedia: “Die Zeitdilatation bewirkt, dass alle inneren Prozesse eines physikalischen Systems relativ zum Beobachter langsamer ablaufen, wenn sich dieses System relativ zum Beobachter bewegt.”) Ein Prozess wird also durch die “Zeitdilatation” kürzer, als im eigenen Bezugssystem gemessen. Das wird bei der Erklärung der bewegten Lichtuhr – bei der auch nur von zeitlichen Entfernungen zwischen Ereignissen ausgegangen wird – seit 100 Jahren wie ein Gebet wiederholt und führen damit alle ein wenig an der Nase herum.

    Über diese Wurzel der Verwirrung stolpert man exemplarisch bei der Erklärung des Dopplereffekts:

    (Ortogonal aus Empfängersicht)
    “Im Empfänger-Bezugssystem gilt:
    Der Sender ist bewegt und unterliegt einer Zeitdilatation. Dazu passend führt ein orthogonaler Lichtstrahl zu einer Rotverschiebung.”

    Die zeitliche Entfernung vom Ereignis der Aussendung des Lichtpulses bis zum Ereignis seines Eintreffens beim Empfänger ist “im Bezugssystem des Empfängers” kleiner als “im Bezugssystem des Senders” (zu diesem Befund kommen sowohl Sender als auch Empfänger). Da aus der Sicht des Empfängers im physikalischen System des Senders mehr Zeit vergangen ist, müsste die Zeit beim Sender schneller und nicht langsamer vergangen sein. Die Aussage, dass der Sender aus Sicht des Empfängers einer “Zeitdilatation” im genannten Sinn unterliegen würde, ist also verfehlt. Es ist genau umgekehrt.

    Zur Situation einige Augenblicke früher, als der Empfänger sein Fernrohr noch schräg gegen den entgegenkommenden Sender ausgerichtet hatte, heißt es:

    (Ortogonal aus Sendersicht)
    “Im Sender-Bezugssystem gilt:
    Der Empfänger ist bewegt und unterliegt einer Zeitdilatation. Dazu passend führt ein orthogonaler Lichtstrahl zu einer Blauverschiebung.”

    Die zeitliche Entfernung vom Ereignis der Aussendung des Lichtpulses bis zum Ereignis seines Eintreffens beim Empfänger ist “im Bezugssystem des Empfängers” größer als “im Bezugssystem des Senders” (zu diesem Befund kommen sowohl Sender als auch Empfänger). Da aus der Sicht des Senders im physikalischen System des Empfängers mehr Zeit vergangen ist, müsste die Zeit beim Empfänger schneller und nicht langsamer vergangen sein! Die Aussage, dass der Empfänger aus Sicht des Senders einer “Zeitdilatation” im genannten Sinn unterliegen würde, ist also verfehlt. Es ist genau umgekehrt.

    Beurteilt man die zweite Situation statt aus der Sicht des Senders aus der Sicht des Empfängers, ist die zeitliche Entfernung vom Ereignis der Aussendung des Lichtpulses bis zum Ereignis seines Eintreffens beim Empfänger “im Bezugssystem des Empfängers” ebenfalls größer als “im Bezugssystem des Senders”. Da aus der Sicht des Empfängers im physikalischen System des Senders weniger Zeit vergangen ist, “ist die Zeit beim Sender langsamer vergangen”. Jetzt endlich ist die Aussage, dass der Sender aus Sicht des Empfängers einer “Zeitdilatation” im genannten Sinn unterliegen würde, richtig. Das ist auch die Situation, die der bewegten Lichtuhr tatsächlich zu Grunde liegt. Dass dort der Empfänger sein Fernrohr eigentlich schräg ausrichten müsste und dass dort eine Blauverschiebung, aus Sicht des Empfängers also eine Frequenzerhöhung eintritt, wo doch die Zeit im physikalischen System des Senders angeblich “langsamer vergangen” sein soll, wird unterschlagen, um den Begriff der “Zeitdilatation” keinen unangenehmen Fragen aussetzen zu müssen.

    Deren weitere gäbe es zuhauf. Die Uhren auf der x-Achse “im bewegten System” gehen aus Sicht des ruhenden Beobachters nicht alle langsamer, sondern nur eine (allerdings sehr wichtige), alle anderen zeigen eine entweder weiter oder geringer fortgeschrittene Zeit an. Es gibt keine einheitliche “Zeitdilatation” wie die obigen zwei Beispiele zeigen. Aus der Sicht des Empfängers kommt es in der Situation 1 zu einer Beschleunigung des Zeitablaufs “im System des Senders”, einige Augenblicke früher aber “im System des Senders” zu einer “Zeitdilatation”. Vergeht jetzt bei dem die Zeit langsamer oder schneller oder beides?

    Man sollte – zumindest bei gleichförmigen Bewegungen – auf die Verwendung dieses Begriffs verzichten. Das fällt den Physikern schwer, weil sie – wie eine Kirche – begriffen haben, dass das Festhalten an unbegreiflichen, widersprüchlichen Dogmen vor einem wissensmäßig unterlegenen Publikum Macht bedeutet.

  810. #811 Anonym_2022
    3. Februar 2022

    @MartinB (2. Februar 2022) #809

    “Was daran liegt, dass Einstein 1911 eine Theorie hatte, bei der nur die Zeitdilatation berücksichtigt wurde, aber nicht die Raumkrümmung (er hat quasi ohne es zu wissen mit der Newtonschen Näherung der ART gearbeitet).”

    Johann Georg von Soldner hat im Jahr 1801 die Lichtablenkung nahe der Sonne mit der Newton’schen Gravitationstheorie und dem Teilchenmodell des Lichts berechnet. Es hatte als Ergebnis den gleichen (zu kleinen) Winkel herausbekommen wie Einstein im Jahr 1911.

    Quelle:
    https://de.wikisource.org/wiki/Ueber_die_Ablenkung_eines_Lichtstrals_von_seiner_geradlinigen_Bewegung

  811. #812 MartinB
    3. Februar 2022

    @Anonym_2022
    Ja, klar, die Theorien sind unter den Bedingungen ja dieselben. 1911 hat Einstein ja “nur” versucht, mit Hilfe eines geänderten Zeitablaufs die Newtonsche Gravitation nachzubilden, weil das dann nicht in Konflikt mit der SRT geraten sollte.

  812. #813 Anonym_2022
    3. Februar 2022

    @Peter Strohmayer (3. Februar 2022) #810

    “(Ortogonal aus Empfängersicht)
    “Im Empfänger-Bezugssystem gilt:
    Der Sender ist bewegt und unterliegt einer Zeitdilatation. Dazu passend führt ein orthogonaler Lichtstrahl zu einer Rotverschiebung.”
    Die zeitliche Entfernung vom Ereignis der Aussendung des Lichtpulses bis zum Ereignis seines Eintreffens beim Empfänger ist “im Bezugssystem des Empfängers” kleiner als “im Bezugssystem des Senders (zu diesem Befund kommen sowohl Sender als auch Empfänger). Da aus der Sicht des Empfängers im physikalischen System des Senders mehr Zeit vergangen ist, müsste die Zeit beim Sender schneller und nicht langsamer vergangen sein.”

    Nein! Bei dieser Argumentation fehlt die Bedingung ∆x’ = 0 im Sendersystem S’ um eine Zeitdilatation des (bewegten) Senders im Empfängersystem zu berechnen, siehe Kommentar #806.

  813. #814 Peter Strohmayer
    3. Februar 2022

    @Anonym
    Es “fehlt die Bedingung ∆x’ = 0 im Sendersystem S’ “.

    Diese Bedingung ist bei zwei beliebigen Ereignissen erfüllt, die aus der Sicht eines Systems am selben Ort stattfinden und zeitlich im Ausmaß irgendwelcher “zwei Ticks” voneinander entfernt sind. Solche beliebigen Ereignisse finden “in jedem System” ununterbrochen statt. Warum sollte man gerade solche Ereignisse willkürlich herausgreifen und “berücksichtigen”, und wozu soll man, um den Dopplereffekt erklären zu können, eine Rechenübung zur Ermittlung des zeitlichen und räumlichen Abstands gerade dieser Ereignisse vornehmen? Wir sprachen immer nur vom Ereignis des Aussendens und dem des Empfangs des Lichtpulses, dessen Frequenz sich ändert. Das reichte, um den Dopplereffekt zu erklären und zu berechnen (#804). Welche konkreten zwei Ereignisse, die die Bedingung ∆x’ = 0 im Sendersystem S’ erfüllen, schlägst Du zur notwendigen Berücksichtigung, die bei mir angeblich fehlt, vor?

    Weil Du aber über diese Bedingung und die von ihr vorausgesetzten “zwei Ticks” die Lichtuhr wieder ins Spiel bringst: Wie oben zu #810 erwähnt, bildet die Lichtuhr die Situation “orthogonal aus Sendersicht” ab. Es vergeht zwischen den besagten Ereignissen aus Sicht des Empfängers mehr Zeit als aus Sicht des Senders. Wenn die Relativgeschwindigkeit des Senders höher wird, so verlängert sich diese Zeitspanne nur “im Empfängersystem”, während sie “im Sendersystem” mit seinen zwei Ticks gleich bleibt. Was ist jetzt mit der “Zeitdilatation”? Die Zeit soll doch beim Sender immer langsamer vergehen, je schneller er unterwegs ist.

    Das geht noch dazu mit einer zunehmenden Blauverschiebung beim Empfang des Photons einher (so im Ergebnis auch #792), was mit einer angeblichen Verlangsamung des Zeitablaufs beim Sender nicht zu vereinbaren ist.

  814. #815 Anonym_2022
    4. Februar 2022

    @Peter Strohmayer (3. Februar 2022) #814

    “Warum sollte man gerade solche Ereignisse willkürlich herausgreifen und “berücksichtigen””

    Weil Deine Argumentation für “Da aus der Sicht des Empfängers im physikalischen System des Senders mehr Zeit vergangen ist, müsste die Zeit beim Sender schneller und nicht langsamer vergangen sein.” falsch ist. Grund: Diese stützt sich auf ein Ereignispaar, für das ∆x = 0 im Empfängersystem und nicht ∆x’ = 0 im Sendersystem gilt. Im Sendersystem bewegt sich das Licht “schräg”.

    “Welche konkreten zwei Ereignisse, die die Bedingung ∆x’ = 0 im Sendersystem S’ erfüllen, schlägst Du zur notwendigen Berücksichtigung, die bei mir angeblich fehlt, vor?”

    Beispiel: Die elektromagnetische Schwingung im punktförmigen Sendersystem erreicht einen Scheitelwert der Amplitude zum (n)ten und zum (n+1)ten mal. Das ist wie das Ticken einer Uhr. Dann gelangt man zu dem Berechnungsergebnis, dass die Periodendauer der Schwingung im Empfängersystem größer (und damit die Frequenz kleiner) ist:
    ∆t = γ ∆t’

    “Wenn die Relativgeschwindigkeit des Senders höher wird, so verlängert sich diese Zeitspanne nur “im Empfängersystem”, während sie “im Sendersystem” mit seinen zwei Ticks gleich bleibt. Was ist jetzt mit der “Zeitdilatation”? Die Zeit soll doch beim Sender immer langsamer vergehen, je schneller er unterwegs ist.”

    Wer behauptet das?

  815. #816 Peter Strohmayer
    4. Februar 2022

    @Anonym
    “Weil Deine Argumentation … falsch ist.“

    Ob – in der Diktion der Verfechter des Begriffs der Zeitdilatation – im physikalischen System des Senders die Zeit anders (“schneller” oder “langsamer”) vergeht, richtet sich nach dem zu Grunde liegenden Denkansatz nach dem Verhältnis von systemabhängigen zeitlichen Entfernungen zwischen zwei beliebigen Ereignissen („wir beobachten einen Lichtstrahl“).

    Die Verhältnisse der zeitlichen Entfernung zweier Ereignisse, die in einem System am selben Ort stattfinden, ist nur ein – allerdings wichtiger – Sonderfall.

    Du erklärst nun die Ergebnisse der Beurteilung von Ereignispaaren, die nicht am selben Ort stattfinden, für „falsch“, und hältst Lichtstrahlen, die sich „im System des Senders“ schräg bewegen (und so diese Ereignisse definieren), für ungeeignete Beurteilungsobjekte. Grund: Du möchtest die Beurteilung auf den Sonderfall zweier am selben Ort stattfindender Ereignisse einengen, um die ausschlaggebende Rolle der „Zeitdilatation“ zu beweisen.

    Wir beurteilten bisher nur einen Lichtpuls bzw. nur ein Ereignis der Aussendung und ein Ereignis des Eintreffens – beide eben nicht am selben Ort. Dieser Lichtpuls kam „rot- oder blauverschoben beim Empfänger an“ (zB #791 #800 #802).

    Nun erklärst Du die auf die Verhältnisse dieses eines Lichtpulses bezogene Argumentation (#810) für „falsch“ und änderst zugleich den Sachverhalt: „Die elektromagnetische Schwingung … wie das Ticken einer Uhr (…)“. Statt von einem Lichtpuls, der zwei räumlich entfernte Ereignisse konstituiert, ist nun von zwei aus Sicht des Senders am selben Ort stattfindenden Ereignissen die Rede, die die Prozessdauer ∆t’ zeitlich voneinander entfernt sind und die „im Systems des Empfängers“ einen größere zeitliche Entfernung ∆t = γ ∆t’ voneinander aufweisen, deren niedrigere Frequenz als Dopplereffekt bezeichnet wird.

    Der Dopplereffekt entsteht aber nicht dadurch, dass „im Sendersystem“ am selben Ort stattfindende Ereignisse im Empfängersystem verzeichnet werden, sondern hier dadurch, dass bei jedem dieser Ereignisse je ein Lichtpuls ausgesendet wird, deren Eintreffen am selben Ort „im Empfängersystem“ zeitlich verzeichnet wird.

    Man könnte die Beurteilung dieser Vorgänge auf den Punkt bringen und – ohne auf zwei Lichtpulse abzustellen – dem Gesetz folgen, dass sich die Rot- bzw. Blauverschiebung eines Lichtpulses „im System des Empfängers“ betragsmäßig exakt aus dem Verhältnis der systembezogen verschiedenen zeitlichen Entfernungen des Ereignisses der Aussendung vom Ereignis des Eintreffens des Lichtpulses ergibt. Diese Zeitspanne ist in der vorliegenden Konstellation „orthogonal aus Empfängersicht“ „im System des Senders“ länger. Daher ist die Frequenz aus Sicht des Empfängers entsprechend niedriger. Um mit den Anhängern der Vorstellung einer „im anderen System anders vergehenden Zeit“ zu sprechen: die Zeit ist während dieses Prozesses im System des bewegten Senders schneller vergangen, weshalb der Prozess der Lichtausbreitung (nicht der Zeit zwischen zwei Ticks am selben Ort beim Sender!) aus dortiger Sicht länger dauerte, was zum betreffenden Dopplereffekt führt.

    Das irritiert die Anhänger der „Zeitdilatation“, bei denen die Zeit im anderen System nicht nur anders (was schon problematisch genug ist), sondern partout immer nur langsamer vergehen muss, und die sich in diese Vorstellung dadurch einzementieren, dass sie es für unzulässig halten, auf einen Prozess zwischen zwei Ereignissen abzustellen, die aus Sicht des bewegten Beobachters nicht am selben Ort stattfinden.

    Um dem nicht auszuweichen, beurteilen wir die genannten Vorgänge also ein wenig umständlicher, so wie #804 für longitudinale Lichtpulse. Dort wurde gesagt:

    „Aus Sicht des Senders 1 dauert der Prozess zwischen zwei gesendeten Signalen (zwei Ereignissen am selben Ort) die Zeit ts. (…) Aus Sicht des Empfängers 2 finden die beiden Ereignisse an verschiedenen Orten statt. Die zeitliche Entfernung zwischen den beiden Ereignissen (die Dauer des genannten Prozesses) und die räumliche Entfernung zwischen den beiden Ereignissen (die Strecke der Annäherung des Senders) verlängern sich gegenüber den Werten ts bzw. v*ts zeitlich bzw. räumlich um den Faktor 1/sqr(1-v2). (…) Der Effekt der Verkürzung der Signallaufzeit überwiegt den Effekt der Verlängerung der zeitlichen Entfernung zwischen den Ereignissen der Aussendung der Signale.“

    In der vorliegenden Konstellation (orthogonal aus Empfängersicht) haben die beiden Ereignisse des Aussendens aus der Sicht des Empfängers zwar auch den räumlichen Abstand von v*ts/sqr(1-v2), der aber die Signallaufzeit nicht verändert, weil er im Verhältnis zum orthogonalen Abstand Sender-Empfänger infinitesimal klein gesetzt werden kann. Damit fällt der Effekt einer Verkürzung/Verlängerung der Signallaufzeit weg und übrig bleibt der Effekt der Verlängerung der zeitlichen Entfernung zwischen den Ereignissen der Aussendung der Signale aus Sicht des Empfängers im Ausmaß der „Zeitdilatation“.

    Kann nun die Zeit beim bewegten Sender zugleich schneller (siehe oben) und langsamer (siehe hier) vergehen? Ja, weil sich die Vorstellung einer anders vorgehenden Zeit, die ich in Ansehung der Aussagekraft zeitlicher Entfernungen zwischen Ereignissen für völlig überflüssig halte, nur auf einen konkreten Prozess beziehen kann. Die Einengung dieser Überflüssigkeit auf Prozesse mit Ereignissen am selben Ort dient nur der Aufrechterhaltung des Dogmas der „Zeitdilatation“.

    @Anonym #815 „Wer behauptet das?“
    Behauptest Du das Gegenteil, nämlich dass mit zunehmender Geschwindigkeit die Dauer des Prozesses zwischen zwei Ticks „im Empfängersystem“ gleich bliebe, während der Prozess „im Sendersystem“ kürzer werden würde (“weil ja die Zeit immer langsamer vergeht”)?
    Oder behauptest Du das Gegenteil, nämlich dass die Zeit mit zunehmender Geschwindigkeit „im Sendersystem“ nicht immer langsamer vergeht (das würde ich sehr begrüßen)?

  816. #817 Anonym_2022
    4. Februar 2022

    @Peter Strohmayer (4. Februar 2022) #816

    “Das irritiert die Anhänger der „Zeitdilatation“, bei denen die Zeit im anderen System nicht nur anders (was schon problematisch genug ist), sondern partout immer nur langsamer vergehen muss, und die sich in diese Vorstellung dadurch einzementieren, dass sie es für unzulässig halten, auf einen Prozess zwischen zwei Ereignissen abzustellen, die aus Sicht des bewegten Beobachters nicht am selben Ort stattfinden.

    Ich habe mich in die Vorstellung einzementiert, dass der folgende Term für die ZD nicht größer als 1 sein kann:

    dτ/dt = √{1-v²/c²}

  817. #818 Peter Strohmayer
    4. Februar 2022

    Das Verhältnis der zeitlichen Entfernung zweier Ereignisse am selben Ort (als Zeitanzeige von dort befindlichen synchronisierten Uhren des Systems S’ “Eigenzeit” τ genannt, weil mit der unmittelbar beim Beobachter befindlichen “eigenen” Uhr gemessen) zur zeitlichen Entfernung dieser Ereignisse an verschiedenen Orten (als Zeitanzeige von dort befindlichen synchronisierten Uhren des Systems S “Koordinatenzeit” t genannt, weil mit Uhren auf bestimmten Koordinatenpunkten gemessen), muss immer kleiner als 1 sein.

    Trotzdem gehen die synchronisierten Uhren dieser Systeme untereinander und im Vergleich der Systeme alle gleich “schnell”, so wie sich ja auch die Lichtpulse in diesen beiden Systemen “gleich schnell” ausbreiten. Das ist die Voraussetzung dafür, dass sie die Eigenzeit bzw. die Koordinatenzeit richtig messen können. So elementar das Verhältnis dτ/dt = √{1-v²} ist (ohne das überflüssigerweise mitgeschleppte c²), so schädlich ist die dem Verstand widerstreitende, aber mit Mannschaftsdisziplin seit 100 Jahren verteidigte Redensart, die Formel zeige, dass die Eigenzeit (angezeigt von allen Uhren des Systems S’) und die Koordinatenzeit (angezeigt von allen Uhren des Systems S) verschieden rasch vergingen. Eigenzeiten und Koordinatenzeiten sind einzelne Messergebnisse und keine Zeiten (im Sinn von Substanzen), die vergehen könnten, und schon gar keine, die verschieden rasch vergingen.

  818. #819 Anonym_2022
    5. Februar 2022

    @Peter Strohmayer (4. Februar 2022) #818

    Diese Definition von” Zeitdilatation” halte ich für gut formuliert:

    … a clock moving uniformly with velocity v through an inertial frame S goes slow by a γ factor relative to the synchronized standard clocks at rest in S.

    This ‘time dilation’, like length contraction, is no accident of convention but a real effect. Moving clocks really do go slow.

    Quelle:
    http://www.scholarpedia.org/article/Special_relativity:_kinematics#Special_relativistic_kinematics

  819. #820 Anonym_2022
    5. Februar 2022

    @Peter Strohmayer (4. Februar 2022) #816

    “Nun erklärst Du die auf die Verhältnisse dieses eines Lichtpulses bezogene Argumentation (#810) für „falsch“ und änderst zugleich den Sachverhalt: „Die elektromagnetische Schwingung … wie das Ticken einer Uhr (…)“. Statt von einem Lichtpuls, der zwei räumlich entfernte Ereignisse konstituiert, ist nun von zwei aus Sicht des Senders am selben Ort stattfindenden Ereignissen die Rede, die die Prozessdauer ∆t’ zeitlich voneinander entfernt sind und die „im Systems des Empfängers“ einen größere zeitliche Entfernung ∆t = γ ∆t’ voneinander aufweisen, deren niedrigere Frequenz als Dopplereffekt bezeichnet wird.”

    Das stimmt. Die Messung zu einem Zeitpunkt war eine Idealisierung. In Wirklichkeit ist so eine Messung eine Mittelung über einen sehr kleinen, aber endlichen Zeitraum. Dadurch wird eine gewisse Ungenauigkeit in Kauf genommen. Solche Messungen wurde durchgeführt:

    Direct measurement of transverse Doppler effect

    The advent of particle accelerator technology has made possible the production of particle beams of considerably higher energy than was available to Ives and Stilwell. This has enabled the design of tests of the transverse Doppler effect directly along the lines of how Einstein originally envisioned them, i.e. by directly viewing a particle beam at a 90° angle. For example, Hasselkamp et al. (1979) observed the Hα line emitted by hydrogen atoms moving at speeds ranging from 2.53×10⁸ cm/s to 9.28×10⁸ cm/s, finding the coefficient of the second order term in the relativistic approximation to be 0.52±0.03, in excellent agreement with the theoretical value of 1/2.[p 10]

    Quelle:
    https://en.wikipedia.org/wiki/Relativistic_Doppler_effect#Direct_measurement_of_transverse_Doppler_effect

  820. #821 Peter Strohmayer
    5. Februar 2022

    @Anonym #819

    Diese Definition ist klug. Ihr Wert besteht darin, dass nur von einer einzigen Uhr des Systems S gesprochen wird. Die anderen Uhren des Systems S bleiben ausgeklammert.

    Diese Definition behauptet nicht, dass “alle Uhren” des bewegten Systems langsamer vergingen, und schon gar nicht, dass die Zeit dort langsamer verginge.

    Vgl. aber zB Markus Pössel https://www.einstein-online.info/spotlight/LichtuhrZeitdilatation/:

    “Wie schon gesagt: Alle Uhren, die relativ zu der bewegten Raumstation ruhen, schlagen im gleichen Takt wie die bewegte Lichtuhr. Alle Uhren, die in meiner eigenen Raumstation ruhen, schlagen im gleichen Takt wie meine eigene Lichtuhr. Die Betrachtung der Lichtuhr ist daher lediglich ein Beispiel für einen viel allgemeineren Umstand, die Zeitdilatation der Speziellen Relativitätstheorie: Von meiner Raumstation aus beurteilt laufen alle Uhren der relativ zu mir bewegten Raumstation langsamer als meine eigenen Uhren. Ebenso wie die bewegten Uhren langsamer gehen, laufen auch alle Vorgänge auf der anderen Raumstation für mich langsamer ab – Fünf-Minuten-Eier kochen länger und haben am Ende doch die richtige Konsistenz, und der Pianist an Bord der anderen Station, der den Minutenwalzer spielt, benötigt dafür deutlich mehr Zeit, als es der üblichen Aufführungspraxis entspricht.”

    Diese missverständlichen Unsinnigkeiten werden vom Max-Planck-Institut für Gravitationsphysik (Albert-Einstein-Institut) in zerfahrener Sprache verbreitet (es ist kein Zufall, dass Du Dich an die englische Fassung von Wikipedia halten musst). Zuerst wird von “Alle Uhren, die relativ zu der bewegten Raumstation ruhen” gesprochen, dann auf “alle Uhren der relativ zu mir bewegten Raumstation” umgestiegen, um mit der Verwendung des Plurals “… Ebenso wie die bewegten Uhren langsamer gehen” fortzufahren. Die Feinheiten dieser Widersprüche gehen bei wissensmäßig unterlegenen Zuhörern ohnehin unter. Sie glauben einfach das Dogma, dass “die Zeit” im bewegten System langsamer vergeht.

    Die Universitäten helfen mit. Franz Embacher:
    https://homepage.univie.ac.at/franz.embacher/SRT/Zeitdilatation.html
    “Da nun eine solche Lichtuhr dazu benutzt werden kann, die Zeitdauer beliebiger anderer Prozesse zu messen (…), ist dieser Effekt nicht auf Lichtuhren beschränkt, sondern betrifft den Zeitfluss in beiden Systemen ganz generell.”

    Bei den Deutschen vergeht die Zeit bei allen Uhren des bewegten Systems langsamer (Embacher: “Zeitfluss ganz generell”). Die Amerikaner, denen es nicht einfällt, ihre Wissenschaft Dogmatikern zu opfern, haben die Behauptung “goes slow” auf eine einzige Uhr des bewegten Systems eingeschränkt. Ein entscheidender Fortschritt. Nun ist klar, dass alle anderen synchronisierten Uhren des Systems S aus der Sicht des Systems S’ entweder eine frühere oder eine spätere Zeit anzeigen, also – in der überflüssigen Diktion der Zeitdilationisten – angeblich entweder noch langsamer oder eben schneller laufen als die mit dem Gammafaktor “langsamer laufende” Uhr. Das aber ist bei einem Satz synchronisierter Uhren denkunmöglich oder zumindest eine Verstiegenheit. Damit ist die Vorstellung einer einheitlich langsamer vergehenden Zeit vom Tisch. In Anbetracht der Aussagekraft unterschiedlicher zeitlicher Entfernungen aus der Sicht verschiedener Systems ist es überflüssig, diese – auch nur bei einer einzigen der vielen Uhren – auf ein “langsamer Vergehen” der Zeit zurückzuführen. Statt
    “Moving clocks really do go slow” wird es einmal heißen “Encounter clocks really show different times”, und damit kann es sein Bewenden haben.

    #820
    “Die Messung zu einem Zeitpunkt war eine Idealisierung.”

    Die Messung der Energie eines einzigen Lichtpulses (im Idealfall eines Photons) reicht, um einen Dopplereffekt zu bestimmen.

  821. #822 Anonym_2022
    6. Februar 2022

    @Peter Strohmayer (5. Februar 2022) #821

    “Nun ist klar, dass alle anderen synchronisierten Uhren des Systems S aus der Sicht des Systems S’ entweder eine frühere oder eine spätere Zeit anzeigen, also – in der überflüssigen Diktion der Zeitdilationisten – angeblich entweder noch langsamer oder eben schneller laufen als die mit dem Gammafaktor “langsamer laufende” Uhr.”

    Das stimmt nicht. Die synchronisierten Uhren des Systems S haben im Systems S’ zwar untereinander einen konstanten Zeitversatz, aber die gleiche Tick-Rate (um Gammafaktor “langsamer laufend”).

    Wenn z.B. zwei dieser Uhren mit dem Abstand ∆x im System S synchron laufen, gilt für ihre synchronen Tick-Ereignisse jeweils:
    ∆t = 0.
    Einsetzen in die LT für die Zeit ergibt:

    ∆t’ = γ (∆t – v/c² ∆x) = -γv/c² ∆x.

    D.h. ihre betrachteten Tick-Ereignisses haben im System S’ einen konstanten Zeitversatz, da v als konstant angenommen wurde.

    Und Zeit ist das, was eine Uhr anzeigt. Insofern hat Markus Pössel recht.

  822. #823 Peter Strohmayer
    7. Februar 2022

    @Anonym
    “Wenn z.B. zwei dieser Uhren mit dem Abstand ∆x im System S synchron laufen, gilt für ihre synchronen Tick-Ereignisse jeweils:
    ∆t = 0.
    Einsetzen in die LT für die Zeit ergibt:
    ∆t’ = γ (∆t – v/c² ∆x) = -γv/c² ∆x.“

    Der Abstand zwischen den Tick-Ereignissen in S’ wächst nicht mit der Entfernung der Uhren. Wenn ich nicht irre, ist das nicht ist die Tickrate, sondern der Zeitversatz der beiden in S gleichzeitigen Ereignisse aus Sicht S’. Ich glaube aber zu verstehen, worauf Du hinaus willst.

    Zwei Uhren, die nach #822 aus der Sicht des Systems S räumlich voneinander entfernt sind und dort die gleiche Zeit anzeigen (zB an den beiden Enden einer Rakete), weisen – wie alle anderen Uhren gegenüber dem System S’ gleichförmig bewegten Uhren des Systems S – aus Sicht S’ die “gleiche Tickrate” (die gleiche zeitliche Entfernung zwischen zwei aus Sicht des Systems S’ bei allen Uhren zeitlich gleich weit entfernten Tickereignissen) auf. Dazu muss man aber die jeweils zwei Anzeigen dieser beiden Uhren des Systems S aus Sicht des Systems S’ nicht zur gleichen Zeit, sondern früher bzw. später mit dem synchronen Stand der Uhren des Systems S’ vergleichen. Die genannten Ereignispaare finden aus Sicht des Systems S’ zu verschiedenen Zeitabschnitten statt. Du beschreibst nicht, wie sich ein Geschehen im System S aus der Sicht einer einheitlichen Gegenwart des Systems S’ darstellt, sondern Du setzt das Ergebnis einer aus Sicht des Systems S’ „zeitdilatierten“ Anzeige der beiden genannten Uhren aus zeitlich dem Feststellungszeitpunkt nach aufgefächerten Messergebnissen zusammen. Gleich lägere Tickraten zweier Uhren zu aus Sicht S’ verschiedenen Zeiten sind kein Nachweis einer aus Sicht S’ im System S gleichförmig anders vergehenden Zeit.

    Zu gleichen längeren Tickraten kommt es auch in in der umgekehrten Konstellation, wenn die aus Sicht des Systems S verschiedenen zwei Zeitpunkte einer Anwesenheit der Enden der Rakete aus Sicht des Systems S’ als gleichzeitige Ereignisse verzeichnet und von diesen Zeitpunkten an für beide Uhren die gleichen um den Gammafaktor längeren Tickraten ermittelt werden. Gegenstand dieser Verzeichnung von Ereignissen im System S’ ist eine aus Sicht des Systems S der Länge nach zu verschiedenen Zeitpunkten existierende Rakete. Deshalb “passt die Leiter in die bewegte Garage“. Gleich längere Tickraten zweier Uhren zu aus Sicht S verschiedenen Zeiten sind kein Nachweis einer aus Sicht S’ im System S gleichförmig anders vergehenden Zeit.

    Es ist ja richtig und spricht auf den ersten Blick für Deinen Standpunkt, dass alle synchronisierten und synchron tickenden Uhren des Systems S, deren Standorte und Zeitanzeigen im System S’ verzeichnet werden, bei ihrer Bewegung durch das System S’ die gleiche längere Tickrate’ aufweisen, weshalb man geneigt ist zu glauben, dies wäre auf ein gleichförmiges, langsameres Vergehen der Zeit im System S’ zurückzuführen.

    Die Apologeten eines langsameren Zeitflusses im anderen System gehen aber darüber hinweg, dass die Uhren des Systems S im unmittelbaren Vergleich mit den Uhren des Systems S’ (bei der Begegnung der jeweiligen Uhren aus S und S’) bereits zum Zeitpunkt der Ursprungsdeckung der Systeme in Bewegungsrichtung umso mehr vorgingen bzw. gegen die Bewegungsrichtung umso mehr nachgingen, je weiter sie vom Ursprung des Bezugssystems entfernt waren. Diese Verkippung, diese zeitlichen Vorsprünge und Verspätungen (und nicht die gleichen Tickraten) sind der überwiegende Faktor bei der Analyse der zeitlichen Verhältnisse der beiden Systeme zueinander.

    Erst in Anbetracht dieser symmetrischen, mit zunehmender räumlichen Entfernung zunehmenden zeitlichen Verkippung der Anzeigen der Uhren des Systems S ist es logisch konsistent, dass sowohl die Uhr im Ursprung des Systems S’ als auch die Uhr im Ursprung des Systems S gegenüber der Begegnungsuhr des jeweils gegenbeteiligten Systems um den inversen Gammafaktor nachgeht. Das folgt aus dem Zusammenspiel der zeitlichen und räumlichen Entfernungen aus Sicht der verschiednen Systeme und ist ist auf dieser Basis errechenbar und verstehbar.

    Sobald man aber versucht, diese Vorgänge an einem angeblich langsameren Zeitfluss im anderen System festzumachen, kann man es vielleicht immer noch berechnen, aber mit dem Verständnis ist es vorbei. Das ist die gegenwärtige Situation der populärwissenschaftlichen Physik.

    Auf den Punkt gebracht: Die Relativität der Gleichzeitigkeit ist nicht die Folge der Zeitdilatation, sondern deren Widerlegung.

    Es gibt kein gleichförmig langsameres Vergehen der Zeit in Markus Pössels ausgedehnter Rakete, gerade wenn Zeit nur das sein soll, was die Uhren der beiden Systeme in ihrem komplizierten Zusammenspiel anzeigen. Der Begriff der Zeitdilatation (#810) ist – wie die Dreifaltigkeit – ein antiaufklärerisches paradoxes Dogma zum Zweck priesterlicher Machtausübung über Laien.

  823. #824 Anonym_2022
    7. Februar 2022

    @Peter Strohmayer (7. Februar 2022) #823

    “Auf den Punkt gebracht: Die Relativität der Gleichzeitigkeit ist nicht die Folge der Zeitdilatation, sondern deren Widerlegung.”

    Weder noch! Beide sind in der LT für die Zeit enthalten:

    • Die LT für x enthält den Faktor γ für die Längenkontraktion und den Term “-vt” für die “Relativität der Gleichortigkeit”.
    • Die LT für die Zeit enthält den Faktor γ für die Zeitdilatation und den Term “-vx/c²” für die “Relativität der Gleichzeitigkeit”.

    “Es gibt kein gleichförmig langsameres Vergehen der Zeit in Markus Pössels ausgedehnter Rakete, gerade wenn Zeit nur das sein soll, was die Uhren der beiden Systeme in ihrem komplizierten Zusammenspiel anzeigen.”

    Markus Pössel hat in Deinem Zitat in #821 auch nicht “gleichförmig langsameres Vergehen der Zeit in der ausgedehnten Rakete” geschrieben. Welcher Satz von ihm ist denn genau aus welchem Grunde falsch?

    “Der Begriff der Zeitdilatation (#810) ist – wie die Dreifaltigkeit – ein antiaufklärerisches paradoxes Dogma zum Zweck priesterlicher Machtausübung über Laien.”

    Ich empfehle, die Definition der ZD nicht von Wikipedia zu übernehmen wie in #810, sondern von etablierten, aktuellen SRT-Lehrbüchern, entsprechend der Definition, die ich von Wolfgang Rindler in #819 zitiert habe. Die ZD ist nicht paradox, weil die Eigenzeit einer bewegten Uhr ins Verhältnis zur Koordinatenzeit des Bezugssystems gesetzt wird. Das ist nicht symmetrisch.

    Bei der ZD werden nicht Koordinatenzeiten zweier Koordinatensysteme verglichen und es werden auch nicht direkt Eigenzeiten zweier Uhren verglichen. Für das zweite ist u.a. das Zwilling”paradoxon” zuständig.

  824. #825 Peter Strohmayer
    8. Februar 2022

    @Anonym
    “Beide sind in der LT für die Zeit enthalten.”
    Terme sind keine Entitäten.

    “Markus Pössel”:
    Er: “alle Uhren der relativ zu mir bewegten Raumstation …”

    “Ich empfehle …”
    Danke für den Tipp.

    “Das ist nicht symmetrisch.”
    Sowohl die Uhr im Ursprung des Systems S’ als auch die Uhr im Ursprung des Systems S geht gegenüber der Begegnungsuhr des jeweils gegenbeteiligten Systems um den inversen Gammafaktor nach. Dies im direkten Vergleich der Koordinatenzeiten beider Systeme aus der Sicht jedes der Systeme.

    “Bei der ZD werden nicht … verglichen …”
    Der Ausdruck “Eigenzeit einer bewegten Uhr ins Verhältnis zur Koordinatenzeit des Bezugssystems gesetzt” ist der Vergleich je einer Koordinatenzeit verschiedener Bezugssysteme (#779, #780).

  825. #826 Anonym_2022
    8. Februar 2022

    Peter Strohmayer (8. Februar 2022) #825

    “Terme sind keine Entitäten.”

    Das ist eine ontologische Aussage und damit in der Physik off-topic.

    ““Markus Pössel”: Er: “alle Uhren der relativ zu mir bewegten Raumstation …””

    Das ist ein Ausschnitt aus seinem Satz “Von meiner Raumstation aus beurteilt laufen alle Uhren der relativ zu mir bewegten Raumstation langsamer als meine eigenen Uhren.” Aus welchem Grund ist die Aussage dieses Satzes falsch?

    “Sowohl die Uhr im Ursprung des Systems S’ als auch die Uhr im Ursprung des Systems S geht gegenüber der Begegnungsuhr des jeweils gegenbeteiligten Systems um den inversen Gammafaktor nach. Dies im direkten Vergleich der Koordinatenzeiten beider Systeme aus der Sicht jedes der Systeme.”

    Nein. Ein “nachgehen” bezieht sich nicht auf Tick-Raten, sondern auf Uhrzeit-Anzeigen. Ein solcher direkter Uhrenvergleich ist nur zwischen Uhren möglich, wenn sie sich begegnen. Beispiel:

    Links im Labor ruhe die Uhr U_A, rechts im Labor ruhe die (im Labor-System) dazu synchrone Uhr U_B. Die Uhr U_C bewege sich mit konstanter Geschwindigkeit v von links nach rechts. Sie trifft erst die Uhr U_A (=Ereignis 1) und dann die Uhr U_B (=Ereignis 2). Angenommen, zum Ereignis 1 zeigen U_A und U_C zufällig die gleichen Uhrzeit “Null” an. Dann wird zum Ereignis die Uhr 2 U_C nachgehen und “1/Gamma” mal die Uhrzeit von U_B anzeigen. Hier wurde also die Eigenzeit einer bewegten Uhr mit der Koordinatenzeit des Laborsystems verglichen. Das ist nicht symmetrisch.

    “Der Ausdruck “Eigenzeit einer bewegten Uhr ins Verhältnis zur Koordinatenzeit des Bezugssystems gesetzt” ist der Vergleich je einer Koordinatenzeit verschiedener Bezugssysteme (#779, #780).”

    Die Kommentare #779, #780 beziehen sich auf ein beschleunigtes Bezugssystem. Das ist eine andere Situation.

  826. #827 Peter Strohmayer
    9. Februar 2022

    @Anonym
    “Terme sind keine Entitäten” ist zugegeben verkürzt. Die L-T ergibt nach #824 eine zeitlich und räumlich aufgesplitterte Welt von S, keine einheitlich verlangsamten Klavierspieler oder Eierkocher.

    “ ‘Von meiner Raumstation aus beurteilt laufen alle Uhren der relativ zu mir bewegten Raumstation langsamer als meine eigenen Uhren.’ Aus welchem Grund ist die Aussage dieses Satzes falsch?”

    Weil er mit “alle Uhren” die anspricht, die in der bewegten Raumstation entlang der x-Achse angeordnet sind und die gleiche Zeit anzeigen, und sodann mit “beurteilt” zum Ausdruck bringt, dass er die angeblich im selben Ausmaß zurückgebliebene Anzeige dieser Uhren zu zwei bestimmten Zeitpunkten gemeinsam beurteilen würde.

    Würde der Satz dahin lauten, dass “alle Uhren” des anderen Systems zu zwei Zeitpunkten gemeinsam beurteilt langsamer laufen, so handelt es sich nicht um Uhren, die in der bewegten Raumstation entlang der x-Achse angeordnet sind und die gleiche Zeit anzeigen.

    “Nein. Ein ‘Nachgehen’ bezieht sich nicht auf Tick-Raten, sondern auf Uhrzeit-Anzeigen. Ein solcher direkter Uhrenvergleich ist nur zwischen Uhren möglich, wenn sie sich begegnen.”

    Die L-T handelt nur vom Standort und von der Zeitanzeige von jeweils synchronisierten Uhren, die sich begegnen.

    “Hier wurde also die Eigenzeit einer bewegten Uhr mit der Koordinatenzeit des Laborsystems verglichen. Das ist nicht symmetrisch.”

    Die Eigenzeit einer bewegten Uhr ist die Zeit in deren Koordinatensystem (alle Uhren gehen mit der Uhr im Ursprung synchron), also auch ihre Koordinatenzeit. Mit dem Gegensatzpaar “Koordinatenzeit” und “Eigenzeit” wird nur betont, dass der Stand einer vom Ursprung räumlich entfernten Uhren mit dem Stand einer im Ursprung befindlichen Uhr des anderen Systems, die ihr begegnet, verglichen wird.

    Zur Symmetrie: Aus Sicht des Labors #826 kommt im Augenblick des Ereignisses 2 der Begegnung der Uhr U-C mit der Uhr U-B gerade eine (im System der U-C neben dieser auf der x-Achse ruhende und mit ihr synchron gehende) Uhr U-D an der Uhr U-A (Ursprung, weil dort die Nullstellung erfolgte) vorbei und zeigt eine um Gamma weiter fortgeschrittene Zeit an, umgekehrt symmetrisch zu den Anzeigen der Uhren U-B / U-C.

    Allein deswegen ist Behauptung eines langsameren “Zeitflusses” (Embacher) im System der Uhren U-C und U-D eine Verstiegenheit (das U ist mühsam).

    Könnte es sein, dass manche Physiker Freude daran finden, mit dem paradoxen Dogma einer ein ganzes System befallenden “Zeitdilatation” den Widerspruch des “gesunden Menschenverstandes” herauszufordern, um ihn anschließend mit überlegenem Sachwissen der Lächerlichkeit preiszugeben? Auf welche Zielgruppe haben sie es dabei abgesehen?

    P.S. Wie geht kursiver Text?

  827. #828 MartinB
    9. Februar 2022

    “Könnte es sein, dass manche Physiker Freude daran finden, mit dem paradoxen Dogma einer ein ganzes System befallenden “Zeitdilatation” den Widerspruch des “gesunden Menschenverstandes” herauszufordern, um ihn anschließend mit überlegenem Sachwissen der Lächerlichkeit preiszugeben?”
    Nein. Das lässt sich auch dadurch belegen, dass PhysikerInnen dann im internen Umgang diese Begriffe nicht verwenden würden, wenn sie nur zur Verwirrung der Außenwelt dienen sollten. Tun sie aber. Zilgruppe für solche Begriffe sind die PhysikerInnen selbst.
    Es mag sein, dass wir nicht immer sauber mit diesen Begriffen operieren, wenn wir versuchen, Dinge populärwissenschaftlich darzustellen. Daraus bösen Willen abzuleiten, finde ich extrem unverschämt.
    Statt einzusehen, dass *Du* ein Verständnisproblem mit dem Begriff hast, unterstellst du lieber eine Verschwörung.

  828. #829 Anonym_2022
    9. Februar 2022

    @Peter Strohmayer (9. Februar 2022) #827

    “P.S. Wie geht kursiver Text?”

    Text <i>kursiv</i&#62

    ergibt:
    Text kursiv

  829. #830 Anonym_2022
    9. Februar 2022

    @Peter Strohmayer (9. Februar 2022) #827

    Korrektur:

    <i>kursiv</i> Text2

    ergibt:
    kursiv Text2

  830. #831 Peter Strohmayer
    9. Februar 2022

    @Martin

    Kein Verständnisproblem, sondern ein Begriffsproblem.

    Zeitdilatation bedeutet nicht, dass alle Uhren eines synchronen Uhrensatzes des anderen Systems zwischen zwei Zeitpunkten „langsamer gehen“, sondern dass einzelne Uhren verschiedener synchronisierter Uhrensätze des andern Systems zwischen zwei Zeitpunkten „langsamer gehen“. Das andere System wird als eine aus unterschiedlichen Zeit- und Raumschichten zusammengesetzte Wirklichkeit verzeichnet.

    Zwischen diesen beiden Bedeutungen liegen Welten. Letztere hält dem Leser vor Augen, dass er sich Wissen aneignen muss, bevor er darüber urteilen kann. Die erstere veranlasst, an Wunder zu glauben, oder provoziert den bekannten Widerspruch, an dem sich die „Einstein-Versteher“, Welträtsellöser“ und „Verschwörungstheoretiker“ abarbeiten.

    Den Bedeutungsunterschied wider besseres Wissen herunterzuspielen oder zu verschweigen, ist aber keine Verschwörung, sondern Folge einer ganz gewöhnlichen Charaktereigenschaft.

    („PhysikerInnen“: Ich hoffe Du verübelst es in Faschingszeiten nicht, wenn ich die Frage weiterleite, wie man ein herrenloses Damenfahrrad gendergerecht meldet.)

    @Anonym
    Vielen Dank.

  831. #832 Anonym_2022
    9. Februar 2022

    @Peter Strohmayer (9. Februar 2022) #827

    “Zur Symmetrie: Aus Sicht des Labors #826 kommt im Augenblick des Ereignisses 2 der Begegnung der Uhr U-C mit der Uhr U-B gerade eine (im System der U-C neben dieser auf der x-Achse ruhende und mit ihr synchron gehende) Uhr U-D an der Uhr U-A (Ursprung, weil dort die Nullstellung erfolgte) vorbei und zeigt eine um Gamma weiter fortgeschrittene Zeit an, umgekehrt symmetrisch zu den Anzeigen der Uhren U-B / U-C.

    D.h. Im Ruhesystem der Uhren U-C und U-D bewegt sich die Uhr U-A von der Uhr U-C zur Uhr U-D und geht beim Treffen mit der Uhr U-D im Vergleich zu dieser nach.

    Dadurch wird klar, dass die Zeitdilatation, wie Wolfgang Rinder sie definiert, nicht paradox ist.

    Es ist jeweils immer die Eigenzeit der bewegten Uhr, die verlangsamt läuft, bezogen auf die Koordinatenzeit des Bezugssystems.

  832. #833 Peter Strohmayer
    9. Februar 2022

    @Anonym
    “Die Zeitdilatation, wie Wolfgang Rinder sie definiert, …” (kursiv schaff’ ich’s nicht ..)

    Ich stimme Dir zu, diese Definition ist nicht paradox, ich schätze sie sehr (#821). Würde man nur sie heranziehen und keine Geschichten über Minutenwalzer oder Eierkochen erzählen, würde sich die Wahrscheinlichkeit wertvoller Diskurse erhöhen.

    Embacher hält sich zB in seinem Lehrbuch (Elemente, Bd. I, 2.2.2) klug zurück wie Rindler. Feynman, Pössels Pate, geht einen anderen Weg (Vorlesungen I, 15-4). Er leistet sich zunächst den Denkfehler, dass “in der bewegten Uhr die effektive Zeit zwischen den Klicks länger ist” und somit “die bewegte Uhr … langsamer zu laufen scheint” (die Wurzel dieses Denkfehlers liegt darin, dass das Photon in der bewegten Lichtuhr blauverschoben ist, also das Gegenteil passiert, als was zu beweisen versucht wird.).

    Und er setzt dann fort:
    “Wenn alle bewegten Uhren langsamer gehen, wenn keine Meßmethode der Zeit etwas anderes als eine langsamere Rate ergibt, so müssen wir lediglich in einem gewissen Sinn sagen, dass die Zeit selber in einem Raumschiff langsamer erscheint. Alle vorkommenden Phänomene – des Mannes Pulsschlag, sein Denkprozess, die zum Anzünden einer Zigarette benötigte Zeit, wie lange es dauert aufzuwachsen und älter zu werden – alle diese Dinge müssen im gleichen Verhältnis langsamer werden, weil er nicht feststellen kann, dass er sich bewegt. Die Biologen und Mediziner sagen manchmal, dass es nicht gewiss ist, dass die zur Entwicklung von Krebs benötigte Zeit im Raumschiff länger ist, aber vom Standpunkt der modernen Physik ist es nahezu Gewissheit; andernfalls könnten wir die Entstehungsdauer des Krebses verwenden, um die Geschwindigkeit im Schiff zu bestimmen.” (Zigaretten und Krebs waren damals noch salonfähig.)

    Ich möchte nicht behaupten, dass Feynman dies wieder besseres Wissen machte. Meine Hoffnung ist indes, dass solche verfälschenden Übertreibungen in Bezug auf “alle bewegten Uhren … in einem Raumschiff” den, der sich ihrer bedient, einer zunehmend kritischen Betrachtung aussetzt.

  833. #834 Anonym_2022
    10. Februar 2022

    @Peter Strohmayer (9. Februar 2022) #833

    “(kursiv schaff’ ich’s nicht ..)”

    Dann markiere mit der Maus die komplette folgende Zeile …

    <i>“Die Zeitdilatation, wie Wolfgang Rinder sie definiert”</i> (kursiv schaff’ ich’s)

    und kopiere sie in dein Kommentarfeld per copy/paste. Dann prüfe die Vorschau.

    “Würde man nur sie heranziehen und keine Geschichten über Minutenwalzer oder Eierkochen erzählen, würde sich die Wahrscheinlichkeit wertvoller Diskurse erhöhen.”

    Was mit den Geschichten wohl (richtigerweise) gesagt werden soll:

    Die Zeitdilatation tritt nicht nur bei einer bewegten Lichtuhr auf, sondern bei jeder beliebigen Art von bewegter Uhr:
    • Lichtuhr
    einfaches Masse/Feder-System
    Unruh-Uhr
    • Eierkocher
    • Klavierspieler
    • Zwilling
    Quartz-Uhr
    • Atomuhr

  834. #835 Peter Strohmayer
    10. Februar 2022

    @Anonym
    Danke, ich habe nicht auf die Vorschau geachtet.

    “Was mit den Geschichten wohl (richtigerweise) gesagt werden soll:”

    Ich stimme Dir zu, wenn klar würde, dass damit nur eine beliebige (Alltags)Uhr an einem Punkt im Raum gemeint ist.

    Tatsächlich ist es aber in seiner Semantik und gewollten Suggestivität für jeden Durchschnittsmenschen die Schilderung eines verlangsamten Alltags in einem bewegten menschenbewohnten Lebensraum. Die ständige Wiederholung dieser “Sensationsnachricht” bewirkt, dass ein mehr oder weniger großer Teil der Öffentlichkeit unter “Zeitdilatation” eine Verlangsamung a la Dornröschenschlaf versteht. Das war vor hundert Jahren so, das ist heute noch so und das kann man jederzeit durch Befragungen nachprüfen. Selbst aus dem richtigen Satz “bewegte Uhren gehen langsamer” ziehen die meisten falsche Schlüsse.

    Es handelt sich dabei nicht um ein unwichtiges Detail in einem komplexen Wissensgebiet, sondern aus der Warte der Aufklärung um eine kulturelle Katastrophe.

    Da “die Zeitdilatation” (im Sinn Rindlers) nur von Spinnern bezweifelt werden kann, ist “die Zeitdilatation” (wie sie dem Publikum geschildert wird) zu einem paradoxen Dogma geworden, das – wie alle von Uneingeweihten nicht widerlegbaren Dogmen – zu Machtzwecken instrumentalisiert werden kann.

  835. #836 Anonym_2022
    12. Februar 2022

    @Peter Strohmayer (10. Februar 2022) #835

    “Ich stimme Dir zu, wenn klar würde, dass damit nur eine beliebige (Alltags)Uhr an einem Punkt im Raum gemeint ist.”

    Damit ist nicht nur eine Alltagsuhr gemeint, sondern jeder physikalische, chemische oder biologische Vorgang an einem (bewegten) Punkt im Raum, da das Zeitintvall selbst transformiert wird.

    Die ZD betrifft z.B. das Zeitintervall zwischen dem Ereignis “die Mitte des Eis beginnt zu kochen” und dem Ereignis “die Mitte des Eis nimmt die Konsistenz eines hartgekochten 5-Minuten-Eis an”.

  836. #837 Karl-Heinz
    Graz
    12. Februar 2022

    @Anonym_2022

    Ist eine Diskussion mit Peter Strohmayer nicht mühsam? Warum nicht die Diskussion mit Peter Strohmayer auf wenige Fakten in leicht verständlicher Form reduzieren. Entspricht seine Antwort nicht unseren Erwartungen, da würde ich persönlich die Sinnlosigkeit einer Fortführung in Betracht ziehen. 😉

  837. #838 Peter Strohmayer
    12. Februar 2022

    @Anonym
    “Damit ist nicht nur eine Alltagsuhr gemeint”
    Natürlich nicht, sondern jeder eichfähige Prozess.

    “da das Zeitintervall selbst transformiert wird”
    Sexl ua, Relativität, Gruppen, Teilchen3, S 32:
    “In der Form [t’=t*Gamma] und [t=t’*Gamma] geschrieben erscheint die Zeitdilatation zunächst paradox, wenn man diese Formeln als Transformationsformeln für t bzw. t’ missversteht. Das ist natürlich nicht der Fall … . (…) [t’=t*Gamma] und [t=t’*Gamma] sind vielmehr Relationen zwischen bestimmten Zeitintervallen (und nicht Zeitkoordinaten) (…). Die Literatur über dieses Missverständnis, [t’=t*Gamma] und [t=t’*Gamma] als Transformationsformeln anzusehen, ist sehr umfangreich.”

    Da wird also kein Zeitintervall transformiert.

    Es gibt iZm dem Begriff der “ZD” nur ein einziges, kärgliches Faktum und eine einzige Erklärung: bei Begegnung zweier jeweils synchronisierter Uhren verschiedener Systeme weichen deren Anzeigen voneinander ab. Dieses unbestreitbare Faktum ist auf die Konstanz und die Endlichkeit der “Lichtgeschwindigkeit” zurückzuführen.

    Alles andere ist hinzugedichtet, insbesondere die Unterstellung, dass die Abweichung der Anzeigen auf ein “anderes Vergehen vonZeit” (mit den üblichen Vorbehalten) zurückzuführen wäre. Jeder, der eine solche Meinung teilt, ist über den unbrauchbaren Zeitbegriff des 18. Jhdts nicht hinausgekommen.

  838. #839 MartinB
    12. Februar 2022

    @Karl-Heinz
    Deswegen bin ich ja schon lange größtenteils raus; insbesondere, wenn dann auch noch Dogma, der Versuch, Menschen zu verwirren etc. zitiert wird.
    Je nach Koordinatensystem haben Strecken entlang der Koordinatenachsen unterschiedliche Längen. Gilt im Raum und in der Raumzeit. Wenn sich das Zeitintervalle zwischen zwei Ereignissen unterscheidet, nennt man das halt Zeitdilatation, weil es ein Phänomen ist, für das man einen Namen braucht, weil es oft physikalisch relevant ist.
    Ich habe ehrlich keine Ahnung, was es da zu diskutieren gibt.

  839. #840 Anonym_2022
    13. Februar 2022

    @Peter Strohmayer (12. Februar 2022) #838

    “Die Literatur über dieses Missverständnis, [t’=t*Gamma] und [t=t’*Gamma] als Transformationsformeln anzusehen, ist sehr umfangreich.”
    Da wird also kein Zeitintervall transformiert.”

    Mit der LT wird ein Zeitintervall transformiert (auch für einen nicht-eichfähigen Prozess), und zwar wie folgt:

    Meistens findet man in Physikbüchern die LT für die Zeit in folgender Notation:

    t’ = γ (t-vx/c²)

    Dann wird vorausgesetzt, dass die Ursprünge beider Koordinatensysteme zum Zeitpunkt t=t’=0 am selben Ort waren.

    Mir gefällt eine andere Notation besser, die ich wie folgt berechne. Zunächst wende ich die o.g. LT auf zwei Ereignisse E₁ und E₂ an:

    t₁’ = γ (t₁-vx₁/c²)
    t₂’ = γ (t₂-vx₂/c²)

    Nun subtrahiere ich die erste Gleichung von der zweiten. Mit
    ∆t := t₂ – t₁
    ∆t’ := t₂’ – t₁’
    ∆x := x₂ – x₁
    (∆x’ := x₂’ – x₁’)
    gilt:

    t₂’ – t₁’ =
    ∆t’ = γ (∆t – v/c² ∆x).

    Am “∆”-Symbol sieht man, dass ich zeitliche Intervalle und räumliche Abstände transformiere.

    Wenn die Uhr im System S ruht, dann gilt für zwei ihrer Tick-Ereignisse:
    ∆x = 0.
    Unter dieser (wichtigen) Einschränkung handelt es sich bei ∆t nur um das Eigenzeitintervall ∆τ.

    D.h.

    ∆t’ = γ (∆τ – v/c² ∆x) = γ ∆τ.

    Das Koordinaten-Zeitintervall in System S’ ist größer als das Eigenzeitintervall ∆τ.

    Jetzt ist die Notation so gewählt, dass man einfach erkennt, dass auch bei der Transformation in Gegenrichtung wieder das Koordinatenzeitintervall des neuen Bezugssystems größer als das Eigenzeitintervall am bewegten Ort ist.

    “weichen deren Anzeigen voneinander ab. Dieses unbestreitbare Faktum ist auf die Konstanz und die Endlichkeit der “Lichtgeschwindigkeit” zurückzuführen.”

    Das als einzige Bedingung reicht nicht und das Wort “Lichtgeschwindigkeit” ist missverständlich.

    Die ZD folgt aus der LT, wenn die Konstante c in der LT endlich ist. Wäre sie unendlich, dann würde die LT mit der Galilei-Transformation übereinstimmen und es würde Newton’s absolute Zeit gelten.

  840. #841 Anonym_2022
    13. Februar 2022

    @Karl-Heinz (12. Februar 2022) #837

    “Ist eine Diskussion mit Peter Strohmayer nicht mühsam?”

    Sie ist relativ einfach, weil er die Lorentz-Transformation akzeptiert (anders, als einige andere SRT-Kritiker).

    Auf die Verschwörungstheorie ist ja @MartinB schon eingegangen.

  841. #842 Karl-Heinz
    Graz
    13. Februar 2022

    @Anonym_2022

    Ich befürchte du zerstört gerade die Reputation von Peter Strohmayer. Kennst du seine Internetseite? Weißt du, dass er auch ein Buch geschrieben hat mit dem Titel “Relationale Symmetrie des Lichts – Annäherung an die spezielle Relativitätstheorie”.
    Glaubst du wirklich, dass er seinen Fehler einsehen wird?

  842. #843 Anonym_2022
    14. Februar 2022

    @Karl-Heinz (13. Februar 2022) #842

    “Glaubst du wirklich, dass er seinen Fehler einsehen wird?”

    Das wird sich herausstellen. Er hat ja in #799 die Diskussion nach kurzer Pause wieder aufgenommen. Fortschritte kann ich erkennen.

  843. #844 Anonym_2022
    14. Februar 2022

    Hier ein Link zur weltweit schnellsten Herleitung der LT:

    http://www.faculty.luther.edu/~macdonal/LorentzT.pdf

    via:
    http://www.faculty.luther.edu/~macdonal/

    Er setzt dabei die ZD (Gamma ungleich 1) nicht voraus. Er leitet die Formel für Gamma zusammen mit der LT her.

  844. #845 MartinB
    14. Februar 2022

    Sehr nett, diese Herleitung.

  845. #846 Peter Strohmayer
    14. Februar 2022

    @Anonym #840

    Natürlich bleibt unter der Voraussetzung Delta x=0 rechnerisch das Zeitintervall als Gegenstand der Transformation übrig. Aber deswegen verschwindet der genuine Zusammenhang mit den Raumkoordinaten nicht. Das ist das Missverständnis, vor dem Sexl warnt.

    Was den umstrittenen und schwankenden Begriff der ZD betrifft, so respektiere ich, wenn für Dich das Verstehen der Welt mit dem Auffinden eines mathematischen Formalismus zusammenfällt, der sich experimentell bewährt. Dann musst Du nicht weiter darüber nachdenken.

    @Martin
    „… nichts zu diskutieren“:

    Ich habe für diesen Standpunkt grundsätzlich Verständnis. Begriffe sind in der Physik anstrengender als Formeln.

    @Martin: „Wenn sich [aus der Sicht zweier Systeme] die Zeitintervalle zwischen zwei Ereignissen unterscheiden, nennt man das Zeitdilatation.“

    Nach dieser Definition kann es je nach räumlicher Anordnung der Ereignisse sowohl zu einer Verlängerung als auch zu einer Verkürzung des Zeitintervalls zwischen ihnen kommen. Bisher wurde vermieden, das als ZD zu bezeichnen. Ihr würde ein Satz teils langsamer teils schneller gehenden Uhren entsprechen. Dies würde den tradierten Begriff der ZD sprengen.

    „Wenn sich aus der Sicht zweier Systeme die Zeitintervalle zwischen zwei Ereignissen, die in einem System am selben Ort stattfinden, unterscheiden, nennt man das Zeitdilatation.“

    Es gibt bei zeitartigen Abständen immer ein System, in dem zwei Ereignisse am selben Ort stattfinden. Das angesprochene Problem der Raumabhängigkeit der Richtung der Veränderung des Zeitintervalls würde dadurch nicht entschärft. Aber zumindest wären raumartige und lichtartige Abstände bei dem Geschehen, das man mit dem Begriff ansprechen will, bei dieser Formulierung ausgeschlossen.

    „Wenn sich aus der Sicht zweier Systeme die Zeitintervalle zwischen zwei Ereignissen, die in einem der zu vergleichenden Systeme am selben Ort stattfinden, unterscheiden, nennt man das Zeitdilatation.“

    Es gibt bei zwei zu vergleichenden Systemen immer zwei Konstellationen, in denen zwei Ereignisse am selben Ort stattfinden, nämlich entweder im System S oder im System S’. Finden die Ereignisse zB beide im Ursprung des System S statt, so kommt es aus Sicht dieses Systems zu einer Verlängerung des Zeitintervalls im System S’, was als „schneller gehende Uhr“ interpretiert werden müsste. Dieser Aspekt soll aber ausgeklammert werden, weil klar würde, dass die L-T mit langsamer gehenden Uhren nichts zu tun haben kann.

    „Wenn sich aus der Sicht zweier Systeme die Zeitintervalle zwischen zwei Ereignissen, die im anderen System am selben Ort stattfinden, unterscheiden, nennt man das Zeitdilatation.“

    Das scheint den angestrebten Begriff zu treffen und stimmt mit der Anmerkung Anonyms überein, man vergleiche eine Koordinatenzeit des eigenen Systems mit der Eigenzeit des anderen. Diese Definition deckt aber nicht das ab, was der frühe Einstein („die Zeit vergeht am Äquator langsamer“), Feynman, Pössel usw als langsameres Vergehen der Zeit in einer Kulisse menschlichen Erlebens beschreiben. Von den vielen in der Kulisse gleichzeitig stattfindenden Ereignispaaren des anderen Systems mit den gleichen Zeitintervallen kann im eigenen System zu einem bestimmten Zeitpunkt immer nur ein einziges dieser Ereignispaare ausgemacht werden, bei dem sich das Zeitintervall so verändert, wie dies der Begriff „Zeitdilatation“ beschreibt.

    Spannend in Bezug auf ein angebliches „anderes Vergehen der Zeit“ wird es erst, wenn ein oder mehrere Massepunkte das Bezugssystem wechseln (Bell’sche Raumschiffe; Zwillingsparadoxon). Aber davon soll bei der ZD gleichförmig bewegter Beobachter nicht die Rede sein.

    @Anonym #844:
    Ich behauptete, dass die Abweichung der Anzeigen zweier Begegnungsuhren nur auf die Konstanz und die Endlichkeit der “Lichtgeschwindigkeit” zurückzuführen ist (#838“). Die Ausbreitung eines Lichtpulses ist nämlich eine Uhr. Verschiedene Längen der gemeinsamen Ausbreitung zweier zugleich ausgesendeter Lichtpulse führen zu verschiedenen zeitlichen Entfernungen aus der Sicht zweier Systeme. Damit ist unmittelbar klar, dass zwei Begegnungsuhren (bis auf wenige Ausnahmen in der “relativistischen Mitte”) nicht dieselbe Zeit anzeigen können. Der Unterschied wird durch den mathematischen Formalismus der L-T beschrieben. Deswegen gehen sie aber nicht unterschiedlich schnell. Die absolut gleiche “Ablaufgeschwindigkeit” wird ja bei der Ausbreitung der Lichtpulse, die diese Uhr sind, ausdrücklich vorausgesetzt. Ein langsameres Gehen der Uhren ist ein Widerspruch zur Konstanz von c (die üblichen Einschränkungen versuchen nur, den verfehlten Ansatz abzuschwächen).

    Ich sehe nicht, dass Macdonald’s bei seiner l-T Ableitung noch weitere Annahmen treffen müsste:

    „Eine Uhr, die sich mit konstanter Geschwindigkeit v in einem Inertialsystem I bewegt, läuft in Bezug auf die synchronisierten Uhren von I, die sie passiert, [wegen des Relativitätsprinzips] mit einer konstanten Rate anders.“

    Auch er geht von Begegnungsuhren aus und setzt den eigentlich neuen Gedanken, deren unterschiedlichen Gang, von vornherein voraus. Kann man machen. Ist aber gewöhnungsbedürftig.

    Stellt man auf die eigentlichen Uhren, die Ausbreitung der Lichtpulse ab, so ist die besagte Rate der unterschiedlichen Länge der Ausbreitung von Lichtpulsen (Stand der beiden Uhren) die Folge davon, dass sich Photonen gegenseitig nicht überholen können. Das ist der Grund des Relativitätsprinzips. Ob man aber die gemeinsame Ausbreitung von Lichtpulsen oder – wahrscheinlich ohne zu verstehen – gleich die unterschiedliche Anzeige von Begegnungsuhren voraussetzt, ist für die mathematische Ableitung der L-T bedeutungslos.

  846. #847 Anonym_2022
    14. Februar 2022

    @MartinB (14. Februar 2022) #845

    “Sehr nett, diese Herleitung.”

    T+X und T-X sind Lichtkegel-Koordinaten.

    Quelle:
    https://www.physicslog.com/blog/2019/03/lightcone-coordinates/

  847. #848 Anonym_2022
    14. Februar 2022

    @Peter Strohmayer (14. Februar 2022) #846

    “Was den umstrittenen und schwankenden Begriff der ZD betrifft, so respektiere ich, wenn für Dich das Verstehen der Welt mit dem Auffinden eines mathematischen Formalismus zusammenfällt, der sich experimentell bewährt. Dann musst Du nicht weiter darüber nachdenken.”

    Dazu passend gibt es gute Anmerkungen von R. Feynman im folgenden Video “Mathematicians versus Physicists”:



    “Ich sehe nicht, dass Macdonald’s bei seiner l-T Ableitung noch weitere Annahmen treffen müsste: … Auch er geht von Begegnungsuhren aus und setzt den eigentlich neuen Gedanken, deren unterschiedlichen Gang, von vornherein voraus. Kann man machen. Ist aber gewöhnungsbedürftig.”

    Er setzt letztendlich die beiden SRT-Postulate voraus. Weitere Annahmen von ihm sind also
    (A) die Invarianz der LG (gleiche LG in jedem Bezugssystem) und
    (B) das Relativitätsprinzip, das schon Galilei formuliert hat (Jedes inertiale Bezugssystem kann glechberechtigt als “das ruhende System” definiert werden).

    Er setzt den unterschiedlichen Gang gerade nicht voraus, siehe seine Fußnote ².

    Assume:
    (A) The speed of light is the same in all inertial frames. (Take c = 1.)
    (B) A clock moving with constant velocity v in an inertial frame I runs at a constant rate γ = γ(|v|) with respect to the synchronized clocks of I which it passes.²

    Assumption (B) follows directly from the relativity principle.

    ²This does not assume time dilation: as far as (B) is concerned, γ could be identically 1.

    Quelle:
    http://www.faculty.luther.edu/~macdonal/LorentzT.pdf

  848. #849 Anonym_2022
    14. Februar 2022

    @Peter Strohmayer (14. Februar 2022) #846

    Ergänzung zu #848

    Das Relativitätsprinzip beinhaltet, dass in allen Inertialsystemen die gleichen physikalischen Gesetzte gelten.

    Daraus folgt, falls im ungestrichenen System eine mit |v| bewegte Uhr um γ verlangsamt läuft, dann läuft auch im gestrichenen System eine mit |v| bewegte Uhr um γ verlangsamt. Wobei γ zunächst nicht als ungleich 1 vorausgesetzt wird. D.h. es wird keine ZD vorausgesetzt. Die wird erst im Laufe der Herleitung der LT mit hergeleitet.

  849. #850 Anonym_2022
    14. Februar 2022

    @Peter Strohmayer (14. Februar 2022) #846

    “Das scheint den angestrebten Begriff zu treffen und stimmt mit der Anmerkung Anonyms überein, man vergleiche eine Koordinatenzeit des eigenen Systems mit der Eigenzeit des anderen. Diese Definition deckt aber nicht das ab, was der frühe Einstein („die Zeit vergeht am Äquator langsamer“), … als langsameres Vergehen der Zeit in einer Kulisse menschlichen Erlebens beschreiben.”

    Doch, diese Definition deckt das ab, was der frühe Einstein („die Zeit vergeht am Äquator langsamer“) 1905 geschrieben hat. Er hat die SRT-Zeitdilatation der Äquator-Uhr mit dem 1/Gamma-Faktor berechnet, siehe Seiten 904 und 905:

    https://de.wikibooks.org/wiki/A._Einstein:_Kommentare_und_Erl%C3%A4uterungen:_Zur_Elektrodynamik_bewegter_K%C3%B6rper:_Kinematischer_Teil:_%C2%A74

    Bei Einstein ruht die Nordpol-Uhr nicht nur im rotierenden System, sondern auch im nicht-rotierden Inertiasystem (ECI). Er kann daher im ECI rechnen und für die Äquator-Uhr die SRT-Zeitdilatation ansetzen (nur bei Vernachlässigung von gravitativer Zeitdilatation).

    Wenn man im Einstein-Szenario allerdings zusätzlich noch gravitative Zeitdilatation berücksichtigt (die er 1905 nicht kannte), dann wird dessen Zeitdilatation zwischen der Uhr am Nordpol und der am Äquator dadurch wegen der Erdabplattung fast exakt kompensiert.

    Siehe auch:
    “Einsteins Theorien: Spezielle und Allgemeine Relativitätstheorie für interessierte Einsteiger zur Wiederholung”, Springer Spektrum
    von Bernd Sonne, Reinhard Weiß

    Kapitel 14.3 “Uhrenvergleich am Äquator und am Pol”

    Die Seite ist bei mir mit dem Browser “Chrome” sichtbar:

    https://books.google.de/books?id=SEckBAAAQBAJ&lpg=PA168&ots=EcmAK7b1UI&dq=einstein%20zeitdilatation%20nordpol%20%C3%A4quator&hl=de&pg=PA171#v=onepage&q=einstein%20zeitdilatation%20nordpol%20%C3%A4quator&f=false

  850. #851 Peter Strohmayer
    14. Februar 2022

    @Anonym
    Einsteins Vorstellung war damals schwärmerisch. Er verlautbarte ohne festes Tatsachensubstrat die Sensation einer anders vergehenden Zeit “am Äquator”. Zwischen einer geradlinigen gleichförmigen Bewegung und einer Kreisbewegung bestehen aber Unterschiede, die es auch ohne Gravitation ausschließen müssten, daran analogisierende Spekulationen auf der Ebene der SRT anzuknüpfen.

    Der Rede von einer langsamer vergehenden Lebbenswirklichkeit im Rahmen der SRT stehe ich im Hinblick auf die “aufgesplitterte Realität” die aus der Sicht des anderen Systems abgebildet wird, und wegen der von Dir auch oben wieder eingeräumten Symmetrie ablehnend gegenüber. Du teilst die Bedenken nicht, und damit kann es sein Bewenden haben.

    Die von Dir hervorgeholten Materialien lassen bei dem, der sie zitiert, das Gefühl entstehen, die Wahrheit zu sagen. Auf diese Weise wird allerdings nicht immer auf den zu beurteilenden Gedankengang eingegangen. Macht aber nichts. Diene Materialen sind wirklich gut und eine Bereicherung. Dein Zugang, die Diskussion nicht emotional zu führen, sollte beispielgebend sein.

  851. #852 Anonym_2022
    15. Februar 2022

    Peter Strohmayer (14. Februar 2022) #851

    “Zwischen einer geradlinigen gleichförmigen Bewegung und einer Kreisbewegung bestehen aber Unterschiede, die es auch ohne Gravitation ausschließen müssten, daran analogisierende Spekulationen auf der Ebene der SRT anzuknüpfen.”

    Das ist falsch. Wie Einstein 1905 auf Seite 904 geschrieben hat, kann man viele kleine geradlinig gleichförmige Bewegungen zu einer beliebigen polygonalen Linie zusammensetzen und den Grenzübergang zu einer stetig gekrümmten Kurve durchführen, also auch zu einer Kreisbahn. Solange der Geschwindigkeitsbetrag konstant bleibt, kann man in einem inertialen Bezugssystem die Zeitdilatation mit einem konstanten Gamma berechnen, genauso wie bei einer geradlinig gleichförmigen Bewegung.

    Quelle (siehe Seite 904):
    https://de.wikibooks.org/wiki/A._Einstein:_Kommentare_und_Erl%C3%A4uterungen:_Zur_Elektrodynamik_bewegter_K%C3%B6rper:_Kinematischer_Teil:_%C2%A74

    P.S.
    Und er schreibt auch, dass die Punkte A und B im betrachteten Bezugssystem am selben Ort sein können. Dann wird mit der stetig gekrümmten Bahn von A nach B das Zwillings”paradoxon” realisiert, bei dem daher derselbe 1/Gamma-Faktor die Alterung der bewegten Uhr bestimmt, bezogen auf die Alterung der ruhenden Uhr.

  852. #853 Karl-Heinz
    Graz
    15. Februar 2022

    @Anonym_2022

    Dein Beitrag in #852 müsste eigentlich ein Schock für Peter Strohmayer sein. So ein Teilchen auf der Kreisbahn unterliegt ja andauernd einer Richtungsänderung und damit einer Beschleunigung. Für die Berechnung der Zeitdilatation bis zur Rückkehr wird aber nur die Geschwindigkeit benötigt. Widerspricht also dem, was Peter uns weismachen will.

  853. #854 Karl-Heinz
    Graz
    15. Februar 2022

    @Anonym_2022

    Und was Peter Strohmayer noch gerne macht ist, dass er den Dopplereffekt mit ins Spiel bringt. Wenn ich ein System von Uhren habe, die synchronisiert sind, dann werden Ereignisse instant zu den jeweiligen Orten erfasst. Mit anderen Worten, es werden Ereignisse und nicht primär die Farbe des Lichtes erfasst. Kannst du das Peter irgendwie schonend beibringen? Ich weiß, muss hart für Peter sein, vor allem, wenn es von einem Laien, wie mir, kommt. 😉

  854. #855 Anonym_2022
    15. Februar 2022

    @Karl-Heinz (15. Februar 2022) #853

    “Dein Beitrag in #852 müsste eigentlich ein Schock für Peter Strohmayer sein. So ein Teilchen auf der Kreisbahn unterliegt ja andauernd einer Richtungsänderung und damit einer Beschleunigung. Für die Berechnung der Zeitdilatation bis zur Rückkehr wird aber nur die Geschwindigkeit benötigt. Widerspricht also dem, was Peter uns weismachen will.”

    Das will er uns bestimmt nicht mehr weismachen, da er inzwischen die Kommentare #133 und #182 gelesen hat.

  855. #856 Anonym_2022
    15. Februar 2022

    In seiner populären Veröffentlichung “The Special and General Theory” von 1916 hat Einstein die SRT-ZD-Formel benutzt, um die gravitative Zeitdilatation im rotierenden Bezugssystem auf einer rotierenden Scheibe zu berechnen. Eine Uhr befindet sich im Zentrum der Scheibe, die andere auf dem Rand. Durch die Fliehkraft entsteht eine nach außen gerichtete Pseudogravitation mit einer gravitativen ZD. Die Uhr auf dem Rad geht verlangsamt gegenüber der Uhr im Zentrum.

    Quelle (siehe Abschnitt “c Displacement of Spectral Lines Towards the Red”):

    https://en.wikisource.org/wiki/Relativity:_The_Special_and_General_Theory/Appendix#(c)_Displacement_of_Spectral_Lines_Towards_the_Red

  856. #857 stone1
    16. Februar 2022

    WTactualF?! Naja, derart kann man sich auch die Zeit vertreiben, wenn draußen nicht viel los ist.

    Habts eure internen Uhren eh abgstimmt, nicht dass einer mehr Gravitation ausgesetzt ist als der Rest von euch wilden Physikern, sonst dila… dilata… dilatierts.

    Nö, bin gar nicht betrunken. Bloß gut aufgelegt und hab schon 2 Kaffee intus um 1/2 6 in der Früh… 🙂

  857. #858 Peter Strohmayer
    16. Februar 2022

    @Anonym
    #852, #856: Zeitdilatation am Äquator. Durch Aufgliederung eines Bogens in infinitesimale Streckenabschnitte wird aus einem Kreis keine Gerade. Das Verhältnis zweier zueinander gleichförmig bewegter Beobachter kann nicht auf das Verhältnis zweier Beobachter in einem rotierenden Bezugssystem übertragen werden. Dort ist die Situation nicht mehr reziprok. Du verweist selbst auf die Anwendbarkeit des Zwillingsparadoxons dem keine gleichförmigen Bewegungen zu Grunde liegen und das ohne (infinitesimale) Übertritte von einem Inertialsystem in ein anderes (=Beschleunigung) nicht darstellbar ist. .

    Vor allem: Ich wende mich gegen eine Mystifizierung der “Zeitdilatation” und habe das am Bespiel übertriebener Behauptungen demonstriert. Du greifst einen eher zufällig ins Spiel gekommenen Aspekt (Zeitdilatation am Äquator) heraus und “widerlegst” mich mit diversen Zitaten, die wieder andere (diskussionswürdige) Themen ins Spiel bringen. So bist Du immer ein linientreuer Verteidiger der Wahrheit, ohne auf die merkwürdige Verstiegenheit des Dogmas eingehen zu müssen.

    Die Verteidigung solcher Bilder wie “fünf Minuten Eierkochen” usw folgt damit der Struktur theologischer Diskussionen. Dort wird belesen und korrekt auf die Bibel und auf Kirchenlehrer verwiesen, ohne jemals auf die Verstiegenheit des Dogmas einzugehen. Man kann keinem Katholiken die Jungferngeburt ausreden.

    Die Verteidigung betrifft auch das Hilfsdogma, das Zwillingsparadoxon komme ohne Beschleunigungen aus oder gleichförmig bewegte Myonen würden langsamer altern bzw. länger leben. Ohne (infinitesimale) Übertritte von einem Inertialsystem in ein anderes (=Beschleunigung) bzw. ohne Umweg in der Raumzeit gibt es aber keine langsamere Alterung. Zum erwartbaren Hinweis auf das Uhrenparadoxon merke ich an, dass die informelle Übertragung der Zeitanzeigen von Begegnungsuhren keine Physik, sondern nur physikalische Buchhaltung ist.

    Mit diesen unterschiedlichen Standpunkten kann es sein Bewenden haben. Du bekommst ja auch Applaus. Dem würde ich allerdings mißtrauen.

  858. #859 Anonym_2022
    17. Februar 2022

    @Peter Strohmayer (16. Februar 2022) #858

    “Du greifst einen eher zufällig ins Spiel gekommenen Aspekt (Zeitdilatation am Äquator) heraus und … So bist Du immer ein linientreuer Verteidiger der Wahrheit, ohne auf die merkwürdige Verstiegenheit des Dogmas eingehen zu müssen.”

    In Experimenten hielten sich auch im Kreis bewegte Müonen an das “Dogma” (ZD im Laborsystem und ZP mit γ=65):

    Im Speicherring kehren die Teilchen immer wieder zu ihrem Ausgangspunkt zurück. Das Einstein’sche Zwillingsparadoxon ist hier in der Tat realisiert: die schnell bewegten Teilchen leben immer noch, wenn ihre ruhenden Brüder schon längst ’gestorben’ sind. Dies wird schematisch in Abb. 4 dargestellt.

    Quelle:
    https://www.desy.de/~pschmues/Relativitaetstheorie.pdf

  859. #860 Peter Strohmayer
    17. Februar 2022

    @Anonym
    Führen im Kreis bewegte Myonen eine gleichförmige Bewegung aus, auf die sich meine Kritik übertreibender Dogmatik bezog?

    Du unterstellst mir, wie die Zitate zeigen, dass ich die SRT in Frage stellen würde. Das ist nicht der Fall. Ich wende mich nur gegen den populistischen Missbrauch der Theorie durch manche Physiker, und gegen solche, die sich nicht davon distanzieren und bei dem Spiel mitmachen.

    Trotzdem: ich lese die von Dir gefundenen Belegstellen gern. Sie sind meist eine Bereicherung.

  860. #861 Karl-Heinz
    Graz
    17. Februar 2022

    @Peter Strohmayer

    Weißt Peter. Philosophie ist eine Sache und Mathematik eine andere. Wie gut beherrscht du die Mathematik?
    Ich will damit andeuten, dass man mit Hilfe der Mathematik manch philosophischen Unsinn sehr schnell entlarven kann.

  861. #862 Peter Strohmayer
    18. Februar 2022

    @Karl-Heinz
    Mathematik, Philosophie und Theologie umfassen unser menschliches Dasein. Nach meiner Erfahrung sind das keine Gegner, die man gegenseitig ausspielen müsste. Man muss sich nur ihrer Grenzen bewusst sein.

  862. #863 Anonym_2022
    18. Februar 2022

    @Peter Strohmayer (17. Februar 2022) #860

    “Führen im Kreis bewegte Myonen eine gleichförmige Bewegung aus, auf die sich meine Kritik übertreibender Dogmatik bezog?”

    Ich hatte #858 so verstanden, dass du “Dogmatik” auch auf die (SRT-) “Zeitdilatation am Äquator” beziehst, also auf eine Uhr, die sich im (nicht-rotierenden) ECI-System auf einer Kreisbahn bewegt, während die Nordpol-Uhr darin ruht.

    Im selben Paper ist aber auch eine nahezu gleichförmige Bewegung von Müonen beschrieben, bezogen auf das Ruhesystem der Erdoberfläche:

    Das bekannteste Beispiel sind die μ-Teilchen der Höhenstrahlung.

    Der beobachtete hohe Fluss von Myonen der Höhenstrahlung wäre unverständlich. Die Lösung des Rätsels liegt in der Lebensdauerverlängerung. Die Teilchen entstehen mit hoher Energie, der Lorentzfaktor liegt im Mittel bei γ = 16.

    Die Strecke von 20000 m entspricht dann nur noch 2.7 Halbwertszeiten, und die Wahrscheinlichkeit, diese Strecke ohne Zerfall zurückzulegen, ist recht groß.

    Quelle:
    https://www.desy.de/~pschmues/Relativitaetstheorie.pdf

  863. #864 Peter Strohmayer
    19. Februar 2022

    @Anonym 2022

    Wenn an einem gleichförmig bewegten Myon, also ohne Wechsel von einem inerten Bezugssystem in ein anderes (bzw. eine infinitesimale Folge derartiger Wechsel), die besagten Effekte eintreten würden, wäre mein Verständnis der SRT falsch und ich müsste einen neuen Zugang zu ihr finden.

    Ich habe allerdings beim Myonenflug stets angenommen, dass es sich um eine Art “Zwilling” handelt, der in der Stratosphäre (in äußerst kurzer Zeit) auf nahe c beschleunigt wird, und dass seine Detektion auf der Erde, zumindest so lange man von einem Myon als Masseteilchen sprechen kann, mit einer negativen Beschleunigung verbunden ist. Ich käme in Verlegenheit, wenn das nicht der Fall wäre.

  864. #865 Anonym_2022
    19. Februar 2022

    @Peter Strohmayer (19. Februar 2022) #864

    “Wenn an einem gleichförmig bewegten Myon, also ohne Wechsel von einem inerten Bezugssystem in ein anderes (bzw. eine infinitesimale Folge derartiger Wechsel), die besagten Effekte eintreten würden, wäre mein Verständnis der SRT falsch und ich müsste einen neuen Zugang zu ihr finden.”

    Der “besagte Effekt” ist ZD im Ruhesystem der Erdoberfläche und die Myonen sind nahezu gleichförmig (mit “fast c”) bewegt. Sie befinden sich im freien Fall und erfahren daher auch keine Eigenbeschleunigung.

    “Ich habe allerdings beim Myonenflug stets angenommen, dass es sich um eine Art “Zwilling” handelt, der in der Stratosphäre (in äußerst kurzer Zeit) auf nahe c beschleunigt wird, und dass seine Detektion auf der Erde, zumindest so lange man von einem Myon als Masseteilchen sprechen kann, mit einer negativen Beschleunigung verbunden ist. Ich käme in Verlegenheit, wenn das nicht der Fall wäre.”

    Selbst wenn die Myonen in der Stratosphäre und an der Erdoberfläche jeweils für kurze Zeit ruhen würden (was nicht der Fall ist), wäre das kein ZP, bei dem sich Zwillingen zu 2 Ereignissen treffen würden. Außerdem betrifft die ZD auch Myonen, die nicht gemessen werden.

    Was bei den Myonen der ZD unterliegt, ist deren Halbwertszeit.

    Die Halbwertszeit … ist die Zeitspanne, nach der eine mit der Zeit abnehmende Größe die Hälfte des anfänglichen Werts (…) erreicht.

    Folgt die Abnahme einem Exponentialgesetz (siehe Abbildung), dann ist die Halbwertszeit immer die gleiche, auch wenn man die Restmenge, die nach einer beliebigen Zeit übrig ist, als neue Anfangsmenge nimmt. Bei exponentieller Abnahme charakterisiert daher die Halbwertszeit den zugrunde liegenden Prozess als solchen.

    Quelle:
    https://de.wikipedia.org/wiki/Halbwertszeit

    D.h. es wird der Myonenfluss in verschiedenen Höhen gemessen um über die e-Funktion die (um γ verlängerte) Halbwertszeit, ausgedrückt in der Koordinatenzeit des Bezugssystems, zu bestimmen. Dazu muss eine statistisch signifikant große Anzahl von Myonen detektiert werden. Es geht nicht um die Betrachtung eines einzelnen Myons, da dessen Zerfallszeitpunkt vom Zufall abhängt und die Halbwertszeit eine statistisch zu bestimmende Größe ist.

    Dieses Szenario zeigt daher schön, dass die ZD die Zeit selbst betrifft.

  865. #866 MartinB
    19. Februar 2022

    @Peter
    Dann *bist* du in Verlegenheit. Die Myonen werden nicht im eigentlichen Sinne “beschleunigt” oder “abgebremst”, sondern entstehen quasi direkt mit der jeweiligen Geschwindigkeit. Ist natürlich komplizierter, weil man eigentlich die QFT berücksichtigen muss, aber generell ist es ja so, dass man sich den Kollisionsprozess, bei dem die Myonen in der Atmosphäre entstehen, auch im Ruhesystem der Pionen anschauen kann, die die Myonen erzeugen. Das pion zerfällt und schickt ein Myon und ein Myonneutrino in seinem Ruhesystem in entgegengesetzte Richtungen aus. Bei der Absorption ist es im Prinzip dasselbe, bei diesen Quantenprozessen von einer Bescheuligung auszugehen, ist in meinen Augen ein falsches Bild.

    Hier ein Dokument, das den Myonenkram erklärt:
    https://fp.physik.unibas.ch/documents/AnleitungMyonen.pdf

    “Wenn an einem gleichförmig bewegten Myon, also ohne Wechsel von einem inerten Bezugssystem in ein anderes (bzw. eine infinitesimale Folge derartiger Wechsel), die besagten Effekte eintreten würden”
    Das tun sie ja definitiv. Man kann ja prinzipiell auch experimentell Myonen erzeugen und dann die Strecke messen, nach der sie zerfallen, ohne dass man sie irgendwie bremst. Ich weiß nicht, ob das genau so schon mal gemacht wurde, aber möglich ist es und die Theorien geben auch ne eindeutige Antwort, was das Ergebnis sein muss.

  866. #867 Anonym_2022
    20. Februar 2022

    @Peter Strohmayer (12. Januar 2020) #735

    “Fehlt noch etwas? Ach ja, das Myon! Bei gleichförmiger Eigengeschwindigkeit legt ein Materiepunkt in Eigenzeit eine relativistisch verkürzte Reisestrecke zurück, so wie der entgegenkommende Materiepunkt (das Ziel) in gleicher Eigenzeit eine relativistisch gleich verkürzte Reisestrecke zurücklegt (auch hier herrscht Symmetrie).

    Zur Klarstellung sollte man bei der Längenkontraktion immer dazu sagen, auf welches Bezugssystem man sich bezieht.

    • Im Bezugssystem der Myonen ist die (bewegte) Strecke von der Stratosphäre bis zur Erdoberfläche kontrahiert.

    • Im Bezugssystem eines Materiepunktes auf der Erdoberfläche ist die (ruhende) Strecke von der Stratosphäre bis zur Erdoberfläche nicht kontrahiert.

    Also keine Symmetrie!

    Wenn man die Symmetrie der Längenkontraktion diskutiert, muss man hier auch eine konstante Strecke ins Spiel bringen, die im Bezugssystem der Myonen ruht und im Bezugssystem des Materiepunktes auf der Erdoberfläche bewegt ist. Im Bezugssystem der Myonen altert der (bewegte) Materiepunkt auf der Erdoberfläche verlangsamt.

    “Die Eigengeschwindigkeit (ein Rechenwert) kann die Lichtgeschwindigkeit überschreiten, deshalb gelangt das Myon auf die Erde, es altert aber – Gravitation ist ausgeblendet – nicht anders als diese.”

    Mit Eigengeschwindigkeit (u) meist du vermutlich die x-Komponente der Vierergeschwindigkeit, also:
    u = ∆x / ∆τ = γ ∆x / ∆t.

    Daraus folgt aber ∆t = γ ∆τ, also die Zeitdilatation im Koordinatensystem der Erdoberfläche.

    Du hast also nur verklausuliert formuliert, dass das Myon wegen der Zeitdilatation auf die Erde gelangt.

  867. #868 Karl-Heinz
    Graz
    20. Februar 2022

    Vielleicht sollte man sich auch im klaren sein, wie man in einem Bezugsystem Längen misst.
    Warum erscheint uns ein bewegter Körper verkürzt? Tipp: siehe Gleichzeitigkeit.
    Ps: Was meine ich mit verkürzt?
    Ein Körper der sich in Ruhe zu mir befindet, hat die Länge L0. Bewegt sich dieser Körper, so erscheint er mir verkürzt.

    Umgelegt auf das Myon.
    Die Länge bis in die äußere Atmosphäre sei 10 km. Das ist die Länge im Ruhesystem. Für das Myon bewegt sich die Erde auf ihn zu. Diese Länge erscheint, dem Myon um den Lorenzfaktor verkürzt. Zum Beispiel von 10 km auf 1 km.

  868. #869 Anonym_2022
    20. Februar 2022

    @Karl-Heinz

    “Warum erscheint uns ein bewegter Körper verkürzt?”

    Man könnte das noch klarer formulieren, indem man das “erscheint uns” durch “ist” ersetzt.

    Bei der LK handelt es sich nicht um einen reinen Beobachtungseffekt, sondern um einen realen Effekt. Die kontrahierte Leiter passt – im Ruhesystem der Garage – in die Garage.

  869. #870 Karl-Heinz
    Graz
    20. Februar 2022

    Auch bezüglich Symmetrie muss man aufpassen, daß man keine falschen Schlüsse zieht. Ich nehme als Beispiel wieder das Myon und die Erdoberfläche. Wir postuliert, dass das Zeitintervall im jeweiligen Ruhesystem für beide gleich ist. Das Myon wird erkennen, dass das Zeitintervall auf der Erde gegegenüber seinem Zeitintervall um den Lorentzfaktor verkürzt erscheint. Aus Symmetriegründen gilt, das auch umgekehrt.
    Wie bestimmt man ein Zeitintervall?
    Erde: Ereignisse (Zeit, Ort), Index e steht für Erde
    E1_e … Start (0,0)
    E2_e … Ende (1,1)
    Daraus folgt
    E1_m und E2_m , Index m steht für Myon.
    Aber Achtung: Wechselt man aus Symmetriegründe die Positionen, haben die neuen Ereignisse nichts mehr mit dem ursprünglichen Ereignis zu tun.

    Jetzt wieder zurück zur eigentlichen Aufgabe
    Es gibt hier nur zwei Ereignisse
    E1 … Myon entsteht bzw. hat Endgeschwindigkeit erreicht (wenn man ohne QT argumentiert)
    E2 … Myon erreicht Erdoberfläche.
    Hier darf man nicht den Fehler machen und die Symmetrie ins Spiel bringen.
    Ich kann es bei Wunsch gerne mal vor rechnen.

    Verwendete Begriffe: Ereignis, Zeitintervall im Ruhesystem und beobachtetes Zeitintervall vom bewegten Körper, Symmetrie, …

  870. #871 Karl-Heinz
    Graz
    20. Februar 2022

    @Anonym_2022

    Danke für den Hinweis. Du hast recht. “Ist” ist die bessere Formulierung. 😉

    PS: Das schlimme ist, dass ich nach ein paar Monaten wahrscheinlich alles wieder vergessen haben werde und ich mich frage, wie ging das schnell nochmal. Was ich, so denke ich, nicht vergessen werde ist, dass die Theorie von Einstein in sich konsistent ist. 😉

  871. #872 Karl-Heinz
    Graz
    20. Februar 2022

    @Peter Strohmayer

    Bitte jetzt nicht böse sein. Wenn ich die Interpretation auf deiner Homepage versuche nachzuvollziehen, dann hackt es schon bei den ersten Abschnitten. Es hackelt mi aus. 😉

  872. #873 MartinB
    20. Februar 2022

    @Karl-Heinz
    Aber Kap. 9 von Isaac nordet einen schnell wieder ein, ging mir auch gerade so…

  873. #874 Karl-Heinz
    Graz
    20. Februar 2022

    @MartinB
    Glücklicherweise habe ich dein Buch. Sobald ich Zeit habe, werde ich mit Hilfe deines Buches die Grundlagen nochmals auffrischen.

    PS: Als ich in jungen Jahren ein Buch über die Relativitätstheorie gelesen habe, da kam auch eine Lichtuhr vor. Es wurde auch von einem Beobachter, der am Bahnsteig steht, gesprochen. Dass der Beobachter aus sich selbst und einem Satz synchronisierter Uhren besteht, die im Raum verteilt sind, wurde nirgendwo erwähnt oder ich habe es irgendwie überlesen. In meiner Vorstellung passierte dann irgendwo ein Ereignis und diese Information wurde per Lichtgeschwindigkeit zum Beobachter übertragen. Dass ich mit dieser Ansicht nicht weit kam liegt auf der Hand. Mein Gott, ich hätte dann eventuell sogar, gleich wie Peter Strohmayer, den Dopplereffekt zur damaligen Zeit berücksichtigen müssen. 😉

  874. #875 MartinB
    20. Februar 2022

    @Karl-Heinz
    Ja, ich habe mir sehr viel Mühe gegeben, solche Dinge einigermaßen akribisch zu erklären (ganz besonders in Kap. 6, 9 und bei den Schwarzschild-Koordinaten, wo sich das alles schon überpingelig liest), weil daher ja viele Missverständnisse kommen (und welche das sind, habe ich z.T. hier auf dem Blog aus Kommentaren gelernt).

  875. #876 Karl-Heinz
    Graz
    20. Februar 2022

    @MartinB
    Vielleicht machst mal eine Seite:
    Buchbesprechung: Isaac oder Die Entdeckung der Raumzeit

    wo die Leute zu den einzelnen Kapiteln diskutieren können.

    ○ Geodäten
    • Beschleunigungen und Kräfte
    • Gekrümmte Räume
    • Mehr über Koordinatensyste me Karten und Koordinaten
    • Die kürzeste Verbindung zwischen zwei
    Punkten
    • Die Metrik
    • Koordinatenbeschleunigungen
    • Geschwindigkeit und Koordinatenbeschleunigung
    • Abweichung benachbarter Geodäten
    usw.

    Bei Fragen oder Diskussionen würde ich dann halt zu Beginn das entsprechende Kapitel anführen.

  876. #877 Karl-Heinz
    Graz
    20. Februar 2022

    @Peter Strohmayer

    Wir könnten dann ja mit dem Buch von MarinB einen Neustart bezüglich Verständnis machen.
    Was hältst du von dieser Idee?

  877. #878 MartinB
    20. Februar 2022

    Ich hatte bisher nicht das Gefühl, dass da viel kommen würde, die Zahl der Rückmeldungen/Fragen zum Buch, de ich bekommen habe, ist…überschaubar…

  878. #879 Karl-Heinz
    Graz
    20. Februar 2022

    @MartinB
    Ich dachte da ja nur an eine Seite, wo man was posten kann. Der Nachteil ist halt, dass man das Buch haben muss um mitdiskutieren zu können. War nur so ein Gedanke.

  879. #880 stone1
    21. Februar 2022

    @Peter Strohmayer

    Ich hab das Buch Isaac oder Die Entdeckung der Raumzeit von MartinB auch und empfehle es ebenfalls zur Lektüre. Danach wird sicherlich Vieles beim Thema Raumzeit, was derzeit offensichtlich Probleme bereitet, besser verstanden sein.

  880. #881 MartinB
    21. Februar 2022

    @Karl-Heinz
    Ich hatte das ursprünglich auch überlegt, aber wie gesagt hatte ich bisher nicht den Eindruck, dass das wirklich gebraucht wird.
    Vielleicht mache ich aber trotzdem die Tage eine Seite auf, auf der kann ich dann auch Fehler sammeln, die mir inzwischen aufgefallen sind.

  881. #882 Anonym_2022
    21. Februar 2022

    @MartinB (21. Februar 2022) #881

    “Vielleicht mache ich aber trotzdem die Tage eine Seite auf, auf der kann ich dann auch Fehler sammeln, die mir inzwischen aufgefallen sind.”

    Eine ähnliche Seite gibt es von Harald Lesch und Josef M. Gaßner.

    moderiertes Forum mit Sammlung von Fehlermeldungen zu den Büchern:
    https://www.umwelt-wissenschaft.de/forum

    Seite für Fragen/Feedback zu den Büchern:
    https://urknall-weltall-leben.de/fragen/

  882. #884 Peter Strohmayer
    21. Februar 2022

    @Martin #866

    Von Anonym [#859 “… die schnell bewegten Teilchen leben immer noch, wenn ihre ruhenden Brüder schon längst ’gestorben’ sind”] nahm ich offenbar unberechtigt an, er gehe (so wie ich) von einem Eintreffen des Myons auf der Erde als Masseteilchen aus, was ich nicht als gleichförmige Bewegung gelten lassen wollte. Ich habe das in #864 durch die Einschränkung “zumindest so lange man von einem Myon als Masseteilchen sprechen kann” zum Ausdruck bringen wollen.

    Wenn diese Bedingung (trotz sich aufhebender Impulse von Myon und Myonneutrino) wegen angeblichen “Geborenwerdens mit einer Geschwindigkeit” nicht erfüllt ist, dann habe ich mit den aus einem solchen Sachverhalt erwachsenden Konsequenzen für Uhrenanzeigen kein Problem.

    Das Myon als Uhr begegnet einer Uhr des Erdystems, deren Zeitanzeige weiter fortgeschritten ist. Die Detektion ist kein Eintreffen des Myons als Massepunkt mit einem bestimmten Alter, sondern eine messelose (energetische) Mitteilung darüber, dass das Myon noch nicht zerfallen war. Dass die zwei Begegnungsuhren verschiedener Systems verschiedene Zeiten anzeigen und darüber Mitteilungen möglich sind, war immer meine Meinung.

    Die Uhr des Myons zeigt also die Lebenszeit des Myons ab Ursprungsdeckung (Start) an, während die begegnende Erdenuhr eine längere Zeitspanne anzeigt.

    Auch die Erdenuhr begegnet nach Ablauf der Lebenszeit eines Myons einer Uhr des Myonensystems, die genau so wie oben die Erdenuhr eine weiter vorangeschrittene Zeit anzeigt.

    Erst wenn Massepunkte mit den bei ihnen ruhenden Uhren eine Beschleunigung erfahren, wird diese Symmetrie der SRT aufgehoben.

    Es ist meine felsenfeste Überzeugung, dass keine der unbeschleunigten synchronisierten Begegnungsuhren ab Ursprungsdeckung jünger oder älter sein kann als eine andere. Sie zeigen verschiedene Zeiten an, ja, aber sie sind im Übrigen alle untereinander und gegenüber denen aller anderen Systeme gleichberechtigt. Der Name dafür heißt: Relativitätsprinzip. Mir ist rätselhaft, wie man diesen festen Boden verlassen kann, nur um angebliche Veränderungen eines “Zeitflusses” verkünden zu können.

    Man kann niemals zutreffend von langsamer gehenden, gleichförmig bewegten Uhren sprechen. Wer es trotzdem tut, ist unbedarft und äußert einen wissenschaftlich unfruchtbaren Gedanken, aber er richtet wenigstens (noch) keinen Schaden in Form von Eierkochen oder Minutenwalzern an, so lange er diese Aussage auf die zwei in #846 herausgegriffenen Uhren („Wenn sich aus der Sicht zweier Systeme die Zeitintervalle zwischen zwei Ereignissen, die im anderen System am selben Ort stattfinden, unterscheiden, nennt man das Zeitdilatation.“) bezieht.

  883. #885 Peter Strohmayer
    21. Februar 2022

    @Martin
    In der von dir in #866 zitierten Quelle wird allerdings gebremst; ist kein Vorwurf, du meinst sicher das Richtige, es ist halt alles sehr verwickelt …:
    “Die Myonen werden in einer Anordnung von drei u ̈bereinander angeordneten Szintil- latoren mit Photomultipliern detektiert. Zwischen den ersten beiden Szintillatoren ist eine Bleischicht angebracht, um die Myonen zu bremsen.”

    (Ändert nichts an #884.)

  884. #886 Karl-Heinz
    Graz
    22. Februar 2022

    Ich denke, es hat nicht sehr viel Sinn, die Diskussion mit Peter Strohmayer weiter fortzuführen. Würde ich eine Berechnung von ihm benötigen, würde Peter kläglich scheitern. Ich finde es komisch, dass Peter sich dessen nicht bewusst ist. Ich kenne einige Leute, die so sind wie Peter. Und keiner von denen hat anders reagiert als Peter.
    Es bleibt mir nur noch eines zu sagen. Schade um die Zeit. Und damit schließe ich meine persönliche Diskussion mit Peter mit den Worten der verlorenen Zeit.

  885. #887 MartinB
    22. Februar 2022

    @Peter
    Seufz, ich muss Karl-Heinz zustimmen.
    Erst sagst du “Wenn man an einem Teilchen ohne Beschleunigung die Dilatation sehen kann, stimmt meine Idee nicht”, dann zeige ich dir, dass das so ist, dann plötzlich verschiebst du die Torpfosten woanders hin…

    PS: ja, man kann die Myonen bremsen, muss man aber ja nicht tun.

  886. #888 Peter Strohmayer
    22. Februar 2022

    @Martin
    Es ist ein fundamentaler Unterschied, ab man einen im Vergleich zu Zwillingen weniger gealterten Massepunkt “sehen kann” oder ob man eine Mitteilung über den Stand einer bewegten Uhr bekommt. Wenn dieser Unterschied von Dir ohne irgend ein Wort der Begründung nicht akzeptiert wird, so nehme ich das zur Kenntnis.

  887. #889 MartinB
    22. Februar 2022

    @Peter
    Ich habe keinen Schimmer, was du mit “einen im vergleich zu Zwillingen weniger gealtertem Massenpunkt” meinst.
    Nimm einen Haufen Myonen auf gerader Strecke mit nahezu Lichtgeschwindigkeit. Miss die Strecke, die sie zurücklegen und die Zeit, die sie brauchen, bis die Hälfte zerfallen ist. Vergleiche mit der Halbwertszeit ruhender Myonen. Der Unterschied ist die Zeitdilatation.
    Was du hier oder auf deiner Internetseite betreibst, ist sophistisches Spielen mit unscharf definierten Begriffen, nicht mehr.

  888. #890 Peter Strohmayer
    22. Februar 2022

    @Martin
    Ich weise darauf hin, dass zwischen dem Vergleich des Alters zweier im selben Bezugssystem ruhender Massepunkte (“Zwillinge”), von denen einer eine Reise hinter sich hat, und dem Vergleich der mitgeteilten Anzeigen zweier Uhren, die zwei zueinander bewegte Massepunkte mit sich führen, ein fundamentaler Unterschied besteht. Ich nehme zur Kenntnis, dass dieser Satz für dich aus unscharf definierten Begriffen besteht, mit denen sophistisch gespielt wird.

  889. #891 Anonym_2022
    22. Februar 2022

    @Peter Strohmayer (22. Februar 2022) #889

    “Es ist ein fundamentaler Unterschied, ab man einen im Vergleich zu Zwillingen weniger gealterten Massepunkt “sehen kann” oder ob man eine Mitteilung über den Stand einer bewegten Uhr bekommt.”

    Das ZP und die ZD sind beide real, weil beide experimentell bestätigt sind (s.o.).

    Der Unterschied:
    • Der Altersunterschied beim ZP ist invariant.
    • Die ZD, also der verlangsamte Zeitfluss an einem bewegten Ort, ist Bezugssystem-abhängig.

    Was in der Physik “fundamental” ist, weiß niemand. Da helfen auch keine unbelegten philosophischen Behauptungen. Vielleicht findet man in Zukunft eine tiefere Erklärungsebene für die Raumzeit, z.B. mit einer Theorie der Quantengravitation.

  890. #892 Peter Strohmayer
    22. Februar 2022

    @Anonym

    Ich schätze Deine sachlich bleibende Argumentation sehr.
    Du bleibst zwar bei Deinen Begriffen, aber in der Sache bringst Du den von mir angesprochenen Unterschied ausgezeichnet auf den Punkt.

  891. #893 Peter Strohmayer
    28. Februar 2022

    @Anonym #891:

    Inhaltlich dürfte Einigkeit bestehen:
    “Der Altersunterschied beim ZP ist invariant.
    Die ZD, also der verlangsamte Zeitfluss an einem bewegten Ort, ist Bezugssystem-abhängig.”

    Der zweite Satz mit der Behauptung eines “verlangsamten Zeitflusses” ist eine der wissenschaftlichen Überprüfung entzogene Spekulation.

    Begründung: Was in der Physik “die Zeit” oder “ein Zeitfluss” oder ein “verlangsamter Zeitfluss” ist, weiß niemand (nicht einmal Augustinus). Da helfen auch keine unbelegten philosophischen Behauptungen des Autors dieses Satzes. Daher sollte man sich beim Umgang mit dem Begriff der Zeit vorerst (bis die Physiker die seit drei Jahrtausenden gesuchte Antwort in einer tieferen Erklärungsebene für die Raumzeit ausgegraben haben werden) auf das beschränken, was eine Uhr anzeigt:

    Dazu ein Vorschlag:
    “Die ZD, also die längere zeitliche Entfernung zwischen zwei in einem Massepunkt stattfindenden Ereignissen aus der Sicht eines relativ dazu bewegten Bezugssystems, ist Bezugssystem-abhängig.”

    Da wir uns nun – zumindest inhaltlich – auf diesen Unterschied geeinigt haben, fragt sich, unter welchen Voraussetzungen es zum ZP – zu einem invarianten Altersunterschied – kommt und unter welchen Voraussetzungen bloß Bezugssystem-abhängige Phänomene auftreten, die sich in L-T konformen Messergebnissen von Ereignisentfernungen erschöpfen.

    Und das führt mich zurück zum Artikel von Martin, der den Unterschied zwischen ZP und ZD mit folgendem Argument relativiert:

    “Das Zwillingsparadoxon kommt also durch einen Wechsel des Bezugssystems zustande – und da ist es egal, ob der durch Beschleunigung, Bewusstseinstransfer oder durch drei Brüder stattfindet.”

    Das ist mE falsch: der “Wechsel des Bezugssystems” bezieht sich beim ZP auf das Ruhen eines Massepunktes, nur daraus kann ein invarianter Altersunterschied erwachsen. Davon ist der “Wechsel eines Bezugssystems”, der sich auf eine willkürlich wählbare Perspektive des Betrachters bezieht und bei dem die variante ZD auftritt, zu unterscheiden.

    Ein bloßer Transfer einer Information über den zeitlichen Abstand zweier Ereignisse ist physikalische Buchhaltung und kann niemals, auch nicht im Gedankenexperiment, dazu führen, dass ein Unterschied zeitlcher Entfernungen zwischen Ereignissen (“Altersunterschied”) invarant wird. Eine bloße Information kann zwar alt sein (lange Zeit unverändert eine Aussage beinhalten), aber nicht altern. Das kann nur Masse.
    Mit anderen Worten: Ohne Beschleunigung kein ZP. Ohne Masse keine Beschleunigung. Ohne Masse kein ZP.

    Die Weltlinien, die Martin zur Erklärung des ZP zeigt (mit “unendlich großen Beschleunigungen”), entsprechen nicht der physikalischen Realität.

    Martin:
    “Entscheidend ist aber – und das macht dieses Diagramm sehr schön deutlich -, dass es einen weiten Bereich von Ereignissen bei Teresa gibt, die für Serena mit gar nichts gleichzeitig sind.”

    Das ist mE nicht entscheidend, sondern – im Gegenteil – der Pferdefuß der Argumentation. Es ist die irreführende Konsequenz daraus, dass das Modell eine physikalisch nicht denkbare Situation zeigt (“Für unendlich große Beschleunigungen werden unsere Überlegungen sinnlos”, könnte man sagen). Es ist – der Ausdruck sei verziehen – ein physikalischer Treppenwitz, dass gerade dieser wunde Punkt in der Argumentation als Ursache für für das Zustandekommen des ZP ins Treffen geführt wird.

  892. #894 MartinB
    28. Februar 2022

    @Peter
    Warum ich das so sehe, steht ganz am Anfang der Kommentare in meiner Diskussion mit Tox.
    Muss ich nicht nochmal aufwärmen.

  893. #895 Anonym_2022
    28. Februar 2022

    Peter Strohmayer (28. Februar 2022) #894

    “Daher sollte man sich beim Umgang mit dem Begriff der Zeit vorerst (…) auf das beschränken, was eine Uhr anzeigt”

    Genau. Zeit ist das, was die Uhr anzeigt. Und was zeigt eine verlangsamt gehende, bewegte Uhr an?

  894. #896 Peter Strohmayer
    28. Februar 2022

    @Anonym
    “Und was zeigt eine verlangsamt gehende, bewegte Uhr an?”

    Die These, dass eine zur Messung herangezogene Uhr “anders” bzw. “verlangsamt” gehen müsse, weil die gemessenen zeitlichen Entfernungen zwischen zwei Ereignissen aus der Sicht verschiedener Bezugssysteme voneinander abweichen, ist eine der wissenschaftlichen Überprüfung nicht zugängliche und außerdem in Anbetracht der Symmetrie der ZD ziemlich abwegige Spekulation.

    Begründung: Was “die Zeit” sonst noch ist, außer die Anzeige einer Uhr, wissen wir immer noch nicht. Die Uhren in den verschiedenen Bezugssystemen zeigen verschiedene Zeitspannen an. Punkt.

    1. Anmerkung: Die Abweichung zeitlicher Entfernungen aus der Sicht verschiedener Bezugssysteme könnte spekulierend auch damit erklärt werden, dass die Zeit keine verfließende Substanz, sondern die Länge der Ausbreitung eines Lichtpulses ist. Diese mE in Anbetracht der “Lichtuhr” nicht ganz so abwegige Spekulation ist aber für die physikalische Beschreibung des Geschehens nicht erforderlich und sie hat daher hier nichts zu suchen. Ebenso ist Deine Spekulation, es gäbe verlangsamt gehende, bewegte Uhren, für die Beschreibung des Geschehens nicht erforderlich und sie hat hier nichts zu suchen.

    2. Anmerkung: Man könnte die wissenschaftliche Annahme treffen, dass die in den jeweiligen Bezugssystemen verwendeten Uhren gleich sind und gleich gehen (Relativitätsprinzip). Diese Annahme könnte man so lange beibehalten, bis sie falsifiziert ist (bis wir wissen, was Zeit ist). Die Invarianz des Altersunterschieds beim ZP widerlegt diese These nicht.

    @Martin
    Ja, Tox hat Recht, deshalb habe ich ihn, damals als PS, unterstützt.

    Ich sehe es so: Du bezeichnest mit dem Ausdruck “Wechsel des Bezugssystems” sowohl den Wechsel eines Massepunkts in ein anderes Inertialsystem als auch einen Wechsel der Perspektive der Beschreibung eines physikalischen Vorgangs (Stichwort: “Bewusstseinsübertragung”). Wenn Dir Physik damit Spaß macht, habe ich damit kein Problem. Ich erlaube mir nur, auf diese ungewöhnliche Begriffsverwendung hinzuweisen.

  895. #897 Anonym_2022
    1. März 2022

    @Peter Strohmayer (28. Februar 2022) #896

    “Die These, dass eine zur Messung herangezogene Uhr “anders” bzw. “verlangsamt” gehen müsse, weil die gemessenen zeitlichen Entfernungen zwischen zwei Ereignissen aus der Sicht verschiedener Bezugssysteme voneinander abweichen, ist eine der wissenschaftlichen Überprüfung nicht zugängliche und außerdem in Anbetracht der Symmetrie der ZD ziemlich abwegige Spekulation.”

    Das “anders” bzw. “verlangsamt” bezieht sich auf die Ableitung der Eigenzeit der bewegten Uhr nach der Koordinatenzeit des Bezugssystems.

    Dass eine bewegte Uhr entsprechend dτ/dt = 1/γ verlangsamt geht, wurde durch Experimente bestätigt, z.B. mit Atomuhren in Flugzeugen, mit dem im Empfängersystem transversalen Dopplereffekt (Rotverschiebung) und mit im Ringbeschleuniger bewegten Myonen. Deren Zeitdilatation ist laut SRT immer dτ/dt = 1/γ, unabhängig davon, ob sie sich geradeaus oder im Kreis bewegen. Der von SRT-Kritikern häufig hinzugedichtete Einfluss der Eigenbeschleunigung auf dτ/dt existiert experimentell abgesichert nicht (s. Uhrenhypothese).

  896. #898 Peter Strohmayer
    1. März 2022

    @Anonym

    Experimente beweisen unterschiedliche zeitliche Entfernungen zwischen zwei Ereignissen aus der Sicht verschiedener, zueinander gleichförmig bewegter Bezugssysteme. Sie beweisen nicht die Ursache für diese Messergebnisse, und insbesondere nicht die native Vermutung, dass da wohl eine Uhr “langsamer” gegangen sein müsste. Du vergleichst Uhrenstände oder beobachtest Dopplereffekte mit einem anders als bei Ole Römer ausgerichteten Fernrohr. Damit hat es sich.

    Auch wenn Du die unterschiedlichen Zeitspannen in ein Verhältnis setzt, bleibt Dein “langsamer” eine der Spekulationen (von mehreren möglichen), von der ich mich, wie gesagt, hier fern halte, weil sie mit Physik nichts zu tun hat. Du kannst ja gern weiter Physikfolklore betreiben.

  897. #899 Anonym_2022
    1. März 2022

    @Peter Strohmayer (1. März 2022) #898

    “Du vergleichst Uhrenstände oder beobachtest Dopplereffekte mit einem anders als bei Ole Römer ausgerichteten Fernrohr.”

    Ich weiß nicht, was Ole Roemer damit zu tun hat. Vielleicht meinst du ja James Bradley.

    Die relativistische Dopplergleichung ist nichts anderes als der Zeitdilatationsfaktor multipliziert mit dem klassischen Dopplereffekt. Um die ZD zu messen, muss man halt eine longitudinale Geschwindigkeitskomponente vermeiden oder herausrechnen.

    Das Ives-Stilwell-Experiment war das erste Experiment, mit dem der transversale Dopplereffekt und somit die aus der speziellen Relativitätstheorie folgende Zeitdilatation direkt nachgewiesen werden konnte.

    Während bei diesem Test der transversale Dopplereffekt aus dem longitudinalen sozusagen herausgefiltert wurde, konnte 1979 auch ein „rein transversaler“ Test durchgeführt werden.[7]

    Durchführung

    Ives verzichtete darauf, den durch die Zeitdilatation verursachten transversalen Dopplereffekt …, im rechten Winkel zur Bewegungsrichtung der Kanalstrahlen zu beobachten, da ein Einfluss des longitudinalen Dopplereffekts kaum auszuschließen war. Deswegen entwickelte er eine Methode, um den transversalen Dopplereffekt in longitudinaler Ausbreitungsrichtung der Kanalstrahlen zu beobachten. Dabei werden drei Lichtstrahlen verglichen, die aus unbewegten, sich nähernden, und sich entfernenden Kanalstrahlen stammen.

    Gemäß klassischem Dopplereffekt müssten die Frequenzen von sich in- und entgegen der Bewegungsrichtung ausbreitendem Licht um
    f’/f = c/(c ± v) verschoben sein, wo c die Lichtgeschwindigkeit und v die Geschwindigkeit der Kanalstrahlen ist. Wenn dies auf die Wellenlängen übertragen wird, ergibt der klassische Dopplereffekt rot- und blauverschobene Wellenlängen mit den Werten 1+v/c und 1-v/c. Wenn alle drei Wellenlängen (rotverschobene, blauverschobene, unveränderte) auf einer linearen Skala markiert werden, müssten diese Wellenlängen gemäß der klassischen Theorie in völlig gleichen Abständen zu finden sein.

    Berücksichtigt man jedoch die Zeitdilatation, müssten die beiden äußeren Markierungen (bezüglich der unbewegten zentralen Markierung) etwas verschoben sein. Diese Verschiebung müsste exakt derjenigen entsprechen, welche auch in transversaler Richtung auftreten würde. Ives und Stilwell fanden tatsächlich eine signifikante Verschiebung des Schwerpunkts der drei Markierungen, in Übereinstimmung mit dem relativistischen Dopplereffekt mit einer maximalen Abweichung von 10^−2.

    Quelle:
    https://de.wikipedia.org/wiki/Ives-Stilwell-Experiment

  898. #900 Peter Strohmayer
    1. März 2022

    @Anonym

    Deiner Sachlichkeit mein Respekt. Es tut mir leid, ich meinte Bradley. Ich will damit zum Ausdruck bringen, dass sich bei der Bradleyschen Ausrichtung die ZD als Blauverschiebung zeigt, oder, um mit Deinen Worten zu sprechen, man halt auf zweierlei Art eine longitudinale Geschwindigkeitskomponente vermeiden oder herausrechnen kann (aber mit konträrem Ergebnis, wenn man damit “langsamer gehende Uhren” begründen wollte).

    Den von dir zitierten Texten haften keine Spekulationen über langsamer gehende Uhren an. Auch an deiner mathematischen Beobachtung, “Die relativistische Dopplergleichung ist … der Zeitdilatationsfaktor multipliziert mit dem klassischen Dopplereffekt.” ist wissenschaftlich betrachtet nichts auszusetzen. Alles faktenbasierte schöne Physik ohne folkloristische Grenzüberschreitungen.

  899. #901 Anonym_2022
    1. März 2022

    @Peter Strohmayer (1. März 2022) #900

    “Ich will damit zum Ausdruck bringen, dass sich bei der Bradleyschen Ausrichtung die ZD als Blauverschiebung zeigt, oder, um mit Deinen Worten zu sprechen, man halt auf zweierlei Art eine longitudinale Geschwindigkeitskomponente vermeiden oder herausrechnen kann (aber mit konträrem Ergebnis, wenn man damit “langsamer gehende Uhren” begründen wollte).”

    Du meinst damit wohl den Fall, wenn das Licht im Sendersystem transversal zur Bewegungsrichtung des Empfänger ausgerichtet ist.

    In dem Fall misst man halt, dass der Empfanger die im Sendersystem langsamer gehende Uhr ist.

    Die Diskussion dreht sich im Kreis, siehe #791.

  900. #902 Peter Strohmayer
    2. März 2022

    @Anonym
    “Langsamer gehende Uhr”: Woher weißt Du das?

    Besser: “In diesem Fall misst man halt, dass der Empfänger kürzere zeitliche Entfernungen zwischen zwei Sendeereignissen konstatiert.”

    Natürlich drehen wir uns im Kreis. Du wirst von Deinen langsamer gehenden Uhren, vom langsameren Fluss der Zeit (und in weiterer Folge vielleicht auch vom langsameren Eierkochen und Klavierspielen) nicht abrücken. Du bist halt überzeugt, dass die Zeit fließt und dass in diesem Fluss Veränderungen auftreten könnten, so wie früher alle davon überzeugt waren, dass Gleichzeitiges für alle gleichzeitig sein müsse. Auch damals hätte man sich die Frage vorlegen müssen: woher weißt Du das? Und wie damals plötzlich keiner eine Antwort wusste, so ist es auch hier, wenn Du Dich an die Grundregeln wissenschaftlichen Denkens halten würdest.

  901. #903 Anonym_2022
    2. März 2022

    @Peter Strohmayer (2. März 2022) #902

    “Besser: “In diesem Fall misst man halt, dass der Empfänger kürzere zeitliche Entfernungen zwischen zwei Sendeereignissen konstatiert.”

    Das is falsch. Der Empfänger konstatiert, dass die Blauverschiebung von einer longitudinalen Geschwindigkeitskomponente des bewegten Senders im Empfänger-Bezugssystem herrührt (=Beitrag des klassischen Dopplereffekts).

    Der Sender konstatiert, dass der bewegte Empfänger einen verlangsamten Eigenzeitfluss hat, bezogen auf die Koordinatenzeit des Senderbezugssystem, (=ZD) und dass der Empfänger das Licht daher blauverschoben misst.

  902. #904 Peter Strohmayer
    2. März 2022

    @Anonym

    “Das ist falsch”.

    Der Empfänger konstatiert also keine kürzere zeitliche Entfernung (“Wellenlänge”), denn: “das ist falsch”. Der gleiche Empfänger soll aber – wie immer ohne den Hauch einer Begründung – konstatieren, von wo diese nicht existierende (“falsche”) kürzere zeitliche Entfernung (“die Blauverschiebung”) herrührt, nämlich vom “verlangsamten Eigenzeitfluss”, freilich nur aus Sicht des Senders.

    Das ist eine Logik, die hat Schwung. Vielleicht kann ich ihr aber aus eigenem Unvermögen nicht folgen. In diesem Fall bin ich nicht der geeignete Gesprächspartner.

  903. #905 Anonym_2022
    2. März 2022

    @Peter Strohmayer (2. März 2022) #904

    “denn: “das ist falsch”.”

    Also erkläre ich das noch einmal genauer: Der Empfänger kann aufgrund der gemessenen Blauverschiebung nicht den Zeitabstand der zwei Senderereignisse bestimmen. Grund ist der Einfluss der longitudinalen Geschwindigkeitskomponente auf den Dopplereffekt in seinem Bezugssystem. Außer: Er rechnet diesen Einfluss heraus. Dann ergibt sich eine Rotverschiebung.

    “Der gleiche Empfänger soll aber – wie immer ohne den Hauch einer Begründung – konstatieren, von wo diese nicht existierende (“falsche”) kürzere zeitliche Entfernung (“die Blauverschiebung”) herrührt, nämlich vom “verlangsamten Eigenzeitfluss”, freilich nur aus Sicht des Senders.”

    Hier mit unterstellst du mir eine Behauptung, die ich nicht geschrieben habe. Der Empfänger hat in seinem eigenen Bezugssystem keine ZD, weil sein eigenes Gamma in seinem eigenen Bezugssystem 1 ist.

    Der Sender kann in seinem Bezugssystem die ZD des bewegten Empfänger bestimmen aus dem (invarianten) Messergebnis, das der Empfänger ihm per WhatsApp geschickt hat. Im Sendersystem gibt es ja keine störende longitudinale Geschwindigkeitskomponente.

    “Vielleicht kann ich ihr aber aus eigenem Unvermögen nicht folgen.”

    Helfen könnte dir, zu verstehen, was ein Bezugssystem ist.

  904. #906 Peter Strohmayer
    3. März 2022

    @Anonym
    “Der Empfänger kann aufgrund der gemessenen Blauverschiebung nicht den Zeitabstand der zwei Senderereignisse bestimmen.”

    Ich habe in #903 eindeutig von dem Messergebnis der zeitlichen Entfernung zweier aus Sicht des Empfängers eintretender Ereignisse gesprochen, das sich aus dem Stand der beim Empfänger ruhenden Uhr ergibt, und nicht von hinzuspekulierten Gründen für diese Messergebnisse.

    Nun drehst Du mir das Wort im Munde um und sprichst von irgendeiner, keinem Messergebnis entsprechenden zeitlichen Entfernungen zwischen den besagten Ereignissen, die sich nach den von Dir gewünschten Betrachtungen und Berechnungen ergeben sollen und die “Der Empfänger … nicht … bestimmen” kann. Eine tatsächliche Messung erklärst Du für “falsch”, die von Dir herbeigeredete für “richtig”.

    Helfen könnte Dir, zu verstehen, was der Unterschied zwischen einer unbegründeten Spekulation und einer streng an der Faktenlage (dem Experiment) orientierten naturwissenschaftlichen Argumentation ist.

  905. #907 Anonym_2022
    3. März 2022

    @Peter Strohmayer (3. März 2022) #906

    Ich bin jetzt raus, da sich die Diskussion wieder im Kreise dreht.

  906. #908 Anonym_2022
    5. März 2022

    Schon Isaac Newton wusste, dass ein Empfänger die Lichtlaufzeit herausrechnen muss, um den Zeitpunkt eines entfernten Ereignisses zu bestimmen.

    Wenn man ein Ereignis auf der Sonne “jetzt” beobachtet, so findet es nicht “jetzt” statt, sondern es fand ca. 8 Minuten vorher statt.

    Isaac Newton schrieb ca. 1718 in seinem Buch “Opticks”:

    The Queries (or simply Queries) is the third book to English physicist Isaac Newton’s Opticks

    Qu. 21.

    Light moves from the Sun to us in about seven or eight Minutes of Time

    Quelle:
    https://en.wikipedia.org/wiki/The_Queries

    Wenn es daher um ein Zeitintervall geht, muss man die Veränderung der Lichtlaufzeit herausrechnen.

    Das bedeutet für die ZD:

    After compensating for varying signal delays due to the changing distance between an observer and a moving clock (i.e. Doppler effect), the observer will measure the moving clock as ticking slower than a clock that is at rest in the observer’s own reference frame.

    Quelle:
    https://en.wikipedia.org/wiki/Time_dilation

  907. #909 Peter Strohmayer
    6. März 2022

    @Anonym
    Ich bin gerade dabei, die Struktur der Argumentationen in den über 900 Kommentaren zu den großteils gleichen Themen zu vergleichen. Da dreht sich natürlich vieles im Kreis. Aber man sieht rückblickend auch sehr schön, wie und warum man aneinander vorbeiredet.

    Man sieht, wie manche etwas unelegant abgeblockt werden

    (“Tut mir Leid, ich verstehe die Logik wirklich schlicht nicht.” – Ja, dann hat es vielleicht keinen Sinn, weiterzudiskutieren, weil du irgendwas anders siehst als ich (und ich nicht wirklich verstehe, was das ist).” – “Langsam verstehe ich gar nicht mehr, was du sagen willst.” – “Ich habe keine Ahnung, wie du das genau meinst.” – “Wo/wie ist da die Symmetrie gebrochen (was immer du nun wieder in diesem Zusammenhang darunter genau verstehst)?” etc etc)

    und man sieht, dass man bei einigem guten Willen – den ich Dir allerdings bei Deiner unerschütterlichen Sachlichkeit nicht abspreche – gemeinsam zu einem besseren Verständnis hätte gelangen können.

    Wenn man zB Deinen Kommentar #736 (den man aufmerksamer hätte lesen müssen) und #891

    (“Der Altersunterschied beim ZP ist invariant. Die ZD, also der verlangsamte Zeitfluss an einem bewegten Ort, ist Bezugssystem-abhängig.” bzw. in meiner Lesart #893 “Der Altersunterschied beim ZP ist invariant. Die ZD, also die längere zeitliche Entfernung zwischen zwei in einem Massepunkt stattfindenden Ereignissen aus der Sicht eines relativ dazu bewegten Bezugssystems, ist Bezugssystem-abhängig.”)

    den ganzen Diskussionen vorangestellt hätte, hätte man sich manchen Verdruss über die (angeblich) mangelnde Intelligenz seines jeweiligen Gesprächspartners ersparen können.

    Dasselbe gilt zB von Martins Begriff “Wechsel des Bezugssystems”. Hätte man die Doppeldeutigkeit dieses Begriffs von Anfang an klargestellt, verstünden sich die “drei Brüder”, tickten drei Uhren richtig und wären esoterische Bewusstseinsübertragungen zu Mitteilungen über Messergebnisse herabgestuft. Über die Myonen herrschte nach #891 nur eine Meinung, ganz gleich, ob die Ereignisse deren Anwesenheit bei gleichförmigen Vorbeiflug gemessen oder ob sie “symmetriebrechend” bei Entstehung beschleunigt und bei der Detektion durch Bleiplatten abgebremst werden.

  908. #910 Anonym_2022
    7. März 2022

    Im folgenden Video wird die gravitative ZD gut veranschaulicht:

    href=”https://www.youtube.com/watch?v=1ENkP0h8nAg

  909. #911 Peter Strohmayer
    Wien
    27. November 2022

    Anonym 2018 #171 “[Ohne Symmetriebrechung ist keine echte messbare Zeitdilatation möglich.] Dem stimme ich bzgl. des … Zwillingsparadoxons … zu. … Das trifft z.B. nicht beim transversalen Dopplereffekt zu.” – Anonym 2022 #786: „Beim relativistischen transversalen Doppler-Effekt bei gleichförmiger Bewegung kann die Frequenzveränderung nur durch eine Zeitdilatation erklärt werden, weil es keine longitudinale Geschwindigkeits-Komponente gibt.“

    Dieses Argument stammt vom Wikipedia Artikel “Relativistic Doppler effect – Source and receiver are at their points of closest approach https://en.wikipedia.org/wiki/Relativistic_Doppler_effect“. Ihm zufolge ist der relativistische Dopplereffekt (DE) der klassische DE korrigiert durch “Zeitdilatation”. Er geht davon aus, dass sich bei 90° Ausstrahlung aus Sicht des Senders (Fig. 2b; entspricht einem im Äther bewegter Empfänger) bzw. bei 90° Einstrahlung aus Sicht des Empfängers (Fig. 3; entspricht einem im Äther bewegter Sender) Sender und Empfänger die Punkte ihrer größte Annäherung erreichen und es daher keine “longitudinale Geschwindigkeitskomponente” und auch keinen transversalen DE gibt. Dass es bei Licht zu einem transversalen DE kommt, wird der “Zeitdilatation” zugeschrieben.

    Licht im Vakuum hat aber mit einer Wellenausbreitung im Äther nichts zu tun. Es gibt bei Licht weder für eine Aussendung zu 90° noch für einen Empfang aus 90° ein zu erwartendes Nullergebnis, das durch “Zeitdilatation” korrigiert werden müsste. Die zu Fig. 2a des Wikipedia Artikels für eine Aussenderichtung von 90° aus Sicht des Senders (für einen im Äther ruhenden Empfänger) gegebene und zu Recht nicht weiterverfolgte, weil falsche Erklärung, dass die durch Zeitdilatation beim Sender hervorgerufene Rotverschiebung durch eine Blauverschiebung auf Grund einer longitudinalen Geschwindigkeitskomponente nur ausgeglichen werde (“would be offset”), spricht für sich.

    Beim Licht spielt das Verhältnis der Signalwege aus der Sicht des Senders bzw. des Empfängers und nicht der kürzeste Signalweg (in den Punkten der größten Annäherung bei angeblich fehlender “longitudinaler Geschwindigkeitskomponente”) eine Rolle. Die Darlegung des Wikipedia Eintrags, wonach der relativistische DE unterbleibt, wenn der Signalweg am kürzesten ist (“Null frequency shift occurs for a pulse that travels the shortest distance from source to receiver”) ist falsch. Der Signalweg ist für jeden Beobachter ein anderer.

    Der relativistische DE beruht vielmehr auf den unterschiedlichen Signalwegen aus Sicht von Sender und Empfänger. Er bleibt nicht aus, wenn “der Signalweg am kürzesten ist” (das ist er an zwei Stellen für jeweils einen Beobachter bei 90°), sondern wenn der Signalweg aus der Sicht des Senders gleich lang ist wie der Signalweg aus der Sicht des Empfängers. Beim longitudinalen DE ist der Unterschied der Längen der Signalwege maximal, in den beiden Fällen des transversalen DE entspricht ihr Verhältnis dem Lorentz-Faktor, bei fehlendem DE ist der Unterschied null.

    Beim transversalen Dopplereffekt wird daher nicht „die SRT-Zeitdilatation“ gemessen (Anonym 2018 #171, #199, #796, Anonym 2022 #899), sondern eine bei Aussendung oder Empfang zu 90° vorhandene (relativistische) positive bzw. negative longitudinale Geschwindigkeitskomponente des Senders in Bezug auf den Empfänger.

  910. #912 Peter Strohmayer
    Wien
    1. Dezember 2022

    Anonym 2022 #819: „Diese Definition von” Zeitdilatation” [von Rindler] halte ich für gut formuliert: … a clock moving uniformly with velocity v through an inertial frame S goes slow by a γ factor relative to the synchronized standard clocks at rest in S. …
This ‘time dilation’, like length contraction, is no accident of convention but a real effect. Moving clocks really do go slow.“

    Mit “real effect” und “really” wird eindeutig ein unterschiedliches Voranschreiten der Eigenzeiten der Uhren behauptet. Um diese Behauptung zu überprüfen, muss man daher – entgegen Anonym 2022 #824 und #832 – das Voranschreiten der Eigenzeiten der Uhren mit Hilfe von Minkowski-Diagrammen zweier zueinander bewegter Systeme vergleichen. So kann der Beweis erbracht werden, dass alle Uhren gleich schnell gehen und dies nicht im Widerspruch zu einem allfälligen Abweichen der Anzeigen von Begegnungsuhren steht.

    Eine rote Uhrenreihe wird im System rot entlang der x-Achse aufgestellt. Eine grüne Uhrenreihe wird im System grün entlang der x’-Achse aufgestellt. Das System rot bewegt sich aus Sicht des Systems grün mit v=0.6c entlang der x-Achse nach links. Bei Ursprungsdeckung der Systeme werden vom jeweiligen Ursprung aus entlang den jeweiligen Uhrenreihen ein roter bzw. ein grüner Lichtpuls nach links ausgesendet. Die jeweiligen Lichtpulse erscheinen im jeweiligen Weg-Zeit-Diagramm als unterschiedlich lange Weltlinien mit einem Winkel von 45°.

    Der bei Ursprungsdeckung (Ereignis 1) ausgesendete Lichtpuls rot des “bewegten” Systems rot wird zur Zeit t=3 am Ort x=-3 (links vom Ursprung) reflektiert (Ereignis 2). Gemäß der L-T wird der komplementäre Lichtpuls grün des “ruhenden” Systems grün beim Ereignis 2 zur Zeit t’=6 am Ort x’=-6 reflektiert. (Beim Ereignis 2 begegnen sich also eine rote und eine grüne Uhr, deren Anzeigen nach dem Gesagten im Verhältnis der Längen der Ausbreitung der beiden Lichtpulse (3:6) voneinander abweichen.)

    Nach der Reflektion kehrt der Lichtpuls rot nach rechts zur Zeit t=6 an den Ort x=0 (an den Ursprung rot) zurück (Ereignis 3). Beim Ereignis 3 befindet sich der komplementäre Lichtpuls grün nach der L-T zur Zeit t’=7,5 am Ort x’=-4,5. Die “bewegte” Uhr im Ursprung des Systems rot, die die Zeit 6 zeigt, begegnet also einer “ruhenden” Uhr des Systems grün, die die Zeit 7,5 zeigt.

    Daraus wird die Behauptung abgeleitet, dass die nachgehende “bewegte” Uhr rot um den (Lorentz-)Faktor 1,25 “langsamer gegangen” wäre als die Begegnungsuhr grün.

    Nach dem Ereignis 3 breiten sich die komplementären Lichtpulse weiter nach rechts aus. Der Lichtpuls grün trifft nun auf die im Ursprung des Systems grün ruhende Uhr (x’=0), die die Zeit t’=12 anzeigt (Ereignis 4). Der komplementäre Lichtpuls rot befindet sich beim Ereignis 4 nach der L-T zur Zeit t=15 am Ort x=9. Die “ruhende” Uhr im Ursprung des Systems grün, die die Zeit 12 anzeigt, begegnet also der “bewegten” Uhr des Systems rot, die die Zeit 15 anzeigt.

    Im Widerspruch zur obigen Behauptung müsste nun behauptet werden, dass die nachgehende “ruhende” Uhr grün um den (Lorentz-)Faktor 1,25 “langsamer gegangen” wäre als die Begegnungsuhr rot. (Die Uhren haben ihre Rollen als “ruhend” bzw. “bewegt” bei der Ausbreitung der Lichtpulse immer beibehalten).

    Keine dieser Behauptungen trifft zu. Die unterschiedlichen Anzeigen der Begegnungsuhren sind auf die unterschiedlichen Längen der Ausbreitung der nach der L-T koordinierten Lichtpulse (auf die unterschiedlichen Signalwege) zurückzuführen. Inwieweit diese Anzeigen abweichen, hängt nur vom Standort einer Uhr in ihrem System ab, nicht aber davon, ob sie als “bewegt” oder als “ruhend” betrachtet wird.

    Die Anzeigen der roten bzw. grünen Uhren repräsentieren die Ausbreitung des jeweiligen Lichtpulses rot bzw. grün ab Ursprungsdeckung bis zu einem Ereignis des Eintreffens. Der Grund für die Abweichung der Anzeigen von Begegnungsuhren ist die vom Ereignisort abhängige unterschiedliche Länge der Gesamtausbreitung des Lichtpulse rot bzw. grün (die unterschiedliche Länge der Signalwege rot bzw. grün). Auf dem Weg ihrer Ausbreitung gibt es nach einem Richtungswechsel im Übrigen immer einen Ereignisort der “relativistischen Mitte”, an denen die Signalwege wieder lang geworden sind, wo also die Begegnungsuhren rot und grün die gleiche Zeit anzeigen.

    “Moving” clocks really do’nt go slower.

  911. #913 Anonym_2022
    18. Dezember 2022

    @Peter Strohmayer (27. November 2022) #911
    “Beim transversalen Dopplereffekt wird daher nicht „die SRT-Zeitdilatation“ gemessen (Anonym 2018 #171, #199, #796, Anonym 2022 #899), sondern eine bei Aussendung oder Empfang zu 90° vorhandene (relativistische) positive bzw. negative longitudinale Geschwindigkeitskomponente des Senders in Bezug auf den Empfänger.”

    Diese Aussage ist bezugssystemabhängig und stimmt im Ruhesystem des Senders. Sie stimmt nicht im Ruhesystem des Empfängers, weil es bei einem 90 Grad-Winkel (im Empfängersystem) keine longitudinale Geschwindigkeitskomponente des Senders gibt.

  912. #914 Anonym_2022
    18. Dezember 2022

    @Peter Strohmayer (1. Dezember 2022) #912

    “Mit “real effect” und “really” wird eindeutig ein unterschiedliches Voranschreiten der Eigenzeiten der Uhren behauptet.”

    Nein. So etwas wird nur für das ZP behauptet, nicht für die ZD.

  913. #915 Peter Strohmayer
    Wien
    20. Dezember 2022

    @Anonym 2022 #914

    “Nein. So etwas wird [] nur für das ZP behauptet, nicht für die ZD.”

    Ergänzung: [von Anonym (und mit Vorbehalten auch von Peter Strohmayer), nicht jedoch von der populärwissenschaftlichen Literatur]

    Allerdings würde ich beim ZP nicht von einem unterschiedlichen Voranschreiten der Eigenzeit, sondern von infinitesimalen Wechseln des Beschleunigenden in andere Inertialsysteme mit infinitesimal weiter vorangeschrittenen Zeiten sprechen würde, wovon das Voranschreiten der Eigenzeit des Beschleunigenden ebenfalls nicht berührt wird. Aber das ist eine andere Geschichte.

  914. #916 Peter Strohmayer
    Wien
    21. Dezember 2022

    @Anonym 2022 #913

    Bei 90° aus Sicht des Empfängers (und bei 90° aus Sicht des Senders) würde eine longitudinale Geschwindigkeitskomponente nur dann fehlen, wenn “Äther” vorhanden wären.

    Bei einem Masseteilchen, das in einem Winkel von 90° einfällt (oder in einem Winkel von 90° ausgesendet wird), gibt es jeweils eine longitudinale Geschwindigkeitskomponente: die Relativgeschwindigkeit des Teilchens gegenüber dem Sender ist in diesen beiden Konstellationen eine andere als die gegenüber dem Empfänger. Die longitudinale Geschwindigkeitskomponente fehlt in einer dazwischen liegenden Konstellation.

    Der Unterschied zu Licht liegt nur im Ausmaß (in der Aberration), nicht im Grundsatz.

  915. #917 Anonym_2022
    22. Dezember 2022

    @Peter Strohmayer #916

    “Bei 90° aus Sicht des Empfängers (und bei 90° aus Sicht des Senders) würde eine longitudinale Geschwindigkeitskomponente nur dann fehlen, wenn “Äther” vorhanden wären.”

    Nein, siehe Figure 3:
    https://en.wikipedia.org/wiki/Relativistic_Doppler_effect#Receiver_sees_the_source_as_being_at_its_closest_point

  916. #918 Peter Strohmayer
    Wien
    22. Dezember 2022

    @Anonym #917
    Eben weil Fig. 3 einen Äther unterstellt, fehlt nach ihr bei 90° eine longitudinale Geschwindigkeitskomponente.

    Die nachteiligen Folgen des Ätherdenkens zeigen sich im folgenden Abschnitt” Point of null frequency shift”:

    “As seen in Fig. 4, null frequency shift occurs for a pulse that travels the shortest distance from source to receiver.”

    Ein DE unterbleibt aber nicht bei dem Lichtpuls, der den kürzesten Weg von der Quelle zum Empfänger zurücklegt (aus welcher Sicht?), sondern bei dem Lichtpuls, der sowohl aus der Sicht des Empfängers als auch aus der Sicht des Senders den gleichen Weg von der Quelle zum Empfänger zurücklegt.

    Allgemein formuliert: Beim Licht tritt eine longitudinale Geschwindigkeitskomponente bzw. ein DE genau dann und genau in dem Ausmaß auf, in dem der Lichtpulsweg aus der Sicht des Empfängers von dem Lichtpulsweg aus der Sicht des Senders abweicht.

  917. #919 Anonym_2022
    23. Dezember 2022

    @ Peter Strohmayer #918

    “Eben weil Fig. 3 einen Äther unterstellt, fehlt nach ihr bei 90° eine longitudinale Geschwindigkeitskomponente.”

    Quark. Wo steht das mit dem Äther? In der Überschrift des Wikipedia-Artikels kommt “Relativistic” vor und die SRT unterstellt keinen Äther.

    Die ZD und der Winkel sind bezugssystemabhängig. Fig. 3 zeigt den Winkel mit Bezug auf das Ruhesystem des Empfängers. Das sieht man daran, dass nur der Sender als bewegt eingezeichnet ist.

  918. #920 Peter Strohmayer
    Wien
    23. Dezember 2022

    @Anonym ‘919

    Außen steht “relativistic” drauf, aber es ist kein “relativistic” drin. Die SRT unterstellt keinen Äther, der Artikel leider schon.

    Seine Argumentation lautet: die longitudinale Geschwindigkeitskomponente fehlt, wenn der Lichtweg aus Sicht des Empfängers beim Transit extremal wird.

    Das kann nicht stimmen. Die longitudinale Geschwindigkeitskomponente betrifft das absolute Geschwindigkeitsverhältnis zwischen Sender und Empfänger (Sender und Empfänger können bezüglich ihrer Relativgeschwindigkeit nicht verschiedener Meinung sein). Also kann bei der Beurteilung, ob eine solche für beide gültige Relativgeschwindigkeit vorhanden ist, nicht auf nur einen Extremalwert des Lichtwegs bei bei nur einem der beiden abgestellt werden. Den Umstand, dass er beim anderen nicht extremal ist, wird im Artikel unter den Tisch gekehrt und stillschweigend ein Äther mitgedacht, bei dem der Lichtweg auch aus der Sicht des anderen extremal ist. Das ist non relativistic, ätherbasiertes Denken. Es führt im Artikel zum falschen Schluss, eine longitudinale Geschwindigkeitskompunente würde fehlen und der DE wäre auf eine geheimnisvolle “Zeitdilatation” zurückzuführen. Diese ist aber ganz profan nur ein Rechenfaktor, der den klassischen vom relativistischen DE unterscheidet.

    Das Gleichnis mit dem Masseteilchen (#916) kommt der Sache näher.

  919. #921 Anonym_2022
    23. Dezember 2022

    @ Peter Strohmayer #920

    “Also kann bei der Beurteilung, ob eine solche für beide gültige Relativgeschwindigkeit vorhanden ist, nicht auf nur einen Extremalwert des Lichtwegs bei bei nur einem der beiden abgestellt werden.”

    Im Empfänger-Ruhesystem (Fig. 3) ist der Lichtweg extremal (d.h. der Winkel zwischen Lichtrichtung und Bewegungsrichtung des Senders beträgt 90 Grad).

    Im Sender-Ruhesystem ist der Lichtweg dagegen nicht extremal (d.h. der Winkel zwischen Lichtrichtung und Bewegungsrichtung des Empfängers beträgt nicht 90 Grad).

    Das nennt man Aberration und es ist nicht widersprüchlich.

  920. #922 Peter Strohmayer
    Wien
    23. Dezember 2022

    @Anonym #921

    Die Relativgeschwindigkeit (die longitudinale Geschwindigkeitskomponente = lG) zum Zeitpunkt des Eintritts eines bestimmten Ereignisses (zB Aussenden des Lichtpulses) muss für beide gleich sein. Dann sehe ich aber einen Widerspruch, wenn nicht für beide zu diesem Zeitpunkt der an dieses Aussenden anschließende Lichtweg als extremal zu beurteilen ist, weil dann fehlt die lG bei einem, während sie beim anderen (noch) da sein soll. Weitere Frage: hier haben wir zwei Ereignisse (Aussenden und Eintreffen). Die Relativgeschwindigkeit ändert sich dazwischen. Welches ist für ein Fehlen einer lG maßgebend?

    Ich kann dieses ganze Argumentationskonzept nicht nachvollziehen.

    Es würde mich freuen, sie auch nach dem Jahreswechsel nicht als Gesprächspartner zu verlieren.

  921. #923 Anonym_2022
    24. Dezember 2022

    @ Peter Strohmayer #922

    “Die Relativgeschwindigkeit (die longitudinale Geschwindigkeitskomponente = lG) zum Zeitpunkt des Eintritts eines bestimmten Ereignisses (zB Aussenden des Lichtpulses) muss für beide gleich sein.”

    Ja.

    “Dann sehe ich aber einen Widerspruch, wenn nicht für beide zu diesem Zeitpunkt der an dieses Aussenden anschließende Lichtweg als extremal zu beurteilen ist, weil dann fehlt die lG bei einem, während sie beim anderen (noch) da sein soll.”

    Ich sehe keinen Widerspruch. Zum Aussendeereignis ist für beide lG=0. Zum Empfanszeitpunkt ist für beide lG ungleich 0.

    Die fehlende Fig. 3 (b), die das Szenario der Fig. 3 im Ruhesystem der Lichtquelle zeigen sollte, muss wie folgt aussehen.

    Die fehlende Fig. 3 (b) hat zur Fig. 3 folgende Unterschiede:
    Die Lichtquelle ruht, der Empfänger bewegt sich nach links. Zum Aussendeereignis befindet sich der Empfänger dabei senkrecht unter dem Sender.
    Der Lichtimpuls wir schräg nach links unten gesendet und erreicht den sich nach links bewegende Empfänger.

    Im Ruhesystem des Senders führt der longitudinale Dopplereffekt zu einer Rotverschiebung mit dem Faktor 1/γ². Durch die Zeitdilatation des bewegten Empfängers wird der Faktor durch eine Blauverschiebung auf 1/γ abgemildert.

  922. #924 Anonym_2022
    28. Dezember 2022

    @ Peter Strohmayer #922

    Direct observation of the transversal Doppler-shift
    Hasselkamp, D. ; Mondry, E. ; Scharmann, A.
    Abstract
    An experiment is reported in which the second order Doppler-shift has been determined by the observation of the Hα-line emitted by linearly moving hydrogen atoms of velocities 2.53×108 cm/s-9.28×108 cm/s. In contrast to previous experiments the direct transversal observation has been used for the first time. The coefficient of the second order term in the relativistic approximation is found to be 0.52±0.03 which compares good with the theoretical value of 1/2.

    Publication:
    Zeitschrift füur Physik A Atoms and Nuclei, Volume 289, Issue 2, pp.151-155
    Pub Date: June 1979

    Quelle:
    https://ui.adsabs.harvard.edu/abs/1979ZPhyA.289..151H/abstract

  923. #925 Anonym_2022
    28. Dezember 2022

    @ Peter Strohmayer #922

    korrigierter Link:
    https://ui.adsabs.harvard.edu/abs/1979ZPhyA.289..151H/abstract

  924. #926 Peter Strohmayer
    Wien
    28. Dezember 2022

    @Anonym 2022 # …

    Bin einige Tage verhindert.

    Ich bitte, eine E-mail Adresse beim Administrator zu hinterlegen, damit ich nach Abschaltung der Seite direkt Stellung nehmen kann.

  925. #927 Anonym_2022
    28. Dezember 2022

    @Peter Strohmayer #926

    “Ich bitte, eine E-mail Adresse beim Administrator zu hinterlegen, damit ich nach Abschaltung der Seite direkt Stellung nehmen kann.”

    Laut dem folgenden Bericht wird ScienceBlogs doch nicht kurzfristig abgeschaltet.

    Die Konradin-Mediengruppe, die im Frühjahr 2014 die deutschen ScienceBlogs übernommen hatte, wird nun doch nicht einfach alles abschalten. Wahr ist, dass die Kosten auf ein Minimum reduziert werden, und wahr ist, dass sich einige Bloggerinnen und Blogger inzwischen nach neuen Plattformen umgesehen haben. Aber die Seite wird, so wie sie ist, erhalten bleiben – was bedeutet, dass alle bisherigen Links und alle bisherigen Kommentare an der gleichen Stelle zu finden sein werden, wo sie bisher zu finden waren. Und einige Bloggerinnen oder Blogger werden auch ihre Blogs weiterhin aktiv pflegen.

    Quelle:
    https://scienceblogs.de/2022/12/15/die-berichte-ueber-meinen-tod-sind-uebertrieben-scienceblogs-macht-doch-weiter/

  926. #928 Peter Strohmayer
    Wien
    2. Januar 2023

    @Anonym 2022 #923

    Es gibt zwei Arten des klassischen Dopplereffekts.

    Die eine beruht auf der Ausbreitung von Wellen in einem Äther. Das Ausmaß des DE in Abhängigkeit von der Relativgeschwindigkeit hängt davon ab, ob der Äther beim Sender oder beim Empfänger ruht.

    Die andere Art beruht auf der Aussendung von Teilchen.

    In der Diskussion über den Dopplereffekt bei Licht blieb bis Einstein umstritten, welche der beiden Konstellationen bei Licht vorliegt (Korpuskulartheorie bzw. ballistische Theorie versus Wellentheorie).

    Bei einem ruhenden homogenen Äther gibt es keine Aberration. Um die Aberration darzustellen, musste der Äther „verrückte“ Eigenschaften annehmen (vgl. https://www.physicsforums.com/threads/stellar-aberration-and-fresnel-ether-drag.145559/ mwN).

    Der englische Wikipedia Eintrag über den DE geht von einem ruhenden Äther aus, einmal ruhend beim Sender (Fig. 2b) und einmal ruhend beim Empfänger (Fig. 3).

    Bei Fig. 2b wird der kürzeste Abstand zwischen Sender und Empfänger (Extremalwert) aus Sicht des Senders zum Zeitpunkt des Eintreffens des Lichtpulses beim Empfänger erreicht. Die lG ist am Ende der Wellenausbreitung Null. Das Signal hat aus der Sicht beider im Äther die kürzesten Weg zwischen Sender und Empfänger zurückgelegt. Die Geschwindigkeit der Welle im Äther wird durch die Relativbewegung nicht beeinflusst. Die Relativbewegung des Empfängers zum Zeitpunkt des Eintreffens der Welle erfolgt senkrecht zur Wellenfront.

    Bei Fig. 3 wird der kürzeste Abstand zwischen Sender und Empfänger (Extremalwert) aus Sicht des Empfängers zum Zeitpunkt des Aussenden des Lichtpulses vom Sender erreicht. Die lG ist am Beginn der Wellenausbreitung Null. Das Signal wird aus der Sicht beider im Äther die kürzesten Weg zwischen Sender und Empfänger zurücklegen. Die Geschwindigkeit der Welle im Äther wird durch die Relativbewegung nicht beeinflusst. Die Relativbewegung des Sender zum Zeitpunkt des Absendens der Welle erfolgt senkrecht zur Wellenfront.

    In beiden Fällen wäre die Argumentation mit der fehlenden longitudinalen Geschwindigkeitskomponente (lG) zwischen Sender und Empfänger zu den genannte Zeitpunkten schlüssig, wenn es sich tatsächlich um eine Wellenausbreitung im Äther handeln würde.

    Bei Licht gibt es aber keinen Äther. Damit ist das angebliche Fehlen einer lG unschlüssig und damit auch die angebliche Korrektur des klassischen Nullergebnisses beim DE durch „Zeitdilatation“.

    Eine korrekte Argumentation müsste sich mit dem klassischen DE mit Teilchen auseinandersetzen. Damit kommen neue Begriffe ins Spiel:

    „Signalgeschwindigkeit eines Teilchens“: Relativgeschwindigkeit eines Teilchens relativ zum Sender bzw. relativ zum Empfänger.

    „Signalweg eines Teilchens“: Weg, den ein Teilchen vom Sender zum Empfänger aus der Sicht des Senders bzw. des Empfängers zurücklegt.

    „Aberration“: unterschiedliche Richtung des Geschwindigkeitsvektors des Teilchens in Bezug auf Sender bzw. Empfänger.

    Eine lG fehlt, wenn die Teilchengeschwindigkeit relativ zum Sender gleich groß ist wie die Teilchengeschwindigkeit relativ zum Empfänger. Das ist dann der Fall, wenn der Signalweg eines Teilchens aus Sicht des Senders der gleiche ist wie aus Sicht des Empfängers. (In Geschwindigkeitsvektoren ausgedrückt: wenn das Vektordreieck gebildet aus dem Geschwindigkeitsvektor der Relativgeschwindigkeit zwischen Sender und Empfänger, dem Geschwindigkeitsvektor des ausgesendeten Teilchens aus Sicht des Senders und dem Geschwindigkeitsvektor des empfangenen Teilchens aus Sicht des Empfängers gleichseitig wird.) In eine ähnliche Richtung argumentiert übrigens auch der Wikipedia Eintrag, wenngleich die Anmerkung „Null frequency shift occurs for a pulse that travels the shortest distance from source to receiver.“ zeigt, dass der Kern der Sache nicht verstanden wird.

    Wird das Teilchen ausgesendet, wenn der Abstand zwischen Sender und Empfänger extremal ist, so ist ebenso eine lG vorhanden (Rotverschiebung), als wenn das Teilchen eintrifft, wenn der Abstand zwischen Sender und Empfänger extremal ist (Blauverschiebung). Keine lG tritt auf, wenn das Teilchen abgesendet wird, bevor der Abstand extremal wird, und wenn es eintrifft, nachdem der Abstand extremal war. Bei Sender und Empfänger besteht im gleichen Ausmaß eine gegensinnige Aberration.

    Nun zurück zum Wikipedia Eintrag:
    Die Fig. 2a beschreibt eine Situation, die nur bei einem DE mit Teilchen auftreten kann (Aberration). Der Artikel argumentiert also einmal mit einer Wellenausbreitung im Äther und einmal mit einer Ausbreitung von Teilchen. Die Argumentation mit Teilchen wird gleich wieder aufgegeben, weil sie unnötig kompliziert wäre („we find that the analysis is more complicated than it should be“). Denkt man sie zu Ende, sieht man, dass das durch die genannten Geschwindigkeitskomponenten gebildete Dreieck nach klassischer Auffassung (Bradley) spitzer ist (tan alpha = v) als nach relativistischer (sin alpha = v). Damit ist der Unterschied der Signalwege aus klassischer Sicht geringer als aus relativistischer, ebenso die lG und damit der transversale DE (Blauverschiebung). Eine „Korrektur“ durch eine rotverschiebende Zeitdilatation führt nicht zum gewünschten Ergebnis des relativistischen transversalen DE. Dieser ergibt sich vielmehr aus dem genannten Dreieck in seiner korrekten relativistischen Form (aus dem Verhältnis der beiden Signalwege aus relativistischer Sicht).

    Eben der selbe Einwand richtet sich gegen die in #923 diskutierte Situation nach einer Fig. 3b (die der Wikipedia Eintrag gar nicht behandelte). Auch hier führt der klassischen Aberrationswinkel nicht zu einem klassischen transversalen DE mit Teilchen (Rotverschiebung), der durch Zeitdilation „durch eine Blauverschiebung auf 1/γ“ auf den richtigen relativistischen Wert korrigiert würde.

    Vielleicht verstehe ich aber Ihre Argumentation besser, wenn ich nachvollziehen kann, wie Sie beim klassischen DE mit Teilchen bei senkrechtem Teilcheneinfall zu einer “Rotverschiebung mit dem Faktor 1/γ2“ kommen (und wie sie bei Fig 2a bei senkrechter Teilchenaussendung zu einer Blauverschiebung mit dem Faktor 1/γ2“ kämen).

  927. #929 Anonym_2023
    2. Januar 2023

    @Peter Strohmayer #928

    In der SRT funktioniert die Aberration sowohl beim Teilchenmodell als auch beim Wellenmodell des Lichts. In beiden Fällen lässt sich die Aberration mit der Lorentz-Transformation herleiten.

    Da sich Licht in jedem Bezugssystem mit c bewegt, sieht der “klassische” Term in der relativistischen Doppler-Formel in jedem Bezugssystem immer so aus wie die klassische Formel im Ruhesystem des Äthers.

    Ich habe einfach beschrieben, was in der Doppler-Formel im Sender-Ruhesystem steht.

    Die Doppler-Formel im Ruhesystem des Empfängers ist Eq. 6. Der Faktor 1/γ steht für die ZD aufgrund des bewegten Senders, die Klammer dahinter für den klassischen (longitudinalen) Dopplereffekt.

    Die Doppler-Formel im Ruhesystem des Senders ist Eq. 7. Der Faktor γ steht für die ZD aufgrund des bewegten Empfängers, die Klammer dahinter für den klassischen (longitudinalen) Dopplereffekt.

    Beide Formeln liefern natürlich das gleiche Ergebnis. Die Umrechnung des Winkels zwischen Sender- und Empfängersystem steht in Eq. 8.

    Quelle:
    https://en.wikipedia.org/wiki/Relativistic_Doppler_effect#Motion_in_an_arbitrary_direction

  928. #930 Anonym_2023
    3. Januar 2023

    @Peter Strohmayer #928

    “Bei einem ruhenden homogenen Äther gibt es keine Aberration.”

    In der SRT dagegen funktioniert die Aberration wie gesagt auch beim Wellenmodell des Lichts.

    In der SRT stehen die Licht-Wellenfronten immer senkrecht auf der Ausbreitungsrichtung:
    > Im Bezugssystem S’ der Lichtuhr sind die Wellenfronten waagerecht ausgerichtet.
    > Im Beobachtersystem S, in dem die Lichtuhr bewegt ist, sind die Wellenfronten um v/c gekippt.

    Herleitung mit der inversen Lorentz-Transformation der Zeit für einen nach unten bewegten Lichtimpuls:
    Δt = γ(Δt’ + v/c² * Δx’)

    => Im Bezugssystem S’ der Lichtuhr trifft eine Wellenfront der Breite Δx’= Δx₂’ – Δx₁’ die x’-Achse rechts und links gleichzeitig, d.h. Δt’=0. Daraus folgt Δt = γ v/c² * Δx’ und mit der Längenkontaktion Δt = v/c² * Δx.

    => Im Beobachtersystem S, in dem die Lichtuhr bewegt ist, trifft die linke Seite der Wellenfront die x-Achse um Δt früher als die rechte Seite der Wellenfront die x-Achse trifft (“Relativität der Gleichzeitigkeit”).

    D.h. wenn die rechte Seite der Wellenfront die x-Achse trifft, hat die linke Seite der Wellenfront die x-Achse schon um Δs=c * Δt passiert.

    Δs=c * Δt = c * v/c² * Δx = v/c * Δx.

    D.h. die Wellenfront ist gekippt um Δs/Δx = v/c. Bei der Galilei-Transformation wäre das nicht der Fall.

  929. #931 Peter Strohmayer
    Wien
    3. Januar 2023

    @Anonym #929

    “Ich habe [in #923] einfach beschrieben, was in der Doppler-Formel im Sender-Ruhesystem steht.”

    Das ist ja dann nur die klassische Ätherformel für den Schall bei einem im Äther ruhenden Sender, korrigiert durch den blauverschiebenden Zeitdilatationsfaktor. (Im Wikipedia Zitat wird die klassische Ätherformel für den Schall bei einem im Äther ruhenden Empfänger, korrigiert durch den rotverschiebenden Zeitdilatationsfaktor dargestellt).

    Das ist nicht die Überleitung vom klassischen ballistischen Dopplereffekt auf den relativistischen Fall unter Berücksichtigung der Aberration, von der in #928 die Rede war und bei der nur die Verhältnisse der Signalwege (mit einem Rechenfaktor Gamma) ausschlaggebend sind (aber nicht eine geheimnisvolle “Zeitdilatation”).

    Zu #930:
    “In der SRT dagegen funktioniert die Aberration wie gesagt auch beim Wellenmodell des Lichts.”

    Ja. Die Aberration ergibt sich aus der Theorie (den Signalwegen bzw. den lichtartigen Abständen), ohne sich zwischen Teilchen und Welle entscheiden zu müssen.

  930. #932 Anonym_2023
    3. Januar 2023

    @Peter Strohmayer #931

    “Das ist nicht die Überleitung vom klassischen ballistischen Dopplereffekt auf den relativistischen Fall unter Berücksichtigung der Aberration, von der in #928 die Rede war und bei der nur die Verhältnisse der Signalwege (mit einem Rechenfaktor Gamma) ausschlaggebend sind (aber nicht eine geheimnisvolle “Zeitdilatation”).”

    Doch.

    Wenn man in Eq. 6 als Winkel 90 Grad einsetzt, dann erhält man die Rotverschiebung entsprechend der ZD des im Empfängersystem bewegen Senders:
    f_r = f_s / γ.

    In Eq. 7 bekommt man dieselbe Rotverschiebung, wenn man für den Klammerausdruck 1/γ² einsetzt.

    Quelle:
    https://en.wikipedia.org/wiki/Relativistic_Doppler_effect#Motion_in_an_arbitrary_direction

  931. #933 Anonym_2023
    3. Januar 2023

    @Peter Strohmayer #931

    Ergänzung:

    Ich setze in Eq. 7 wegen der Aberration :

    cos (θ_s) = v/c

    Dann wird der Klammerausdruck (1-v²/c²) = 1/γ².

    Quelle:
    https://en.wikipedia.org/wiki/Relativistic_Doppler_effect#Motion_in_an_arbitrary_direction

  932. #934 Peter Strohmayer
    Wien
    7. Januar 2023

    @Anonym #923, #932 und #933:

    Keine der von Ihnen zitierten Formeln beschreibt den “klassischen Dopplereffekt mit Teilchen” (nach der Emissionstheorie).

    Seit vielen Monaten argumentieren Sie damit, dass der klassische Dopplereffekt bei 90° ein Nullergebnis haben würde, was durch „Zeitdilatation“ auf den relativistischen Wert korrigiert würde. Anders wäre das entsprechende Versuchsergebnis nicht zu erklären. Das wäre der Beweis für die Existenz der ZD.

    Ich hielt dem entgegen, dass es zwei Formeln für den klassischen Dopplereffekt gibt. Die einen – bekannteren – basieren auf der in der Natur vorkommenden Äthersituation (Schallwellen) ohne Aberration.

    Die anderen beruhen auf der Emission von Teilchen, die in Bezug auf den Sender immer mit der gleichen Geschwindigkeit ausgesendet werden (c=1) und bei einem relativ zum Sender mit v bewegten Empfänger für diesen mit entsprechend anderer Geschwindigkeit eintreffen. Nur dabei tritt Aberration auf. Daher ist der klassische DE mit Teilchen viel eher mit dem relativistischen DE zu vergleichen als mit dem im Äther.

    Meine Argumentation war nun, dass diese Art des klassischen Dopplereffekts bei 90 ° keineswegs ein Nullergebnis zeitigt und dass der dortige Wert des transversalen klassischen Dopplereffekts keineswegs durch den Faktor 1/Gamma auf den relativistischen Wert gebracht wird.

    Und vor allem: dass die Argumentation mit der angeblich beim transversalen DE fehlenden longitudinalen Geschwindigkeitskomponente zwischen Sender und Empfänger nicht stimmt, weil beim transversalen DE mit Teilchen sehr wohl eine longitudinale Geschwindigkeitskomponente auftritt, wie das Verhältnis der relativen Teilchengeschwindigkeiten zeigt. Sie fehlt nur in Ätherkonstellationen, weil sich dort die Signale nur nach dessen Bedingungen fortpflanzen.

    Die angeblich fehlende longitudinale Geschwindigkeitskomponente war aber Ihr Hauptargument dafür, dass beim transversalen relativistischen DE die reine “Zeitdilatation” beobachtet würde. Man könne beobachten, wie bei “Stillstand” zwischen Sender und Empfänger die Zeit beim Sender langsamer vergeht

    Meiner Argumentation mit dem “klassischen DE mit Teilchen” treten sie mit Formeln entgegen (“Doch” usw in #932), die dem engl. Wikipedia Artikel entnommen sind und die alle nur den relativistischen DE abbilden.

    Aus diesen ergibt sich keineswegs, dass der “klassische DE mit Teilchen” bei 90° einen Wert 1/Gamma2 hätte, der mit dem Faktor Gamma auf den relativistischen Wert gebracht werden könnte.

    Die Formel für den klassischen DE mit Teilchen lautet vielmehr (bei beta = Aussendewinkel in Bezug auf die Bewegungsachse; der Empfangswinkel ergibt sich über die Aberration)):

    fe = fs * sqr(1 + v2 + 2*v*cosbeta).

    Bei 0° zeigt sich der longitudinale DE mit

    fe = fs * (1+v).

    Bei 90° zeigt sich der transversale DE mit

    fe = fs * sqr(1 + v2). (Hier gibt es eben keinen „Nulleffekt“ als „Normalfall“).

    Bei Teilchen ist der Gedanke, man könne eine Korrektur mit „Zeitdilatation“ vornehmen, von vornherein Unsinn. Dass sie beim klassischen DE mit Äther als Rechenfaktor funktioniert, ist auf strukturell-mathematische Verwandtschaften, nicht aber auf reale physikalische Verhältnisse zurückzuführen.

    Das Verhältnis von fe zu fs ist übrigens für alle klassischen Konstellationen mit Teilchen das der Signalwege bzw. der Teilchengeschwindigkeiten relativ zu Sender bzw. Empfänger. Genau so beim relativistischen DE, nur mit dem Unterschied, dass hier sich das Verhältnis der Signalwege nicht aus der Gallileitransformation, sondern aus der Lorentztransfpormation ergibt.

    Eine „Zeitdilatation“, die den „Normalfall“ mit fehlender longitudinaler Geschwindigkeitskomponente (auch für den relativistischen Fall stellen Sie diese falsche Behauptung auf) korrigiert, gibt es nicht, weil es keinen „Normalfall“ gibt.

    Es ist daher nicht richtig, eine mathematische Korrektur des transversalen DE im Äther als Beweis für die Existenz der Zeitdilatation heranzuziehen.

  933. #935 Karl-Heinz
    Graz
    8. Januar 2023

    @Peter Strohmayer

    Kann es sein, dass dir die Zeitdilatation und damit die Relativitätstheorie ein Dorn im Auge ist?

  934. #936 Anonym_2023
    9. Januar 2023

    @Peter Strohmayer #934

    ” fe = fs * sqr(1 + v2 + 2*v*cosbeta). “

    Diese Formel kenne ich nicht. Wie wird sie hergeleitet?

  935. #937 Peter Strohmayer
    Wien
    9. Januar 2023

    @Anonam 2023 #936

    Im Gegensatz zur Äthertheorie ist bei der Emissionstheorie ausschließlich die Relativbewegung von Quelle und Empfänger für den Dopplereffekt fe = fs * (1+v/c), als auch für die Aberration verantwortlich. (Quelle: https://de.wikipedia.org/wiki/Korpuskeltheorie).
    (Die Formel entspricht dem longitudinalen Dopplereffekt bei einem im Äther ruhenden Sender.)

    Da die Korpuskeltheorie (mit additiver Teilchengeschwindigkeit) für das Licht zu Recht verworfen wird, andererseits aber der Fall von Masseteilchen, die in bestimmter Frequenz ausgesendet werden, praktisch nicht vorkommt, habe ich eine Verallgemeinerung der genannten Formel für beliebige Einfallswinkel nicht gefunden (wie übrigens auch nicht für den Dopplereffekt im Äther).

    Ich habe die Formel für Teilchen aus aus dem Kosinussatz abgeleitet (die Geschwindigkeitsvektoren v und c beim Sender bilden für alle Richtungen ein Parallelogramm mit gleichen Seitenlängen, der Geschwindigkeitsvektor c’ beim Empfänger ist dessen Diagonale).

  936. #938 Peter Strohmayer
    Wien
    10. Januar 2023

    @Anonym 2023 Nachtrag zu #930

    “In der SRT stehen die Licht-Wellenfronten immer senkrecht auf der Ausbreitungsrichtung.”

    Klassischer Dopplereffekt im Äther:

    Bei im Äther ruhendem Sender steht die eintreffende Wellenfront normal zur Bewegungsrichtung des Empfängers. Daher keine Frequenzänderung.

    Bei im Äther ruhendem Empfänger steht die ausgesendete Wellenfront normal zur Bewegungsrichtung des Empfängers. Daher keine Frequenzänderung.

    Relativistischer Dopplerereffekt:

    Wenn im Wellenmodell bei Licht (wie soll das ohne Äther gehen?) bei 90° eine Aberration aufträte (ich kann die Argumentation nicht nachvollziehen, weil sie ihr Ergebnis voraussetzt, ist aber nicht so wichtig), so wäre das – wie bei den Teilchen – auch ein Beweis für das Vorhandensein einer longitudinalen Geschwindigkeitskomponente (Empfänger schneidet die ankommenden Wellenfronten, wodurch sich die Frequenz für ihn ändert).

  937. #939 Anonym_2023
    10. Januar 2023

    @Peter Strohmayer #934

    “Und vor allem: dass die Argumentation mit der angeblich beim transversalen DE fehlenden longitudinalen Geschwindigkeitskomponente zwischen Sender und Empfänger nicht stimmt, weil beim transversalen DE mit Teilchen sehr wohl eine longitudinale Geschwindigkeitskomponente auftritt, wie das Verhältnis der relativen Teilchengeschwindigkeiten zeigt.”

    Wenn im Ruhesystem des Empfänger der Winkel 90 Grad ist – ohne Doppler-Frequenzunterschied – und im Ruhesystem des Senders ungleich 90 Grad, also mit Doppler-Frequenzunterschied, dann muss die verwendete Doppler-Formel falsch sein. Die Berechnungen in beiden Bezugssystemen müssen dasselbe Ergebnis für den Doppler-Frequenzunterschied liefern.

    In der SRT liefern die Berechnungen in beiden Bezugssystem dasselbe Ergebnis für den Doppler-Frequenzunterschied. Das liegt am γ-Faktor in den Formeln, also an der ZD.

  938. #940 Peter Strohmayer
    Wien
    10. Januar 2023

    @Anonym 2023 #939

    “Wenn im Ruhesystem des Empfänger der Winkel 90 Grad ist – ohne Doppler-Frequenzunterschied –”

    Das ist unrichtig. Die angegebene Formel stellt auf den Winkel in Bezug auf den Sender ab (eine Formel, die auf den Winkel in Bezug auf den Empfänger abstellt, habe ich noch nicht abgeleitet). Beispiel: Bei v= 0,8 und einem Abstrahlwinkel aus Sicht des Senders von 127° ergibt sich ein Frequenzunterschied fe=fs*0,8 (analog zur Rotverschiebung beim relativistischen DE) und beim Empfänger einen Aberrationswinkel von 90° (analog zum Aberrationswinkel beim relativistischen DE), also mit dem von Ihnen genannten Wert. Es tritt also bei 90° in Bezug auf den Empfänger sehr wohl ein Dopplereffekt auf (was in Anbetracht des genannten Geschwindigkeitsvektorenparallelogramms auf den ersten Blick klar ist).

    ” … und im Ruhesystem des Senders ungleich 90 Grad, also mit Doppler-Frequenzunterschied, …”

    das ist wieder richtig

    ” … dann muss die verwendete Doppler-Formel falsch sein. …”

    das ist wie gesagt falsch.

    “Die Berechnungen in beiden Bezugssystemen müssen dasselbe Ergebnis für den Doppler-Frequenzunterschied liefern.”

    Das tun sie selbstverständlich, wenngleich ich die Formel, die auf den Winkel aus Sicht des Empfängers abstellt, noch nicht abgeleitet habe, was aber eine unnötige Umformungsübung ist, weil sich das aus dem Parallelogramm und aus einer Beispielsrechnung wie der obigen ohnehin ergibt.

    “In der SRT liefern die Berechnungen in beiden Bezugssystem dasselbe Ergebnis für den Doppler-Frequenzunterschied. Das liegt am γ-Faktor in den Formeln, also an der ZD.”

    Dasselbe Ergebnis habe ich nicht in Zweifel gezogen. Gamma ist ein Rechenfaktor, der gemeinsam mit der Formel für den klassischen Dopplereffekt im Äther die richtige Formel für den relativistischen DE ergibt.

    Da die Situation bei Licht aber aus den genannten Gründen mit dem klassischen DE im Äther nichts zu tun hat (im klassischen Fall gibt es keine Aberration, jedoch eine vom Äther diktierte Signalgeschwindgkeit aus der Sicht eines der Beobachter und daher für den jeweils anderen keine longitudinale Geschwindigkeitskomponente), handelt es sich nur um eine mathematisch-strukturelle Verwandtschaft der beiden Formeln. Wieso der Faktor “Zeitdialatation” getauft werden soll, wo jeder weiß, dass es eine wechselseitige “Zeitdilatation”, und sohin eben keine wäre und die mit einem anderen Vergehen von Zeit nichts zu tun hat, kann ich mir nur damit erklären, dass manche Physiker als alttestamentarische Magier gesehen werden wollen. Es ist eine priesterliche Überheblichkeit, die den Verstand beleidigt.

  939. #941 Karl-Heinz
    Graz
    10. Januar 2023

    Hi darf ich bei euch auch mitdiskutieren?

  940. #942 Anonym_2023
    10. Januar 2023

    @Karl-Heinz #941

    “Hi darf ich bei euch auch mitdiskutieren?”

    Ja, gerne.

  941. #943 Karl-Heinz
    Graz
    11. Januar 2023

    @Anonym_2023

    Ich würde dann so vorgehen.
    Den Dopplereffekt nach der Korpuskeltheorie herleiten. Aufzeigen was dafür alles zu beachten ist.

    Wenn man das so einigermaßen überzeugend findet, würde ich dann den Transversaler Doppler-Effekt für unsere Raumzeit herleiten.
    Wie sich unsere Raumzeit benimmt, ist ja durch die Lorentztransformation, in der die maximale Geschwindigkeit die Lichtgeschwindigkeit ist, genau definiert. Ich würde dann auch auf einige Begriffe eingehen mit dem Versuch Missverständnisse auszuräumen. Wäre das für dich auch Ok lieber Peter Strohmayer?

  942. #944 Peter Strohmayer
    Wien
    11. Januar 2023

    @Karl-Heinz #943

    Genau das ist der richtige Weg. Ich hoffe, ich habe den DE nach der Korpuskeltheorie richtig abgeleitet.

  943. #945 Karl-Heinz
    11. Januar 2023

    @Peter Strohmayer

    Meinst du diese Formel?

    ” fe = fs * sqr(1 + v2 + 2*v*cosbeta). “

    SQR bedeutet hoch 2
    SQRT bedeutet Wurzel
    meinst du sqr oder sqrt ?
    mit v2 meinst du da v² bzw. v^2 ?
    mit v meinst du da normierte Geschwindigkeit v auf c?

    Der Teufel steckt im Detail. 😉

  944. #946 Peter Strohmayer
    Wien
    11. Januar 2023

    Danke für den Hinweis.

    fe = fs * sqrt(1 + v^2 + 2*v*cosbeta).

    c ist der Geschwindigkeitsvektor der Teilchen in Bezug auf den Sender. Er wird betragsmäßig mit “1” angenommen.

    Sender und Empfänger bewegen sich mit v auf parallelen Bewegungsachsen gleichförmig aneinander vorbei. v als Vektor verläuft entlang der Bewegungsachse des Senders. Der Betrag von v ist ein Bruchteil von c (v = 0,8 = 0,8*c).

    v’ wäre der Geschwindigkeitsvektor der Teilchen in Bezug auf den Empfänger mit einem Betrag als Bruchteil von c.

  945. #947 Peter Strohmayer
    Wien
    11. Januar 2023

    Korrektur: c’ wäre der Geschwindigkeitsvektor der Teilchen in Bezug auf den Empfänger mit einem Betrag als Bruchteil von c.

  946. #948 Karl-Heinz
    Graz
    12. Januar 2023

    Peter Strohmayer

    (I) fe = fs * sqrt(1 + v^2 + 2*v*cosbeta)

    Plausibilitätsprüfung:
    beta = 0° und 90°
    Erwartet wird:
    fe = fs * (1±v/c)
    fe = fs

    Nach Formel (I) ergibt sich für
    —————————————-
    beta = 0°
    fe(0°) = fs * sqrt(1 + v^2 + 2*v)
    —————————————–
    beta = 90°
    fe(90°) = fs * sqrt(1 + v^2)
    —————————————-

    Plausibilitätsprüfung ist negativ.
    Der Teufel steckt im Detail. 😉

  947. #949 Karl-Heinz
    Graz
    12. Januar 2023

    @Peter Strohmayer

    Merke: Es ist sinnvoll abgeleitete Formeln auf ihre Richtigkeit und Plausibilität zu prüfen. 😉

  948. #950 Anonym_2023
    12. Januar 2023

    @Karl-Heinz #943

    “Ich würde dann so vorgehen.
    Den Dopplereffekt nach der Korpuskeltheorie herleiten. Aufzeigen was dafür alles zu beachten ist.”

    OK. In Fig. 3 in Wikipedia bewegt sich das Photon im Ruhesystem des Empfängers mit c in (-y) Richtung (90 Grad Winkel). Daher ist sein Viererimpuls:
    P = (p_t, p_x, p_y, p_z) = E/c² (c, 0, -c, 0) = hf/c² (c, 0, -c, 0).

    Lozentz-Transformation des Viererimpulses in das Bezugssystem des mit v nach rechts bewegten Senders:

    p_t’ = γ (p_t – β * p_x)
    p_x’ = γ (p_x – β * p_t)
    p_y’ = p_y
    p_z’ = p_z

    =>

    p_x’ = γ (0 – β * hf/c) = = – γ hf/c * v/c
    p_y’ = -hf/c

    => Betrag des Dreierimpules im Sendersystem:

    p’ = √ {(p_x’)² + (p_y’)²} = hf/c √ {γ² v²/c² +1} = γ hf/c.

    hf’/c = γ hf/c.

    f = f’ / γ.

  949. #951 Karl-Heinz
    Graz,11.Bez.:Mariatrost
    12. Januar 2023

    @Anonym_2023

    Oh, sieht cool aus.
    Ich bin begeistert 🙂
    Das ist natürlich nur was für Fortgeschrittene.
    Bist du ein Physiker?

  950. #952 Karl-Heinz
    Graz
    13. Januar 2023

    @Peter Strohmayer

    Ich möchte zunächst den Begriff Frequenz näher erläutern.
    f … Frequenz
    T… Periodendauer

    f =1/T

    Frage: Ist Umdrehung pro Minute eine Frequenz?

    Wechselstrom bzw. Drehstrom hat eine Periodendauer von 20 ms
    Damit macht ein Drehstrommotor mit einem Polpaar in der Minute 3000 Umdrehungen
    = 60 Sekunden/(0,02 Sekunden pro Umdrehung) =3000 Umdrehungen.

    3000 Umdrehungen pro Minute = 3000/(60 Sekunden) = 50 Umdrehung pro Sekunde = 50 Hertz.

    Wir sehen, dass der Begriff Frequenz = 1/Periodendauer Sinn macht.

    Sorry wegen diesem Einschub, der zu den Grundlagen gehört.

  951. #953 Peter Strohmayer
    Wien
    13. Januar 2023

    @Karl-Heinz

    #949
    “Merke: Es ist sinnvoll abgeleitete Formeln auf ihre Richtigkeit und Plausibilität zu prüfen.”

    Die Prüfung wurde in #934 durchgeführt. Ergebnis: die Formel entspricht dem Experiment = der Erwartung. Bei Teilchen ist der transversale DE wegen der Aberration und der unterschiedlichen Teilchengeschwindigkeit in Bezug auf Empfänger bzw. Sender nicht null.

    #948
    “beta = … und 90°
    Erwartet wird:
    … fe = fs”

    Nicht die Formel ist umplausibel, sondern ihre “Erwartung”.

  952. #954 Karl-Heinz
    Graz
    13. Januar 2023

    @Peter Strohmayer

    Nö … du schreibst

    fe = fs * sqr(1 + v^2 + 2*v*cosbeta).

    Bei 0° zeigt sich der longitudinale DE mit

    fe = fs * (1+v).

    Jetzt abgesehen davon das cos(0°) = 1 und cos (90°) = 0

    hast du ein v^2 drinnen und was ist mit der Wurzel?

    Darum möchte ich mit dir die Herleitung gemeinsam durchgehen. 🙂

  953. #955 Karl-Heinz
    Graz
    13. Januar 2023

    Wegen der Aberration (Astronomie), da müssen oder sollten wir das auch ausdiskutieren. Falls du keine Lust dazu hast, sag einfach ein lautes NEIN. 🙂

  954. #956 Peter Strohmayer
    Wien
    13. Januar 2023

    @Karl-Heinz #954

    0°:

    fe = fs * sqrt(1 + v^2 + 2*v*1).

    fe = fs * sqrt((1 + v)^2)

    fe = fs * (1 + v)

  955. #957 Karl-Heinz
    Graz
    13. Januar 2023

    @Peter Strohmayer

    Stimmt, du hast Recht. Hatte ich vollkommen übersehen. Ich habe da so einen leisen Verdacht, wie du auf die untenstehende Formel kommst.
    Mal schauen. Eventuell ist es durch deine Aberration (Astronomie). Muss ich erst prüfen. 🙂

    fe(90°) = fs * sqrt(1 + v^2)

  956. #958 Anonym_2023
    13. Januar 2023

    @Peter Strohmayer #956

    0°:
    fe = fs * sqrt(1 + v^2 + 2*v*1).
    fe = fs * sqrt((1 + v)^2)
    fe = fs * (1 + v)

    Warum hast du nicht, um beliebige Winkel zu berücksichtigen, in der unteren Gleichung (Kosinussatz) das v mit cosbeta multipliziert?

    Dann wäre in der oberen Gleichung das v an beiden Stellen durch v*cosbeta ersetzt.

    Das Ergebnis wäre dann für 90 Grad
    fe = fs * 1, was genauso dem Experiment widerspräche, wie
    fe(90°) = fs * sqrt(1 + v^2). Das enthält wegen das Pluszeichen weder γ noch 1/γ.

    Die SRT sagt dagegen wegen das ZD in Übereinstimmung mit dem Experiment voraus:
    fe (90° im Sendersytem) = fs * γ
    fe (90° im Empfangersytem) = fs * 1/γ.

  957. #959 Peter Strohmayer
    Wien
    13. Januar 2023

    @Anonym #950

    Der akzeptierte Vorschlag von Karl-Heinz in #943 lautete:

    “Den Dopplereffekt nach der Korpuskeltheorie herleiten [den klassischen DE mit Teilchen herleiten].
    (…) würde ich dann den Transversaler Doppler-Effekt für unsere Raumzeit herleiten [den relativistischen DE herleiten].”

    Sie leiten den relativistischen DE für 90° ab, wie das in vielen Lehrbüchern beschrieben wird. Wozu? Das ist ja unstrittig.

    Unser Ziel war es herauszufinden, welchem Umstand beim Übergang vom klassischen zum relativistischen DE Bedeutung zukommt.

    Sie behaupten: hier waltet die “Zeitdilatation”.

    Ich behaupte: hier waltet das unterschiedliche Verhältnis der Signalwege nach der Galilei-Transformation bzw. nach der L-T.

    Der Unterschied ist nicht auf ein anderes Vergehen von Zeit beim “bewegten” Beobachter (den es objektiv gar nicht gibt) zurückzuführen, sondern auf die unterschiedliche Länge der Ausbreitung von Lichtpulsen, die zwei sich einander begegnende Beobachter aussenden. Wessen Signalweg länger ist (und wie nach dem Verhältnis der DE ausfällt), hängt nur von der Aussenderichtung, nicht aber von einem anderen Vergehen von Zeit ab.

    Auch der englische Wikipedia Eintrag stellt letztlich auf Signalwege ab, wie man bei seinem Versuch, das Nullergebnis zu erklären, sieht. Im Übrigen stiftet er aber mit seiner Vermischung von klassischem DE mit Teilchen (Fig. 2a, wovon er letztlich die Finger lässt, weil er “unnötig kompliziert” sei) mit dem klassischen DE im Äther (Fig. 2b und Fig 3) nur Verwirrung.

  958. #960 Peter Strohmayer
    Wien
    13. Januar 2023

    @Anonym #959

    Ich verstehe überhaupt nichts und bitte um eine nähere Erklärung (“obere” Gleichung, “untere” Gleichung …?): was habe ich nicht mit cosbeta multipliziert (hier handelt es sich doch um die Anwendung der Formel für den Fall cosbeta=1). Die Formel steht in #946. Wie soll bei 90° fe=fs herauskommen?

    Ich habe noch immer den Verdacht, dass Sie den klassischen DE mit Teilchen ausblenden und beim relativistischen DE eine Formel beweisen wollen, die niemand bestreitet.

    Der Punkt ist doch: beim klassischen Dopplereffekt mit Teilchen – der den Verhältnissen bei Licht wegen der ebenfalls auftretenden Aberration eher entspricht als der klassische DE im Äther – entsteht ein transversaler DE, dessen Wert sich nicht durch den Faktor Gamma vom relativistischen Wert unterscheidet. Daher ist der Übergang vom klassischen zum relativistischen Fall nicht durch die Einwirkung einer Zeitdilatation auf das “klassische Verhältnis” zu erklären. Wir könnten darüber streiten, wer von den beiden der “wirkliche”, der “maßgebliche” klassische DE ist. Worüber ich aber nicht mehr streiten wollte, wäre die Behauptung, dass es nur eine Art des klassischen DE gäbe.

  959. #961 Karl-Heinz
    Graz
    13. Januar 2023

    Ich weiß, wo der Vergleich hinkt.

    Das kommt, wenn man von 90° spricht und übersieht, dass jeder einen unterschiedlichen Winkel meint.

    Mein Winkel und auch der Winkel von Anonym, ist ein Winkel zwischen Bewegungsrichtung und Wellenfront.

    @Peter Strohmayer
    Verzeih Peter
    Du packst dein Fernrohr aus und dein Winkel ist der Winkel zwischen Bewegungsrichtung und Ausrichtung des Fernrohres. Bei dir fließt die Aberration mit ein, was bei uns nicht der Fall ist.

    Was lernen wir daraus? 90° ist nicht unbedingt 90°.
    Ich werde das noch grafisch veranschaulichen.

  960. #962 Karl-Heinz
    Graz
    13. Januar 2023

    @Peter Strohmayer

    Wir befinden uns auf x=-10 und y=0.
    Wir bewegen uns in y-Richtung nach oben.

    siehe Bild

  961. #963 Peter Strohmayer
    Wien
    14. Januar 2023

    @Karl-Heinz #961

    In der Formel fe = fs * sqrt(1 + v^2 + 2*v*1) bedeutet beta den “Aussendewinkel in Bezug auf die Bewegungsachse” (#940). (Bewegungsachse = Geschwindigkeitsvektor v des Empfängers aus Sicht des Senders). v ist positiv, wenn sich der Sender aus Sicht des Empfängers nach rechts und daher der Empfänger aus Sicht des Senders nach links bewegt.

    Der Winkel beta beträgt 0°, wenn das Teilchen vom Sender direkt zum entgegenkommenden Empfänger gesendet wird (longitudinaler DE in Annäherung). Der Winkel beta öffnet sich im weitern Verlauf aus Sicht des Senders im Uhrzeigersinn nach unten und erreicht 90°, wenn der Sender die Teilchen normal zum Geschwindigkeitsvektor v nach unten sendet (so, dass das Teilchen vom Empfänger aus gesehen schräg von links in einem spitzen Winkel eintrifft, wenn die Entfernung zwischen Sender und Empfänger ein Minimum erreicht).

    Aberrativ dazu gestaltet sich der Winkel alpha, in dem die Teilchen vom Empfänger aus gesehen in Bezug auf die Bewegungsachse auf ihn zukommen. Er beginnt mit dem longitudinalen Fall ebenfalls bei 0° und öffnet sich dann im Uhrzeigersinn nach oben und erreicht 90°, wenn der Winkel ß die 90° bereits überschritten hat.

  962. #964 Pter Strohmayer
    Wien
    14. Januar 2023

    Korrektur: fe = fs * sqrt(1 + v^2 + 2*v*cosbeta) natürlich.

  963. #965 Peter Strohmayer
    Wien
    14. Januar 2023

    Hier nun auch die Formel des DE in Abhängigkeit vom Empfangswinkel alpha iSd #963 (DE mit „Empfängerformel“):

    fe = fs * (v*cosalpha + sqrt(v^2(cosalpha^2-1) + 1))

    Beispiel:

    v=0,8, alpha=90° (cosalpha=0):

    fe = fs * sqrt(-0,64 + 1) = fs * 0,6.

    fe = fs * 0,6.

    In diesem Fall, beträgt der Aussendewinkel iSd #963 aus Sicht des Senders beta 143°.

    Probe (DE mit „Senderformel“):

    v=0,8, beta=143° (cosbeta=-0,8):

    fe = fs * sqrt(1 + v^2 + 2*v*cosbeta)

    fe = fs * sqrt(1 + 0,64 – 2*0,8*0,8)

    fe = fs * sqrt(1,64 – 1,28)

    fe = fs * 0,6.

    Aus beider Sicht ist der DE gleich.

  964. #966 Anonym_2023
    14. Januar 2023

    @Peter Strohmayer #965

    “Aus beider Sicht ist der DE gleich.”

    In der SRT berechnet man die Frequenz eines Photons aus dessen (bezugssystem-abhängiger) Energie:
    E = hf.

    Woraus berechnet man die Frequenz Ihres klassischen Korpuskels?

  965. #967 Karl-Heinz
    Graz
    14. Januar 2023

    @Peter Strohmayer

    Hi, ich muss mir das Ganze nochmals überlegen. Habe gerade bemerkt, dass sich der lokale Wellenvektor drehen kann. Mal gucken, was für Auswirkungen der Wellenvektor auf meine Übrlegung hat.

  966. #968 Anonym_2023
    15. Januar 2023

    @Pter Strohmayer #964

    “Korrektur: fe = fs * sqrt(1 + v^2 + 2*v*cosbeta) natürlich.”

    Das kann für beta = 90 Grad nicht stimmen.

    Nicht-relativistische “Korpuskel”-Beschreibung vom Szenarion in Wikipedia Fig. 2(b):

    Der ruhende Sender schickt mit dem kleinen Zeitabstand Δt zwei Korpuskeln nach senkrecht unten mit Geschwindigkeit c aus. Ein punktförmiger Empfänger könnte nur eine Korpuskel davon empfangen, weil er sich nach links bewegt. Um beide zu empfangen, müsste der Empfänger eine endliche Ausdehnung v*Δt haben. Diese kann aber beliebig klein sein, wenn man das Δt beliebig klein wählt.

    Das Empfangs-Δt ist dann gleich dem Sende-Δt (gleiches c, gleiche Entfernungen), also kein Doppler-Effekt.

    Bei relativistischer Rechnung käme der ZD-Faktor γ hinzu.

    https://en.wikipedia.org/wiki/Relativistic_Doppler_effect#Source_and_receiver_are_at_their_points_of_closest_approach

    @Peter Strohmayer #956
    ” 0°:

    fe = fs * (1 + v) ”

    Beim relativistischen longitudinalen Dopplereffekt kommt – experimentell bestätigt – auch der ZD-Faktor γ hinzu.

  967. #969 Karl-Heinz
    15. Januar 2023

    @Anonym_2023 und Peter Strohmayer

    Ich stelle mal meine Erkenntnisse bezüglich Dopplerefekt für die Korpuskeltheorie vor.
    Gegeben sei ein Stern, welcher mit der Periodendauer T für einen sehr kurzen Augenblick Lichtquanten (Korpuskeln) mit der Geschwindigkeut c, aussenden. Der Stern als Sender befindet sich in Ruhe, wobei sich der Empfänger einmal

    Fall a) mit der Geschwindigkeit v auf den Stern sich zubewegt

    Fall b) senkrecht mit der Geschwindigkeit v am äusseren Ende der Verbindungslinie “Empfänger-Stern” sich am Stern vorbeibewegt.

    Bezeichnungen
    c … Geschwindigkeit der Korpuskeln(Photonen) zum Stern
    v … Geschwindigkeit Empfänger zum ruhenden Stern

    T … Periodendauer für Lichtaussendung Stern
    T’ …gemessene Periodendauer beim Empfänger

    λ …Wellenlänge im System Stern
    λ’ …gemessene Wellenlänge im beim Empfänger

    f … Frequenz im System Stern
    f’ …gemessene Frequenz beim Empfänger

    Wichtige Beziehungen
    f = 1/T
    f’ =1/T’
    allgemein: Weg = Geschwindigkeit * Zeit

    Für Fall a: Beobachter bewegt sich mit der Geschwindigkeit v auf den Stern zu.

    (I): f * λ = c
    (II): f’ * λ’ = c + v
    (III): f’ = (1+v/c) * f
    aus obigen Gleichungen folg:
    (IV): λ’ = λ

    Für Fall b)Empfänger bewegt sich mit der Geschwindigkeit v senkrecht am äusseren Ende der Verbindungslinie “Beobachter-Stern” am Stern vorbei.

    (I): f * λ = c
    (II): f’ * λ’ = √(c² + v²)
    (III): f’ = f
    (IV): Aberrationwinkel (klassisch) = arctan(v/c)
    aus obigen Gleichungen folg:
    (V): λ’ = √(1 + v²/c²) * λ

  968. #970 Peter Strohmayer
    Wien
    16. Januar 2023

    @Anonym #966

    “In der SRT berechnet man die Frequenz eines Photons aus dessen (bezugssystem-abhängiger) Energie”

    Dieser Zugang ist für mich sowieso der genialste. Der Dopplereffekt zeigt sich schon an einem einzigen Photon (man braucht keine Teilchen oder Wellen, die in einem bestimmten zeitlichen Abstand ausgesendet werden). Er ist aus der Konstanz des Produkts der Dauer eines Wechselwirkungsvorgangs (einmal aus der Sicht des Senders, einmal aus der Sicht des Empfängers) mit der dabei übertragenen Energie abzuleiten. Die beim Auftreffen eines Photons umgesetzte Energie ist umso größer, je kürzer der Wechselwirkungsvorgang dauert. Die Frequenz des Photons ist proportional zur Energie und damit umgekehrt proportional zur Dauer des Wechselwirkungsvorgangs. Die Dauer des Wechselwirkungsvorgangs aus der Sicht des jeweiligen Bezugssystems ergibt sich aus den Längen der Signalwege aus der Sicht des jeweiligen Bezugssystems, und diese wiederum aus der L-T. Für die SRT genügt es, die auf der Begrenztheit der Wirkungsausbreitung beruhenden unterschiedlichen Längen der Signalwege (die L-T) zu verstehen.

    Dasselbe (die Bedeutung der Signalwege) lässt sich übrigens auch für herkömmliche Signalfolgen (für Photonen, die in gleichen zeitlichen Entfernungen ausgesendet werden) zeigen, wo es auf die unterschiedlichen Taktlängen ankommt.

    “Woraus berechnet man die Frequenz Ihres klassischen Korpuskels?”

    Eben auch aus den Signalwegen, die im Verhältnis der Galilei-Transformation stehen, weil sich die unterschiedlichen Teilchengeschwindigkeiten in Anbetracht des gleichen Zeitmaßes im Verhältnis der Signalwege niederschlagen. (Bei dieser Betrachtung findet eine klassische und nicht eine relativistische Addition der Geschwindigkeitsvektoren statt.)

  969. #971 Peter Strohmayer
    Wien
    16. Januar 2023

    @Anonym #968

    “Ein punktförmiger Empfänger könnte nur eine Korpuskel davon empfangen, weil er sich nach links bewegt. …”

    Dieses Argument ist zu hinterfragen, denn es würde in weiterer Folge bedeuten, dass bei Teilchen nur ein longitudinaler DE auftreten kann. Sobald irgendein Winkel zur Bewegungsachse besteht, würde “Das Empfangs-Δt dann gleich dem Sende-Δt (gleiches c, gleiche Entfernungen)” sein und kein DE auftreten. Dass dem nicht so ist, beweist das aus Teilchen bestehende Licht, das sein Ziel auch bei einem punktförmig gedachten Empfänger findet.

  970. #972 Anonym_2023
    16. Januar 2023

    @Peter Strohmayer #971

    “Dieses Argument ist zu hinterfragen, denn es würde in weiterer Folge bedeuten, dass bei Teilchen nur ein longitudinaler DE auftreten kann. Sobald irgendein Winkel zur Bewegungsachse besteht, würde “Das Empfangs-Δt dann gleich dem Sende-Δt (gleiches c, gleiche Entfernungen)” sein und kein DE auftreten. Dass dem nicht so ist, beweist das aus Teilchen bestehende Licht, das sein Ziel auch bei einem punktförmig gedachten Empfänger findet.”

    Dass kein DE auftreten würde, ist ja nur das Ergebnis der nicht-relativistischen Rechnung.

    In der Praxis, bei Experimenten, gibt es kein Problem, weil Sender, Lichtsignale und z.B. das Teleskop des Empfängers jeweils eine endliche Ausdehnung haben. Und die ZD wurde, wie gesagt, beim transversalen DE mit hoher Genauigkeit gemessen.

  971. #973 Peter Strohmayer
    Wien
    16. Januar 2023

    @Anonym #972

    “Dass kein DE auftreten würde, ist ja nur das Ergebnis der nicht-relativistischen Rechnung.”

    Nein, diese Rechnung (siehe #965) ergibt, dass ein transversaler DE auftritt.

    Auch das Gedankenexperiment ergibt, dass ein DE auftritt, wenn man statt Lichtteilchen Masseteilchen setzt (so wie Bradley es getan haben muss).

    Es ist Ihr Gedankenexperiment, in dem kein DE auftritt.

  972. #974 Peter Strohmayer
    Wien
    16. Januar 2023

    Nachtrag:
    Beim transversalen DE mit Teilchen kommt es zu einer longitudinalen Geschwindigkeitskomponente, die bei Ihrem Gedankenexperiment unter den Tisch fällt.

    Beispiel c=1; v=0,6; c’=Teilchengeschwindigkeit aus Sicht des Empfängers:
    90° Aussendung aus Sicht des Senders – 31° Eintreffen aus Sicht des Empfängers (“Vorhaltewinkel”).

    “Entfernung Sender Empfänger” >> “Signalabstand*v”: zwei aufeinanderfolgende Signale kommen für den Empfänger aus der gleichen Richtung.

    v*t, c*t und c’*t bilden ein rechtwinkeliges Dreieck. Es kommt zu einer Blauverschiebung, weil sich von einem Signal zum nächsten die Entfernung zwischen Sender und Empfänger um c’*t – c*t verringert. Das bilden die Formeln #965 ab.

  973. #975 Anonym_2023
    16. Januar 2023

    @Peter Strohmayer #974

    “Es kommt zu einer Blauverschiebung”

    Nein. In #974 beschreiben Sie dasselbe Szenario, das ich in #968 beschrieben habe, nur diesmal im Ruhesystem des Empfängers. Das ändert nichts am Ergebnis: Ein punktförmiger Empfänger kann nicht beide Korpuskeln empfangen. Wenn man dagegen beide Empfangsorte auf einem ausgedehntem Empfänger berücksichtig, gibt es keinen DE.

    Schlussfolgerung: Eine nicht-relativistische Rechnung mach bei Korpuskeln keinen Sinn.

    Sinn macht dagegen eine nicht-relativistische Rechnung bei Schallwellen. Dafür gibt es auch eine Formel für beliebige Winkel, und zwar die Formel unter der Formel (2) in Wikipedia:
    https://de.wikipedia.org/wiki/Doppler-Effekt#Beobachter_bewegt,_Signalquelle_in_Ruhe

    Das dort angegebene Skalarprodukt aus dem Geschwindigkeitsvektor und dem angegebenen Einheitsvektor in Richtung von Sender zum Empfänger berechnet sich wie folgt:
    https://de.wikipedia.org/wiki/Skalarprodukt

  974. #976 Peter Strohmayer
    Wien
    16. Januar 2023

    @Anonym #975

    “Ein punktförmiger Empfänger kann nicht beide Korpuskeln empfangen. (…) Eine nicht-relativistische Rechnung macht bei Korpuskeln keinen Sinn.”

    Doch: Ein bewegter Stern kann für den Empfänger auch jahrelang stillstehen, wenn er weit genug entfernt ist.
    Auch ein infinitesimales Nachjustieren des Aussendewinkels (das Szenario einer großen Anzahl von radial ausgesendeten Teilchen) würde ihr “Problem” lösen, ohne am Prinzip des DE mit Teilchen Substantielles zu ändern.

    Es ist leichter, Fakten zu negieren, als Vorurteile zu revidieren. Wir sind bei #960 Schlusssatz angelangt. Der Beweis nach #943 wird abgesagt. Es kann nicht sein, was nicht sein darf.

  975. #977 Anonym_2023
    16. Januar 2023

    @Peter Strohmayer #976

    “Auch ein infinitesimales Nachjustieren des Aussendewinkels (das Szenario einer großen Anzahl von radial ausgesendeten Teilchen) würde ihr “Problem” lösen, ohne am Prinzip des DE mit Teilchen Substantielles zu ändern.”

    Wenn sich z.B. im Ruhesystem des Empfängers der Winkel nur infinitesimal ändern soll, dann ist auch der Unterschied der Weglängen aufeinanderfolgender mit c’ bewegten Korpuskeln nur infinitesimal klein, also kein unterschiedliches Δt, kein DE.

    Sie habe das Problem in #971 schon richtig beschrieben:

    Dieses Argument ist zu hinterfragen, denn es würde in weiterer Folge bedeuten, dass bei Teilchen nur ein longitudinaler DE auftreten kann.

  976. #978 Peter Strohmayer
    Wien
    17. Januar 2023

    @Anonym #977

    “Wenn sich z.B. im Ruhesystem des Empfängers der Winkel nur infinitesimal ändern soll, dann ist auch der Unterschied der Weglängen aufeinanderfolgender mit c’ bewegten Korpuskeln nur infinitesimal klein …”

    Sie irren. Ich empfehle, die (bei “unendlicher” Entfernung parallelen) Teilchenbahnen (mit gedachtem Schnittpunkt beim punktförmigen Empfänger) in der Nähe des Senders aufzuzeichnen, dann ergibt sich die Änderung der Länge der Teilchenbahnen zwischen zwei Takten auf den ersten Blick. Das kann auch nicht anders sein, denn bei Ihrer Argumentation würden auch Photonen den punktförmigen Empfänger verfehlen. Allerdings ist bei diesem Modell das Verhältnis der Teilchenwege nicht exakt das der Beträge der Geschwindigkeitsvektoren c und c’, sondern um den Rechenfaktor Gamma mehr, was ich bei den Formeln noch nicht berücksichtigt habe. (Bitte glauben Sie jetzt nicht, dass diese Korrektur auf die Zeitdilatation zurückzuführen wäre.)

  977. #979 Karl-Heinz
    Graz
    17. Januar 2023

    @Peter Strohmayer

    Bitte glauben Sie jetzt nicht, dass diese Korrektur auf die Zeitdilatation zurückzuführen wäre.

    Da wären wir also wieder bei der Lichtuhr. Und so schließt sich der Kreis. Aber sowas kann jemanden wie dich nicht erschüttern, stimmts?. 🙂

  978. #980 Anonym_2023
    18. Januar 2023

    @Peter Strohmayer #978

    “Bitte glauben Sie jetzt nicht, dass diese Korrektur auf die Zeitdilatation zurückzuführen wäre.”

    Die Diskussion über das γ können wir glaube ich vereinfachen, weil die ZD auch beim relativistischen, longitudinalen Dopplereffekt mit dem Winkel 0 Grad vorhanden ist. Wenn wir den Fall 0 Grad betrachten, können wir uns die Diskussion über Aberration sparen und in diesem Spezialfall sowohl bei der relativistischen als auch bei der klassischen Rechnung wahlweise mit Teilchen oder mit Wellen argumentieren.

    In #805 habe ich mit der LT die Formel für den relativistischen, longitudinalen Dopplereffekt hergeleitet:

    f’/f = γ * (1-v/c)
    Diese Gleichung ist mathematisch äquivalent zu
    f’/f = (1/γ) * 1/(1+v/c)

    Die erste Gleichung ist das Produkt aus der klassischen Dopplergleichung für ruhenden Sender mit dem ZD-Faktor γ, weil in diesem Bezugssystem der Empfänger bewegt ist und damit eine ZD hat.

    Die zweite Gleichung ist das Produkt aus der klassischen Dopplergleichung für ruhenden Empfänger mit dem ZD-Faktor 1/γ, weil in diesem Bezugssystem der Sender bewegt ist und damit eine ZD hat.

    Bei den klassischen (nicht-relativistischen) Gleichungen fehlt das γ, weil diese fälschlicherweise davon ausgehen, das Licht habe jeweils nur in einem der beiden Bezugssysteme die Geschwindigkeit c.

  979. #981 Karl-Heinz
    Graz
    18. Januar 2023

    @Anonym_2023

    Ok, ich mache mal die Umformung, damit Peter aus Wien sie auch begreift.
    ——————————————————–
    Relativistische Longitudinaler Dopplereffekt
    f’ = 1/γ * 1/(1-v/c) *f

    f’ = 1/γ * 1/(1-v/c) *f = 1/γ * ((1+v/c))/[(1-v/c) * (1+v/c)] * f = 1/γ * ((1+v/c))/[(1-v²/c²)] *f = γ²/γ * ((1+v/c))* f = γ * ((1+v/c)) *f
    f’ = γ *(1+v/c)* f

    ——————————————————————
    Longitudinaler Dopplereffekt nach Korpuskeltheorie

    f’ = (1+v/c) * f

  980. #982 Karl-Heinz
    Graz
    18. Januar 2023

    Können Philosophen die Relativitätstheorie verstehen?
    Ich habe da meine Zweifel, weil Philosophen glauben, neue abartige Begriffe und abartige Thesen einführen zu müssen, die nur ein Philosoph versteht.

  981. #983 MartinB
    19. Januar 2023

    Ja, können Philosophen, zumindest einige. Ich habe z.B. mal einen sehr guten Artikel von einem Philosophen über Begriffe in der SRT gelesen. Und ich weiß ja, dass Philosophie-Bashing unter Physikern sehr in ist (manchmal auch durchaus zu Recht), aber Philosophie pauschal für nutzlos zu erklären, halte ich für vollkommen falsch.

    Siehe auch z.B. diesen Artikel:
    https://scienceblogs.de/hier-wohnen-drachen/2018/10/03/ist-das-naturwissenschaftliche-weltbild-rational/

  982. #984 Karl-Heinz
    Graz
    19. Januar 2023

    @MartinB

    Oh, … Sorry
    Natürlich hatte ich nie die Absicht, die Philosophie herabzusetzen. Sie nimmt auch für mich einen sehr hohen Stellenwert ein.

    Was Philosophen halt nicht machen sollten, ist drüberfahren bzw. andere Ansichten beinhart zu ignorieren. Das geht mit ziemlicher Sicherheit schief. 🙂

  983. #985 Peter Strohmayer
    Wien
    19. Januar 2023

    @Anonym 2023 #980

    “Wenn wir den Fall 0 Grad betrachten, können wir uns die Diskussion über Aberration sparen und in diesem Spezialfall sowohl bei der relativistischen als auch bei der klassischen Rechnung wahlweise mit Teilchen oder mit Wellen argumentieren.”

    Bei 0° kommt die Frage nicht in den Blick, ob es sich bei Gamma um eine Rechenfaktor im Rahmen raumzeitlicher Entfernungen bzw. Abstände oder um eine geheimnisvolle “Zeitdialatation” handelt. Bei 90° vermochte eine oberflächliche Argumentation damit zu beeindrucken, es würde “normalerweise”, also ohne langsamer Vergehen von Zeit”, kein DE auftreten, und es bedürfte keiner gesonderten Betrachtung der dabei auftretenden räumlichen Verhältnisse. Dass es sich beim tr. rel. DE um ein ebenbürtiges Zusammenwirken von Raum und Zeit handelt (und nicht um ein bloßes “wer hat an der Uhr gedreht”), wird ausgeblendet. Daher lohnt die Auseinandersetzung mit dem klassischen transversalen DE mit Teilchen.

  984. #986 MartinB
    19. Januar 2023

    @Karl-Heinz
    Das stimmt und passiert gelegentlich – ich erinnere mich immer mit Grausen an den PhilosophieDozenten, der erzählte, die QM würde nicht gelten, weil ja die Bellsche Ungleichung verletzt sei, und alle Physiker würden das totschweigen….

  985. #987 Anonym_2023
    19. Januar 2023

    @Karl-Heinz #982

    “Können Philosophen die Relativitätstheorie verstehen?”

    Der Philosoph Henri Bergson, der sich mit Einstein zu einer öffentlichen Diskussion getroffen hatte, hat die die Relativitätstheorie nicht verstanden. Das sieht man z.B. an dem Unsinn, den er ab Seite 93 seines Buches “Dauer und Gleichzeitigkeit” über das Zug/Bahndamm-Gedankenexperiment von Einstein geschrieben hat:
    https://archive.org/details/DurationAndSimultaneityHenriBergson/page/n137/mode/2up

    Dagegen hat z.B. Hans Reichenbach die SRT verstanden, er war allerdings sowohl Philosoph als auch in erster Linie Physiker.

  986. #988 Anonym_2023
    19. Januar 2023

    @Peter Strohmayer #985

    “Bei 0° kommt die Frage nicht in den Blick, ob es sich bei Gamma um eine Rechenfaktor im Rahmen raumzeitlicher Entfernungen bzw. Abstände oder um eine geheimnisvolle “Zeitdialatation” handelt.”

    Die ZD ist genauso eine bezugssystemabhängige Größe wie z.B. die Einweg-Geschwindigkeit v. Ich sehe keinen Grund, warum das eine “geheimnisvoll” sein soll und das andere nicht.

  987. #989 Karl-Heinz
    Graz
    19. Januar 2023

    @Anonym_2023

    öffentlichen Diskussion getroffen

    Ich schließe mich dem an, was ein Leser bereits geschrieben hat.

    Jetzt hätte ich gerne nochmal den selben Artikel, nur halt von einem Physiker geschrieben, nicht von einem Philosophen.

    😉

  988. #990 Peter Strohmayer
    Wien
    20. Januar 2023

    @Anonym 2023 #988

    “Die ZD ist genauso eine bezugssystemabhängige Größe wie z.B. die Einweg-Geschwindigkeit v”.

    Dieser Satz ist unverständlich:
    – niemand stellt den ersten Aussageteil in Frage
    – der Betrag der Relativgeschwindikeit v ist nicht bezugssystemabhängig
    – die Eigenschaft “Einweg” macht bei v keinen Sinn
    – die “Einweglichtgeschwindigkeit” c wäre auch nicht bezugssystemabhängig.

    Was sagt uns dieser Vergleich? Unser Meinungsaustausch über die ZD hat sich erschöpft. Ich habe aber einiges gelernt.

    #987
    Wieder stoßen wir auf merkwürdige Erklärungen von Physikern, die Priestern gleichen und die Mysterien verteidigen, bis Macht und Glaube übrig bleiben:

    Sie tun Bergson unrecht. Er scheitert (wie viele andere), weil Einsteins Zugbeispiel unverständlich ist.

    Würde der Blitz bei Ursprungsdeckung in M und in M’ einschlagen und würden sich Photonen von M und von M’ aus in beide Richtungen ausbreiten, so kann die Relativität der Gleichzeitigkeit verstanden werden. Einige Lehrbücher stellen das auch so dar.

    Lässt man aber, wie Einstein, zwei Blitze an den Enden des Zuges und des Bahndammes “gleichzeitig” einschlagen und zieht man dann a l l e i n aus dem Umstand, dass ein Lichtpuls das Zusende früher, den Zugsanfang aber später erreicht, als Begründung für das Auseinanderfallen der Gleichzeitigkeit heran, so ist das unverständlich. Die korrekte Begründung wäre vielmehr die, dass an den Punkten der beiden Bezugssysteme, an denen die Blitze einschlagen sollen, bereits zum Zeitpunkt des Einschlagens keine Gleichzeitigkeit gegeben ist (wie die L-T zeigt) und dass das spätere Ausbreiten zu den Mittelpunkten nur noch hinzutritt. Wie soll man etwas verstehen, was stattgefunden hat, bevor das eigentliche Gedankenexperiment, dass die SRT erklären soll, einsetzt? Diese von vornherein gegebene Ungleichzeitigkeit fällt unter den Tisch. Ist Ihnen das nie aufgefallen?

    Bergson ist nicht der einzige, der davon verwirrt wurde, aber sicher einer, der sich am geistreichsten gegen diese Zumutung zur Wehr gesetzt hat. Konformist Reichenbach hat uns mM weniger zu sagen.

  989. #991 Peter Strohmayer
    Wien
    20. Januar 2023

    Korrektur:
    “… zieht man dann a l l e i n aus dem Umstand, dass ein Lichtpuls vom Zugsende den Mittelpunkt später, vom Zugsanfang aber früher erreicht, als Begründung für das Auseinanderfallen der Gleichzeitigkeit heran, so ist das unverständlich …”

  990. #992 Karl-Heinz
    Graz
    20. Januar 2023

    @Peter Strohmayer

    Wie misst du die Länge eines bewegten Objektes?
    Welche Möglichkeiten gibt es dafür?

  991. #993 Anonym_2023
    20. Januar 2023

    @Peter Strohmayer #990

    “Diese von vornherein gegebene Ungleichzeitigkeit fällt unter den Tisch. Ist Ihnen das nie aufgefallen?

    Nein, mir ist aber das Gegenteil davon aufgefallen:

    Are two events (e.g. the two strokes of lightning A and B) which are simultaneous with reference to the railway embankment also simultaneous relatively to the train? We shall show directly that the answer must be in the negative.

    Quelle:
    https://en.wikisource.org/wiki/Relativity:_The_Special_and_General_Theory/Part_I#Section_9_-_The_Relativity_of_Simultaneity

  992. #994 Karl-Heinz
    Graz
    20. Januar 2023

    @Anonym_2023

    Sind zwei Ereignisse (z. B. die beiden Blitze A und B), die in Bezug auf den Bahndamm gleichzeitig sind, auch in Bezug auf den Zug gleichzeitig? Wir werden direkt zeigen, dass die Antwort negativ ausfallen muss.

    Ja Genau,
    man kann die Gleichzeitigkeit in einem Bezugssystem exakt definieren. Anschließend kann man danach fragen, ob diese definierte Gleichzeitigkeit auch in einem anderen Bezugsystem als gleichzeitig wahrgenommen wird.

    Ich denke, Peter fühlt sich so überlegen, dass im dieser wirklich einfacher Sachverhalt gar nicht aufgefallen ist.

  993. #995 Peter Strohmayer
    Wien
    20. Januar 2023

    @Anonym 2023 #993

    “mir ist aber das Gegenteil davon aufgefallen”

    Das Gegenteil von was? Von der im Vorhinein gegebenen Ungleichzeitigkeit? Bestreiten Sie die im Ernst?

  994. #996 Karl-Heinz
    Graz,11.Bez.:Mariatrost
    20. Januar 2023

    @Peter Strohmayer

    Na lieber Peter.
    Bleiben wir mal bei der Gleichzeitigkeit. Ist einfacher.
    Braucht man den Begriff Gleichzeitigkeit oder braucht man ihn nicht?

    PS: zum Nachdenken
    Raumschiff A starte vom Ort A gleichzeitig wie Raunschiff B vom Ort B.
    Ein anderes Bezugsystem findet eventuell den Start der beiden Raketen nicht mehr gleichzeitig. Wie ist sowas möglich?

  995. #997 Karl-Heinz
    20. Januar 2023

    @Peter Strohmayer

    Was passt dir an der speziellen Relativitätstheorie nicht?
    Kanns du mir das kurz und bündig mit einfachen Worten erklären? Aus dem was du da schreibst werde ich nicht schlau.
    Deine Ansicht bzw. Privattheorie

  996. #998 Peter Strohmayer
    Wien
    20. Januar 2023

    Die beanstandete Argumentation Schritt für Schritt:

    – Wir (Einstein) werden nun „direkt zeigen“, dass zwei Blitzeinschläge, die in Bezug auf den Bahndamm gleichzeitig sind, in Bezug auf den Zug nicht gleichzeitig sind.
    – Beim Einschlag der Blitze fällt der Punkt M mit dem Punkt M’ zusammen.
    – Würde sich M’ nicht bewegen, dann würden ihn die Lichtpulse gleichzeitig erreichen.
    – M’ bewegt sich aber mit dem Zug nach rechts.
    – M’ eilt auf den Lichtstrahl von B zu, während er dem Lichtstrahl von A vorauseilt.
    – M’ sieht also den Lichtstrahl von B früher als den von A.
    – Damit wurde “direkt gezeigt”, dass die Ereignisse A und B nicht gleichzeitig stattfinden.

    M’ sieht die Lichtpulse aber nicht ungleichzeitig, weil er dem Licht davon- oder nachlaufen würde, sondern weil die Lichtpulse tatsächlich nicht gleichzeitig gestartet sind.

    Das, was Einstein “direkt zeigt”, ist, dass zwei Lichtpulse, die nicht gleichzeitig starten, nicht gleichzeitig in der Mitte ankommen.

    Das hätte der eine oder andere vielleicht auch schon vorher gewusst.

    Ich habe Einstein aber in Verdacht, er wollte nicht nur “direkt zeigen”, dass … , sondern warum die Ereignisse nicht gleichzeitig stattgefunden haben.

    Es gibt viele glückliche Geister, und vielleicht gehören auch Sie (Anonym) dazu, die überzeugt sind, mit dem “dass” auch das “warum” verstanden zu haben. Sie haben vielleicht auch kein Problem damit, dass er dabei mit Lichtpulsen Fangen spielt und ihnen davonläuft.

  997. #999 Karl-Heinz
    Graz
    20. Januar 2023

    Endlich ein konkretes Beispiel. Damit kann man super zeigen, dass die Argumentation von Einstein genial und plausibel ist.

  998. #1000 Anonym_2023
    21. Januar 2023

    @Peter Strohmayer #998

    “– Beim Einschlag der Blitze fällt der Punkt M mit dem Punkt M’ zusammen.”

    Diese Aussage enthält bereits die Annahme, dass beide Blitz-Ereignisse A und B gleichzeitig stattfinden. Daher muss es sich um eine bezugssystemabhängige Aussage handeln. Die Angabe eines Bezugssystems fehlt aber bei Ihnen.

    Sie habe in Ihrem Zitat die Fußnote “8” von Einstein unterschlagen, in der er das Bezugssystem genannt hat:

    Just when the flashes[8] of lightning occur, this point M’ naturally coincides with the point M but it moves towards the right in the diagram with the velocity v of the train

    8. As judged from the embankment

    Quelle:
    https://en.wikisource.org/wiki/Relativity:_The_Special_and_General_Theory/Part_I#Section_9_-_The_Relativity_of_Simultaneity

    “M’ sieht die Lichtpulse aber nicht ungleichzeitig, weil er dem Licht davon- oder nachlaufen würde, sondern weil die Lichtpulse tatsächlich nicht gleichzeitig gestartet sind.

    Hier vermischen Sie zwei Bezugssysteme bei bezugssystemabhängigen Größen. Das passiert durchgehend in Ihren Kommentaren. Den gleichen fatalen Fehler machte Henri Bergson.

  999. #1001 Peter Strohmayer
    Wien
    21. Januar 2023

    @Anonym 2023 #1000

    “Sie haben in Ihrem Zitat die Fußnote “8” von Einstein unterschlagen, in der er das Bezugssystem genannt hat:”

    Ich würde keinen Wert darin sehen, durch Unterschlagungen zu überzeugen.

    “Hier vermischen Sie zwei Bezugssysteme bei bezugssystemabhängigen Größen.”

    Nein, ich gebe Einstein wieder. Aber ich kann das auch ausführlicher machen:

    1. Aus der Sicht von M treffen die Lichtstrahlen der beiden Blitzeinschläge gleichzeitig bei ihm ein. Die Lichtstrahlen müssen gleichzeitig gestartet sein, denn sie haben für ihren jeweils gleich langen Weg zu M die gleiche Zeit benötigt. Daher haben die Blitze gleichzeitig eingeschlagen.

    2. Aus der Sicht von M’ müssen die Lichtstrahlen der beiden Blitzeinschläge nacheinander bei ihm eintreffen. Es kann hier keine Abweichung von der Auffassung des M geben, wonach Lichtstrahlen, die bei ihm gleichzeitig eintreffen, bei M’, der sich nach rechts bewegt, zu einem früheren bzw. späteren Zeitpunkt vorbeikommen.

    Um aber Rückschlüsse auf die Einschlagzeiten der Blitze selbst zu ziehen, benötigt M’ die Längen der Lichtwege von den Orten, an denen aus seiner Sicht die Einschläge stattfinden, bis zu ihm. Wo befinden sich diese Orte?

    Aus der Sicht des M ist er M’ begegnet, als die Blitze einschlugen, wobei er sich aus seiner Sicht in der Mitte zwischen den Einschlagorten befand, an denen aber gleichzeitig auch die Spitze bzw. das Ende des Zuges vorbeikamen. M’, der sich aus seiner Sicht in der Mitte des Zuges befindet, befindet sich damit ebenfalls in der Mitte zwischen den Einschlagorten, unabhängig davon, wann und wie weit entfernt die Blitze aus seiner Sicht einschlagen. (Sollte aus Sicht M’ das Vorbeikommen von Zusende bzw. Zugsanfang an den Einschlagstellen zu verschiedenen Zeitpunkten stattfinden, wäre die Entfernung der Einschlagsorte voneinander aus Sicht M eine andere als die Entfernung der Einschlagorte voneinander aus Sicht des M’. Beide würden sich aber immer und unter allen Umständen aus ihrer jeweiligen Sicht in der Mitte zwischen den Einschlagorten befinden.) Dass sich M und M’ während der Ausbreitung der Lichtstrahlen voneinander entfernen, sodass sie sich aus der Sicht des jeweils anderen bei dem Eintreffen der Lichtstrahlen nicht mehr in der Mitte befinden, ändert nichts an den genannten Einschlagsorten.

    Fazit: Aus der Sicht von M’ treffen die Lichtstrahlen der beiden Blitzeinschläge nacheinander bei ihm ein. Die Lichtstrahlen müssen nacheinander gestartet sein, denn sie haben für ihren jeweils gleich langen Weg zu M’ die gleiche Zeit benötigt. Daher haben die Blitze nicht gleichzeitig eingeschlagen.

    Einstein hat damit sein Versprechen eingehalten und direkt gezeigt, dass die Blitze aus Sicht des M gleichzeitig, jedoch aus Sicht des M’ nicht gleichzeitig eingeschlagen haben.

    3. Problematisch an Einsteins Darstellung finde ich, dass er das Schwergewicht auf das Zueilen bzw. Vorauseilen vor Lichtstrahlen, also auf eine scheinbare Relativgeschwindigkeit zwischen Beobachter und Lichtstrahl legt, wie sie im Rahmen der Verzeichnung von Weltlinien in einem Bezugssystem auftritt. Dieser Aspekt war sein Argument, weshalb die Lichtstrahlen nacheinander bei M’ eintreffen. Er taugt aber nicht auch noch als Argument dafür, dass aus der Sicht des M’ die Blitze selbst nacheinander einschlagen. Der mit dem Vor- und Nacheilen betonte Umstand, dass jeder Beobachter den jeweils anderen bei Eintreffen der Lichtpulse nicht in der Mitte der Einschlagsorte sieht, ist das glatte Gegenteil des Angelpunkts der Argumentation, wonach sich jeder Beobachter unter allen Umständen in der Mitte der Einschlagorte befindet und dass die Lichtwege und Lichtlaufzeiten jeweils gleich lang sind.

    Des Weiteren finde ich an dieser Darstellung problematisch, dass sie den Eindruck erweckt, als würde die Relativität der Gleichzeitigkeit mit der Ausbreitung der Lichtstrahlen von den Einschlagsorten zum jeweiligen Beobachter bzw. mit deren Bewegungen aus der Mitte zu tun haben. Demgegenüber zeigten die synchronisierten Uhren der beiden Systeme (die nach Einstein in den Punkten A, B, C bzw. A’, B’, C’ usw aufgestellt sind), die sich bei den Einschlagereignissen begegneten, bereits bei der Begegnung von M und M’ nicht die gleichen Zeiten an. Das nachfolgende Ausbreiten der Lichtstrahlen und nachfolgenden Bewegungen der Beobachter aus ihren Mitten haben darauf keinen Einfluss. Das Auseinanderfallen der Gleichzeitigkeit ist sohin bereits passiert, bevor dieses Gedankenexperiment einsetzt. Um nachzuvollziehen, wie es zu den unterschiedlichen Anzeigen der Begegnungsuhren kommt, muss man einfacher beginnen und die Ausbreitung von Lichtspulsen diskutieren, die von zwei Ursprüngen bei ihrer Begegnung ausgesendet werden (vgl. den Anhang des zur Rede stehenden Buches „Über die spezielle und allgemeine Relativitätstheorie (Gemeinverständlich)“, in dem bei der Ableitung der L-T von zwei sich in dieselbe Richtung ausgehenden Lichtstrahlen ausgegangen wird). Das Gedankenexperiment mit den zwei außerhalb der Ursprünge einschlagenden Blitzen zeigt zwar, dass etwas mit der absoluten Gleichzeitigkeit nicht stimmen kann, es ist aber sehr komplex, sodass es bei vielen wohl bloß das Gefühl vermittelt, etwas verstanden zu haben.

  1000. #1002 Karl-Heinz
    Graz
    22. Januar 2023

    Wenn man den Hilfsatz (I,II) verwenden, lässt sich obiges Gedankenspiel sehr leicht beweisen.

    (I): Ereignisse die am gleichen Ort gleichzeitig auftreten, treten in allen Bezugsystemen gleichzeitig auf.
    (II): Ereignisse die am gleichen Ort nicht gleichzeitig auftreten, treten in allen Bezugsystemen nicht gleichzeitig auf.

    Der kurze Beweis geht in etwa so.
    Der Blitz schlägt aus Sicht des Bahnsteiges in den beiden Enden des fahrenden Zuges gleichzeitig ein. Dann kann der Beobachter am Bahnsteig durch Überlegungen ableiten, das die beiden Lichtsignale nicht gleichzeitig in M’ eintreffen werden. Dann wendet man Hilfssatz (II) an und schon weiß man, ob sich die Blitze aus Sicht des Beobachters im Zug gleichzeitig in M’ eintreffen werden. Treffen aus Sicht des Beobachters im Zug die Blitze nicht gleichzeitig in M’ ein, bedeutet dies, dass aus Sicht des Beobachters im Zug, die Blitze an den beiden Enden nicht gleichzeitig gestartet sind.

    Bin echt gespannt, ob Peter dazu fähig ist diesen trivialen Beweis zu verstehen. 😉

  1001. #1003 Karl-Heinz
    Graz
    22. Januar 2023

    @Peter Strohmayer

    Das Gedankenexperiment mit den zwei außerhalb der Ursprünge einschlagenden Blitzen zeigt zwar, dass etwas mit der absoluten Gleichzeitigkeit nicht stimmen kann, es ist aber sehr komplex, sodass es bei vielen wohl bloß das Gefühl vermittelt, etwas verstanden zu haben.

    Das meinst du aber jetzt nicht im Ernst, oder etwa doch? Ich denke schon, dass jene die sich länger damit beschäftigen, auch das Warum verstehen.

  1002. #1004 Karl-Heinz
    Graz
    22. Januar 2023

    @Peter Strohmayer

    Du darfst den Beweis in #1002 gerne Kritisieren. Ich denke, dass dadurch weitere Missverständnisse deinerseits/meinerseits aufgedeckt werden.

  1003. #1005 Karl-Heinz
    Graz
    22. Januar 2023

    Jedenfalls Danke für die Diskussion. Man lernt immer etwas Neues dazu. Es ist denke an der Zeit sich was anderem zuzuwenden. Ich hoffen ich war in der Hitze des Gefechtes nicht allzu schroff.

  1004. #1006 Anonym_2023
    23. Januar 2023

    @Peter Strohmayer #1001

    “3. Problematisch an Einsteins Darstellung finde ich”

    Man kann Einsteins Gedankenexperiment Bahndamm/Zug in Kapitel 9 besser verstehen, wenn man zuvor auch das zugehörige Kapitel 8 “On the Idea of Time in Physics” gelesen hat. Praktisch ist das eine einfachere Variante der Einstein’schen Uhrensynchronisation, als sie in seinem Paper von 1905 beschrieben war.

    That light requires the same time to traverse the path A -> M as for the path B -> M is in reality neither a supposition nor a hypothesis about the physical nature of light, but a stipulation which I can make of my own freewill in order to arrive at a definition of simultaneity.”

    Quelle:
    https://en.wikisource.org/wiki/Relativity:_The_Special_and_General_Theory/Part_I#Section_8_-_On_the_Idea_of_Time_in_Physics

  1005. #1007 Anonym_2023
    24. Januar 2023

    @Peter Strohmayer #998

    “Sie haben vielleicht auch kein Problem damit, dass er dabei mit Lichtpulsen Fangen spielt und ihnen davonläuft.”

    Nein, dadurch wird vielmehr anschaulich gezeigt, dass sie beiden Lichtsignale ungleichzeigt bei M’ ankommen. Dass man daraus auf die Ungleichzeitigkeit der Blitzereignisse in Bezug auf das Ruhesystem des Zuges schließen kann, geht aus der Definition der Gleichzeitigkeit im vorhergehenden Kapitel 8 hervor.

  1006. #1008 Peter Strohmayer
    Wien
    24. Januar 2023

    @Anonym 2023 #1007

    Einsteins Argumentation ist in hohem Maße verwickelt:

    Beispiel: Blitze A und B schlagen in M gleichzeitig ein. Bahndamm 8, Zug v=0,6.

    ERSTER TEIL:
    Die Zeitpunkte der Blitzeinschläge werden, wie Einstein in § 8 ausführt, dadurch ermittelt, dass im jeweiligen Bezugssystem überall Uhren aufgestellt werden und die Zeitangabe derjenigen Uhr abgelesen wird, die dem jeweiligen Blitzeinschlag am nächsten ist.

    M:
    A x=-4, t=0
    B x=4, t=0

    Gleichzeitig.

    M’:
    A x’=-5, t’=3
    B x’=5, t’=-3

    Ungleichzeitig.

    ZWEITER TEIL:

    Dem beschriebenen Ablesen bei einer Uhr ist nach Einstein (§ 8) eine ursprünglichere Methode äquivalent, nach der die Einschlagzeiten über Lichtstrahlen ermittelt werden, die sich von den Einschlagereignissen über eine bestimmte Länge zu M bzw. M’ ausbreiten (MA=Eintreffen des Lichtstrahls A bei M, MB = Eintreffen des Lichtstrahls B bei M, usw):

    M
    MA: x=0, t=3
    MB: x=0, t=3

    Gleichzeitiges Eintreffen.

    M’
    MA’: x’=0, t’=8
    MB’: x’=0, t’=2

    Ungleichzeitiges Eintreffen.

    Aus den Zeiten des Eintreffens der Lichtsignale kann der Beobachter im Ursprung des jeweiligen Bezugssystems die Einschlagzeiten wie oben mit den Uhren ermitteln (bei M’ -3/3). Die Lichtsignale sind neutral. Sie beeinflussen die gegebene zeitlichen Lagerung der Einschlagereignisse nicht.

    2. Um das Auseinanderfallen der Gleichzeitigkeit („erster Teil“) zu demonstrieren, hätte Einstein ein Gedankenexperiment wählen können, in dem bei Zusammenfallen der Punkte M und M’ von M aus Lichtstrahlen an die beiden Enden des Fahrdamms gesendet werden, sodass mit deren gleichzeitigen Eintreffen Blitze einschlagen, wenn gerade die Zugsenden vorbeikommen. Das hätte gezeigt, dass und warum Ereignisse (hier: des Ankommens dieser Lichtstrahlen) im System M gleichzeitig und im System M’ nacheinander erfolgen müssen. Das Gedankenexperiment wäre mit dem „ersten Teil“ (oben) abgeschlossen.

    3. Einsteins Gedankenexperiment beschäftigt sich aber ausschließlich mit dem „zweiten Teil“ (oben), nämlich mit der – an die Blitzeinschläge anschließenden – Phase der Feststellung der Einschlagszeiten durch Lichtsignale. Mit einer Analyse der Ausbreitung der Boten-Lichtsignale von den gleichzeitigen Ereignissen hin zu M weist er nach, dass die Ereignisse in M’ – gegen jede Intuition – ungleichzeitig sein müssen. Statt den direkten Nachweis zu führen (oben 2.), wählt er einen sehr komplizierten.

    Werden statt Lichtsignalen gleichförmig bewegte Massepunkte mit bekannter Geschwindigkeit als Signale der Einschlagereignisse verwendet, so treffen die Signale aus Sicht des M nicht zugleich bei M’ ein. Die Intuition der Gleichzeitigkeit der Einschlagereignisse aus Sicht des M’ wird nun nicht widerlegt, wenn die Geschwindigkeiten der Masseteilchen und des M’ aus Sicht des M klassisch addiert werden, die Signale aus Sicht des M’ gleiche Wege in unterschiedlichen Zeiten durchlaufen und die Rückrechnung wieder Gleichzeitigkeit ergibt. Addiert man die Geschwindigkeiten relativistisch, ergibt sich aus den Eintreffenszeiten der Signale aus Sicht des M’ ein Nacheinander der Blitzeinschläge.

    Bei Lichtsignalen gibt es keinen großen Unterschied in der Argumentation: addiert man c klassisch zur Relativgeschwindigkeit, wird aus Sicht M’ die intuitive Gleichzeitigkeit bestätigt. Wird – nach Einsteins Definition der Gleichzeitigkeit – korrekt relativistisch addiert (es bleibt bei c), so MUSS bei den gegebenen Prämissen der Blitzeinschlag aus Sicht des M’ ungleichzeitig erfolgen. Es macht (außer für den Dopplereffekt) keinen Unterschied, ob die Lichtstrahlen von M oder von M’ ausgehen.

    Mit anderen Worten: das relativistische Zusammenspiel der Ausbreitung von Lichtpulsen, das dem „ersten Teil“ zu Grunde lag und zu einem Auseinanderfallen der Gleichzeitigkeit führte (vgl. oben Pkt. 2), tritt natürlich auch bei den Lichtsignalen des „zweiten Teils“ auf, was zur Folge hat, dass solche Signale nicht widerspruchsfrei von Ereignissen ausgehen können, die in beiden Systemen gleichzeitig sind.

    Respekt an alle, die das durchschauen.

    Ich bleibe dabei: Bergson scheiterte, weil Einsteins Zugsbeispiel unverständlich ist.

  1007. #1009 Karl-Heinz
    Graz
    24. Januar 2023

    @#1008

    Eigentlich wollte ich dazu nichts mehr schreiben. Aber was solls.
    Warum hat Einstein das Beispiel für Normalbürger so gewählt?

    * An beiden Zug-Enden (vorne und hinten) schlägt aus klassischer Sicht der Blitz gleichzeitig ein.

  1008. #1010 Karl-Heinz
    Graz
    24. Januar 2023

    Bei Sir 1008Peter schlägt ein Blitz genau in der Mitte vom Zug und Bahnsteig ein. Erreicht dann der Blitz in beiden Bezugssystem die beiden Enden vom Zug (vorne und hinten) gleichzeitig oder nicht?
    Sir Peter soll uns aufklären.

  1009. #1011 Anonym_2023
    24. Januar 2023

    @Peter Strohmayer #1008

    “Werden statt Lichtsignalen gleichförmig bewegte Massepunkte mit bekannter Geschwindigkeit als Signale der Einschlagereignisse verwendet, so treffen die Signale aus Sicht des M nicht zugleich bei M’ ein.”

    Da die Geschwindigkeit eines Massepunktes bezugssystemabhängig ist, fehlt hier die Angabe, auf welches System sich die “bekannte Geschwindigkeit” bezieht. Was ist hier das Bezugssystem?

  1010. #1012 Peter Strohmayer
    Wien
    25. Januar 2023

    @Anonym 2023 #1011

    Da das Zugsbeispiel Einsteins die Lichtsignale ja auch zuerst aus der Sicht M verzeichnet, kann man die Kontrollbetrachtungen mit Massepunkten damit beginnen, die Bewegungen der als Signale verwendeten Massepunkte von A bzw. von B zum Mittelpunkt M mit der gleich großen Geschwindigkeit w sowie die Bewegung von M’ mit v aus der Sicht des Systems M zu beurteilen.

  1011. #1013 Karl-Heinz
    25. Januar 2023

    @Anonym 2023

    Kannst du mir erklären warum P. die Geschwindigkeit w ins Spiel bringt?
    Die Geschwindigkeit w ist ja abhängig vom Bezugssystem während c invariant (bei veränderten Bedingungen unverändert bleibend) gegenüber Bezugssysteme ist.
    Besser ich sage jetzt nix was ich mir denke.

  1012. #1014 Anonym_2023
    25. Januar 2023

    @Peter Strohmayer #1008

    “Respekt an alle, die das durchschauen.”

    Ihr Szenario mit den Massepunkten ist ja auch schwerer zu durschauen als Einsteins Gedankenexperiment mit den Lichtsignalen.

    Die Lorentz-Transformation und das aus ihr abgeleitete Additionstheorem der Geschwindigkeiten kommen in seinem Buch erst in späteren Kapiteln vor.

    Eine Voraussetzung für die Gültigkeit der LT ist die Definition der Gleichzeitigkeit entsprechend Kapitel 8, falls man Zeit definiert als das, was eine Uhr anzeigt.

    Man kann die Gleichzeitigkeit auch anders definieren als in Kapitel 8, wie es z.B. Hans Reichenbach mit seinem ε beschrieben hat. Dann würde aber die LT verkompliziert.

  1013. #1015 Peter Strohmayer
    Wien
    27. Januar 2023

    @Anonym 2023 #1014

    „Die Lorentz-Transformation und das aus ihr abgeleitete Additionstheorem der Geschwindigkeiten kommen in seinem Buch erst in späteren Kapiteln vor.“

    Die Definition der Gleichzeitigkeit ist – anders als Sie meinen – nicht der für das Auseinanderfallen der Gleichzeitigkeit entscheidende Punkt. Entscheidend ist das Additionstheorem. Es ist von Anfang an das zentrale Thema (§ 7).

    Die Gleichzeitigkeit kann mit beliebigen Signalen definiert werden, sei es mit Lichtstrahlen, sei es mit Massepunkten. Dazu brauche ich keine Relativitätstheorie. Wenn die Signale von Quellen ausgesendet werden, die im System M’ ruhen – was nach der Definition der Gleichzeitigkeit in § 8 völlig korrekt wäre – , so wäre sich der fiktive Gesprächspartner Einsteins sicher, dass die Signale gleichzeitig bei M’ ankommen und dass daher die Blitze auch für M’ gleichzeitig einschlagen.

    Wenn aber Lichtstrahlen oder Massepunkte, die von einer im System M ruhenden Quelle ausgesendet werden („von den Blitzorten A und B ausgehende Lichtstrahlen“), von M’ als Signale verwendet werden, wird das das Additionstheorem benötigt, um dem Gedankenexperiment folgen zu können. Nur unter dieser Voraussetzung wird das gleichzeitige Einschlagen der Blitze bei M’ zum Widerspruch.

    Ich habe oben erwähnt, dass es (bei korrekt angewendetem Additionstheorem) experimentell keinen Unterschied machen würde, ob die Lichtstrahlen von Quellen des M oder denen des M’ ausgesendet werden. In beiden Fällen würde sich herausstellen, dass die Blitze bei M’ nacheinander einschlagen (was mit oben Teil 1 fest steht).

    Bei Lichtstrahlen würde M’ nicht am Ergebnis des Experiments, sondern nur am Dopplereffekt aus beiden Richtungen merken, dass die Lichtsignale nicht von seinen eigenen, sondern von den Quellen des M stammen. Also kann der Dopplereffekt nichts mit der Frage dr Gleichzeitigkeit zu tun haben. Sie sind ja ein Jünger der „Zeitdilatation“. Hat das langsamere Vergehen von Zeit – mit oder ohne Dopplereffekt – mit dem Auseinanderfallen der Gleichzeitigkeit zu tun, und wenn ja, was?

  1014. #1016 Karl-Heinz
    Graz
    27. Januar 2023

    @Peter Strohmayer

    Die Gleichzeitigkeit kann mit beliebigen Signalen definiert werden, sei es mit Lichtstrahlen, sei es mit Massepunkten.

    NEIN! NEIN! NEIN!
    Nehmen wir zwei Massenpunkte, die vom Mittelpunkt M sich mit gleicher Geschwindigkeit w entgegengesetzt wegbewegen. Für den Beobachter M mag damit eine Gleichzeitigkeit definiert sein. Für den bewegten Beobachter M’ aber auf keinen Fall, da sich für ihn die beiden Massenpunkte mit unterschiedlicher Geschwindigkeit bewegen. Licht bewegt sich aber immer mit c.

    PS: Es freut mich sehr, dass Anonym_2023 eine so ausgesprochen Geduld mit ihnen hat. 🙂

  1015. #1017 Anonym_2023
    27. Januar 2023

    @Peter Strohmayer #1015

    “Die Definition der Gleichzeitigkeit ist – anders als Sie meinen – nicht der für das Auseinanderfallen der Gleichzeitigkeit entscheidende Punkt.”

    Ich habe nicht geschrieben, dass ich das meine.

    “Entscheidend ist das Additionstheorem. Es ist von Anfang an das zentrale Thema (§ 7).”

    In §6 und §7 ist nur vom nicht-relativistischen Additionstheorem die Rede. Das relativistische Additionstheorem wird erst in §13 aus der LT hergeleitet.

    “Wenn die Signale von Quellen ausgesendet werden, die im System M’ ruhen – was nach der Definition der Gleichzeitigkeit in § 8 völlig korrekt wäre – , so wäre sich der fiktive Gesprächspartner Einsteins sicher, dass die Signale gleichzeitig bei M’ ankommen und dass daher die Blitze auch für M’ gleichzeitig einschlagen.”

    Bedingung dafür wäre, dass die Aussende-Ereignisse gleichzeitig in Bezug auf das Ruhesystem des Zuges erfolgten, was Einsteins Szenario widerspricht.

    “Hat das langsamere Vergehen von Zeit – mit oder ohne Dopplereffekt – mit dem Auseinanderfallen der Gleichzeitigkeit zu tun, und wenn ja, was?”

    Beide erscheinen als unterschiedliche Terme in der LT.

    Die LT für x enthält Terme für:
    (1) Längenkontraktion
    (2) Relativität der GleichOrtigkeit,

    die LT für die Zeit enthält Terme für:
    (3) Zeitdilatation
    (4) Relativität der Gleichzeitigkeit.

    Das Gedankenexperiment “Lichtuhr” ist durch die transversale Richtung der Lichtausbreitung so definiert, dass von den o.g. Effekten der LT nur folgende auftreten:
    (2) Relativität der GleichOrtigkeit
    (3) Zeitdilatation.
    Dadurch kommt die Argumentation für die ZD mit den beiden SRT-Postulation aus, ohne die volle LT bemühen zu müssen.

    Einsteins Gedankenexperiment “Relativität der Gleichzeitigkeit” ist durch die longitudinale Richtung der Lichtausbreitung so definiert, dass von den o.g. Effekten der LT nur folgende auftreten:
    (1) Längenkontraktion
    (4) Relativität der Gleichzeitigkeit.
    Dadurch kommt auch die Argumentation für die “Relativität der Gleichzeitigkeit” mit den beiden SRT-Postulation aus, ohne die volle LT bemühen zu müssen.

    Beide Gedankenexperiment sind komplementär zueinander.

    Gedankenexperiment “Lichtuhr”:
    Die zwei betrachteten Ereignisse (Reflexionen des Lichtimpulses am unteren Spiegel) finden in einem Bezugssystem am selben Ort, aber zu unterschiedlichen Zeiten statt.

    Gedankenexperiment “Relativität der Gleichzeitigkeit”:
    Die zwei betrachteten Ereignisse (Blitzereignisse A und B) finden in einem Bezugssystem zur selben Zeit, aber an unterschiedlichen Orten statt.

    Beide Gedankenexperiment benutzen außerdem Lichtimpulse statt Massepunkten, sodass das relativistische Additionstheorem für Geschwindigkeiten nicht benötigt wird, sondern stattdessen einfach SRT-Postulat 2.

  1016. #1018 Anonym_2023
    27. Januar 2023

    @Karl-Heinz #1016

    “NEIN! NEIN! NEIN!”

    Ich könnte zur Klärung dem @Peter Strohmayer folgende Ergänzung zu Einsteins Szenario vorschlagen.

    Vier Soldaten sind mit gleichartigen Gewehren und mit gleichartiger Munition ausgerüstet. Einer steht auf dem Bahnsteig am Ort A, einer auf dem Bahnsteig am Ort B, einer im Zug am Ort A’ und einer im Zug am Ort B’.

    Die Soldaten auf dem Bahnsteig erhalten jeweils den Befehl, auf den Beobachter am Ort M zu schießen, wenn sie vom Blitz getroffen werden.

    Die Soldaten im Zug erhalten jeweils den Befehl, auf den Beobachter am Ort M’ zu schießen, wenn sie vom Blitz getroffen werden.

    Die folgende Beschreibung bezieht sich immer auf das Ruhesystem des Bahnsteiges.

    Der linke Blitz trifft die beiden Soldaten bei A und A’.
    Der rechte Blitz trifft die beiden Soldaten bei B und B’.

    Der Beobachter bei M wird zunächst gleichzeitig von beiden Lichtsignalen getroffen und kurz danach gleichzeitig von 2 Gewehrkugeln.

    Der Beobachter bei M’ wird zunächst ungleichzeitig von beiden Lichtsignalen getroffen und kurz danach ungleichzeitig von 2 Gewehrkugeln.

    Davon kann sich @Peter Strohmayer überzeugen, wenn er die Geschwindigkeiten der von A’ und B’ abgeschossenen Gewehrkugeln mit dem relativistischen Additionstheorem vom Ruhesystem des Zuges ins Ruhesystem des Bahnsteiges transformiert. Dann kann er im Ruhesystem des Bahnsteiges das “Fangenspielen und davonlaufen” (siehe #998) mit den Gewehrkugeln berechnen.

  1017. #1019 Peter Strohmayer
    Wien
    28. Januar 2023

    @Anonym 2023 #1018

    “Der Beobachter bei M wird zunächst gleichzeitig von beiden Lichtsignalen getroffen und kurz danach gleichzeitig von 2 Gewehrkugeln [von im System M ruhenden Gewehren].

    Der Beobachter bei M’ wird zunächst ungleichzeitig von beiden Lichtsignalen getroffen und kurz danach ungleichzeitig von 2 Gewehrkugeln [von im System M’ ruhenden Gewehren].”

    Genau so ist es.

    Ich wundere mich, wieso Sie sich zu dieser Ergänzung zu Einsteins Szenario durchringen konnten. Das widerspricht Ihrer sonstigen Gewohnheit, alles an sich bzw. an Zitaten abprallen zu lassen.

    Ihr weiterer Beitrag, wie ich mich von meiner eigenen Meinung überzeugen sollte, passt besser zum bisherigen Eindruck. Wozu sollte man mit dem “relativistischen Additionstheorem” die Bahnen der Gewehrkugeln des gegenbeteiligten Systems im eigenen System berechnen, die man hier gar nicht als Signale verwendet? Das “Fangenspielen und Davonlaufen” ist eines gegenüber den Quellen. Es kommt in diesem Beispiel gar nicht vor, weil die Quellen jeweils ruhen.

  1018. #1020 Karl-Heinz
    Graz
    28. Januar 2023

    Ich hatte gestern mit jemand eine Diskussion, der behauptete, dass der Mond keine Eigenrotation hat. Meine Frage an ihn war dann, in Bezug auf was rotiert der Mond. Er war dann ziemlich verärgert und warf mir an den Kopf, dass Konvektionen ihn nicht interessiere würde und er sich ihrer nicht unterwerfen werde. Ich muß zugeben, ich war ziemlich schockiert von so einem Unverständnis.

    So nun versuche ich mein hämisch Grinsen zu #1019 zu unterdrücken. Ich verschärfe das Ganze noch etwas, indem ich voraussetze, wenn M und M’ am gleichen Ort sind, dass von dort aus jeweils ein Lichtblitz zur jeweils ruhenenden Lichtquelle des Bezugssystems ausgesand wird. Damit ist die Gleichzeitigkeit durch die Konvention eindeutig. Es ist mir nicht möglich über das Addition der Geschwindigkeit bzw. Dopplereffekt etwas rauszurechnen und das in meiner Signallaufzeit zu berücksichtigen. Mit den Gewehrkugeln ist das möglich, solange ihre Geschwindigkeit weit unter der Lichtgeschwindigkeit ist, wodurch die Konventionen bezüglich Gleichzeitigkeit hier natürlich nicht mehr gegeben ist.

  1019. #1021 Anonym_2023
    28. Januar 2023

    @Peter Strohmayer #1019

    “Wozu sollte man mit dem “relativistischen Additionstheorem” die Bahnen der Gewehrkugeln des gegenbeteiligten Systems im eigenen System berechnen, die man hier gar nicht als Signale verwendet?”

    Sie sollten jetzt zur Übung die Lichtsignale ignorieren (z.B. annehmen, die Beobachter bei M und bei M’ seien blind) und die Gewehrkugeln als Signale ansehen.

    “Das “Fangenspielen und Davonlaufen” ist eines gegenüber den Quellen. Es kommt in diesem Beispiel gar nicht vor, weil die Quellen jeweils ruhen.”

    Es kommt sehr wohl in diesem Beispiel vor, weil die Gewehre im Zug bei A’ und bei B’ als Quellen nicht ruhen im genannten Bezugssystem, dem Ruhesystem des Bahnsteigs.

    Dass die Blitzereignisse ungleichzeitig in Bezug auf das Ruhesystem des Zuges stattfinden, soll nicht eine Annahme sein, sondern als Endergebnis einer Berechnung gezeigt werden. Angenommen werden soll nur, dass die Blitzereignisse gleichzeitig in Bezug auf das Ruhesystem des Bahnsteigs stattfinden.

    Als Zwischenergebnis einer Berechnung soll in Bezug auf das Ruhesystem des Bahnsteigs berechnet werden, dass der Beobachter bei M’ ungleichzeitig von 2 Gewehrkugeln getroffen wird. Dafür kann das relativistische Additionstheorem zusammen mit dem “Fangenspielen und Davonlaufen” benutzt werden.

  1020. #1022 Peter Strohmayer
    Wien
    28. Januar 2023

    @ Anonym 2023 #1017

    „Beide [langsameres Vergehen von Zeit und Auseinanderfallen von Gleichzeitigkeit] erscheinen als unterschiedliche Terme in der LT.“

    Was ich von einem Verfechter einer tatsächlich auftretenden „Zeitdilatation“ erwartet hätte, wäre eine Argumentation in der Art „weil bei B die Zeit langsamer vergeht, tritt bei A ein transversaler DE ein“, bzw. „weil bei M’ die Zeit langsamer vergeht, erfolgen bei ihm die Blitzeinschläge nacheinander“. Dass (auch) letzteres nicht funktioniert kann, wäre ein hilfreiches Eingeständnis gewesen.

    Mit der Darstellung der LT, bestätigen Sie, wohl ohne es zu beabsichtigen, dass es sich bei der „Zeitdilatation“ und „Längenkontraktion“ um Rechenfaktoren (Oberbegriff: „Terme“) handelt.

    Ist in einem System die räumliche Komponente eines zeitartigen Abstands zweier Ereignisse null, dann ist sie im anderen nicht. Ist in einem System die zeitliche Komponente eines raumartigen Abstands zweier Ereignisse null, dann ist sie im anderen nicht.

    Die beiden Komponenten der Abstände entwickeln sich in beiden Systemen symmetrisch. Es kommt nicht darauf an, wer sich bewegt, sondern darauf, was sich ereignet. In keinem System „vergeht die Zeit langsamer“, in keinem „zieht sich der Raum zusammen“. Das ist antiquiertes Denken, in der die Spekulationen aus der Zeit Fitzgeralds und Lorentz weiter leben.

    So bei der Lichtuhr: hier haben wir z w e i Raumdimensionen, e i n e Zeitdimension, d r e i Ereignisse, z w e i lichtartige Abstände und e i n e n zeitartigen Abstand, der zeitlich u n d räumlich durch die LT transformiert wird. Die räumliche Komponente dieses Abstands ist in S Null („Gleichortigkeit“) und in S’ nicht Null, die zeitliche Komponente dieses Abstands ist in S kürzer als in S’. Beide Transformationen sind untrennbar miteinander verknüpft, eine kann ohne die andere nicht existieren.

    So beim – amputierten und auf Teil 2 reduzierten – Zugsbeispiel: hier haben wir – im Unterschied zur Lichtuhr – e i n e Raumdimension, e i n e Zeitdimension, z w e i Ereignisse und e i n e n raumartigen Abstand. (Wir hätten in oben Teil 1 – analog zur Lichtuhr – auch ein drittes Ereignis, die Aussendung zweier Lichtpulse vom M, und demgemäß auch z w e i lichtartige Abstände, aber das wird von Einstein ausgeblendet.) Dieser raumartige Abstand wird durch die LT zeitlich und räumlich transformiert. Die zeitliche Komponente dieses Abstands ist in M Null („Gleichzeitigkeit“) und in M’ nicht Null, die räumliche Komponente dieses Abstands ist in M kürzer als in M’. Beide Transformationen sind untrennbar miteinander verknüpft, eine kann ohne die andere nicht existieren.

    Dass von den „Effekten“ der LT bei zeitartigen Abständen nur eine „Zeitdilatation“ (bzw. deren Term) und bei raumartigen Abständen nur eine „Längenkontraktion“ (bzw. deren Term) auftreten würde, „ohne die volle LT bemühen zu müssen“, ist unrichtig. Es zeigt sich auch hier, dass es solche isoliert gedachten und benannten Effekte nicht gibt. Ein solches Denken ist von der newtonschen Mechanik und seinem getrennten Raum- und Zeitbegriff kontaminiert.

  1021. #1023 Peter Strohmayer
    Wien
    28. Januar 2023

    @ Anonym 2023 #1017

    „In §6 und §7 ist nur vom nicht-relativistischen Additionstheorem die Rede. Das relativistische Additionstheorem wird erst in §13 aus der LT hergeleitet.“

    Das „Ausbreitungsgesetz des Lichts“, von dem in § 7 die Rede ist, ist ein relativistisches Additionstheorem.

  1022. #1024 Peter Strohmayer
    Wien
    28. Januar 2023

    @ Anonym 2023 #1021

    “Es [das Fangenspielen und Davonlaufen] kommt sehr wohl in diesem Beispiel [#1018] vor, weil die Gewehre im Zug bei A’ und bei B’ als Quellen nicht ruhen im genannten Bezugssystem, dem Ruhesystem des Bahnsteigs.”

    Jetzt wechseln Sie von ihrer Konstellation mit vier Soldaten (#1018), bei denen jedes System Signale verwendet, die von im eigenen System ruhenden Quellen ausgesendet werden, zu einer Konstellation mit zwei Soldaten, bei denen ein System (M) Signale verwendet, die von im eigenen System ruhenden Quellen ausgesendet werden und das andere System (M’) Signale verwendet, die von Quellen ausgesendet werden, die im anderen System (M) ruhen. In der zweiten Konstellation benötige ich das relativistische Additionstheorem. Das habe ich in #1008 und #1015 gesagt. Bei #1018 benötige ich es nicht.

  1023. #1025 Anonym_2023
    28. Januar 2023

    @Peter Strohmayer #1023

    “Das ‘Ausbreitungsgesetz des Lichts’, von dem in § 7 die Rede ist, ist ein relativistisches Additionstheorem.”

    Nein. Das Ausbreitungsgesetz des Lichts steht im 2. SRT-Postulat. Man kann aus beiden Postulaten die LT herleiten (siehe Anhang 1, die Ergänzung zu §11) und man kann dann aus der LT das relativistische Additionstheorem herleiten (siehe §13).

  1024. #1026 Anonym_2023
    28. Januar 2023

    @Peter Strohmayer #1024

    “Jetzt wechseln Sie von ihrer Konstellation”

    Nein. Ich wechsele nicht die Konstellation.

  1025. #1027 Jolly
    28. Januar 2023

    @Karl-Heinz

    dass Konvektionen ihn nicht interessiere würde und er sich ihrer nicht unterwerfen werde

    Egal ob Sprach-, Schreib-, oder Denkfehler, schockieren kann mich sowas nicht mehr, Konvention hin, Konvektion her. Immerhin war dein Gesprächspartner ja kein Flachmondler.

  1026. #1028 Karl-Heinz
    Graz
    28. Januar 2023

    @Jolly

    Scherzkeks 😉
    Aber du hast Recht. Wenn schon eine flache Erde warum nicht auch einen flachen Mond.

  1027. #1029 Karl-Heinz
    Graz
    28. Januar 2023

    @Jolly

    Die blöde Autokorrektur. Da wird schon mal aus einer Konvention eine Konvektion.

  1028. #1030 Anonym_2023
    28. Januar 2023

    @Peter Strohmayer #1022

    “‘weil bei M’ die Zeit langsamer vergeht, erfolgen bei ihm die Blitzeinschläge nacheinander’. Dass (auch) letzteres nicht funktioniert kann, wäre ein hilfreiches Eingeständnis gewesen.”

    Das funktioniert in der Tat nicht, aber weil diese beiden Ereignisse raumartig getrennt sind, d.h. es kann kein Bezugssystem geben, in dem beide Ereignisse am selben Ort stattfinden. Das ist kein Eingeständnis.

    Ich habe allerdings beim Schreiben von #1017 vergessen, dass es im Gedankenexperiment zur “Relativität der Gleichzeitigkeit” noch mindestens ein weiteres Ereignispaar gibt:
    – Ereignis Eazm: “Das Lichtsignal des linken Blitzes erreicht die Zugmitte”,
    – Ereignis Ebzm: “Das Lichtsignal des rechten Blitzes erreicht die Zugmitte”.

    Für den Zeitabstand zwischen diesen Ereignissen gibt es in der Tat eine ZD mit dem Faktor γ zwischen den beiden Bezugssystemen, weil beide Ereignisse in einem Bezugssystem am selben Ort stattfinden.

    Siehe hierzu auch den letzten Abschnitt von §12.

  1029. #1031 Peter Strohmayer
    Wien
    28. Januar 2023

    @ Anonym 2023 #1030

    “Für den Zeitabstand zwischen diesen Ereignissen [des Eintreffens der Lichtsignale bei M’] gibt es in der Tat eine ZD mit dem Faktor γ”

    Nach den Beträgen in #1008 beträgt der Zeitabstand zwischen dem Eintreffen der Lichtsignale bei M’ “6”. Gamma ist 1,25. Wo ist die zeitliche Entfernung zweier Ereignisse aus der Sicht des Bezugssystems M, die mit dem Faktor 1,25 auf 6 “dilatiert” worden wäre?

    Was Sie möglicherweise zum Ausdruck bringen wollten, ist die Veränderung der zeitlichen Komponente des lichtartigen Abstands des Ereignisses vom Aussenden eines Lichtsignals bis zu dessen Eintreffen (aus Sicht M jeweils “4”, aus Sicht M’ jeweils “5”), die niemand in Zweifel zieht.

    Übrigens ist mit in #1022 ein Irrtum unterlaufen: Der Satz “Ist in einem System die zeitliche Komponente eines raumartigen Abstands zweier Ereignisse null, dann ist sie im anderen nicht.” stimmt nicht immer. Es gibt raumartig entfernte Ereignisse, bei denen sowohl in M als auch in M’ die zeitliche Entfernung null ist, also Ereignisse, die sowohl in M als auch in M’ gleichzeitig stattfinden, wo also die Ereignisse des Eintreffens von Lichtsignalen dieser Ereignisse bei M bzw. bei M’ keine zeitliche Entfernung aufweisen.
    Wo bleibt da die “Zeitdilatation” im Verhältnis bewegter Systeme zueinander?

  1030. #1032 Anonym_2023
    29. Januar 2023

    @Peter Strohmayer #1031

    “Ereignisse, die sowohl in M als auch in M’ gleichzeitig stattfinden

    Wo bleibt da die “Zeitdilatation” im Verhältnis bewegter Systeme zueinander?”

    Da hier zwei Ereignisse zu einem Ereignis zusammenfallen, handelt es sich bzgl. der Zeitdilatation der Beobachters im Zug bei M’ in Bezug auf das Ruhesystem des Bahndamms um den Spezialfall
    0 * γ = 0.

    Herleiten kann mal diese ZD z.B. mit der LT und zwei unterschiedlichen Ticks der Uhr des Beobachters bei M’.

  1031. #1033 Peter Strohmayer
    Wien
    29. Januar 2023

    @ Anonym 2023 #1030

    “Da hier [wenn zwei Ereignisse in M und M’ gleichzeitig sind] zwei Ereignisse zu einem Ereignis zusammenfallen”

    Dann wäre es ja nicht nur kein raumartiger Abstand, sondern gar kein Abstand. Das ist nicht die Situation. Die normal zur Bewegungsachse stehenden Ereignisse, die in M und M’ gleichzeitig sind und die die gleiche räumliche Entfernung voneinander haben, sind räumlich getrennt. Wo bleiben hier ihre isolierten Vorgänge einer Zeitdilatation und einer Raumkontraktion?

    “0*γ = 0.”

    Wenn Sie nur ein Ereignis zur Verfügung haben, um es zu transformieren, können sich nicht die räumlichen und zeitliche Entfernungen zwischen zwei Ereignissen ändern.

    So schon #1017:

    “Die LT für x enthält Terme für:
    (1) Längenkontraktion
    (2) Relativität der GleichOrtigkeit,
    die LT für die Zeit enthält Terme für:
    (3) Zeitdilatation
    (4) Relativität der Gleichzeitigkeit.”

    Die LT, wie Sie sie hier darstellen, bezieht sich nur auf ein Ereignis. Da transformieren sich keine Entfernungen. Um ihre Namensgebung zu rechtfertigen, müssen Sie die LT auf zwei Ereignisse anwenden und dann herausarbeiten, warum Sie meinen, dass ein Term für das zuständig wäre und ein anderer für jenes, und dass diese Terme in der Realität unabhängig voneinander ihr Werk verrichten.

  1032. #1034 Anonym_2023
    29. Januar 2023

    @Peter Strohmayer #1033

    “Dann wäre es ja nicht nur kein raumartiger Abstand, sondern gar kein Abstand. Das ist nicht die Situation.”

    Doch! Ich meinte von diesen Eintreff-Ereignisse die bei bei M’:

    “wo also die Ereignisse des Eintreffens von Lichtsignalen dieser Ereignisse bei M bzw. bei M’ keine zeitliche Entfernung aufweisen.”

  1033. #1035 Anonym_2023
    29. Januar 2023

    @Peter Strohmayer #1001

    “Aus der Sicht des M ist er M’ begegnet, als die Blitze einschlugen, wobei er sich aus seiner Sicht in der Mitte zwischen den Einschlagorten befand, an denen aber gleichzeitig auch die Spitze bzw. das Ende des Zuges vorbeikamen.”

    Einstein hat in §9 nur von einem “sehr langen Zug” gesprochen. Eine genaue Längenangabe war auch nicht nötig.

    Aus dem was Sie schreiben geht aber zusätzlich hervor, dass der in Bezug auf das Ruhesystem des Bahnsteiges fahrende und damit längenkontrahierte Zug genauso lang ist wie der Abstand zwischen A und B in Bezug auf das Ruhesystem des Bahnsteiges. D.h. man kann die beiden Blitzereignisse zusammen mit der LT zur Herleitung der Längenkontraktion benutzen.

    Der Zug kann zur Veranschaulichung der LK dienen, insbesondere, wenn die Blitze Markierungen auf dem Bahnsteig und an den Zugenden hinterlassen.

    Im Ruhesystem des Zuges sind sowohl das zeitliche Intervall als auch der räumliche Abstand zwischen den Blitzereignissen größer als im Ruhesystem des Bahnsteiges.

  1034. #1036 Anonym_2023
    29. Januar 2023

    @Peter Strohmayer #1022

    “So bei der Lichtuhr: hier haben wir z w e i Raumdimensionen, e i n e Zeitdimension, d r e i Ereignisse, z w e i lichtartige Abstände und e i n e n zeitartigen Abstand, der zeitlich u n d räumlich durch die LT transformiert wird. Die räumliche Komponente dieses Abstands ist in S Null („Gleichortigkeit“) und in S’ nicht Null, die zeitliche Komponente dieses Abstands ist in S kürzer als in S’.”

    Richtig! Wenn sich jetzt über der Lichtuhr an der selben x-Koordinate wie die Lichtuhr noch ein Eierkocher befindet, dann braucht das Ei eine bestimmte Anzahl von Ticks der Lichtuhr, bis es hart gekocht ist. D.h. bezogen auf das System S’ braucht das Ei dafür um den Faktor γ mehr Zeit. Die ZD betrifft auch kontinuierlich ablaufende physikalische Prozesse, nicht nur den Zeitabstand zwischen zwei diskreten Ereignissen.

    “Beide Transformationen sind untrennbar miteinander verknüpft, eine kann ohne die andere nicht existieren.”

    Richtig! Damit meinen Sie warscheinlich die Effekte “Relativität der GleichOrtigkeit” und “Zeitdilatation”.

  1035. #1037 Peter Strohmayer
    Wien
    30. Januar 2023

    @Anonym 2023

    #1035: Ja, das ist die Situation nach Einsteins Skizze.

    #1036: Alles schön und gut, aber die daran geknüpfte Erzählung ist irreführend. Ein “Eierkocher” ist räumlich ausgedehnt, und so kommt es für jeden Raumpunkt zu einer zeitlich verschobenen “Zeitdilatation”. Ein raumzeitlich “aufgefächerten” Eierkocher ist kein Küchengerät, in dem Eier langsamer kochen. Das ist schwärmerisches Denken.

    #1017: “Beide Gedankenexperimente sind komplementär zueinander.”

    Das ist korrekt.

    Lichtuhr und Zugsbeispiel sind Versatzstücke des Archetyps des Aussendens, der Reflexion und der Rückkehr von Lichtsignalen, der allen Gedankenexperimenten der SRT zu Grunde liegt:

    Bei nur einer räumlichen Dimension handelt es sich um folgenden archetypische Vorgang:

    1) Von M wird beim Ereignis A nach links und rechts instantan je ein Lichtpuls ausgesendet.

    2) Die beiden Lichtpulse treffen nach einer Ausbreitung von jeweils x gleichzeitig bei den Ereignissen B und C ein.

    3) Dort werden die Lichtpulse reflektiert (bei den Ereignissen B bzw. C wieder ausgesendet).

    4) Die Lichtpulse kehren nach einer Ausbreitung von jeweils x beim Ereignis D instantan bei M ein.

    Aus der Lorentz-Transformation der Längen der Ausbreitung der Lichtpulse (bei gleichem lichtartigen Abstand der Ereignisse des Aussendens und des Eintreffens) von A (bei diesem Ereignis ist auch die Ursprungsdeckung von M und M’ anzusetzen) nach B, B nach D bzw. A nach C, C nach D, ergeben sich die Längen der Ausbreitungen der komplementären Lichtpulse aus der Sicht von M’, die jeweils den gleichen raumzeitlichen Abstand wie die in M haben.

    Unter der Ausbreitung der “komplementären Lichtpulse” sind jene Lichtpulse zu verstehen, die von in M ruhenden Lichtquellen stammen und von M’ “gesehen” werden, bzw. – was raumzeitlich dasselbe ist – jene Lichtpulse, die bei den Ereignissen von in M’ ruhenden Lichtquellen so ausgesendet werden, dass sich die Fronten der Lichtpulse M und M’ gemeinsam ausbreiten.

    Die Zusammensetzung dieser lichtartigen Abstände ergibt dann den r a u m a r t i g e n Abstand zwischen den Ereignissen B und C sowie den z e i t a r t i g e n Abstand zwischen A und D. In den Ereignissen B und C steckt Einsteins (im Hinblick auf die Unterschlagung des Ereignisses A) amputiertes Zugsbeispiel, in den Ereignissen A und D steckt die – hier in die Horizontale gelegte – nicht amputierte Lichtuhr.

    Der einheitliche raumzeitliche Ablauf (ohne die Gedankenkonstrukte einer isolierten Zeitdilatation und einer isolierten Raumkontraktion) wird an den aneinandergefügten, Lorentz-transformierten lichtartigen Abständen deutlich, die bei diesem Ablauf auftreten (vgl. das Zusammenspiel lichtartiger Abstände in #912).

    Das Ganze kann man auch auf zwei Raumdimensionen mit gegenläufigen Lichtpulspaaren in alle Richtungen eines Kreises erweitern, bei der dann auch die normal zur Bewegungsachse stehenden Ereignisse auftreten (#1033), die in beiden Systemen gleichzeitig sind.

  1036. #1038 Peter Strohmayer
    Wien
    30. Januar 2023

    … und wo dann auch die Lichtuhr in vertrauter Gestalt erscheint.

  1037. #1039 Peter Strohmayer
    Wien
    30. Januar 2023

    … und aus denen für alle Einstrahlrichtungen die Dopplereffekte mit ihren Aberrationswinkeln abgelesen werden können.